Chapter 26 Respiratory System, BH7 HEME, L7 CARDIAC, L7 26: Nursing Assessment: Respiratory System, Ch 24: Management of Patients with CPD, Ch 22: Management of Patients with Upper Respiratory Tract Disorders, Ch 21: Respiratory Care Modalities, L9 2...

Pataasin ang iyong marka sa homework at exams ngayon gamit ang Quizwiz!

d (Rationale: The pain and decreased urine output suggest a renal artery embolism, and monitoring of renal function is needed. The data are not consistent with the complications of infection, hypovolemia, or bleeding.)

Several hours after an open surgical repair of an abdominal aortic aneurysm, the patient develops a urinary output of 20 mL/hr for 2 hours. The nurse notifies the health care provider and anticipates orders for A. an additional antibiotic. B. a white blood cell (WBC) count. C. a decrease in IV infusion rate. D. a blood urea nitrogen (BUN) level.

Calcium Channel blocker

nifedipine (Procardia, Adalat), verapamil (Calan, Isoptin), amilodipine ( Norvasc)- treat hypertension- check pulse.

congestive heart failure

older term, not all heart failure causes pulmonary congestion new terms: acute vs. chronic heart failure

Ejection fraction, normal

the amount of blood ejected from the left ventricle with each contraction, normally 55-60%

Tx for a pt w/ uncomplicated aortic dissection is successful when:

Pain is relieved - sign it's stabilized.

A 54-year-old patient admitted with type 2 diabetes, asks the nurse what "type 2" means. Which of the following is the most appropriate response by the nurse? "1. ""With type 2 diabetes, the body of the pancreas becomes inflamed." 2. "With type 2 diabetes, insulin secretion is decreased and insulin resistance is increased." 3. "With type 2 diabetes, the patient is totally dependent on an outside source of insulin." 4. "With type 2 diabetes, the body produces autoantibodies that destroy b-cells in the pancreas.""

"Right Answer: 2 Rationale: In type 2 diabetes mellitus, the secretion of insulin by the pancreas is reduced and/or the cells of the body become resistant to insulin"

What insulin type can be given by IV? Select all that apply: "A. Glipizide (Glucotrol) B. Lispro (Humalog) C. NPH insulin D. Glargine (Lantus) E. Regular insulin

"What insulin type can be given by IV? Select all that apply: "A. Glipizide (Glucotrol) B. Lispro (Humalog) C. NPH insulin D. Glargine (Lantus) E. Regular insulin E) Regular insulinThe only insulin that can be given by IV is regular insulin.

SNS activation (compensatory mechanism)

-triggerend by low CO -least effective -release epinephrine and norepinephrine -increased HR -increased myocardial contractility and vasoconstriction

cachexia

-wasting, muscle loss, atrophy -weakness, fatigue

What are common complications of chronic venous insufficiency?

- DVT - vercisoty

Appropriate teaching for the patient w/ PAD includes (3)

- keep legs and feet warm - walk @ least 30 min/d until the point of discomfort - inspect LEs for pulses, temp, and swelling

Symptoms, Cause, TX of Arteriosclerosis/ Atherosclerosis

- Intermittent claudication,decrease circulation, skin color changes, coolness in extremities, dizziness, loss of memory.- Age, obesity, tobacco, Diabetes. -Wt. control, low cholesterol/fat diet, decrease stress.

What meds tx Bueger's? Surgery?

- Maybe antiplatelets - ABS, sympathectomy to prevent vasoconstriction

What can be done to prevent?

- ankle circles - SCDs - walking - heparin/lovenox

What teaching is appropriate for patients w/ venous ulcers? (6)

- avoid prolonged sitting/standing - use graduated compression hosiery - fx elevation of legs - wet compresses to weeping dermatitis 4x daily for 1 hour - topical corticosteroids for discoloration

What dx are appropriate for pt's w/ varicose veins?

- chronic pain - Risk for impaired skin integrity - Risk for peripheral neurovascular dysfunction.

Venous stasis ulcers occur from what directly?

- damage to leafelets - blood pools - venous P increases - circulation is impaired - cells die r/t hypoxia - venous stasis ulcers occur

A patient has a DVT and is put on heparin and coumadin. When will the clot disentigrate? How long before coumadin is active? What can be done to prevent further edema?

- disintegrate in 7-10 days - active in 5-10 days - elevate leg to prevent further edema

What are diagnostic for venous thrombosis?

- doppler - ultrasound

What dx are used r/t arterial ulcers of the LE? (3)

- doppler (shows plaque location) - ABI - Angiography

What teachings are important w/ PAD of the LEs?

- foot care - elevate HOB on blocks (rest pain) - smoking cessation - regular, progressive exercise (30-45 min/day) - rest w/ claudication - control DM, HTN, lipids

What are options for revascularization?

- non-surgical - PTA (stent, arthrectomy, balloon) - surgical - gravt or bypass

What is dx for Raynaud's? Meds?

- none - CCBs, alpha blockers

What are risk factors for venous thrombi? (3)

- orthopedic procedures - abd/thoracic surgeries - cancers - trauma - oral contraceptives (^ DVT by 50%)

What are the 6 p's of arterial ischemia?

- pain - pallor - paralysis - pulselessness - paresthesia - poikilothermia

What teaching is imp. for endarterectomy?

- procedure - turn/cough/deep breath - incentive spirometry - leg straight - watch for infection - meds

Once blood is restored, plan to:

- put on IV heparin - oral anticoagulants for at least 6 months

What treatments are effective for varicosities?

- sclerotherapy (makes veins not as large) - lasers - x-rays - venous ligation (pull out tortuous vein) * need compression stockings until they develop collateral circulation)

What are the 4 risk factors for peripheral artery disease (PAD)?

- smoking - hyperlipidemia - HTN - DM

Teaching for Bueger's is:

- smoking cessation - prevent vasoconstriction (wear warm clothes) - avoid caffeine

Three important factors in the etiology of venous thrombosis (Virchow's triad) are:

- venous stasis - damage to endothelium - hypercoagulability of blood

What preop teaching for an aortic graft is necessary?

- will be sedated, intubated - turn/cough/db exercises - will clean out guy prior

chest pain (symptom of HF)

-angina, chest pain -decreased coronary artery perfusion from decreased CO

Following an AAA repair, patients are taught to assess peripheral pulses for:

Quality and strength.

RAAS inhibitors

-ACE inhibitors -Angiotensin II receptor blockers -Aldosterone antagonists Monitor K+ levels for hyperkalemia

Dilation (compensatory mechanism)

-Enlargement of heart chambers -Pressure in the chambers, left ventricle, increases over time -Stretch heart muscle fibers initially causes increased CO -over time overstretched elastic fibers and cannot contract effectively

Primary causes of HF

-CAD, MI -HTN, hypertensive crisis -Rheumatic heart disease -congenital heart defects -pulmonary hypertension -cardiomyopathy (viral, postpartum, drug abuse) -hyperthyroidism -valvular disorder (mitral stenosis) -myocarditis

Diagnostic studies, HF

-Endomyocardial biopsy -Echocardigraphy -chest xray -ECG, stress test -cardiac catheterization -labwork, BNP, ABG

Ventricular failure

-HF usually both ventricles fail -one may precede the other in dysfunction -one may fail while the other pumps normally

Natriuretic peptides, counterregulatory mechanisms (2 types)

-Have renal, cardiovascular and -hormone effects -Help counter effects of SNS and RAAS -Released in response to increased blood volume in heart 1) Atrial natriuretic peptide (ANP), released from atria 2) Brain, b-type, natriuretic peptide (BNP), released from ventricles Effects: diuresis, vasodilation, lowered BP

Neurohormonal response, compensatory HF

-Kidneys release renin, RAAS-> fluid retention, vasoconstriction -posterior pituitary releases ADH-> Na and H2O retention -endothelin released by vascular endothelial cell-> vasoconstriction -proinflammatory cytokines

Pulmonary edema, clinical manifestations

-anxious, pale, cyanotic -cool, clammy skin -dyspnea, orthopnea, tachypnea -accessory muscles -cough, blood dinges sputum -crackles, wheezes, rhonchi -tachycardia -hypo/hypertension

Etiology, HF

-anything that interferes with CO -Preload -HR -Contractility -Afterload -Direct damage to the heart -conditions that increase the workload of the heart: anemia, infection, thyroid toxicosis, dysrhythmias

) are a first-line therapy for this type of angina. Lipid-lowering drugs help reduce atherosclerosis (i.e., plaque formation), and β-adrenergic blockers decrease sympathetic stimulation of the heart (i.e., palpitations). Medications or activities that increase myocardial contractility will increase the incidence of angina by increasing oxygen demand.

...

Morphine

-Reduces preload and after load -Relieves dyspnea and anxiety

Compensatory mechanisms

-SNS, chatecholamines -Neurohormonal response -Ventricular dilatation -ventricualr hypertrophy

Contributing factors, HF

-advanced age -diabetes -tobacco use -obesity -high serum cholesterol

ventricular remodeling

-caused by SNS activation and neurohormonal response -hypertrophy of ventricular myocytes -produces large abnormally shaped contractile cells -increases wall tension -increased O2 consumption -impaired contractility -ventricles become less effective at pumping -can lead to dysrhythmias and death

Ventricular assist device (VAD)

-circulatory assist device, HF -bridge to transplant -can be a destination therapy, long term

Intraaortic Balloon Pump (IABP)

-circulatory assist device, HF -counter pulsation device -strict bed rest -increase coronary perfusion -decrease aortic pressure -increase forward blood flow -temporary

HF, Low BP

-decreased CO -poor renal perfusion -activates RAAS -vasoconstriction, retention of Na and H2O -elevates BP -compensatory mechanism -poor renal perfusion can lead to risk of renal failure

Behavior changes (symptom of HF)

-decreased cerebral ciruculation -restlessness, confusion, decreased attention span or memory -secondary to poor gas exchange -often in late stages

Skin changes (symptom of HF)

-dusky skin, increased capillary O2 extraction -swollen shiny lower extremity, diminished or absent hair, pigment changes

Weight changes (symptom of HF)

-fluid retention partially due to renal failure -abdominal fullness, ascites -hepatomegaly-> anorexia and nausea -

Primary risk factors for HF

-hypertension -CAD

Diastolic HF definition

-inability of the ventricles to relax and fill during diastole -high filling pressures, stiff ventricles -decreased filling of the ventricle -> decreased stroke volume -> decreased CO -results in pulmonary and systemic vascular engorgement

Long term effect of SNS activation

-increase failing heart's workload and need for O2 -vasoconstriction casuses increase in preload-> increases venous return to heart-> volume overload worsens ventricular performance

Diastolic HF causes

-most common: left ventricular hypertrophy -myocardial ischemia -valve disease (aortic, mitral) -cardiomyopathy

Neurohormonal response (compensatory mechanism) Proinflammatory cytokines

-local but over time leads to systemic inflammatory response -released by heart cells due to cardiac inj -tumor necrosis factor and interleukin-1 -depress heart function, causes: *hypertrophy *contractile dysfunction *cell death

Ventricular failure causes...

-low BP -low CO -poor renal perfusion -poor exercise tolerance -heart dysrhythmias

ADHF clinical manifestations

-manifest as pulmonary edema -life threatening -alveoli filled with fluid -suffocation -Left HF -pink frothy sputum

Maintenance goals of HF treatment

-treat underlying cause -maximize CO -alleviate SXS -improve ventricular function -improve quality of life -preserve target organ fx (heart, lung, kidney, brain) -improve mortality and morbidity, maximize viable time

Left-sided HF side effects

-prevents normal forward flow of blood -blood backs up into atrium and pulmonary veins -increased pulmonary pressure-> fluid leakage from pulmonary capillaries into interstitum and alveoli -pulmonary congestion and edema

Causes of Right-sided HF

-primary cause is left-sided heart failure -right ventricular infection (acute) -pulmonary embolism (acute) -right ventricular dilation and hypertrophy

INR should be:

0.75 - 1.25 tx level is 2-3 w/ mechanical valve 2.5-3.5

Physiological Integrity SHORT ANSWER 1. When assessing a patient with possible PAD, the nurse obtains a brachial BP of 140/80 and an ankle pressure of 110/70. The nurse calculates the patient's ankle-brachial index (ABI) as ______.

0.78 or 0.79 Rationale: The ABI is calculated by dividing the ankle systolic BP by the brachial systolic BP. Cognitive Level: Application Text Reference: p. 901 Nursing Process: Implementation

warfarin

Match the following anticoagulant drugs with their characteristics. Can choose unfractionated heparin, LMWH, Hirudin derivatives, Warfarin (Coumadin), Thrombin inhibitor (argatroban (Acova)) Vitamin K is antidote

Onset of HF (2 types)

1) abrupt onset (i.e. from an MI) 2) subtle process from slow progressive changes

Renal effects of natriuretic peptides (counterregulatory mechanism)

1) increased GFR 2) excretion of sodium

Hormonal effects of natriuretic peptides (counterregulatory mechanism)

1) inhibition of aldosterone and renin secretion 2) interference with ADH release

Mechanisms that regulate CO

1) preload 2) afterload 3) myocardial contractility 4) heart rate

Two subgroups of HF causes

1) primary causes 2) precipitating causes

Two classifications of HF

1) systolic 2) diastolic

Cardiovascular effects of natriuretic peptides (counterregulatory mechanism)

1) vasodilation 2) increased BP

"A client is admitted to the hospital with signs and symptoms of diabetes mellitus. Which findings is the nurse most likely to observe in this client? Select all that apply: "1. Excessive thirst 2. Weight gain 3. Constipation 4. Excessive hunger 5. Urine retention 6. Frequent, high-volume urination

1, 4, 6 Rationale: Classic signs of diabetes mellitus include polydipsia (excessive thirst), polyphagia (excessive hunger), and polyuria (excessive urination). Because the body is starving from the lack of glucose the cells are using for energy, the client has weight loss, not weight gain. Clients with diabetes mellitus usually don't present with constipation. Urine retention is only a problem is the patient has another renal-related condition.

PT should be?

11 - 16 secons

A nurse is preparing a teaching plan for a client with diabetes Mellitus regarding proper foot care. Which instruction is included in the plan? 1. Soak feet in hot water 2. apply a moisturizing lotion to dry feet but not between the toes 3. Always have a podiatrist cut your toenails, never cut them yourself 4. avoid using mild soap on the feet

2. The client is instructed to use a moisturizing lotion on the feet and to avoid applying the lotion between the toes.

Which electrolyte replacement should the nurse anticipate being ordered by thehealth-care provider in the client diagnosed with DKA who has just been admitted tothe ICD? 1.Glucose. 2.)Potassium. 3.Calcium. 4.Sodium

2.(CORRECT)-->The client in DKA loses potassium from increased urinary output, acidosis, cata-bolic state, and vomiting. Replacement isessential for preventing cardiac dysrhyth-mias secondary to hypokalemia.

In treating varicosities, start with compression stockings at what level?

20 mmHg and move upwards

Physiological Integrity 5. A patient has a dysrhythmia that requires careful monitoring of atrial activity. Which lead will be best to use for continuous monitoring? a. MCL1 b. AVF c. V6 d. I

A Rationale: Leads II and MCL1 are the best leads for visualization of P waves, which reflect atrial activity. The other leads are less commonly used for continuous monitoring, since they do not usually demonstrate the P wave and QRS activity as well. Cognitive Level: Application Text Reference: p. 843 Nursing Process: Implementation

A client is receiving intravenous heparin to prevent blood clots. The order is for heparin 1,200 units per hour. The pharmacy sends 25,000 units of heparin in 500 mL of D5W. At how many mL per hour will the nurse infuse this solution? Enter the correct number ONLY.

24 mL/hr

Normal aPTT should be?

25-35 seconds tx. level is 46 - 70

A client with type I diabetes is placed on an insulin pump. The most appropriate short-term goal when teaching this client to control the diabetes is: "1) adhere to the medical regimen 2) remain normoglycemic for 3 weeks 3) demonstrate the correct use of the administration equipment. 4) list 3 self care activities that are necessary to control the diabetes"

3.) is correct "1) this is not a short-term goal 2) this is measurable, but it's a long-term goal 3) this is a short-term goal, client oriented, necessary for the client to control the diabetes, and measurable when the client performs a return demonstration for the nurse 4) although this is measurable and a short-term goal, it is not the one with the greatest priority when a client has an insulin pump that must be mastered before discharge"

Safe and Effective Care Environment COMPLETION 1. A patient with asthma has a personal best peak expiratory flow rate (PEFR) of 400 L/minute. When explaining the asthma action plan, the nurse will teach the patient that a change in therapy is needed when the PEFR is less than ___ L/minute

320 A PEFR less than 80% of the personal best indicates that the patient is in the yellow zone where changes in therapy are needed to prevent progression of the airway narrowing. DIF: Cognitive Level: Apply (application) REF: 579 TOP: Nursing Process: Evaluation MSC:

While hospitalized and recovering from an episode of diabetic ketoacidosis, the patient calls the nurse and reports feeling anxious, nervous, and sweaty. Based on the patient's report, the nurse should a. obtain a glucose reading using a finger stick. b. administer 1 mg glucagon subcutaneously. c. have the patient eat a candy bar. d. have the patient drink 4 ounces of orange juice.

A Rationale: The patient's clinical manifestations are consistent with hypoglycemia and the initial action should be to check the patient's glucose with a finger stick or order a stat blood glucose.

6. A nurse is evaluating the diagnostic study data of a patient with suspected cystic fibrosis (CF). Which of the following test results is associated with a diagnosis of cystic fibrosis? A) Elevated sweat chloride concentration B) Presence of protein in the urine C) Positive phenylketonuria D) Malignancy on lung biopsy

A Feedback: Gene mutations affect transport of chloride ions, leading to CF, which is characterized by thick, viscous secretions in the lungs, pancreas, liver, intestine, and reproductive tract as well as increased salt content in sweat gland secretions. Proteinuria, positive phenylketonuria, and malignancy are not diagnostic for CF.

When the patient diagnosed with angina pectoris complains that he is experiencing chest pain more frequently even at rest, the period of pain is longer, and it takes less stress for the pain to occur, the nurse recognizes that the patient is describing which type of angina? a) Unstable b) Refractory c) Variant d) Intractable

A. unstable

7. The nurse is caring for a patient whose recent unexplained weight loss and history of smoking have prompted diagnostic testing for cancer. What symptom is most closely associated with the early stages of laryngeal cancer? A) Hoarseness B) Dyspnea C) Dysphagia D) Frequent nosebleeds

A Feedback: Hoarseness is an early symptom of laryngeal cancer. Dyspnea, dysphagia, and lumps are later signs of laryngeal cancer. Alopecia is not associated with a diagnosis of laryngeal cancer.

A client with diabetes mellitus demonstratees acute anxiety when first admitted for the treatment of hyperglycemia. The most appropriate intervention to decrease the client's anxiety would be to 1. administer a sedative 2. make sure the client knows all the correct medical terms to understand what is happening 3. ignore the signs and symptoms of anxiety so that they will soon disappear 4. convey empathy, trust, and respect toward the client

4. The most appropriate intervention is to address the client's feelings related to the anxiety

21. A critical care nurse is caring for a client with an endotracheal tube who is on a ventilator. The nurse knows that meticulous airway management of this patient is necessary. What is the main rationale for this? A) Maintaining a patent airway B) Preventing the need for suctioning C) Maintaining the sterility of the patient's airway D) Increasing the patient's lung compliance

A Feedback: Maintaining a patent (open) airway is achieved through meticulous airway management, whether in an emergency situation such as airway obstruction or in long-term management, as in caring for a patient with an endotracheal or a tracheostomy tube. The other answers are incorrect.

a nurse is interviewing a client with type 2 diabetes mellitus. which statement by the client indicated an understanding of the treatment for this disorder? "1. ""i take oral insulin instead of shots"" 2. ""by taking these medications I am able to eat more"" 3. ""when I become ill, I need to increase the number of pills I take"" 4. ""the medications I'm taking help release the insulin I already make""

4.)Clients with type 2 diabetes mellitus have decreased or imparied insulin secretion. Oral hypoglycemic agents are given to these clients to facilitate glucose uptake. Insulin injections may be given during times of stress-induced hyperglycemia. Oral insulin is not available because of the breakdown of the insulin by digestion. Options 1, 2 and 3 are incorrect

5 yr Mortality rate, HF

50% after diagnosis It is a terminal illness

Which patient should the nurse assign as the roommate for a patient who has aplastic anemia? a. A patient with chronic heart failure b. A patient who has viral pneumonia c. A patient who has right leg cellulitis d. A patient with multiple abdominal drains

ANS: A Patients with aplastic anemia are at risk for infection because of the low white blood cell production associated with this type of anemia, so the nurse should avoid assigning a roommate with any possible infectious process.

Physiological Integrity COMPLETION 1. When analyzing an ECG rhythm strip of a patient with a regular cardiac rhythm, the nurse finds there are 25 small blocks from one R wave to the next. The nurse calculates the patient's heart rate as ______.

60 Rationale: There are 1500 small blocks in a minute, and the nurse will divide 1500 by 25. Cognitive Level: Comprehension Text Reference: p. 844 Nursing Process: Assessment

Patients w/ a stent can have an MRI when?

8 weeks after stent is placed

e

Repair of an aortic aneurysm by placing an aortic graft inside the aneurysm through the femoral artery is called the _______ _______. A. saccular B. synthetic graft C. all D. iliac E. endovascular graft F. renal

What size aneurysm requires surgery?

> 5.5 cm in men > 5 cm in women

When can this patient ambulate?

>24 hours on heparin

Health Promotion and Maintenance 2. To assist the patient with CAD to make the appropriate dietary changes, which of these nursing interventions will be most effective? a. Assist the patient to modify favorite high-fat recipes by using monosaturated oils when possible. b. Provide the patient with a list of low-sodium, low-cholesterol foods that should be included in the diet. c. Instruct the patient that a diet containing no saturated fat and minimal sodium will be necessary. d. Emphasize the increased risk for cardiac problems unless the patient makes the dietary changes.

A Rationale: Lifestyle changes are more likely to be successful when consideration is given to patient's preferences. The highest percentage of calories from fat should come from monosaturated fats. Although low-sodium and low-cholesterol foods are appropriate, providing the patient with a list alone is not likely to be successful in making dietary changes. Removing saturated fat from the diet completely is not a realistic expectation; up to 7% of calories in the therapeutic lifestyle changes (TLC) diet can come from saturated fat. Telling the patient about the increased risk without assisting further with strategies for dietary change is unlikely to be successful. Cognitive Level: Application Text Reference: pp. 792-793 Nursing Process: Implementation

23. The critical care nurse and the other members of the care team are assessing the patient to see if he is ready to be weaned from the ventilator. What are the most important predictors of successful weaning that the nurse should identify? A) Stable vital signs and ABGs B) Pulse oximetry above 80% and stable vital signs C) Stable nutritional status and ABGs D) Normal orientation and level of consciousness

A Feedback: Among many other predictors, stable vital signs and ABGs are important predictors of successful weaning. Pulse oximetry must greatly exceed 80%. Nutritional status is important, but vital signs and ABGs are even more significant. Patients who are weaned may or may not have full level of consciousness.

38. Postural drainage has been ordered for a patient who is having difficulty mobilizing her bronchial secretions. Before repositioning the patient and beginning treatment, the nurse should perform what health assessment? A) Chest auscultation B) Pulmonary function testing C) Chest percussion D) Thoracic palpation

A Feedback: Chest auscultation should be performed before and after postural drainage in order to evaluate the effectiveness of the therapy. Percussion and palpation are less likely to provide clinically meaningful data for the nurse. PFTs are normally beyond the scope of the nurse and are not necessary immediately before postural drainage.

7. A patient is exhibiting signs of a pneumothorax following tracheostomy. The surgeon inserts a chest tube into the anterior chest wall. What should the nurse tell the family is the primary purpose of this chest tube? A) To remove air from the pleural space B) To drain copious sputum secretions C) To monitor bleeding around the lungs D) To assist with mechanical ventilation

A Feedback: Chest tubes and closed drainage systems are used to re-expand the lung involved and to remove excess air, fluid, and blood. The primary purpose of a chest tube is not to drain sputum secretions, monitor bleeding, or assist with mechanical ventilation.

16. A nurse practitioner has provided care for three different patients with chronic pharyngitis over the past several months. Which patients are at greatest risk for developing chronic pharyngitis? A) Patients who are habitual users of alcohol and tobacco B) Patients who are habitual users of caffeine and other stimulants C) Patients who eat a diet high in spicy foods D) Patients who have gastrointestinal reflux disease (GERD)

A Feedback: Chronic pharyngitis is common in adults who live and work in dusty surroundings, use the voice to excess, suffer from chronic chough, and habitually use alcohol and tobacco. Caffeine and spicy foods have not been linked to chronic pharyngitis. GERD is not a noted risk factor.

10. The ED nurse is assessing a young gymnast who fell from a balance beam. The gymnast presents with a clear fluid leaking from her nose. What should the ED nurse suspect? A) Fracture of the cribriform plate B) Rupture of an ethmoid sinus C) Abrasion of the soft tissue D) Fracture of the nasal septum

A Feedback: Clear fluid from either nostril suggests a fracture of the cribriform plate with leakage of cerebrospinal fluid. The symptoms are not indicative of an abrasion of the soft tissue or rupture of a sinus. Clear fluid leakage from the nose would not be indicative of a fracture of the nasal septum.

20. A nurse is caring for a patient who has been admitted with an exacerbation of chronic bronchiectasis. The nurse should expect to assess the patient for which of the following clinical manifestations? A) Copious sputum production B) Pain on inspiration C) Pigeon chest D) Dry cough

A Feedback: Clinical manifestations of bronchiectasis include hemoptysis, chronic cough, copious purulent sputum, and clubbing of the fingers. Because of the copious production of sputum, the cough is rarely dry. A pigeon chest is not associated with the disease and patients do not normally experience pain on inspiration.

18. A pediatric nurse practitioner is caring for a child who has just been diagnosed with asthma. The nurse has provided the parents with information that includes potential causative agents for an asthmatic reaction. What potential causative agent should the nurse describe? A) Pets B) Lack of sleep C) Psychosocial stress D) Bacteria

A Feedback: Common causative agents that may trigger an asthma attack are as follows: dust, dust mites, pets, soap, certain foods, molds, and pollens. Lack of sleep, stress, and bacteria are not common triggers for asthma attacks.

8. The nurse is caring for a patient who needs education on his medication therapy for allergic rhinitis. The patient is to take cromolyn (Nasalcrom) daily. In providing education for this patient, how should the nurse describe the action of the medication? A) It inhibits the release of histamine and other chemicals. B) It inhibits the action of proton pumps. C) It inhibits the action of the sodium-potassium pump in the nasal epithelium. D) It causes bronchodilation and relaxes smooth muscle in the bronchi.

A Feedback: Cromolyn (Nasalcrom) inhibits the release of histamine and other chemicals. It is prescribed to treat allergic rhinitis. Beta-adrenergic agents lead to bronchodilation and stimulate beta-2 adrenergic receptors in the smooth muscle of the bronchi and bronchioles. It does not affect proton pump action or the sodium-potassium pump in the nasal cells.

10. A nurse is caring for a 6-year-old patient with cystic fibrosis. In order to enhance the child's nutritional status, what intervention should most likely be included in the plan of care? A) Pancreatic enzyme supplementation with meals B) Provision of five to six small meals per day rather than three larger meals C) Total parenteral nutrition (TPN) D) Magnesium, thiamine, and iron supplementation

A Feedback: Nearly 90% of patients with CF have pancreatic exocrine insufficiency and require oral pancreatic enzyme supplementation with meals. Frequent, small meals or TPN are not normally indicated. Vitamin supplements are required, but specific replacement of magnesium, thiamine, and iron is not typical.

30. A nurse is planning the care of a client with bronchiectasis. What goal of care should the nurse prioritize? A) The patient will successfully mobilize pulmonary secretions. B) The patient will maintain an oxygen saturation level of 98%. C) The patient's pulmonary blood pressure will decrease to within reference ranges. D) The patient will resume prediagnosis level of function within 72 hours.

A Feedback: Nursing management focuses on alleviating symptoms and helping patients clear pulmonary secretions. Pulmonary pressures are not a central focus in the care of the patient with bronchiectasis. Rapid resumption of prediagnosis function and oxygen saturation above 98% are unrealistic goals.

27. The nurse is providing patient teaching to a patient diagnosed with acute rhinosinusitis. For what possible complication should the nurse teach the patient to seek immediate follow-up? A) Periorbital edema B) Headache unrelieved by OTC medications C) Clear drainage from nose D) Blood-tinged mucus when blowing the nose

A Feedback: Patient teaching is an important aspect of nursing care for the patient with acute rhinosinusitis. The nurse instructs the patient about symptoms of complications that require immediate follow-up. Referral to a physician is indicated if periorbital edema and severe pain on palpation occur. Clear drainage and blood-tinged mucus do not require follow-up if the patient has acute rhinosinusitis. A persistent headache does not necessarily warrant immediate follow-up.

32. A patient has had a nasogastric tube in place for 6 days due to the development of paralytic ileus after surgery. In light of the prolonged presence of the nasogastric tube, the nurse should prioritize assessments related to what complication? A) Sinus infections B) Esophageal strictures C) Pharyngitis D) Laryngitis

A Feedback: Patients with nasotracheal and nasogastric tubes in place are at risk for development of sinus infections. Thus, accurate assessment of patients with these tubes is critical. Use of a nasogastric tube is not associated with the development of the other listed pathologies.

17. The nurse has admitted a patient who is scheduled for a thoracic resection. The nurse is providing preoperative teaching and is discussing several diagnostic studies that will be required prior to surgery. Which study will be performed to determine whether the planned resection will leave sufficient functioning lung tissue? A) Pulmonary function studies B) Exercise tolerance tests C) Arterial blood gas values D) Chest x-ray

A Feedback: Pulmonary function studies are performed to determine whether the planned resection will leave sufficient functioning lung tissue. ABG values are assessed to provide a more complete picture of the functional capacity of the lung. Exercise tolerance tests are useful to determine if the patient who is a candidate for pneumonectomy can tolerate removal of one of the lungs. Preoperative studies, such as a chest x-ray, are performed to provide a baseline for comparison during the postoperative period and to detect any unsuspected abnormalities.

17. A nurse has been asked to give a workshop on COPD for a local community group. The nurse emphasizes the importance of smoking cessation because smoking has what pathophysiologic effect? A) Increases the amount of mucus production B) Destabilizes hemoglobin C) Shrinks the alveoli in the lungs D) Collapses the alveoli in the lungs

A Feedback: Smoking irritates the goblet cells and mucous glands, causing an increased accumulation of mucus, which, in turn, produces more irritation, infection, and damage to the lung.

24. The OR nurse is setting up a water-seal chest drainage system for a patient who has just had a thoracotomy. The nurse knows that the amount of suction in the system is determined by the water level. At what suction level should the nurse set the system? A) 20 cm H2O B) 15 cm H2O C) 10 cm H2O D) 5 cm H2O

A Feedback: The amount of suction is determined by the water level. It is usually set at 20 cm H2O; adding more fluid results in more suction.

40. A nurse is admitting a new patient who has been admitted with a diagnosis of COPD exacerbation. How can the nurse best help the patient achieve the goal of maintaining effective oxygenation? A) Teach the patient strategies for promoting diaphragmatic breathing. B) Administer supplementary oxygen by simple face mask. C) Teach the patient to perform airway suctioning. D) Assist the patient in developing an appropriate exercise program.

A Feedback: The breathing pattern of most people with COPD is shallow, rapid, and inefficient; the more severe the disease, the more inefficient the breathing pattern. With practice, this type of upper chest breathing can be changed to diaphragmatic breathing, which reduces the respiratory rate, increases alveolar ventilation, and sometimes helps expel as much air as possible during expiration. Suctioning is not normally necessary in patients with COPD. Supplementary oxygen is not normally delivered by simple face mask and exercise may or may not be appropriate.

6. The home care nurse is assessing the home environment of a patient who will be discharged from the hospital shortly after his laryngectomy. The nurse should inform the patient that he may need to arrange for the installation of which system in his home? A) A humidification system B) An air conditioning system C) A water purification system D) A radiant heating system

A Feedback: The nurse stresses the importance of humidification at home and instructs the family to obtain and set up a humidification system before the patient returns home. Air-conditioning may be too cool and too drying for the patient. A water purification system or a radiant heating system is not necessary.

16. The nurse is caring for a patient who is ready to be weaned from the ventilator. In preparing to assist in the collaborative process of weaning the patient from a ventilator, the nurse is aware that the weaning of the patient will progress in what order? A) Removal from the ventilator, tube, and then oxygen B) Removal from oxygen, ventilator, and then tube C) Removal of the tube, oxygen, and then ventilator D) Removal from oxygen, tube, and then ventilator

A Feedback: The process of withdrawing the patient from dependence on the ventilator takes place in three stages: the patient is gradually removed from the ventilator, then from the tube, and, finally, oxygen.

32. A patient has been discharged home after thoracic surgery. The home care nurse performs the initial visit and finds the patient discouraged and saddened. The client states, "I am recovering so slowly. I really thought I would be better by now." What nursing action should the nurse prioritize? A) Provide emotional support to the patient and family. B) Schedule a visit to the patient's primary physician within 24 hours. C) Notify the physician that the patient needs a referral to a psychiatrist. D) Place a referral for a social worker to visit the patient.

A Feedback: The recovery process may take longer than the patient had expected, and providing support to the patient is an important task for the home care nurse. It is not necessary, based on this scenario, to schedule a visit with the physician within 24 hours, or to get a referral to a psychiatrist or a social worker.

5. A patient with emphysema is experiencing shortness of breath. To relieve this patient's symptoms, the nurse should assist her into what position? A) Sitting upright, leaning forward slightly B) Low Fowler's, with the neck slightly hyperextended C) Prone D) Trendelenburg

A Feedback: The typical posture of a person with COPD is to lean forward and use the accessory muscles of respiration to breathe. Low Fowler's positioning would be less likely to aid oxygenation. Prone or Trendelenburg positioning would exacerbate shortness of breath.

Physiological Integrity 21. Which assessment finding in a patient admitted with acute decompensated heart failure (ADHF) requires the most immediate action by the nurse? a. Oxygen saturation of 88% b. Weight gain of 1 kg (2.2 lb) c. Heart rate of 106 beats/minute d. Urine output of 50 mL over 2 hours

A A decrease in oxygen saturation to less than 92% indicates hypoxemia. The nurse should administer supplemental oxygen immediately to the patient. An increase in apical pulse rate, 1-kg weight gain, and decreases in urine output also indicate worsening heart failure and require nursing actions, but the low oxygen saturation rate requires the most immediate nursing action. DIF: Cognitive Level: Apply (application) REF: 769-770 OBJ: Special Questions: Prioritization TOP: Nursing Process: Assessment MSC:

Physiological Integrity 25. A patient in the clinic with cystic fibrosis (CF) reports increased sweating and weakness during the summer months. Which action by the nurse would be most appropriate? a. Have the patient add dietary salt to meals. b. Teach the patient about the signs of hypoglycemia. c. Suggest decreasing intake of dietary fat and calories. d. Instruct the patient about pancreatic enzyme replacements.

A Added dietary salt is indicated whenever sweating is excessive, such as during hot weather, when fever is present, or from intense physical activity. The management of pancreatic insufficiency includes pancreatic enzyme replacement of lipase, protease, and amylase (e.g., Pancreaze, Creon, Ultresa, Zenpep) administered before each meal and snack. This patient is at risk for hyponatremia based on reported symptoms. Adequate intake of fat, calories, protein, and vitamins is important. Fat-soluble vitamins (vitamins A, D, E, and K) must be supplemented because they are malabsorbed. Use of caloric supplements improves nutritional status. Hyperglycemia due to pancreatic insufficiency is more likely to occur than hypoglycemia. DIF: Cognitive Level: Apply (application) REF: 605 TOP: Nursing Process: Implementation MSC:

Physiological Integrity 29. When caring for a patient who has just arrived on the medical-surgical unit after having cardiac catheterization, which nursing intervention should the nurse delegate to a licensed practical/vocational nurse (LPN/LVN)? a. Give the scheduled aspirin and lipid-lowering medication. b. Perform the initial assessment of the catheter insertion site. c. Teach the patient about the usual postprocedure plan of care. d. Titrate the heparin infusion according to the agency protocol.

A Administration of oral medications is within the scope of practice for LPNs/LVNs. The initial assessment of the patient, patient teaching, and titration of IV anticoagulant medications should be done by the registered nurse (RN). DIF: Cognitive Level: Apply (application) REF: 758 OBJ: Special Questions: Delegation TOP: Nursing Process: Planning MSC:

Physiological Integrity 41. The nurse reviews the medication administration record (MAR) for a patient having an acute asthma attack. Which medication should the nurse administer first? a. Albuterol (Ventolin) 2.5 mg per nebulizer b. Methylprednisolone (Solu-Medrol) 60 mg IV c. Salmeterol (Serevent) 50 mcg per dry-powder inhaler (DPI) d. Triamcinolone (Azmacort) 2 puffs per metered-dose inhaler (MDI)

A Albuterol is a rapidly acting bronchodilator and is the first-line medication to reverse airway narrowing in acute asthma attacks. The other medications work more slowly. DIF: Cognitive Level: Apply (application) REF: 570 | 576 OBJ: Special Questions: Prioritization TOP: Nursing Process: Implementation MSC:

Physiological Integrity 36. A patient who is experiencing an acute asthma attack is admitted to the emergency department. Which assessment should the nurse complete first? a. Listen to the patient's breath sounds. b. Ask about inhaled corticosteroid use. c. Determine when the dyspnea started. d. Obtain the forced expiratory volume (FEV) flow rate.

A Assessment of the patient's breath sounds will help determine how effectively the patient is ventilating and whether rapid intubation may be necessary. The length of time the attack has persisted is not as important as determining the patient's status at present. Most patients having an acute attack will be unable to cooperate with an FEV measurement. It is important to know about the medications the patient is using but not as important as assessing the breath sounds. DIF: Cognitive Level: Apply (application) REF: 564-565 OBJ: Special Questions: Prioritization TOP: Nursing Process: Assessment MSC:

Physiological Integrity 14. A patient with acute dyspnea is scheduled for a spiral computed tomography (CT) scan. Which information obtained by the nurse is a priority to communicate to the health care provider before the CT? a. Allergy to shellfish b. Apical pulse of 104 c. Respiratory rate of 30 d. Oxygen saturation of 90%

A Because iodine-based contrast media is used during a spiral CT, the patient may need to have the CT scan without contrast or be premedicated before injection of the contrast media. The increased pulse, low oxygen saturation, and tachypnea all indicate a need for further assessment or intervention but do not indicate a need to modify the CT procedure. DIF: Cognitive Level: Apply (application) REF: 492 OBJ: Special Questions: Prioritization TOP: Nursing Process: Implementation MSC:

Health Promotion and Maintenance 17. The clinic nurse teaches a patient with a 42 pack-year history of cigarette smoking about lung disease. Which information will be most important for the nurse to include? a. Options for smoking cessation b. Reasons for annual sputum cytology testing c. Erlotinib (Tarceva) therapy to prevent tumor risk d. Computed tomography (CT) screening for lung cancer

A Because smoking is the major cause of lung cancer, the most important role for the nurse is teaching patients about the benefits of and means of smoking cessation. CT scanning is currently being investigated as a screening test for high-risk patients. However, if there is a positive finding, the person already has lung cancer. Erlotinib may be used in patients who have lung cancer, but it is not used to reduce the risk of developing cancer. DIF: Cognitive Level: Apply (application) REF: 540 TOP: Nursing Process: Planning MSC:

Integrated Process: Nursing Process (Diagnosis) 34. The nurse assesses a client's chest tube and finds continuous bubbling in the water seal chamber. When the nurse clamps the chest tube close to the client's dressing, the bubbling stops. How does the nurse interpret this finding? a. An air leak is present at the chest tube insertion site or in the thoracic cavity. b. An air leak is present in the drainage system. c. More water needs to be added to the water seal. d. The system is functioning appropriately and no intervention is needed.

A Bubbling in the water seal chamber indicates air drainage from the client and usually is seen when the client's intrathoracic pressure is greater than atmospheric pressure, such as during exhalation, coughing, or sneezing. When the air in the pleural space has been sufficiently removed, bubbling stops. Continuous bubbling indicates an air leak. If the air leak is in the thoracic cavity, air and air pressure increase in the thoracic cavity, forcing more air into the water seal chamber. This air movement is prevented when the chest tube is clamped close to the insertion site. DIF: Cognitive Level: Application/Applying or higher REF: N/A TOP: Client Needs Category: Physiological Integrity (Physiological Adaptation—Illness Management)

Physiological Integrity 19. After being hit by a baseball, a patient arrives in the emergency department with a possible nasal fracture. Which finding by the nurse is most important to report to the health care provider? a. Clear nasal drainage b. Complaint of nasal pain c. Bilateral nose swelling and bruising d. Inability to breathe through the nose

A Clear nasal drainage may indicate a meningeal tear with leakage of cerebrospinal fluid. This would place the patient at risk for complications such as meningitis. The other findings are typical with a nasal fracture and do not indicate any complications. DIF: Cognitive Level: Apply (application) REF: 498 OBJ: Special Questions: Prioritization TOP: Nursing Process: Assessment MSC:

Physiological Integrity 34. Which information about a patient who has been receiving thrombolytic therapy for an acute myocardial infarction (AMI) is most important for the nurse to communicate to the health care provider? a. No change in the patient's chest pain b. An increase in troponin levels from baseline c. A large bruise at the patient's IV insertion site d. A decrease in ST-segment elevation on the electrocardiogram

A Continued chest pain suggests that the thrombolytic therapy is not effective and that other interventions such as percutaneous coronary intervention (PCI) may be needed. Bruising is a possible side effect of thrombolytic therapy, but it is not an indication that therapy should be discontinued. The decrease of the ST-segment elevation indicates that thrombolysis is occurring and perfusion is returning to the injured myocardium. An increase in troponin levels is expected with reperfusion and is related to the washout of cardiac markers into the circulation as the blocked vessel is opened. DIF: Cognitive Level: Apply (application) REF: 752 OBJ: Special Questions: Prioritization TOP: Nursing Process: Evaluation MSC:

Physiological Integrity 15. The nurse is caring for a patient with cor pulmonale. The nurse should monitor the patient for which expected finding? a. Peripheral edema b. Elevated temperature c. Clubbing of the fingers d. Complaints of chest pain

A Cor pulmonale causes clinical manifestations of right ventricular failure, such as peripheral edema. The other clinical manifestations may occur in the patient with other complications of chronic obstructive pulmonary disease (COPD) but are not indicators of cor pulmonale. DIF: Cognitive Level: Apply (application) REF: 586 TOP: Nursing Process: Evaluation MSC:

Physiological Integrity 26. The nurse administers prescribed therapies for a patient with cor pulmonale and right-sided heart failure. Which assessment would best evaluate the effectiveness of the therapies? a. Observe for distended neck veins. b. Auscultate for crackles in the lungs. c. Palpate for heaves or thrills over the heart. d. Review hemoglobin and hematocrit values.

A Cor pulmonale is right ventricular failure caused by pulmonary hypertension, so clinical manifestations of right ventricular failure such as peripheral edema, jugular venous distention, and right upper-quadrant abdominal tenderness would be expected. Crackles in the lungs are likely to be heard with left-sided heart failure. Findings in cor pulmonale include evidence of right ventricular hypertrophy on electrocardiogram ECG and an increase in intensity of the second heart sound. Heaves or thrills are not common with cor pulmonale. Chronic hypoxemia leads to polycythemia and increased total blood volume and viscosity of the blood. The hemoglobin and hematocrit values are more likely to be elevated with cor pulmonale than decreased. DIF: Cognitive Level: Apply (application) REF: 555 TOP: Nursing Process: Evaluation MSC:

Physiological Integrity 3. A patient with bacterial pneumonia has rhonchi and thick sputum. What is the nurse's most appropriate action to promote airway clearance? a. Assist the patient to splint the chest when coughing. b. Teach the patient about the need for fluid restrictions. c. Encourage the patient to wear the nasal oxygen cannula. d. Instruct the patient on the pursed lip breathing technique.

A Coughing is less painful and more likely to be effective when the patient splints the chest during coughing. Fluids should be encouraged to help liquefy secretions. Nasal oxygen will improve gas exchange, but will not improve airway clearance. Pursed lip breathing is used to improve gas exchange in patients with COPD, but will not improve airway clearance. DIF: Cognitive Level: Apply (application) REF: 527 TOP: Nursing Process: Implementation MSC:

Physiological Integrity 19. The nurse makes a diagnosis of impaired gas exchange for a patient with COPD in acute respiratory distress, based on the assessment finding of a. a pulse oximetry reading of 86%. b. dyspnea and respiratory rate of 36. c. use of the accessory respiratory muscles. d. the presence of crackles in both lungs.

A Rationale: The best data to support the diagnosis of impaired gas exchange are abnormalities in the ABGs or pulse oximetry. The other data would support a diagnosis of risk for impaired gas exchange. Cognitive Level: Application Text Reference: pp. 650-651 Nursing Process: Diagnosis

Physiological Integrity 4. On auscultation of a patient's lungs, the nurse hears low-pitched, bubbling sounds during inhalation in the lower third of both lungs. How should the nurse document this finding? a. Inspiratory crackles at the bases b. Expiratory wheezes in both lungs c. Abnormal lung sounds in the apices of both lungs d. Pleural friction rub in the right and left lower lobes

A Crackles are low-pitched, bubbling sounds usually heard on inspiration. Wheezes are high-pitched sounds. They can be heard during the expiratory or inspiratory phase of the respiratory cycle. The lower third of both lungs are the bases, not apices. Pleural friction rubs are grating sounds that are usually heard during both inspiration and expiration. DIF: Cognitive Level: Understand (comprehension) REF: 487 | 489 TOP: Nursing Process: Assessment MSC:

Physiological Integrity 25. The nurse is caring for a patient who was admitted to the coronary care unit following an acute myocardial infarction (AMI) and percutaneous coronary intervention the previous day. Teaching for this patient would include a. when cardiac rehabilitation will begin. b. the typical emotional responses to AMI. c. information regarding discharge medications. d. the pathophysiology of coronary artery disease.

A Early after an AMI, the patient will want to know when resumption of usual activities can be expected. At this time, the patient's anxiety level or denial will interfere with good understanding of complex information such as the pathophysiology of coronary artery disease (CAD). Teaching about discharge medications should be done closer to discharge. The nurse should support the patient by decreasing anxiety rather than discussing the typical emotional responses to myocardial infarction (MI). DIF: Cognitive Level: Apply (application) REF: 759-762 TOP: Nursing Process: Planning MSC:

Integrated Process: Nursing Process (Implementation) 19. The home care nurse observes white patches on the oral mucosa of a client with severe, chronic airflow limitation. What is the nurse's best action? a. Ask the client whether he or she uses a steroid inhaler. b. Inquire about any recent viral illnesses. c. Have the client rinse the mouth with salt water. d. Have the client brush the patches with a soft-bristled brush.

A Excessive use of steroid inhalers reduces local immune function and increases the client's risk for oral-pharyngeal infection, including candidiasis, which manifests as white patches on the oral mucosa. The client should not brush the lesions, and salt water will not help the sores. Recent illnesses would have no effect on these lesions. DIF: Cognitive Level: Application/Applying or higher REF: N/A TOP: Client Needs Category: Physiological Integrity (Pharmacological and Parenteral Therapies—Adverse Effects/Contraindications/Interactions/Side Effects)

Physiological Integrity 4. To auscultate for S3 or S4 gallops in the mitral area, the nurse listens with the a. bell of the stethoscope with the patient in the left lateral position. b. diaphragm of the stethoscope with the patient in a supine position. c. bell of the stethoscope with the patient sitting and leaning forward. d. diaphragm of the stethoscope with the patient lying flat on the left side.

A Gallop rhythms generate low-pitched sounds and are most easily heard with the bell of the stethoscope. Sounds associated with the mitral valve are accentuated by turning the patient to the left side, which brings the heart closer to the chest wall. The diaphragm of the stethoscope is best to use for the higher-pitched sounds such as S1 and S2. DIF: Cognitive Level: Apply (application) REF: 697 TOP: Nursing Process: Assessment MSC:

Safe and Effective Care Environment 43. Which finding in a patient hospitalized with bronchiectasis is most important to report to the health care provider? a. Cough productive of bloody, purulent mucus b. Scattered rhonchi and wheezes heard bilaterally c. Respiratory rate 28 breaths/minute while ambulating in hallway d. Complaint of sharp chest pain with deep breathing

A Hemoptysis may indicate life-threatening hemorrhage and should be reported immediately to the health care provider. The other findings are frequently noted in patients with bronchiectasis and may need further assessment but are not indicators of life-threatening complications. DIF: Cognitive Level: Apply (application) REF: 607 OBJ: Special Questions: Prioritization TOP: Nursing Process: Assessment MSC:

Physiological Integrity 2. The nurse assesses the chest of a patient with pneumococcal pneumonia. Which finding would the nurse expect? a. Increased tactile fremitus b. Dry, nonproductive cough c. Hyperresonance to percussion d. A grating sound on auscultation

A Increased tactile fremitus over the area of pulmonary consolidation is expected with bacterial pneumonias. Dullness to percussion would be expected. Pneumococcal pneumonia typically presents with a loose, productive cough. Adventitious breath sounds such as crackles and wheezes are typical. A grating sound is more representative of a pleural friction rub rather than pneumonia. DIF: Cognitive Level: Apply (application) REF: 527 TOP: Nursing Process: Assessment MSC:

Physiological Integrity 6. The nurse is caring for a mechanically ventilated patient with a cuffed tracheostomy tube. Which action by the nurse would best determine if the cuff has been properly inflated? a. Use a manometer to ensure cuff pressure is at an appropriate level. b. Check the amount of cuff pressure ordered by the health care provider. c. Suction the patient first with a fenestrated inner cannula to clear secretions. d. Insert the decannulation plug before the nonfenestrated inner cannula is removed.

A Measurement of cuff pressure using a manometer to ensure that cuff pressure is 20 mm Hg or lower will avoid compression of the tracheal wall and capillaries. Never insert the decannulation plug in a tracheostomy tube until the cuff is deflated and the nonfenestrated inner cannula is removed. Otherwise, the patient's airway is occluded. A health care provider's order is not required to determine safe cuff pressure. A nonfenestrated inner cannula must be used to suction a patient to prevent tracheal damage occurring from the suction catheter passing through the fenestrated openings. DIF: Cognitive Level: Apply (application) REF: 509 TOP: Nursing Process: Implementation MSC:

Physiological Integrity 6. An 80-year-old patient breathing room air has an ABG analysis. The nurse interprets which results as normal? a. pH 7.38, arterial carbon dioxide (PaO2) 82 mm Hg, PaCO2 40 mm Hg, and O2 sat 92% b. pH 7.32, PaO2 85 mm Hg, PaCO2 55 mm Hg, and O2 sat 90% c. pH 7.48, PaO2 90 mm Hg, PaCO2 31 mm Hg, and O2 sat 98% d. pH 7.52, PaO2 91 mm Hg, PaCO2 42 mm Hg, and O2 sat 94%

A Rationale: All the values in this answer are correct. The answer beginning "pH 7.32, PaO2 85 mm Hg" shows respiratory acidosis. The answer beginning "pH 7.48, PaO2 90 mm Hg" indicates respiratory alkalosis, and the answer beginning "pH 7.52, PaO2 91 mm Hg" shows metabolic alkalosis. Cognitive Level: Application Text Reference: p. 514 Nursing Process: Implementation

Physiological Integrity 19. A patient in the intensive care unit with acute decompensated heart failure (ADHF) complains of severe dyspnea and is anxious, tachypneic, and tachycardic. All of the following medications have been ordered for the patient. The nurse's priority action will be to a. give IV morphine sulfate 4 mg. b. give IV diazepam (Valium) 2.5 mg. c. increase nitroglycerin (Tridil) infusion by 5 mcg/min. d. increase dopamine (Intropin) infusion by 2 mcg/kg/min.

A Morphine improves alveolar gas exchange, improves cardiac output by reducing ventricular preload and afterload, decreases anxiety, and assists in reducing the subjective feeling of dyspnea. Diazepam may decrease patient anxiety, but it will not improve the cardiac output or gas exchange. Increasing the dopamine may improve cardiac output, but it will also increase the heart rate and myocardial oxygen consumption. Nitroglycerin will improve cardiac output and may be appropriate for this patient, but it will not directly reduce anxiety and will not act as quickly as morphine to decrease dyspnea. DIF: Cognitive Level: Analyze (analysis) REF: 774 OBJ: Special Questions: Prioritization TOP: Nursing Process: Implementation MSC:

Psychosocial Integrity 26. A patient who has recently started taking pravastatin (Pravachol) and niacin (Nicobid) reports the following symptoms to the nurse. Which is most important to communicate to the health care provider? a. Generalized muscle aches and pains b. Dizziness when changing positions quickly c. Nausea when taking the drugs before eating d. Flushing and pruritus after taking the medications

A Muscle aches and pains may indicate myopathy and rhabdomyolysis, which have caused acute kidney injury and death in some patients who have taken the statin medications. These symptoms indicate that the pravastatin may need to be discontinued. The other symptoms are common side effects when taking niacin, and although the nurse should follow-up with the health care provider, they do not indicate that a change in medication is needed. DIF: Cognitive Level: Apply (application) REF: 738-739 OBJ: Special Questions: Prioritization TOP: Nursing Process: Evaluation MSC:

Physiological Integrity 14. Which action should the nurse include in the plan of care when caring for a patient admitted with acute decompensated heart failure (ADHF) who is receiving nesiritide (Natrecor)? a. Monitor blood pressure frequently. b. Encourage patient to ambulate in room. c. Titrate nesiritide slowly before stopping. d. Teach patient about home use of the drug.

A Nesiritide is a potent arterial and venous dilator, and the major adverse effect is hypotension. Because the patient is likely to have orthostatic hypotension, the patient should not be encouraged to ambulate. Nesiritide does not require titration and is used for ADHF but not in a home setting. DIF: Cognitive Level: Apply (application) REF: 774 TOP: Nursing Process: Planning MSC:

Physiological Integrity 11. An older patient is receiving standard multidrug therapy for tuberculosis (TB). The nurse should notify the health care provider if the patient exhibits which finding? a. Yellow-tinged skin b. Orange-colored sputum c. Thickening of the fingernails d. Difficulty hearing high-pitched voices

A Noninfectious hepatitis is a toxic effect of isoniazid (INH), rifampin, and pyrazinamide, and patients who develop hepatotoxicity will need to use other medications. Changes in hearing and nail thickening are not expected with the four medications used for initial TB drug therapy. Presbycusis is an expected finding in the older adult patient. Orange discoloration of body fluids is an expected side effect of rifampin and not an indication to call the health care provider. DIF: Cognitive Level: Apply (application) REF: 531 TOP: Nursing Process: Implementation MSC:

Safe and Effective Care Environment 18. The nurse is caring for a hospitalized older patient who has nasal packing in place to treat a nosebleed. Which assessment finding will require the most immediate action by the nurse? a. The oxygen saturation is 89%. b. The nose appears red and swollen. c. The patient's temperature is 100.1° F (37.8° C). d. The patient complains of level 8 (0 to 10 scale) pain.

A Older patients with nasal packing are at risk of aspiration or airway obstruction. An O2 saturation of 89% should alert the nurse to further assess for these complications. The other assessment data also indicate a need for nursing action but not as immediately as the low O2 saturation. DIF: Cognitive Level: Apply (application) REF: 499 OBJ: Special Questions: Prioritization TOP: Nursing Process: Assessment MSC:

Physiological Integrity 31. The nurse cares for a patient who has just had a thoracentesis. Which assessment information obtained by the nurse is a priority to communicate to the health care provider? a. Oxygen saturation is 88%. b. Blood pressure is 145/90 mm Hg. c. Respiratory rate is 22 breaths/minute when lying flat. d. Pain level is 5 (on 0 to 10 scale) with a deep breath.

A Oxygen saturation would be expected to improve after a thoracentesis. A saturation of 88% indicates that a complication such as pneumothorax may be occurring. The other assessment data also indicate a need for ongoing assessment or intervention, but the low oxygen saturation is the priority. DIF: Cognitive Level: Apply (application) REF: 550 | 554 OBJ: Special Questions: Prioritization TOP: Nursing Process: Assessment MSC:

Integrated Process: Caring 36. The nurse is assessing a client who has a chest tube. Which assessment finding requires intervention by the nurse? a. Pain at the insertion site b. Bloody drainage in the collection chamber c. Intermittent bubbling in the water seal chamber d. Tidaling in the water seal chamber

A Pain is the priority for the client. Bloody drainage may be normal, depending on the client's condition. Intermittent bubbling in the water seal indicates air escaping as the lung fully expands, and does not need to be addressed immediately. Tidaling often occurs with inspiration and expiration. DIF: Cognitive Level: Application/Applying or higher REF: N/A TOP: Client Needs Category: Physiological Integrity (Physiological Adaptation—Illness Management)

Physiological Integrity 22. A patient experiences a chest wall contusion as a result of being struck in the chest with a baseball bat. The emergency department nurse would be most concerned if which finding is observed during the initial assessment? a. Paradoxic chest movement b. Complaint of chest wall pain c. Heart rate of 110 beats/minute d. Large bruised area on the chest

A Paradoxic chest movement indicates that the patient may have flail chest, which can severely compromise gas exchange and can rapidly lead to hypoxemia. Chest wall pain, a slightly elevated pulse rate, and chest bruising all require further assessment or intervention, but the priority concern is poor gas exchange. DIF: Cognitive Level: Apply (application) REF: 543 TOP: Nursing Process: Assessment MSC:

Physiological Integrity 8. A nurse obtains a health history from a patient who has a 35 pack-year smoking history. The patient complains of hoarseness and tightness in the throat and difficulty swallowing. Which question is most important for the nurse to ask? a. "How much alcohol do you drink in an average week?" b. "Do you have a family history of head or neck cancer?" c. "Have you had frequent streptococcal throat infections?" d. "Do you use antihistamines for upper airway congestion?"

A Prolonged alcohol use and smoking are associated with the development of laryngeal cancer, which the patient's symptoms and history suggest. Family history is not a risk factor for head or neck cancer. Frequent antihistamine use would be asked about if the nurse suspected allergic rhinitis, but the patient's symptoms are not suggestive of this diagnosis. Streptococcal throat infections also may cause these clinical manifestations, but patients with this type of infection will also have pain and a fever. DIF: Cognitive Level: Apply (application) REF: 515 TOP: Nursing Process: Assessment MSC:

Physiological Integrity 9. A patient with COPD has a "barrel chest." The nurse would expect the chest x-ray report to indicate that there is a. overinflation of the alveoli. b. consolidation of lung tissue. c. fluid in the alveoli. d. air in the pleural space.

A Rationale: A barrel chest results from lung hyperinflation and is a common finding in patients with COPD. Consolidation, fluid, and air in the pleural space all would indicate that intervention is needed. Cognitive Level: Application Text Reference: pp. 511, 521 Nursing Process: Assessment

Physiological Integrity 41. A patient with a chronic productive cough and weight loss is receiving a tuberculosis skin test and asks the nurse the reason for the test. Which response should the nurse give? a. The skin test will determine if you have a tuberculosis infection. b. The skin test will indicate whether you have active tuberculosis. c. The skin test is used to decide which antibiotic therapy will work best. d. The skin test is done prior to notification of the public health department.

A Rationale: A positive skin test will indicate whether the patient has been infected with tuberculosis. It does not indicate active infection, which will be established through chest x-ray and sputum culture. Initial drug treatment with 4 antibiotics uses a standardized protocol. Although the public health department should be notified if the patient has TB, the nurse should focus on the patient, rather than on the public health concerns. Cognitive Level: Application Text Reference: p. 571 Nursing Process: Implementation

Physiological Integrity 8. To evaluate both oxygenation and ventilation in a patient with acute respiratory failure, the nurse uses the findings revealed with a. arterial blood gas (ABG) analysis. b. hemodynamic monitoring. c. chest x-rays. d. pulse oximetry.

A Rationale: ABG analysis is useful because it provides information about both oxygenation and ventilation and assists with determining possible etiologies and appropriate treatment. The other tests may also provide useful information about patient status but will not indicate whether the patient has hypoxemia, hypercapnia, or both. Cognitive Level: Comprehension Text Reference: p. 1805 Nursing Process: Assessment

Safe and Effective Care Environment 15. A 36-year-old patient who has a history of thromboangiitis obliterans (Buerger's disease) is admitted to the hospital with a gangrenous lesion of the right small toe. When the nurse is planning expected outcomes for the patient, which outcome has the highest priority for this patient? a. Cessation of smoking b. Maintenance of appropriate weight c. Control of serum lipid levels d. Demonstration of meticulous foot care

A Rationale: Absolute cessation of nicotine use is needed to reduce the risk for amputation in patients with Buerger's disease. Other therapies have limited success in treatment of this disease. Cognitive Level: Application Text Reference: p. 908 Nursing Process: Planning

Physiological Integrity 18. After 2 months of TB treatment with a standard four-drug regimen, a patient continues to have positive sputum smears for acid-fast bacilli (AFB). The nurse discusses the treatment regimen with the patient with the knowledge that a. directly observed therapy (DOT) will be necessary if the medications have not been taken correctly. b. the positive sputum smears indicate that the patient is experiencing toxic reactions to the medications. c. twice-weekly administration may be used to improve compliance with the treatment regimen. d. a regimen using only INH and rifampin (Rifadin) will be used for the last 4 months of drug therapy.

A Rationale: After 2 months of therapy, negative sputum smears would be expected if the TB bacillus is susceptible to the medications and if the medications have been taken correctly. The nurse will need to initiate DOT if the patient has not been consistently taking the medications. Toxic reactions to the medications would not result in a positive sputum smear. Twice-weekly medication administration is not one of the options for therapy. INH and rifampin are used for the last 4 months of drug therapy only if the initial four-drug regimen has been effective as evidenced by negative sputum smears. Cognitive Level: Application Text Reference: pp. 571-572 Nursing Process: Implementation

Physiological Integrity 15. A patient admitted to the coronary care unit (CCU) with an MI and frequent premature ventricular contractions (PVCs) has health care provider orders for continuous amiodarone infusion, IV nitroglycerin infusion, and morphine sulfate 2 mg IV every 10 minutes until there is relief of pain. The patient says, "This is the worst pain I have ever had. Am I going to die?" Based on these data, the nurse identifies a priority nursing diagnosis of a. acute pain related to myocardial ischemia. b. anxiety related to perceived threat of death. c. decreased cardiac output related to cardiogenic shock. d. activity intolerance related to decreased cardiac output.

A Rationale: All the nursing diagnoses may be appropriate for this patient, but the data indicate that the priority diagnosis is pain, a physiologic stressor. The patient's anxiety will also be reduced if the pain is resolved. There are no data indicating that the patient is experiencing cardiogenic shock or activity intolerance. Cognitive Level: Application Text Reference: pp. 806, 811, 813 Nursing Process: Diagnosis

Physiological Integrity 14. In analyzing the results of a patient's blood gas analysis, the nurse will be most concerned about an a. arterial oxygen tension (PaO2) of 60 mm Hg. b. arterial oxygen saturation (SaO2) of 91%. c. arterial carbon dioxide (PaCO2) of 47 mm Hg. d. arterial bicarbonate level (HCO3) of 27 mEq/L.

A Rationale: All the values are abnormal, but the low PaO2 indicates that the patient is at the point on the oxyhemoglobin dissociation curve where a small change in the PaO2 will cause a large drop in the O2 saturation and a decrease in tissue oxygenation. The nurse should intervene immediately to improve the patient's oxygenation. Cognitive Level: Analysis Text Reference: p. 513 Nursing Process: Assessment

Physiological Integrity 12. A patient with myocardial infarction develops symptomatic hypotension. The monitor shows a type 1, second-degree AV block with a heart rate of 30. The nurse administers IV atropine as prescribed. The nurse determines that the drug has been effective on finding a(n) a. increase in the patient's heart rate. b. increase in peripheral pulse volume. c. decrease in ventricular response. d. decrease in premature contractions.

A Rationale: Atropine will increase the heart rate and conduction through the AV node. Because the medication increases electrical conduction, not cardiac contractility, the quality of the peripheral pulses is not used to evaluate the drug effectiveness. Ventricular response will be increased by atropine because of the improvement in AV conduction. Atropine will not decrease PVCs, and the patient does not have PVCs. Cognitive Level: Application Text Reference: p. 853 Nursing Process: Evaluation

Physiological Integrity 31. The emergency department nurse will suspect a tension pneumothorax in a patient who has been in an automobile accident if a. the breath sounds on one side are decreased. b. there are wheezes audible throughout both lungs. c. there is a sucking sound with each patient breath. d. paradoxic movement of the chest is noted.

A Rationale: Breath sounds are decreased on the affected side with tension pneumothorax because air trapped in the pleural space compresses the lung on that side. Wheezes that are heard in both lungs indicate airway narrowing, but not pneumothorax. A sucking sound with inspiration is heard with an open pneumothorax. Paradoxic chest movement is associated with flail chest. Cognitive Level: Application Text Reference: p. 586 Nursing Process: Assessment

Physiological Integrity 13. The nurse has received the laboratory results for a patient who developed chest pain 2 hours ago and may be having a myocardial infarction. The most important laboratory result to review will be a. troponins T and I. b. creatine kinase-MB. c. LDL cholesterol. d. C-reactive protein.

A Rationale: Cardiac troponins start to elevate 1 hour after myocardial injury and are specific to myocardium. Creatine kinase (CK-MB) is specific to myocardial injury and infarction, but it does not increase until 4 to 6 hours after the infarction occurs. LDL cholesterol and C-reactive protein are useful in assessing cardiovascular risk but are not helpful in determining whether a patient is having an acute myocardial infarction. Cognitive Level: Application Text Reference: pp. 751-752 Nursing Process: Assessment

Health Promotion and Maintenance 32. A 19-year-old male patient with CF and his wife are considering having a child. Which statement by the patient indicates that the nurse's teaching has been effective? a. "We will plan on having genetic counseling before we make a decision." b. "My erectile dysfunction will make it more difficult to have a child." c. "It is likely that I will die before any children we have are grown." d. "There should not be any problems as long as I take my medications."

A Rationale: Children of patients with CF are either CF carriers or have the disease. Most men with CF are sterile, but erectile dysfunction is not associated with CF. The life expectancy for CF is getting longer, with a mean age of 35. Despite appropriate therapy, the couple is likely to experience problems becoming pregnant. Cognitive Level: Application Text Reference: p. 659 Nursing Process: Evaluation

Physiological Integrity 7. A patient with COPD is admitted to the hospital with dyspnea and a cough producing yellow sputum. When palpating the patient's thorax, the nurse will expect to find that chest expansion is a. diminished. b. asymmetric. c. normal. d. increased.

A Rationale: Chronic lung hyperinflation, such as occurs in COPD, decreases expansion of the lungs with inspiration. Lung expansion is usually symmetrical with emphysema. Cognitive Level: Application Text Reference: pp. 524, 526 Nursing Process: Assessment

Physiological Integrity 27. A patient with PAD has a new prescription for clopidogrel (Plavix). Which information should the nurse include when teaching the patient about this medication? a. "Call if you notice that your stools are black or have blood in them." b. "Take the Plavix on an empty stomach as soon as you get up." c. "Change position slowly to avoid dizziness while you are taking Plavix." d. "You should never use aspirin while you are taking the Plavix."

A Rationale: Clopidogrel inhibits platelet function and increases the risk for GI bleeding. It can be taken without regard to food. Orthostatic hypotension is not an expected side effect of the medication. Aspirin may be prescribed concurrently with clopidogrel for some patients. Cognitive Level: Application Text Reference: p. 902 Nursing Process: Implementation

Physiological Integrity 19. Which information obtained by the nurse when assessing a patient with acute respiratory distress syndrome (ARDS) who is being treated with mechanical ventilation and high levels of positive end-expiratory pressure (PEEP) indicates a complication of ventilator therapy is occurring? a. The patient has subcutaneous emphysema. b. The patient has a sinus bradycardia, rate 52. c. The patient's PaO2 is 50 mm Hg and the SaO2 is 88%. d. The patient has bronchial breath sounds in both the lung fields.

A Rationale: Complications of positive-pressure ventilation (PPV) and PEEP include subcutaneous emphysema. Bradycardia, hypoxemia, and bronchial breath sounds are all concerns, but they are not caused by PPV and PEEP. Cognitive Level: Application Text Reference: p. 1816 Nursing Process: Assessment

Physiological Integrity 4. To promote airway clearance in a patient with pneumonia, the nurse instructs the patient to a. splint the chest when coughing. b. maintain fluid restrictions. c. wear the nasal oxygen cannula. d. try the pursed-lip breathing technique.

A Rationale: Coughing is less painful and more likely to be effective when the patient splints the chest during coughing. Fluids should be encouraged to help liquefy secretions. Nasal oxygen will improve gas exchange in patients with chronic obstructive pulmonary disease (COPD) but will not improve airway clearance in pneumonia. Cognitive Level: Application Text Reference: p. 568 Nursing Process: Implementation

Physiological Integrity 31. Which of these nursing interventions included in the plan of care for a patient who had an AMI 3 days ago is most appropriate for the RN to delegate to an experienced LPN/LVN? a. Administration of the ordered metoprolol (Lopressor) and aspirin b. Evaluating the patient's response to ambulation in the hallway c. Teaching the patient about the pathophysiology of heart disease d. Completing the documentation for a home health nurse referral

A Rationale: LPN/LVN education and scope of practice include safe administration of medications. Evaluating the patient response to exercise after an AMI requires more education and should be done by the RN. Teaching and discharge planning/documentation are higher level skills that require RN education and scope of practice. Cognitive Level: Application Text Reference: pp. 810-817 Nursing Process: Evaluation

Physiological Integrity 28. Heparin is prescribed for a patient who has dilated cardiomyopathy has been admitted to the hospital with fatigue and orthopnea. Which statement is appropriate for the nurse to use in patient teaching about anticoagulation therapy? a. "Heparin will help prevent blood clots from forming in your heart chambers." b. "Heparin is used to improve the circulation to the muscles in your arms and legs." c. "Heparin has been prescribed to stop blood clots from traveling to your lungs." d. "Heparin makes it easier for your heart to pump and will decrease your symptoms."

A Rationale: Decreased blood flow through the heart causes blood stasis and the formation of blood clots in the ventricles, which then may embolize. Anticoagulant therapy will not improve circulation to the skeletal muscles. The patient with dilated cardiomyopathy who is inactive may be at risk for deep-vein thrombosis and pulmonary emboli, but this is not the usual reason for anticoagulation. There is no indication in the stem that the patient is immobile. Heparin will not decrease cardiac workload or decrease the patient's fatigue or orthopnea. Cognitive Level: Application Text Reference: p. 886 Nursing Process: Implementation

Physiological Integrity 19. The nurse has identified the collaborative problem of potential complication: pulmonary embolism for a patient with left-calf DVT. Which nursing action is appropriate to include in the plan of care? a. Maintain bed rest as ordered. b. Administer oxygen to keep O2 saturation >90%. c. Apply compression gradient stockings. d. Remind the patient to dorsiflex the feet and rotate the ankles.

A Rationale: Decreasing muscle activity of the leg will help prevent thrombus dislodgement. There is no need to administer oxygen unless the patient develops a pulmonary embolism. Compression gradient stockings are ordered after resolution of the DVT to prevent further DVT. Exercising the muscles in the legs may prevent a new DVT, but it may dislodge the current thrombus if the patient has a DVT. Cognitive Level: Application Text Reference: p. 912 Nursing Process: Planning

Physiological Integrity 2. A patient who was admitted to the hospital with pneumococcal pneumonia has a temperature of 101.6° F with a frequent cough and is complaining of chest pain rated 7 on a 10-point scale with deep inspiration. Which of these ordered medications should the nurse give first? a. Azithromycin (Zithromax) b. Acetaminophen (Tylenol) c. Guaifenesin (Robitussin) d. Codeine phosphate (Codeine)

A Rationale: Early initiation of antibiotic therapy has been demonstrated to reduce mortality. The other medications are also appropriate and should be given as soon as possible, but the priority is to start antibiotic therapy. Cognitive Level: Application Text Reference: pp. 563, 566 Nursing Process: Implementation

Physiological Integrity 2. The nurse will monitor for clinical manifestations of hypercapnia when a patient in the emergency department has a. chest trauma and multiple rib fractures. b. carbon monoxide poisoning after a house fire. c. left-sided ventricular failure and acute pulmonary edema. d. tachypnea and acute respiratory distress syndrome (ARDS).

A Rationale: Hypercapnia is caused by poor ventilatory effort, which occurs in chest trauma when rib fractures (or flail chest) decrease lung ventilation. Carbon monoxide poisoning, acute pulmonary edema, and ARDS are more commonly associated with hypoxemia. Cognitive Level: Application Text Reference: p. 1800 Nursing Process: Assessment

Physiological Integrity 18. The health care provider orders a continuous IV heparin infusion for a patient with swelling and pain of the upper leg caused by a DVT. While the patient is receiving the heparin infusion, the nurse should a. avoid any IM medications to prevent localized bleeding. b. notify the health care provider if the partial thromboplastin time (PTT) value is greater than 50 seconds. c. have vitamin K available in case reversal of the heparin is needed. d. monitor posterior tibial and dorsalis pedis pulses with the Doppler.

A Rationale: IM injections are avoided in patients receiving anticoagulation. A PTT of 50 seconds is within the therapeutic range. Vitamin K is used to reverse warfarin. Pulse quality is not affected by DVT. Cognitive Level: Application Text Reference: pp. 913, 916 Nursing Process: Planning

Physiological Integrity 14. When admitting a patient in possible respiratory failure with a high PaCO2, which assessment information will be of most concern to the nurse? a. The patient is somnolent. b. The patient's SpO2 is 90%. c. The patient complains of weakness. d. The patient's blood pressure is 162/94.

A Rationale: Increasing somnolence will decrease the patient's respiratory rate and further increase the PaCO2 and respiratory failure. Rapid action is needed to prevent respiratory arrest. An SpO2 of 90%, weakness, and elevated blood pressure all require ongoing monitoring but are not indicators of possible impending respiratory arrest. Cognitive Level: Application Text Reference: p. 1804 Nursing Process: Assessment

Physiological Integrity 45. All of the following orders are received for a patient who has just been admitted with probable bacterial pneumonia and sepsis. Which one will the nurse accomplish first? a. Obtain blood cultures from two sites. b. Give ciprofloxin (Cipro) 400 mg IV. c. Send to radiology for chest radiograph. d. Administer aspirin suppository.

A Rationale: Initiating antibiotic therapy rapidly is essential, but it is important that the cultures be obtained before antibiotic administration. The chest radiograph and aspirin administration can be done last. Cognitive Level: Application Text Reference: p. 566 Nursing Process: Implementation

Physiological Integrity 11. While teaching a patient with asthma the appropriate use of a peak flow meter, the nurse instructs the patient to a. take and record peak flow readings when having asthma symptoms or an attack. b. increase the doses of long-term control medications for peak flows in the red zone. c. use the flow meter each morning after taking asthma medications. d. empty the lungs and then inhale rapidly through the mouthpiece.

A Rationale: It is recommended that patients check peak flows when asthma symptoms or attacks occur to compare the peak flow with the baseline. Increased doses of rapidly acting 2-agonists are indicated for peak flows in the red zone. Peak flows should be checked every morning before using medications. Peak flows are assessed during rapid exhalation. Cognitive Level: Application Text Reference: pp. 625, 628 Nursing Process: Implementation

Physiological Integrity 4. A patient is admitted with a metabolic acidosis of unknown origin. Based on this diagnosis, the nurse would expect the patient to have a. Kussmaul's respirations. b. slow, shallow respirations. c. a low oxygen saturation (SpO2). d. a decrease in PVO2.

A Rationale: Kussmaul's (deep and rapid) respirations are a compensatory mechanism for metabolic acidosis. Slow, shallow respirations, a low oxygen saturation rate, and a decrease in PVO2 would not be caused by acidosis. Cognitive Level: Analysis Text Reference: p. 524 Nursing Process: Assessment

Physiological Integrity 7. Which finding would be the best indication to the nurse that the patient having an acute asthma attack was responding to the prescribed bronchodilator therapy? a. Wheezes are more easily heard. b. The oxygen saturation is 89%. c. Vesicular breath sounds resolve. d. The respiratory effort decreases.

A Rationale: Louder wheezes indicate that more air is moving through the airways and that the bronchodilator therapy is working. An oxygen saturation level less than 90% indicates continued hypoxemia. Vesicular breath sounds are normal. A decreased respiratory effort may indicate that the patient is becoming too fatigued to breathe effectively and needs mechanical ventilation. Cognitive Level: Application Text Reference: p. 617 Nursing Process: Evaluation

Physiological Integrity 6. After repair of an abdominal aortic aneurysm, the nurse notes that the patient does not have popliteal, posterior tibial, or dorsalis pedis pulses. The legs are cool and mottled. Which action is appropriate for the nurse to take first? a. Review the preoperative assessment form for data about the pulses. b. Notify the surgeon and anesthesiologist. c. Document that the pulses are absent and recheck in 30 minutes. d. Elevate the lower extremities on pillows.

A Rationale: Many patients with aortic aneurysms also have peripheral arterial disease, so the nurse should check the preoperative assessment to determine whether pulses were present before surgery before notifying the health care providers about the absent pulses. Because the patient's symptoms suggest graft occlusion or multiple emboli and a possible need to return to surgery, it is not appropriate to wait 30 minutes before taking action. Elevating the legs will decrease blood flow. Cognitive Level: Application Text Reference: p. 898 Nursing Process: Implementation

Health Promotion and Maintenance 6. A patient in the intensive care unit with ADHF complains of severe dyspnea and is anxious, tachypneic, and tachycardic. All these medications have been ordered for the patient. The first action by the nurse will be to a. administer IV morphine sulfate 2 mg. b. give IV diazepam (Valium) 2.5 mg. c. increase dopamine (Intropin) infusion by 2 mcg/kg/min. d. increase nitroglycerin (Tridil) infusion by 5 mcg/min.

A Rationale: Morphine improves alveolar gas exchange, improves cardiac output by reducing ventricular preload and afterload, decreases anxiety, and assists in reducing the subjective feeling of dyspnea. Diazepam may decrease patient anxiety, but it will not improve the cardiac output or gas exchange. Increasing the dopamine may improve cardiac output but will also increase the heart rate and myocardial oxygen consumption. Nitroglycerin will improve cardiac output and may be appropriate for this patient, but it will not directly reduce anxiety and will not act as quickly as morphine to decrease dyspnea. Cognitive Level: Analysis Text Reference: pp. 828-829 Nursing Process: Implementation

Physiological Integrity 21. A patient with ADHF who is receiving nesiritide (Natrecor) asks the nurse how the medication will work to help improve the symptoms of dyspnea and orthopnea. The nurse's reply will be based on the information that nesiritide will a. dilate arterial and venous blood vessels, decreasing ventricular preload and afterload. b. improve the ability of the ventricular myocardium to contract, strengthening contractility. c. enhance the speed of impulse conduction through the heart, increasing the heart rate. d. increase calcium sensitivity in vascular smooth muscle, boosting systemic vascular resistance.

A Rationale: Nesiritide, a recombinant form of BNP, causes both arterial and venous vasodilation, leading to reductions in preload and afterload. Inotropic medications, such as dopamine and dobutamine, may be used in ADHF to improve ventricular contractility. Nesiritide does not increase impulse conduction or calcium sensitivity in the heart. Cognitive Level: Application Text Reference: p. 829 Nursing Process: Implementation

Physiological Integrity 16. When teaching the patient who is receiving standard multidrug therapy for TB about possible toxic effects of the antitubercular medications, the nurse will give instructions to notify the health care provider if the patient develops a. yellow-tinged skin. b. changes in hearing. c. orange-colored urine. d. thickening of the nails.

A Rationale: Noninfectious hepatitis is a toxic effect of INH, rifampin, and pyrazinamide, and patients who develop hepatotoxicity will need to use other medications. Changes in hearing and nail thickening are not expected with the four medications used for initial drug therapy. Orange discoloration of body fluids is an expected side effect of rifampin and not an indication to call the health care provider. Cognitive Level: Application Text Reference: pp. 572-573 Nursing Process: Implementation

Psychosocial Integrity 27. A patient with COPD is admitted to the hospital. How can the nurse best position the patient to improve gas exchange? a. Sitting up at the bedside in a chair and leaning slightly forward b. Resting in bed with the head elevated to 45 to 60 degrees c. In the Trendelenburg's position with several pillows behind the head d. Resting in bed in a high-Fowler's position with the knees flexed

A Rationale: Patients with COPD improve the mechanics of breathing by sitting up in the "tripod" position. Resting in bed with the head elevated would be an alternative position if the patient was confined to bed, but sitting in a chair allows better ventilation. The Trendelenburg's position or sitting upright in bed with the knees flexed would decrease the patient's ability to ventilate well. Cognitive Level: Application Text Reference: p. 634 Nursing Process: Implementation

Physiological Integrity 8. To assess the patient with pericarditis for the presence of a pericardial friction rub, the nurse should a. place the diaphragm of the stethoscope at the lower left sternal border of the chest. b. ask the patient to stop breathing during auscultation to distinguish the sound from a pleural friction rub. c. use the diaphragm of the stethoscope to listen for a rumbling, low-pitched, systolic sound. d. feel the precordial area with the palm of the hand to detect vibration with cardiac contraction.

A Rationale: Pericardial friction rubs are heard best with the diaphragm at the lower left sternal border. Because dyspnea is one clinical manifestation of pericarditis, the nurse should time the friction rub with the pulse rather than ask the patient to stop breathing during auscultation. Friction rubs are not typically low pitched or rumbling and are not confined to systole. Rubs are not assessed by palpation. Cognitive Level: Comprehension Text Reference: p. 872 Nursing Process: Assessment

Physiological Integrity 23. The occupational nurse at a manufacturing plant where there is high worker exposure to beryllium dust will monitor workers for a. shortness of breath. b. chest pain. c. elevated temperature. d. barrel-chest.

A Rationale: The nurse will monitor for the earliest signs of occupational lung disease, which are dyspnea and a cough. The other symptoms are also consistent with occupational lung disease but would occur much later, after significant lung involvement has occurred. Cognitive Level: Application Text Reference: pp. 577-578 Nursing Process: Assessment

Physiological Integrity 7. A patient is admitted to the hospital with possible acute pericarditis. The nurse will plan to teach the patient about the purpose of a. multiple ECGs. b. daily blood cultures. c. cardiac catheterization. d. pericardiocentesis.

A Rationale: Pericarditis causes changes such as ST segment elevation in multiple leads on the ECG, which evolve over the course of the inflammatory process. Blood cultures are not indicated unless the patient has evidence of sepsis. Cardiac catheterization is not a diagnostic procedure for pericarditis. Pericardiocentesis will not be done unless the patient has symptoms of cardiac tamponade. Cognitive Level: Application Text Reference: p. 872 Nursing Process: Planning

Health Promotion and Maintenance 15. A patient is receiving isoniazid (INH) after having a positive tuberculin skin test. Which information will the nurse include in the patient teaching plan? a. "Take vitamin B6 daily to prevent peripheral nerve damage." b. "Read a newspaper daily to check for changes in vision." c. "Schedule an audiometric examination to monitor for hearing loss." d. "Avoid wearing soft contact lenses to avoid orange staining."

A Rationale: Peripheral neurotoxicity associated can be prevented by taking vitamin B6 when being treated with INH. Visual changes, hearing problems, and orange staining are adverse effects of other TB medications. Cognitive Level: Application Text Reference: p. 573 Nursing Process: Planning

Physiological Integrity 11. The nurse observes a nursing assistant doing all the following activities when caring for a patient with right lower-lobe pneumonia. The nurse will need to intervene when the nursing assistant a. turns the patient over to the right side. b. splints the patient's chest during coughing. c. elevates the patient's head to 45 degrees. d. assists the patient to get up to the bathroom.

A Rationale: Positioning the patient with the left (or "good" lung) down will improve oxygenation. The other actions are appropriate for a patient with pneumonia. Cognitive Level: Application Text Reference: p. 569 Nursing Process: Implementation

Physiological Integrity 24. Postural drainage with percussion and vibration is ordered bid for a patient with chronic bronchitis. The nurse will plan to a. carry out the procedure 3 hours after the patient eats. b. maintain the patient in the lateral positions for 20 minutes. c. perform percussion and vibration before placing the patient in the drainage position. d. give the ordered albuterol (Proventil) after the patient has received the therapy.

A Rationale: Postural drainage, percussion, and vibration should be done 1 hour before or 3 hours after meals. Patients remain in each postural drainage position for 5 to 15 minutes. Percussion and vibration are done after the postural drainage. Bronchodilators are administered before chest physiotherapy. Cognitive Level: Application Text Reference: p. 647 Nursing Process: Planning

Physiological Integrity 22. A patient with COPD asks the home health nurse about home oxygen use. The nurse should teach the patient that long-term home O2 therapy a. can improve the patient's prognosis and quality of life. b. may cause oxygen dependency in patients with COPD. c. is used only for patients who have severe end-stage respiratory disease. d. should never be used at night because the patient cannot monitor its effect.

A Rationale: Research supports the use of home oxygen to improve quality of life and prognosis. Oxygen dependency is not an issue for patients with COPD. Although most patients using home oxygen have SpO2 levels less than 89% on room air, it would not be appropriate to tell the patient that he or she was at the end stage of the disease. Oxygen use at night can improve sleep quality and is frequently recommended. Cognitive Level: Application Text Reference: p. 644 Nursing Process: Implementation

Health Promotion and Maintenance 20. While caring for a patient with aortic stenosis, the nurse establishes a nursing diagnosis of pain related to decreased coronary blood flow. An appropriate intervention by the nurse is to a. promote rest to decrease myocardial oxygen demand. b. teach the patient to use sublingual nitroglycerin for chest pain. c. educate the patient about the need for anticoagulant therapy. d. elevate the head of the bed 40 degrees to decrease venous return.

A Rationale: Rest is recommended to balance myocardial oxygen supply and demand and to decrease chest pain. The patient with aortic stenosis requires higher preload to maintain cardiac output, so nitroglycerin and measures to decrease venous return are contraindicated. Anticoagulation is not recommended unless the patient has atrial fibrillation. Cognitive Level: Application Text Reference: p. 880 Nursing Process: Implementation

Physiological Integrity 37. A patient with primary pulmonary hypertension is receiving nifedipine (Procardia). The nurse will evaluate that the treatment is effective if a. the patient reports decreased exertional dyspnea. b. the blood pressure is less than 140/90 mm Hg. c. the heart rate is between 60 and 100 beats/minute. d. the patient's chest x-ray indicates clear lung fields.

A Rationale: Since a major symptom of PPH is exertional dyspnea, an improvement in this symptom would indicate that the medication was effective. Nifedipine will affect BP and heart rate, but these parameters would not be used to monitor effectiveness of therapy for a patient with PPH. The chest x-ray will show clear lung fields even if the therapy is not effective. Cognitive Level: Application Text Reference: p. 601 Nursing Process: Evaluation

Physiological Integrity 15. The nurse hears a murmur between the S1 and S2 heart sounds at the patient's left 5th intercostal space and midclavicular line. The best way to record this information is a. "systolic murmur heard at mitral area." b. "diastolic murmur heard at aortic area." c. "systolic murmur heard at Erb's point." d. "diastolic murmur heard at tricuspid area."

A Rationale: The S1 sound is created by closure of the mitral and tricuspid valves and signifies the onset of ventricular systole. S2 is caused by the closure of the aortic and pulmonic valves and signifies the onset of diastole. A murmur occurring between these two sounds is a systolic murmur. The mitral area is the intersection of the left 5th intercostal space and the midclavicular line. The aortic area is located at the 2nd intercostal space along the right sternal border. Erb's point is located at the 3rd intercostal space along the left sternal border. The tricuspid area is located at the 5th intercostal space along the left sternal border. Cognitive Level: Application Text Reference: pp. 749-751 Nursing Process: Assessment

c (The patient's peak flow readings indicate good asthma control, and no changes are needed. The other actions would be used for patients in the yellow or red zones for peak flow.)

A patient seen in the asthma clinic has recorded daily peak flows that are 85% of the baseline. Which action will the nurse plan to take? A. Teach the patient about the use of oral corticosteroids. B. Administer a bronchodilator and recheck the peak flow. C. Instruct the patient to continue to use current medications. D. Evaluate whether the peak flow meter is being used correctly.

Physiological Integrity 22. A patient with acute respiratory distress syndrome (ARDS) has progressed to the fibrotic phase. The patient's family members are anxious about the patient's condition and are continuously present at the hospital. In addressing the family's concerns, it is important for the nurse to a. support the family and help them understand the realistic expectation that the patient's chance for survival is poor. b. inform the family that home health nurses will be able to help them maintain the mechanical ventilation at home after patient discharge. c. refer the family to social support services and case management to plan for transfer of the patient to a long-term care facility. d. provide hope and encouragement to the family because the patient's disease process has started to resolve.

A Rationale: The chance for survival is poor when the patient progresses to the fibrotic stage because permanent damage to the alveoli has occurred. Because of continued severe hypoxemia, the patient is not a candidate for home health or long-term care. The fibrotic stage indicates a poor patient prognosis, not the resolution of the ARDS process. Cognitive Level: Application Text Reference: p. 1814 Nursing Process: Implementation

Physiological Integrity 13. A patient with rheumatic fever has subcutaneous nodules, erythema marginatum, and polyarthritis. An appropriate nursing diagnosis based on these findings is a. activity intolerance related to fatigue and arthralgia. b. risk for infection related to open skin lesions. c. risk for impaired skin integrity related to pruritus. d. impaired physical mobility related to permanent joint fixation.

A Rationale: The clinical manifestations of rheumatic fever include fatigue and arthralgia. The skin lesions seen in rheumatic fever are not open or pruritic. Joint inflammation is a temporary clinical manifestation of rheumatic fever. Cognitive Level: Application Text Reference: p. 877 Nursing Process: Diagnosis

Health Promotion and Maintenance 16. While caring for a patient with respiratory disease, the nurse observes that the patient's SpO2 drops from 94% to 85% when the patient ambulates in the hall. The nurse determines that a. supplemental oxygen should be used whenever the patient exercises. b. arterial blood gas analysis should be done to verify the patient's SpO2. c. the response is normal and the patient should continue at this activity level. d. the patient activity should be limited until the disease process is resolved.

A Rationale: The drop in SpO2 to 85% indicates that the patient is hypoxemic and needs supplemental oxygen when exercising. ABG measurements are unnecessary and would increase patient discomfort and expense. The patient will need to continue to ambulate to avoid the many complications of immobility. Cognitive Level: Application Text Reference: p. 515 Nursing Process: Evaluation

Physiological Integrity 18. The nurse administers IV nitroglycerin to a patient with an MI. In evaluating the effect of this intervention, the nurse should monitor for a. relief of chest discomfort. b. a decreased heart rate. c. an increase in BP. d. fewer cardiac dysrhythmias.

A Rationale: The goal of IV nitroglycerin administration in AMI is relief of chest pain by improving the balance between myocardial oxygen supply and demand. Increases in heart rate and a drop in BP are common side effects of nitroglycerin. Nitroglycerin does not directly impact cardiac dysrhythmias. Cognitive Level: Application Text Reference: pp. 806, 808 Nursing Process: Evaluation

Health Promotion and Maintenance 15. The community health nurse involved in programs to prevent rheumatic fever knows that the most important intervention to decrease the incidence of the disease is a. teaching people to seek medical diagnosis and treatment for streptococcal pharyngitis. b. providing prophylactic antibiotics to people with a family history of rheumatic fever. c. immunizing susceptible groups of people with streptococcal vaccine. d. promoting hygienic measures to prevent the transmission of streptococcal infections.

A Rationale: The incidence of rheumatic fever is decreased by treatment of streptococcal infections with antibiotics. Family history is not a risk factor for rheumatic fever. There is no immunization that is effective in decreasing the incidence of rheumatic fever. Hygienic measures may help to decrease the spread of streptococcal infections but is not the primary means of decreasing risk for rheumatic fever. Cognitive Level: Comprehension Text Reference: p. 878 Nursing Process: Planning

Physiological Integrity 11. A patient experiences dizziness and shortness of breath for several days. During cardiac monitoring in the ED, the nurse obtains the following ECG tracing. The nurse interprets this cardiac rhythm as a. third-degree AV block. b. sinus rhythm with premature atrial contractions (PACs). c. sinus rhythm with PVCs. d. junctional escape rhythm.

A Rationale: The inconsistency between the atrial and ventricular rates and the variable PR interval indicate that the rhythm is third-degree AV block. Sinus rhythm with PACs or PVCs will have a normal rate and consistent PR intervals with occasional PACs or PVCs. A junctional escape rhythm will not have P waves. Cognitive Level: Application Text Reference: p. 854 Nursing Process: Assessment

Physiological Integrity 23. The nurse is admitting a patient who has a diagnosis of an acute asthma attack. Which information obtained by the nurse indicates that the patient may need teaching regarding medication use? a. The patient has been using the albuterol (Proventil) inhaler more frequently over the last 4 days. b. The patient became very short of breath an hour before coming to the hospital. c. The patient has been taking acetaminophen (Tylenol) 650 mg every 6 hours for chest-wall pain. d. The patient says there have been no acute asthma attacks during the last year.

A Rationale: The increased need for a rapid-acting bronchodilator should alert the patient that an acute attack may be imminent and that a change in therapy may be needed. The patient should be taught to contact a health care provider if this occurs. The other data do not indicate any need for additional teaching. Cognitive Level: Application Text Reference: p. 517 Nursing Process: Assessment

Physiological Integrity 19. A staff nurse has a TB skin test of 16-mm induration. A chest radiograph is negative, and the nurse has no symptoms of TB. The occupational health nurse will plan on teaching the staff nurse about the a. use and side effects of INH. b. standard four-drug therapy for TB. c. need for annual repeat TB skin testing. d. recommendation guidelines for bacille Calmette-Guérin (BCG) vaccine.

A Rationale: The nurse is considered to have a latent TB infection and should be treated with INH daily for 6 to 9 months. The four-drug therapy would be appropriate if the nurse had active TB. TB skin testing is not done for individuals who have already had a positive skin test. BCG vaccine is not used in the United States and would not be helpful for this individual, who already has a TB infection. Cognitive Level: Application Text Reference: p. 572 Nursing Process: Planning

Physiological Integrity 22. A patient is receiving fibrinolytic therapy 2 hours after developing an AMI. Which assessment information will be of most concern to the nurse? a. No change in the patient's chest pain b. A large bruise at the patient's IV insertion site c. A decrease in ST-segment elevation on the ECG d. An increase in cardiac enzyme levels since admission

A Rationale: The ongoing chest pain indicates continued myocardial injury and necrosis. The nurse should notify the physician. Bruising is a possible side effect of fibrinolytic therapy, but it is not an indication that therapy should be discontinued. The decrease of the ST-segment elevation indicates that fibrinolysis is occurring and perfusion is returning to the injured myocardium. An increase in cardiac enzyme levels is expected even with reperfusion. Cognitive Level: Application Text Reference: p. 808 Nursing Process: Evaluation

Physiological Integrity 47. A patient with a deep vein thrombophlebitis complains of sudden chest pain and difficulty breathing. The nurse finds a heart rate of 142, BP of 100/60, and respirations of 42. The nurse's first action should be to a. elevate the head of the bed. b. administer the ordered pain medication. c. notify the patient's health care provider. d. offer emotional support and reassurance.

A Rationale: The patient has symptoms consistent with a pulmonary embolism; elevating the head of the bed will improve ventilation and gas exchange. The other actions can be accomplished after the head is elevated (and oxygen is started). Cognitive Level: Application Text Reference: p. 599 Nursing Process: Implementation

Psychosocial Integrity 18. A patient has received instruction on the management of a new permanent pacemaker before discharge from the hospital. The nurse recognizes that teaching has been effective when the patient tells the nurse, a. "I won't lift the arm on the pacemaker side up very high until I see the doctor." b. "I will notify the airlines when I make a reservation that I have a pacemaker." c. "I must avoid cooking with a microwave oven or being near a microwave in use." d. "It will be 6 weeks before I can take a bath or return to my usual activities."

A Rationale: The patient is instructed to avoid lifting the arm on the pacemaker side above the shoulder to avoid displacing the pacemaker leads. The pacemaker rarely sets off an airport security alarm and there is no need to notify the airlines when making a reservation. Microwave oven use does not affect the pacemaker. The insertion procedure involves minor surgery that will have a short recovery period. Cognitive Level: Application Text Reference: p. 861 Nursing Process: Evaluation

Physiological Integrity 19. A transesophageal echocardiogram (TEE) is ordered for a patient with possible endocarditis. Which of these actions included in the standard TEE orders will the nurse need to accomplish first? a. Make the patient NPO. b. Start a large-gauge IV line. c. Administer O2 per mask. d. Give lorazepam (Ativan) 1 mg IV.

A Rationale: The patient will need to be NPO for 6 hours preceding the TEE, so the nurse should place the patient on NPO status as soon as the order is received. The other actions will also need to be accomplished, but not until just before or during the procedure. Cognitive Level: Application Text Reference: p. 754 Nursing Process: Implementation

Physiological Integrity 5. A patient is brought to the emergency department unconscious following a barbiturate overdose. Which potential complication will the nurse include when developing the plan of care? a. Hypercapnic respiratory failure related to decreased ventilatory effort b. Hypoxemic respiratory failure related to diffusion limitations c. Hypoxemic respiratory failure related to shunting of blood d. Hypercapnic respiratory failure related to increased airway resistance

A Rationale: The patient with an opioid overdose develops hypercapnic respiratory failure as a result of the decrease in respiratory rate and depth. Diffusion limitations, blood shunting, and increased airway resistance are not the primary pathophysiology causing the respiratory failure. Cognitive Level: Application Text Reference: p. 1800 Nursing Process: Diagnosis

Health Promotion and Maintenance 19. An outpatient who has developed heart failure after having an acute myocardial infarction has a new prescription for carvedilol (Coreg). After 2 weeks, the patient returns to the clinic. The assessment finding that will be of most concern to the nurse is that the patient a. has BP of 88/42. b. has an apical pulse rate of 56. c. complains of feeling tired. d. has 2+ pedal edema.

A Rationale: The patient's BP indicates that the dose of carvedilol may need to be decreased because the mean arterial pressure is only 57. Bradycardia is a frequent adverse effect of -Adrenergic blockade, but the rate of 56 is not as great a concern as the hypotension. -adrenergic blockade will initially worsen symptoms of heart failure in many patients, and patients should be taught that some increase in symptoms, such as fatigue and edema, is expected during the initiation of therapy with this class of drugs. Cognitive Level: Application Text Reference: p. 832 Nursing Process: Assessment

Physiological Integrity Lewis: Medical-Surgical Nursing, 7th Edition Test Bank Chapter 38: Nursing Management: Vascular Disorders MULTIPLE CHOICE 1. A patient with a history of a 4-cm abdominal aortic aneurysm is admitted to the emergency department with severe back pain and bilateral flank ecchymoses. The vital signs are blood pressure (BP) 90/58, pulse 138, and respirations 34. The nurse plans interventions for the patient based on the expectation that treatment will include a. immediate surgery. b. a STAT angiogram. c. a paracentesis when vital signs are stabilized with fluid replacement. d. admission to intensive care for observation and diagnostic testing.

A Rationale: The patient's history and clinical manifestations are consistent with rupture into the retroperitoneal space, and the patient will need immediate surgery to have a chance at survival. The other listed treatments will all be too time consuming. Cognitive Level: Application Text Reference: p. 895 Nursing Process: Planning

Physiological Integrity 14. The nurse hears the cardiac monitor alarm and notes that the patient has a cardiac pattern of undulations of varying contours and amplitude with no measurable ECG pattern. The patient is unconscious with no pulse or respirations. After calling for assistance, the nurse should a. start basic cardiopulmonary resuscitation (CPR). b. administer an IV bolus dose of epinephrine. c. prepare the patient for endotracheal intubation. d. wait for the defibrillator to arrive.

A Rationale: The patient's rhythm and assessment indicate ventricular fibrillation and cardiac arrest; therefore, the initial actions include calling for help, and initiating CPR until defibrillation is possible. If a defibrillator is not immediately available or is unsuccessful in converting the patient to a better rhythm, CPR should be continued and IV medications and endotracheal intubation should be initiated. Cognitive Level: Application Text Reference: p. 855 Nursing Process: Implementation

Physiological Integrity 40. A patient who was admitted the previous day with pneumonia complains of a sharp pain "whenever I take a deep breath." Which action will the nurse take next? a. Listen to the patient's lungs. b. Check the patient's O2 saturation. c. Have the patient cough forcefully. d. Notify the patient's health care provider.

A Rationale: The patient's statement indicates that pleurisy or a pleural effusion may have developed and the nurse will need to listen for a pleural friction rub and/or decreased breath sounds. The re is no indication that the oxygen saturation has decreased The patient is unlikely to be able to cough forcefully until pain medication has been administered. The nurse will want to obtain more assessment data before calling the health care provider. Cognitive Level: Application Text Reference: p. 597 Nursing Process: Assessment

Physiological Integrity 2. The health care provider writes the following admitting orders for a patient with suspected IE who has fever and chills: ceftriaxone (Rocephin) 1.0 g intravenous piggyback (IVPB) q12hr, acetylsalicylic acid (ASA) for temperature above 102° F (38.9° C), and blood cultures 2, complete blood cell count (CBC), and electrocardiogram (ECG). When admitting the patient, the nurse gives the highest priority to a. obtaining the blood cultures. b. initiating the IV antibiotic. c. scheduling the ECG. d. administering the ASA.

A Rationale: Treatment of the IE with antibiotics should be started as quickly as possible, but it is essential to obtain blood cultures before initiating antibiotic therapy to obtain accurate sensitivity results. The ECG and ASA should also be accomplished rapidly, but the blood cultures (and then administration of the antibiotic) have highest priority. Cognitive Level: Application Text Reference: p. 867 Nursing Process: Planning

Physiological Integrity 11. A patient who has had severe chest pain for the last 4 hours is admitted with a diagnosis of possible AMI. Which of these ordered laboratory tests should the nurse monitor to help determine whether the patient has had an MI? a. Troponin levels b. C-reactive protein c. High-density lipoprotein (HDL) cholesterol d. Homocysteine

A Rationale: Troponin levels increase about 3 hours after the onset of MI. The other laboratory data are useful in determining the patient's risk for developing CAD but are not helpful in determining whether an acute MI is in progress. Cognitive Level: Application Text Reference: pp. 805-806 Nursing Process: Assessment

Health Promotion and Maintenance 8. During physical examination of a thin 72-year-old patient, the nurse observes pulsation of the abdominal aorta in the epigastric area just below the xiphoid process. The nurse teaches the patient that this is a. a normal assessment finding for a thin individual. b. likely to be caused by age-related sclerosis and inelasticity of the aorta. c. an indication that an abdominal aortic aneurysm has probably developed. d. evidence of elevated systemic arterial pressure.

A Rationale: Visible pulsation of the abdominal aorta is commonly observed in the epigastric area for thin individuals. More data would be needed to support a diagnosis of aortic sclerosis, aortic aneurysm, or hypertension. Cognitive Level: Application Text Reference: p. 750 Nursing Process: Assessment

Physiological Integrity 20. When performing an assessment of the patient's respiratory system, the nurse uses the following illustrated technique to evaluate a. chest expansion. b. tactile fremitus. c. accessory muscle use. d. diaphragmatic excursion.

A Rationale: When assessing chest expansion on the posterior chest, the nurse will place the hands at the level of the 10th rib, position the thumbs until they meet over the spine, and have the patient breathe deeply. Tactile fremitus is assessed by having the patient repeat a word or phrase such as "ninety-nine" while the nurse uses the palms of the hands to assess for vibration. Accessory muscle use and anterior-posterior diameter are assessed during inspection of the chest and do not require palpation. Cognitive Level: Comprehension Text Reference: p. 522 Nursing Process: Assessment

Physiological Integrity Lewis: Medical-Surgical Nursing, 7th Edition Test Bank Chapter 35: Nursing Management: Heart Failure MULTIPLE CHOICE 1. A patient with a history of chronic heart failure is admitted to the emergency department with severe dyspnea and a dry, hacking cough. The patient has pitting edema in both ankles, blood pressure (BP) of 170/100, an apical pulse rate of 92, and respirations 28. The most important assessment for the nurse to accomplish next is to a. auscultate the lung sounds. b. assess the orientation. c. check the capillary refill. d. palpate the abdomen.

A Rationale: When caring for a patient with severe dyspnea, the nurse should use the ABCs to guide initial care. This patient's severe dyspnea and cough indicate that acute decompensated heart failure (ADHF) is occurring. ADHF usually manifests as pulmonary edema, which should be detected and treated immediately to prevent ongoing hypoxemia and cardiac/respiratory arrest. The other assessments will provide useful data about the patient's volume status and should also be accomplished rapidly, but detection (and treatment) of fluid-filled alveoli is the priority. Cognitive Level: Application Text Reference: pp. 824-825 Nursing Process: Assessment

Physiological Integrity 12. A 32-year-old patient is seen in the clinic for dyspnea associated with the diagnosis of emphysema. The patient denies any history of smoking. The nurse will anticipate teaching the patient about a. 1-antitrypsin testing. b. use of the nicotine patch. c. continuous pulse oximetry. d. effects of leukotriene modifiers.

A Rationale: When emphysema occurs in young patients, especially without a smoking history, a congenital deficiency in 1-antitrypsin should be suspected. Because the patient does not smoke, a nicotine patch would not be ordered. There is no indication that the patient requires continuous pulse oximetry. Leukotriene modifiers would be used in patients with asthma, not with emphysema. Cognitive Level: Application Text Reference: p. 632 Nursing Process: Planning

Physiological Integrity 38. A patient with a pleural effusion is scheduled for a thoracentesis. Prior to the procedure, the nurse will plan to a. position the patient sitting upright on the edge of the bed and leaning forward. b. instruct the patient about the importance of incentive spirometer use after the procedure. c. start a peripheral intravenous line to administer the necessary sedative drugs. d. remove the water pitcher and remind the patient not to eat or drink anything for 8 hours.

A Rationale: When the patient is sitting up, fluid accumulates in the pleural space at the lung bases and can more easily be located and removed. The lung will expand after the effusion is removed; incentive spirometry is not needed to assure alveolar expansion. The patient does not usually require sedation for the procedure and there are no restrictions on oral intake, since the patient is not sedated or unconscious. Cognitive Level: Application Text Reference: p. 596 Nursing Process: Planning

Integrated Process: Nursing Process (Implementation) 41. The nurse assesses an older adult after an upper respiratory infection and notes the following lung sound on auscultation. What is the nurse's best action? (Click the media button to hear the audio clip.) a. Assess the client for the development of asthma. b. Ask the client if he or she finished all the medication. c. Administer oxygen immediately. d. Assess arterial blood gas.

A Scattered wheezes is the sound heard. New-onset asthma can occur in older clients after they recover from an upper respiratory infection or severe cold. The nurse should assess the client for other symptoms such as sputum production and response to activity. Finishing medication would not necessarily cause the client to have wheezing. The nurse should assess oxygen saturation before administering oxygen or assessing arterial blood gas. DIF: Cognitive Level: Application/Applying or higher REF: N/A TOP: Client Needs Category: Health Promotion and Maintenance (Techniques of Physical Assessment)

Physiological Integrity 12. A patient is admitted to the emergency department complaining of sudden onset shortness of breath and is diagnosed with a possible pulmonary embolus. How should the nurse prepare the patient for diagnostic testing to confirm the diagnosis? a. Start an IV so contrast media may be given. b. Ensure that the patient has been NPO for at least 6 hours. c. Inform radiology that radioactive glucose preparation is needed. d. Instruct the patient to undress to the waist and remove any metal objects.

A Spiral computed tomography (CT) scans are the most commonly used test to diagnose pulmonary emboli, and contrast media may be given IV. A chest x-ray may be ordered but will not be diagnostic for a pulmonary embolus. Preparation for a chest x-ray includes undressing and removing any metal. Bronchoscopy is used to detect changes in the bronchial tree, not to assess for vascular changes, and the patient should be NPO 6 to 12 hours before the procedure. Positron emission tomography (PET) scans are most useful in determining the presence of malignancy, and a radioactive glucose preparation is used. DIF: Cognitive Level: Apply (application) REF: 492 TOP: Nursing Process: Planning MSC:

Integrated Process: Nursing Process (Implementation) 40. The nurse assesses the following lung sounds in a client. What is the nurse's best action? (Click the media button to hear the audio clip.) a. Administer a rescue inhaler. b. Administer oxygen. c. Assess vital signs. d. Elevate the client's head.

A Stridor is the sound heard. This sound indicates severe airway constriction. The nurse must administer a bronchodilator to get air into the lungs. Administering oxygen will not help until the client's airways are open. DIF: Cognitive Level: Application/Applying or higher REF: N/A TOP: Client Needs Category: Physiological Integrity (Physiological Adaptation-Pathophysiology)

Physiological Integrity 13. The nurse hears a murmur between the S1 and S2 heart sounds at the patient's left fifth intercostal space and midclavicular line. How will the nurse record this information? a. Systolic murmur heard at mitral area b. Systolic murmur heard at Erb's point c. Diastolic murmur heard at aortic area d. Diastolic murmur heard at the point of maximal impulse

A The S1 signifies the onset of ventricular systole. S2 signifies the onset of diastole. A murmur occurring between these two sounds is a systolic murmur. The mitral area is the intersection of the left fifth intercostal space and the midclavicular line. The other responses describe murmurs heard at different landmarks on the chest and/or during the diastolic phase of the cardiac cycle. DIF: Cognitive Level: Apply (application) REF: 697 TOP: Nursing Process: Assessment MSC:

Integrated Process: Nursing Process (Implementation) 37. The nurse assesses a client who is on fluticasone (Flovent) and notes oral lesions. What is the nurse's best action? a. Teach the client to rinse the mouth after Flovent use. b. Have the client use a mouthwash daily. c. Start the client on a broad-spectrum antibiotic. d. Document the finding as a known side effect.

A The drug reduces local immunity and increases the risk for local infection, especially Candida albicans. Rinsing the mouth after using the inhaler will decrease the risk for developing this infection. Use of mouthwash and broad-spectrum antibiotics is not warranted in this situation. The nurse should document the finding, but the best action to take is to have the client start rinsing his or her mouth after using Flovent. DIF: Cognitive Level: Application/Applying or higher REF: N/A TOP: Client Needs Category: Physiological Integrity (Pharmacological and Parenteral Therapies—Adverse Effects/Contraindications/Interactions/Side Effects)

Physiological Integrity 12. Which action should the nurse take first when a patient develops a nosebleed? a. Pinch the lower portion of the nose for 10 minutes. b. Pack the affected nare tightly with an epistaxis balloon. c. Obtain silver nitrate that will be needed for cauterization. d. Apply ice compresses over the patient's nose and cheeks.

A The first nursing action for epistaxis is to apply direct pressure by pinching the nostrils. Application of cold packs may decrease blood flow to the area, but will not be sufficient to stop bleeding. Cauterization and nasal packing are medical interventions that may be needed if pressure to the nares does not stop the bleeding, but these are not the first actions to take for a nosebleed. DIF: Cognitive Level: Apply (application) REF: 499 OBJ: Special Questions: Prioritization TOP: Nursing Process: Implementation MSC:

Physiological Integrity 20. After receiving change-of-shift report on a heart failure unit, which patient should the nurse assess first? a. A patient who is cool and clammy, with new-onset confusion and restlessness b. A patient who has crackles bilaterally in the lung bases and is receiving oxygen. c. A patient who had dizziness after receiving the first dose of captopril (Capoten) d. A patient who is receiving IV nesiritide (Natrecor) and has a blood pressure of 100/62

A The patient who has "wet-cold" clinical manifestations of heart failure is perfusing inadequately and needs rapid assessment and changes in management. The other patients also should be assessed as quickly as possible but do not have indications of severe decreases in tissue perfusion. DIF: Cognitive Level: Analyze (analysis) REF: 770 OBJ: Special Questions: Prioritization; Multiple Patients TOP: Nursing Process: Assessment MSC:

Physiological Integrity 20. The nurse assesses a patient with chronic obstructive pulmonary disease (COPD) who has been admitted with increasing dyspnea over the last 3 days. Which finding is most important for the nurse to report to the health care provider? a. Respirations are 36 breaths/minute. b. Anterior-posterior chest ratio is 1:1. c. Lung expansion is decreased bilaterally. d. Hyperresonance to percussion is present.

A The increase in respiratory rate indicates respiratory distress and a need for rapid interventions such as administration of oxygen or medications. The other findings are common chronic changes occurring in patients with COPD. DIF: Cognitive Level: Apply (application) REF: 482 OBJ: Special Questions: Prioritization TOP: Nursing Process: Assessment MSC:

Physiological Integrity 18. The standard policy on the cardiac unit states, "Notify the health care provider for mean arterial pressure (MAP) less than 70 mm Hg." The nurse will need to call the health care provider about the a. postoperative patient with a BP of 116/42. b. newly admitted patient with a BP of 150/87. c. patient with left ventricular failure who has a BP of 110/70. d. patient with a myocardial infarction who has a BP of 140/86.

A The mean arterial pressure (MAP) is calculated using the formula MAP = (systolic BP + 2 diastolic BP)/3. The MAP for the postoperative patient in answer 3 is 67. The MAP in the other three patients is higher than 70 mm Hg. DIF: Cognitive Level: Apply (application) REF: 690-691 TOP: Nursing Process: Assessment MSC:

Integrated Process: Nursing Process (Implementation) 17. The nurse is teaching a client with asthma about self-management. Which statement by the nurse is best? a. "Keep a daily symptom and intervention diary." b. "Measure your anterior/posterior diameter weekly." c. "Note your symptoms when you don't take your medications." d. "Exercise before and after taking inhalers and compare tolerance."

A The nurse should tell the client to keep a daily symptom diary. This will help identify triggers and responses to therapy in asthma. Chest circumference is not expected to change in clients with asthma. The client should not be instructed to discontinue medications. Comparing exercise tolerance before and after activity will not give the client the most complete information about his or her asthma. DIF: Cognitive Level: Comprehension/Understanding REF: p. 606 TOP: Client Needs Category: Health Promotion and Maintenance (Self-Care)

Physiological Integrity 27. A patient who is being admitted to the emergency department with intermittent chest pain gives the following list of medications to the nurse. Which medication has the most immediate implications for the patient's care? a. Sildenafil (Viagra) b. Furosemide (Lasix) c. Captopril (Capoten) d. Warfarin (Coumadin)

A The nurse will need to avoid giving nitrates to the patient because nitrate administration is contraindicated in patients who are using sildenafil because of the risk of severe hypotension caused by vasodilation. The other home medications also should be documented and reported to the health care provider but do not have as immediate an impact on decisions about the patient's treatment. DIF: Cognitive Level: Apply (application) REF: 745 OBJ: Special Questions: Prioritization TOP: Nursing Process: Assessment MSC:

Physiological Integrity 38. The nurse in the emergency department receives arterial blood gas results for four recently admitted patients with obstructive pulmonary disease. Which patient will require the most rapid action by the nurse? a. 22-year-old with ABG results: pH 7.28, PaCO2 60 mm Hg, and PaO2 58 mm Hg b. 34-year-old with ABG results: pH 7.48, PaCO2 30 mm Hg, and PaO2 65 mm Hg c. 45-year-old with ABG results: pH 7.34, PaCO2 33 mm Hg, and PaO2 80 mm Hg d. 65-year-old with ABG results: pH 7.31, PaCO2 58 mm Hg, and PaO2 64 mm Hg

A The pH, PaCO2, and PaO2 indicate that the patient has severe uncompensated respiratory acidosis and hypoxemia. Rapid action will be required to prevent increasing hypoxemia and correct the acidosis. The other patients also should be assessed as quickly as possible but do not require interventions as quickly as the 22-year-old. DIF: Cognitive Level: Analyze (analysis) REF: 566 OBJ: Special Questions: Prioritization; Multiple Patients TOP: Nursing Process: Assessment MSC:

Physiological Integrity 19. Three days after experiencing a myocardial infarction (MI), a patient who is scheduled for discharge asks for assistance with hygiene activities, saying, "I am too nervous to take care of myself." Based on this information, which nursing diagnosis is appropriate? a. Ineffective coping related to anxiety b. Activity intolerance related to weakness c. Denial related to lack of acceptance of the MI d. Disturbed personal identity related to understanding of illness

A The patient data indicate that ineffective coping after the MI caused by anxiety about the impact of the MI is a concern. The other nursing diagnoses may be appropriate for some patients after an MI, but the data for this patient do not support denial, activity intolerance, or disturbed personal identity. DIF: Cognitive Level: Apply (application) REF: 757 TOP: Nursing Process: Diagnosis MSC:

The community health nurse is planning health promotion teaching targeted at preventing coronary artery disease (CAD). Which ethnic group would the nurse select as the highest priority for this intervention?

A White male

Physiological Integrity 35. A patient who is experiencing an asthma attack develops bradycardia and a decrease in wheezing. Which action should the nurse take first? a. Notify the health care provider. b. Document changes in respiratory status. c. Encourage the patient to cough and deep breathe. d. Administer IV methylprednisolone (Solu-Medrol).

A The patient's assessment indicates impending respiratory failure, and the nurse should prepare to assist with intubation and mechanical ventilation after notifying the health care provider. IV corticosteroids require several hours before having any effect on respiratory status. The patient will not be able to cough or deep breathe effectively. Documentation is not a priority at this time. DIF: Cognitive Level: Apply (application) REF: 565 OBJ: Special Questions: Prioritization TOP: Nursing Process: Implementation MSC:

Physiological Integrity 15. Which patient in the ear, nose, and throat (ENT) clinic should the nurse assess first? a. A 23-year-old who is complaining of a sore throat and has a muffled voice b. A 34-year-old who has a "scratchy throat" and a positive rapid strep antigen test c. A 55-year-old who is receiving radiation for throat cancer and has severe fatigue d. A 72-year-old with a history of a total laryngectomy whose stoma is red and inflamed

A The patient's clinical manifestation of a muffled voice suggests a possible peritonsillar abscess that could lead to an airway obstruction requiring rapid assessment and potential treatment. The other patients do not have diagnoses or symptoms that indicate any life-threatening problems. DIF: Cognitive Level: Analyze (analysis) REF: 507 OBJ: Special Questions: Prioritization; Multiple Patients TOP: Nursing Process: Assessment MSC:

Physiological Integrity 42. A patient who was admitted the previous day with pneumonia complains of a sharp pain of 7 (based on 0 to 10 scale) "whenever I take a deep breath." Which action will the nurse take next? a. Auscultate breath sounds. b. Administer the PRN morphine. c. Have the patient cough forcefully. d. Notify the patient's health care provider.

A The patient's statement indicates that pleurisy or a pleural effusion may have developed and the nurse will need to listen for a pleural friction rub and/or decreased breath sounds. Assessment should occur before administration of pain medications. The patient is unlikely to be able to cough forcefully until pain medication has been administered. The nurse will want to obtain more assessment data before calling the health care provider. DIF: Cognitive Level: Apply (application) REF: 525 OBJ: Special Questions: Prioritization TOP: Nursing Process: Assessment MSC:

Physiological Integrity 6. During a visit to a 78-year-old with chronic heart failure, the home care nurse finds that the patient has ankle edema, a 2-kg weight gain over the past 2 days, and complains of "feeling too tired to get out of bed." Based on these data, the best nursing diagnosis for the patient is a. activity intolerance related to fatigue. b. disturbed body image related to weight gain. c. impaired skin integrity related to ankle edema. d. impaired gas exchange related to dyspnea on exertion.

A The patient's statement supports the diagnosis of activity intolerance. There are no data to support the other diagnoses, although the nurse will need to assess for other patient problems. DIF: Cognitive Level: Apply (application) REF: 770 TOP: Nursing Process: Diagnosis MSC:

Psychosocial Integrity 11. The nurse completes discharge instructions for a patient with a total laryngectomy. Which statement by the patient indicates that additional instruction is needed? a. "I must keep the stoma covered with an occlusive dressing at all times." b. "I can participate in most of my prior fitness activities except swimming." c. "I should wear a Medic-Alert bracelet that identifies me as a neck breather." d. "I need to be sure that I have smoke and carbon monoxide detectors installed."

A The stoma may be covered with clothing or a loose dressing, but this is not essential. An occlusive dressing will completely block the patient's airway. The other patient comments are all accurate and indicate that the teaching has been effective. DIF: Cognitive Level: Apply (application) REF: 517 TOP: Nursing Process: Evaluation MSC:

1. Following assessment of a patient with pneumonia, the nurse identifies a nursing diagnosis of ineffective airway clearance. Which assessment data best supports this diagnosis? a. Weak, nonproductive cough effort b. Large amounts of greenish sputum c. Respiratory rate of 28 breaths/minute d. Resting pulse oximetry (SpO2) of 85%

A The weak, nonproductive cough indicates that the patient is unable to clear the airway effectively. The other data would be used to support diagnoses such as impaired gas exchange and ineffective breathing pattern. DIF: Cognitive Level: Apply (application) REF: eNCP 28-1 TOP: Nursing Process: Diagnosis MSC:

Integrated Process: Teaching/Learning 30. A client recently diagnosed with lung cancer is being taught by the nurse. What information does the nurse teach the client? a. "You will receive 6 weeks of daily radiation therapy." b. "Lung cancer has a very good prognosis." c. "Further testing is not needed because lung cancer rarely metastasizes." d. "It is very likely that surgery will be curative."

A This is the only statement that is accurate. Small doses of radiation given over long periods are an effective routine treatment. Lung cancer does not have a good prognosis, and it often metastasizes. Surgery often is only palliative. DIF: Cognitive Level: Comprehension/Understanding REF: p. 633 TOP: Client Needs Category: Health Promotion and Maintenance (Principles of Teaching/Learning)

Safe and Effective Care Environment 39. Which nursing action for a patient with chronic obstructive pulmonary disease (COPD) could the nurse delegate to experienced unlicensed assistive personnel (UAP)? a. Obtain oxygen saturation using pulse oximetry. b. Monitor for increased oxygen need with exercise. c. Teach the patient about safe use of oxygen at home. d. Adjust oxygen to keep saturation in prescribed parameters.

A UAP can obtain oxygen saturation (after being trained and evaluated in the skill). The other actions require more education and a scope of practice that licensed practical/vocational nurses (LPN/LVNs) or registered nurses (RNs) would have. DIF: Cognitive Level: Apply (application) REF: 592 OBJ: Special Questions: Delegation TOP: Nursing Process: Planning MSC:

Physiological Integrity 48. The nurse is caring for a patient who has a right-sided chest tube after a right lower lobectomy. Which nursing action can the nurse delegate to the unlicensed assistive personnel (UAP)? a. Document the amount of drainage every eight hours. b. Obtain samples of drainage for culture from the system. c. Assess patient pain level associated with the chest tube. d. Check the water-seal chamber for the correct fluid level.

A UAP education includes documentation of intake and output. The other actions are within the scope of practice and education of licensed nursing personnel. DIF: Cognitive Level: Apply (application) REF: 15 OBJ: Special Questions: Delegation TOP: Nursing Process: Planning MSC:

Physiological Integrity 8. A young adult patient who denies any history of smoking is seen in the clinic with a new diagnosis of chronic obstructive pulmonary disease (COPD). It is most appropriate for the nurse to teach the patient about a. 1-antitrypsin testing. b. use of the nicotine patch. c. continuous pulse oximetry. d. effects of leukotriene modifiers.

A When COPD occurs in young patients, especially without a smoking history, a genetic deficiency in 1-antitrypsin should be suspected. Because the patient does not smoke, a nicotine patch would not be ordered. There is no indication that the patient requires continuous pulse oximetry. Leukotriene modifiers would be used in patients with asthma, not with COPD. DIF: Cognitive Level: Apply (application) REF: 582-583 TOP: Nursing Process: Planning MSC:

Physiological Integrity 21. A few days after experiencing a myocardial infarction (MI) and successful percutaneous coronary intervention, the patient states, "I just had a little chest pain. As soon as I get out of here, I'm going for my vacation as planned." Which reply would be most appropriate for the nurse to make? a. "What do you think caused your chest pain?" b. "Where are you planning to go for your vacation?" c. "Sometimes plans need to change after a heart attack." d. "Recovery from a heart attack takes at least a few weeks."

A When the patient is experiencing denial, the nurse should assist the patient in testing reality until the patient has progressed beyond this step of the emotional adjustment to MI. Asking the patient about vacation plans reinforces the patient's plan, which is not appropriate in the immediate post-MI period. Reminding the patient in denial about the MI is likely to make the patient angry and lead to distrust of the nursing staff. DIF: Cognitive Level: Apply (application) REF: 757 TOP: Nursing Process: Implementation MSC:

Physiological Integrity 7. Which statement by the patient indicates that the teaching has been effective for a patient scheduled for radiation therapy of the larynx? a. "I will need to buy a water bottle to carry with me." b. "I should not use any lotions on my neck and throat." c. "Until the radiation is complete, I may have diarrhea." d. "Alcohol-based mouthwashes will help clean oral ulcers."

A Xerostomia can be partially alleviated by drinking fluids at frequent intervals. Radiation will damage tissues at the site being radiated but should not affect the abdominal organs, so loose stools are not a usual complication of head and neck radiation therapy. Frequent oral rinsing with non-alcohol-based rinses is recommended. Prescribed lotions and sunscreen may be used on radiated skin, although they should not be used just before the radiation therapy. DIF: Cognitive Level: Apply (application) REF: 515-516 TOP: Nursing Process: Evaluation MSC:

a

A ________ aneurysm may be surgically treated by excising only the weakened area and suturing the artery closed. A. saccular B. synthetic graft C. all D. iliac E. endovascular graft f. renal

a (Physical therapy after a rotator cuff repair begins on the first postoperative day to prevent "frozen shoulder." A shoulder immobilizer is used immediately after the surgery, but leaving the arm immobilized for several days would lead to loss of range of motion, ROM. The drop-arm test is used to test for rotator cuff injury, but not after surgery. The patient may be able to return to pitching after rehabilitation.)

A 20-year-old baseball pitcher has an arthroscopic repair of a rotator cuff injury performed in same-day surgery. When the nurse plans postoperative teaching for the patient, which information will be included? A. "You have an appointment with a physical therapist for tomorrow." B. "You can still play baseball but you will not be able to return to pitching." C. "The doctor will use the drop-arm test to determine the success of surgery." D. "Leave the shoulder immobilizer on for the first few days to minimize pain."

b (The nurse's initial response should be to assess the patient's knowledge level and need for information. Although the lifespan for patients with CF is likely to be shorter than normal, it would not be appropriate for the nurse to address this as the initial response to the patient's comments. The other responses are accurate, but the nurse should first assess the patient's understanding about the issues surrounding pregnancy.)

A 20-year-old patient with cystic fibrosis (CF) tells the nurse that she is considering having a child. Which initial response by the nurse is best? A. "Are you aware of the normal lifespan for patients with CF?" B. "Do you need any information to help you with the decision?" C. "You will need to have genetic counseling before making a decision." D. "Many women with CF do not have difficulty in conceiving children."

a

A 32-year-old patient who denies any history of smoking is seen in the clinic with a new diagnosis of emphysema. The nurse will anticipate teaching the patient about A. a1-antitrypsin testing. B. use of the nicotine patch. C. continuous pulse oximetry. D. effects of leukotriene modifiers.

b (The initial action by the nurse should be further assessment. The other three responses might be appropriate based on the information the nurse obtains with further assessment.)

A 35-year-old patient with three school-age children who has recently been diagnosed with rheumatoid arthritis (RA) tells the nurse that the inability to be involved in many family activities is causing stress at home. Which response by the nurse is most appropriate? A. "You may need to see a family therapist for some help." B. "Tell me more about the situations that are causing stress." C. "Perhaps it would be helpful for you and your family to get involved in a support group." D. "Your family may need some help to understand the impact of your rheumatoid arthritis."

b (Compression stockings are applied with the legs elevated to reduce pressure in the lower legs.Walking is recommended to prevent recurrent varicosities. Sitting and standing are both risk factors for varicose veins and venous insufficiency. An aspirin a day is not adequate to preventvenous thrombosis and would not be recommended to the patient who had just hadsclerotherapy)

A 42-year-old service-counter worker undergoes sclerotherapy for treatment of superficial varicose veins at an outpatient center. Before discharging the patient, the nurse teaches the patient that A. sitting at the work counter, rather than standing, is recommended. B. compression stockings should be applied before getting out of bed. C. exercises such as walking or jogging cause recurrence of varicosities. D. taking one aspirin daily will help prevent clotting around venous valves.

a (Absolute cessation of nicotine use is needed to reduce the risk for amputation in patients with Buerger's disease. Other therapies have limited success in treatment of this disease.)

A 46-year-old is diagnosed with thromboangiitis obliterans (Buerger's disease). When the nurse is planning expected outcomes for the patient, which outcome has the highest priority for this patient? A. Cessation of smoking B. Control of serum lipid levels C. Maintenance of appropriate weight D. Demonstration of meticulous foot care

c (Significant risk factors for peripheral artery disease include tobacco use, hyperlipidemia, elevated levels of high-sensitivity C-reactive protein, diabetes mellitus, and uncontrolled hypertension; the most important is tobacco use. Other risk factors include family history, hypertriglyceridemia, hyperuricemia, increasing age, obesity, sedentary lifestyle, and stress.)

A 50yr old woman weighs 85 kg and has a history of cigarette smoking, high blood pressure, high sodium intake, and sedentary lifestyle. When developing an individualized care plan for her, the nurse determines that the most important risk factors for peripheral artery disease (PAD) that need to be modified are: A. weight and diet B. activity level and diet C. cigarette smoking and high blood pressure D. sedentary lifestyle and high blood pressure

2 (Rationale: Outpatient drug therapy options for a healthy person with community-acquired pneumonia will be macrolides, erythromycin, or doxycycline. If the patient is allergic to macrolides, doxycycline would be prescribed.)

A 56-year-old normally healthy patient at the clinic is diagnosed with community-acquired pneumonia. Before treatment is prescribed, the nurse asks the patient about an allergy to 1. amoxicillin. 2. erythromycin. 3. sulfonamides. 4. cephalosporins.

d (Progression of osteoporosis can be slowed by increasing calcium intake and weight-bearing exercise. Estrogen replacement therapy does help prevent osteoporosis, but it is not the only treatment and is not appropriate for some patients. Corticosteroid therapy increases the risk for osteoporosis.)

A 58-year-old woman who has a family history of osteoporosis is diagnosed with osteopenia following densitometry testing. In teaching the woman about her osteoporosis, the nurse explains that A. estrogen replacement therapy must be started to prevent rapid progression to osteoporosis. B. continuous, low-dose corticosteroid treatment is effective in stopping the course of osteoporosis. C. with a family history of osteoporosis, there is no way to prevent or slow gradual bone resorption. D. calcium loss from bones can be slowed by increasing calcium intake and weight-bearing exercise.

d (A pulmonary embolism is the most common complication of hip surgery because of high vascularity and the release of fat cells from the bone marrow)

A 72 yr old male client has a total hip replacement for long-standing degenerative bone disease of the hip. When assessing this client postoperatively, the nurse considers that the most common complication of hip surgery is: A. Pneumonia B. Hemorrhage C. Wound Infection D. Pulmonary Embolism

When teaching the patients with angina about taking sublingual nitroglycerin tablets, the nurse instructs the patient to do which of the following? A) To lie still and place one tablet under the tongue when chest pain occurs B) To take the tablet with a large amount of water so it will dissolve right away C) That if one table does not relieve the pain in 15 minutes, the patient should go to the hospital D)That if the tablet causes dizziness and a headache, the medication should be stopped

A IS THE CORRECT ANSWER B) You don't want it to dissolve C)No you wait 5 minutes, not 15 minutes D) Dizziness/headache are expected- don't stop cardiac meds either

When planning emergent care for a patient with a suspected MI, what should the nurse anticipate administrating?

A Oxygen, nitroglycerin, aspirin, and morphine

When providing nutritional counseling for patients at risk for CAD, which foods would the nurse encourage patients to include in their diet (select all that apply)?

A Tofu B Walnuts C Tuna fish

c (Pursed-lip breathing prolongs exhalation and prevents bronchiolar collapse and air trapping. Diaphragmatic breathing emphasizes the use of the diaphragm to increase maximum inhalation, but it has not been shown to be helpful for patients with COPD. Thoracic breathing is not as effective as diaphragmatic breathing and is the method most naturally used by patients with COPD. Huff coughing is a technique used to increase coughing patterns to remove secretions)

A breathing technique the nurse should teach the patient with COPD to promote exhalation is: A. huff coughing B. thoracic breathing C. pursed-lip breathing D. diaphragmatic breathing

d (Treatment for repetitive strain syndrome includes changing the ergonomics of the activity. Elbow injections and surgery are not initial options for this type of injury. A wrist splint might be used for hand or wrist pain.)

A checkout clerk in a grocery store has repetitive strain syndrome in the left elbow. The nurse will plan to teach the patient about A. surgical options. B. elbow injections. C. utilization of a left wrist splint. D. modifications in arm movement.

b (Eating is an effort for the patients with COPD, and frequently these patients do not eat because of fatigue, dyspnea, and difficulty holding their breath while swallowing. Foods that require much chewing cause more exhaustion and should be avoided. A low-carb diet is indicated if the patient has hypercapnia because carbs are metabolized into carbon dioxide. Fluids should be avoided to prevent a full stomach, and cold foods give less of a sense of fullness)

A dietary modification that helps meet the nutritional needs of patients with COPD is: A. eating a high-carb, low fat diet B. avoiding foods that require a lot of chewing C. preparing most foods of the diet to be eaten hot D. drinking fluids with meals to promote digestion

b (The clinical manifestations of a ruptured abdominal aortic aneurysm include severe back pain, back or flank ecchymosis, Grey Turner's sign, and hypovolemic shock, tachycardia, hypotension, pale clammy skin, decreased urine output, altered level of consciousness, and abdominal tenderness.)

A patient is admitted to the hospital with a diagnosis of abdominal aortic aneurysm. which of the following S/S would suggest that his aneurysm has ruptured? A. sudden shortness of breath and hemoptysis B. sudden, severe low back pain and bruising along his flank C. gradually increasing substernal chest pain and diaphoresis D. sudden, patchy blue mottling on his feet and toes and rest pain

c (The patient's dry eyes are consistent with Sjögren's syndrome, a common extraarticular manifestation of RA. Symptomatic therapy such as OTC eye drops is recommended. Dry eyes are not a side effect of methotrexate. Although dry eyes are common in RA, it is more helpful to offer a suggestion to relieve these symptoms than to offer reassurance. The dry eyes are not caused by RA treatment, but by the disease itself.)

A home health patient with rheumatoid arthritis (RA) complains to the nurse about having chronically dry eyes. Which action by the nurse is most appropriate? A. Reassure the patient that dry eyes are a common problem with RA. B. Teach the patient more about adverse affects of the RA medications. C. Suggest that the patient start using over-the-counter (OTC) artificial tears. D. Ask the health care provider about lowering the methotrexate (Rheumatrex) dose.

a (Patients on anticoagulants should be taught to monitor and report any signs of bleeding, which can be a serious complication. Other important patient teaching includes reduction of vitamin K intake and routine coagulation laboratory studies if taking warfarin.)

A key aspect of teaching for the patient on anticoagulant therapy includes which instructions? A. monitor for and report any signs of bleeding B. do not take acetaminophen for a headache C. decrease your dietary intake of foods containing vitamin K D. arrange to have your blood drawn regularly to check drug levels

d (Diuretic use increases uric acid levels and can precipitate gout attacks. The other medications are safe to administer.)

A long-term care patient who takes multiple medications develops acute gouty arthritis. The nurse will consult with the health care provider before giving the prescribed dose of A. sertraline (Zoloft). B. famotidine (Pepcid). C. oxycodone (Roxycodone). D. hydrochlorothiazide (HydroDiuril).

A nurse is caring for a cient with type 1 diabetes mellitus. which client complaint would alert the nurse to the presence of a possible hypoglycemic reaction? "1. Tremors 2. Anorexia 3. Hot, dry skin 4. Muscle cramps

A nurse is caring for a cient with type 1 diabetes mellitus. which client complaint would alert the nurse to the presence of a possible hypoglycemic reaction? "1. Tremors 2. Anorexia 3. Hot, dry skin 4. Muscle cramps 1) tremorsdecreased blood glucose levels produce autonomic nervous system symptoms, which are manifested classically as nervousness, irritability, and tremors. option 3 is more likely for hyperglycemia, and options 2 and 4 are unrelated to the signs of hypoglycemia.

c ( The compression dressing/bandage supports the soft tissues, reduces edema, hastens healing, minimizes pain, and promotes residual limb shrinkage. If the dressing is left off, edema will form quickly and may delay rehabilitation. Elevation and ice will no be as effective at preventing the edema that will form. Dressing the incision with dry gauze will not provide the benefits of a compression dressing)

A patient 24 hours after a below-the-knee amputation uses the call system to tell the nurse his dressing (a compression bandage) has fallen off. What action should the nurse take? A. apply ice to the site B. cover the incision with dry gauze C. reapply the compression dressing D. elevate the extremity on a couple of pillows

a (Immediate care after a sprain or strain injury includes the application of cold and compression to the injury to minimize swelling. The other actions should be taken after the ankle is wrapped with a compression bandage and ice is applied.)

A patient arrives in the emergency department with ankle swelling and severe pain after twisting the ankle playing soccer. Which of these prescribed collaborative interventions will the nurse implement first? A. Wrap the ankle and apply an ice pack. B. Administer naproxen (Naprosyn) 500 mg PO. C. Give acetaminophen with codeine (Tylenol #3). D. Take the patient to the radiology department for x-rays.

a (Rationale: The nurse should assess for other clinical manifestations of peripheral arterial disease in a patient who describes intermittent claudication. Changes in skin color that occur in response to cold are consistent with Raynaud's phenomenon. Tortuous veins on the legs suggest venous insufficiency. Unilateral leg swelling, redness, and tenderness point to deep vein thrombosis, DVT)

A patient at the clinic says, "I have always taken an evening walk, but lately my leg cramps and hurts after just a few minutes of walking. The pain goes away after I stop walking, though." The nurse should A. attempt to palpate the dorsalis pedis and posterior tibial pulses. B. check for the presence of tortuous veins bilaterally on the legs. C. ask about any skin color changes that occur in response to cold. D. assess for unilateral swelling, redness, and tenderness of either leg.

b (Rationale: Three important factors, called Virchow's triad, in the etiology of venous thrombosis are, 1, venous stasis, 2, damage of the endothelium, inner lining of the vein, and, 3, hypercoagulability of the blood. The patient at risk for venous thrombosis usually has predisposing conditions for these three disorders, see Table 38-7. The 32-year-old woman has the highest risk: long trips without adequate exercise, venous stasis, cigarette smoking, and use of oral contraceptives (especially in women older than 35 years who smoke, the likelihood of hypercoagulability of blood is increased.)

A patient at the highest risk for venous thromboembolism, VTE, is: A. a 62 year old man with spider veins who is having arthroscopic knee surgery B. a 32 year old woman who smokes, takes oral contraceptives, and is planning a trip to Europe C. a 26 year old woman who is 3 days postpartum and received maintenance IV fluid for 12 hours during her labor D. an active 72 year old man at home recovering from transuerthral resection of the prostate for benign prostatic hyperplasia

b ( Phantom sensation or phantom pain may occur following amputation, especially if pain was present in the affected limb preoperatively. The pain is a real sensation to the patient and should be treated with analgesics and other pain interventions. As recovery and ambulation progress, phantom limb sensation usually subsides)

A patient complains of pain in the foot that was recently amputated. The nurse recognizes that the pain A. is caused by swelling at the incision B. should be treated with ordered analgesics C. will become worse with the use of a prosthesis D. can be managed with diversion because it is psychologic

b (A patient with class 3TB has clinically active disease, and airborne infection isolation is required for active diseases until the patient has been on drug therapy for at least 2 weeks or until smears are negative on different days. Cardiac monitoring and observation will need to be done with the patient in isolation. The nurse will administer the antitubercular drugs after the patient is in isolation. There should be no need for suction or extra linens after the TB patient is receiving drug therapy.)

A patient diagnosed with class 3TB one week ago is admitted to the hospital with symptoms of chest pain. Initially, the nurse gives the highest priority to: a)administering the patient's antitubercular drugs b)admitting the patient to an airborne-infection isolation room c)preparing the patient's room with suction equipment and extra linens d)placing the patient in an intensive care unit where he can be closely monitored

b (Rationale: Difficulty swallowing may occur with a thoracic aneurysm because of pressure on the esophagus. Abdominal tenderness or changes in bowel habits are consistent with an abdominal aneurysm. Dizziness or weakness may occur if there is blood loss from the aneurysm, but this aneurysm was discovered accidentally, not because the patient was symptomatic.)

A patient has a 5-cm thoracic aortic aneurysm that was discovered during a routine chest x-ray. When obtaining a nursing history from the patient, it will be most important to ask about A. back or lumbar pain. B. difficulty swallowing. C. abdominal tenderness. D. changes in bowel habits.

b (Until a plaster cast has dried, placing pressure on the cast should be avoided to prevent creating areas inside the cast that could place pressure on the arm. The left arm should be elevated to prevent swelling. The edges of the cast may be petaled once the cast is dry, but padding the edges before that may cause the cast to be misshapen. The cast should not be covered until it is dry because heat builds up during drying.)

A patient has a long-arm plaster cast applied for immobilization of a fractured left radius. Until the cast has completely dried, the nurse should A. keep the left arm in a dependent position. B. handle the cast with the palms of the hands. C. place gauze around the cast edge to pad any roughness. D. cover the cast with a small blanket to absorb the dampness.

c (Rationale: A patient with a venous ulcer should have a balanced diet with adequate protein, calories, and micronutrients; this type of diet is essential for healing. Nutrients most important for healing include protein, vitamins A and C, and zinc. Foods high in protein, e.g., meat, beans, cheese, tofu;vitamin A, green leafy vegetables; vitamin C; citrus fruits, tomatoes, cantaloupe, and zinc; meat, seafood; must be provided. For patients with diabetes mellitus, maintaining normal blood glucose levels assists the healing process. For overweight individuals and no active venous ulcer, a weight-loss diet should be considered.)

A patient has a venous ulcer related to chronic venous insufficiency. The nurse should provide education on which type of diet for this patient? A.1200-calorie-restricted diet B. High-carbohydrate diet C. High-protein diet D. Low-fat diet

b (Rationale: To prevent hip flexion contractures, patients should lie on their abdomen for 30 minutes three or four times each day and position the hip in extension while prone. Patients should avoid sitting in a chair for more than 1 hour with hips flexed or having pillows under the surgical extremity. The patient should avoid dangling the residual limb over the bedside to minimize edema.)

A patient has an amputation of the left leg below the knee. Which intervention should the nurse include in the plan of care for this patient? a. Elevate the residual limb on a pillow for 4 to 5 days after surgery. b. Lay prone with hip extended for 30 minutes four times per day. c. Dangle the residual limb for 20 to 30 minutes every 6 hours. d. Sit in a chair for 1 to 2 hours three times each day.

pain, pallor, pulselessness, parathesia, paralysis, poikilothermia

A patient has chronic atrial fib and develops an acute occlusion at the iliac artery bifurcation. What are the six P's of acute arterial occlusion the nurse may find in the patient?

b (Foot drop is an indication that the foot is not being supported in a neutral position by a splint. Using crutches and monitoring the oral temperature are appropriate self-care activities. Frustration with the length of treatment is not an indicator of ineffective health maintenance of the osteomyelitis.)

A patient has chronic osteomyelitis of the left femur, which is being managed at home with administration of IV antibiotics. The nurse chooses a nursing diagnosis of ineffective health maintenance when the nurse finds that the patient A. takes and records the oral temperature twice a day. B. is unable to plantar flex the foot on the affected side. C. uses crutches to avoid weight bearing on the affected leg. D. is irritable and frustrated with the length of treatment required.

c (Rationale: The diagnostic criteria for CF involve a combination of clinical presentation, sweat chloride testing, and genetic testing to confirm the diagnosis. The sweat chloride test is performed by placing pilocarpine on the skin and carried by a small electric current to stimulate sweat production. This takes about 5 minutes, and the patient will feel a slight tingling or warmth. The sweat is collected on filter paper or gauze and then analyzed for sweat chloride concentrations, for about 1 hour. Values above 60 mmol/L for sweat chloride are consistent with the diagnosis of CF. However, a second sweat chloride test is recommended to confirm the diagnosis unless genetic testing identifies two CF mutations. Genetic testing is used if the results from a sweat chloride test are unclear.)

A patient has clinical manifestations consistent with cystic fibrosis (CF) and is scheduled for a sweat chloride test. The nurse should include which instruction when teaching the patient about this diagnostic test? a. "The test measures the amount of sodium chloride (or salt) present in your sweat." b. "If the sweat chloride test is positive on two occasions, genetic testing will be necessary." c. "Sweat chloride greater than 60 mmol/L is consistent with a diagnosis of CF." d. "If sweating occurs after an oral dose of pilocarpine, the test for CP is positive.

b (Leaning over would flex the hip at greater than 90 degrees and predispose the patient to hip dislocation. The other patient actions are appropriate and do not require any immediate action by the nurse to protect the patient.)

A patient has hip replacement surgery using the posterior approach. Which patient action requires rapid intervention by the nurse? A. The patient uses crutches with a swing-to gait. B. The patient leans over to pull shoes and socks on. C. The patient sits straight up on the edge of the bed. D. The patient bends over the sink while brushing the teeth.

c (Resting with the head elevated and knees flexed will reduce the strain on the back and decrease muscle spasms. Twisting from side to side will increase tension on the lumbar area. A pillow placed under the upper back will cause strain on the lumbar spine. Alternate application of cold and heat should be used to decrease pain.)

A patient has muscle spasms and acute low back pain. An appropriate nursing intervention for this problem is to teach the patient to A. avoid the use of cold because it will exacerbate the muscle spasms. B. keep both feet flat on the floor when prolonged standing is required. C. keep the head elevated slightly and flex the knees when resting in bed. D. twist gently from side to side to maintain range of motion in the spine.

d (Rationale: Intermittent claudication is an ischemic muscle ache or pain that is precipitated by a consistent level of exercise, resolves within 10 minutes or less with rest, and is reproducible. Angina is the term used to describe chest pain. Paresthesia is the term used to describe numbness or tingling in the toes or feet. Reactive hyperemia is the term used to describe redness of the foot; if the limb is in a dependent position the term is dependent rubor.)

A patient has peripheral artery disease. Which symptom, if experienced by the patient, indicates to the nurse that the patient is experiencing intermittent claudication? A. Patient complains of chest pain with strenuous activity B. Patient has numbness and tingling of the toes and feet C. Patient states the feet become red if in a D. Patient reports muscle leg pain that occurs with exercise

d (The first goal of collaborative management is realignment of the knee to its original anatomic position, which will require anesthesia or monitored anesthesia care, MAC, formerly called conscious sedation. Immobilization, gentle range of motion, ROM, exercises, and discussion about activity restrictions will be implemented after the knee is realigned.)

A patient in the emergency department who is experiencing severe pain is diagnosed with a patellar dislocation. The initial patient teaching by the nurse will focus on the need for A. a knee immobilizer. B. gentle knee flexion. C. activity restrictions. D. monitored anesthesia care (conscious sedation).

a (current research indicates that statin use by patients with PAD improves multiple outcomes. There is no research that supports the use of the other medication categories in PAD.)

A patient in the outpatient clinic has a new diagnosis of peripheral artery disease (PAD). Which medication category will the nurse plan to include when providing patient teaching about PAD management? A. Statins B. Vitamins C. Thrombolytics D. Anticoagulants

b (Diminished or absent breath sounds may indicate a significant decrease in air movements resulting from exhaustion and an inability to generate enough muscle force to ventilate and is an ominous sign. The other symptoms are expected in an asthma attack)

A patient is admitted to the emergency department with an acute asthma attack. Which of the following assessments of the patient is of greatest concern to the nurse? A. The presence of a pulsus paradoxus B. Markedly diminished breath sounds with no wheezing C. Use of accessory muscles of respiration and a feeling of suffocation D. A respiratory rate of 34 breaths/min and increased pulse and blood pressure

d (The initial nursing action should be assessment of the neurovascular status of the injured leg. After assessment, the nurse may need to splint and elevate the leg, based on the assessment data. Information about tetanus immunizations should be done if there is an open wound.)

A patient is admitted to the emergency department with possible left lower leg fractures. The initial action by the nurse should be to A. elevate the left leg. B. splint the lower leg. C. obtain information about the tetanus immunization status. D. check the popliteal, dorsalis pedis, and posterior tibial pulses.

a (Because the edema associated with venous insufficiency increases when the patient has beenstanding, shoes will feel tighter at the end of the day. The other patient statements arecharacteristic of peripheral artery disease, PAD.)

A patient is admitted to the hospital with a diagnosis of chronic venous insufficiency. Which of these statements by the patient is most consistent with the diagnosis? A. "I can't get my shoes on at the end of the day." B. "I can never seem to get my feet warm enough." C. "I wake up during the night because my legs hurt." D. "I have burning leg pains after I walk three blocks."

a (The patient with pneumonococcal pneumonia is acutely ill with fever and the systemic manifestations of fever, such as chills, thirst, headache, and malaise. Interventions that monitor temperature and aid in lowering body temperature are appropriate. Ineffective airway clearance would be manifested by adventitious breath sounds and difficulty producing secretions. Disorientation and confusion are not noted in this patient and are not typical unless the patient is very hypoxemic. Pleuritic pain is an acute pain that is due to inflammation of the pleura.)

A patient is admitted to the hospital with fever, chills, a productive cough with rusty sputum, and pleuritic chest pain. Pneumonococcal pneumonia is suspected. An appropriate nursing diagnosis for the patient based on the patient's manifestations is: a)hyperthermia related to acute infectious process b)chronic pain related to ineffective pain management c)risk for injury related to disorientation and confusion d)ineffective airway clearance related to retained secretions

b (Rationale: Unfractionated heparin can be given by continuous intravenous, IV, for VTE treatment. When given IV, heparin requires frequent laboratory monitoring of clotting status as measured by activated partial thromboplastin time, aPTT.)

A patient is admitted with venous thromboembolism (VTE) and prescribed unfractionated heparin. What laboratory test should the nurse assess while the patient is receiving this medication? A. International normalized ratio (INR) B. Activated partial thromboplastin time (APTT) C. Anti-factor Xa D. Platelet count

b (The patient will be on IV antibiotics for several months, and the patient will need to recognize signs of infection at the IV site and how to care for the catheter during daily activities such as bathing. IV antibiotics rather than oral antibiotics are used for acute osteomyelitis. Patients are instructed to avoid exercise and heat application because these will increase swelling and the risk for spreading infection.)

A patient is being discharged after 2 weeks of IV antibiotic therapy for acute osteomyelitis in the left leg. Which information will be included in the discharge teaching? A. How to apply warm packs safely to the leg to reduce pain B. How to monitor and care for the long-term IV catheter site C. The need for daily aerobic exercise to help maintain muscle strength D. The reason for taking oral antibiotics for 7 to 10 days after discharge

a (Liquid oxygen reservoirs will last approximately 7-10days when used at 2L/min, and the portable units will hold about 6-8hrs of oxygen. Compressed oxygen comes in various tank sizes but generally requires weekly deliveries of four to five large tanks to meet a 7-10 day supply. Oxygen concentrators or extractors are more expensive; they continuously supply oxygen, but they must be kept out of the bedroom because they are noisy and interfere with sleep)

A patient is being discharged with plans for home oxygen therapy provided by a liquid oxygen reservoir with a refillable portable unit. In preparing the patient to use the equipment, the nurse teaches the patient that: A. the portable tank filled from the reservoir will last about 6-8hrs at 2L/min B. the unit concentrates oxygen from the air, providing continuous oxygen supply C/ the unit should be kept out of the bedroom and extension tubing used at night because of the noise D. weekly delivery of one large cylinder of oxygen will be necessary for a 7-10 day supply of oxygen

a (Immobilization of the affected leg helps decrease pain and reduce the risk for pathologic fractures. Weight-bearing exercise increases the risk for pathologic fractures. NSAIDs are frequently prescribed to treat pain. Flexion of the affected limb is avoided to prevent contractures.)

A patient is hospitalized for initiation of regional antibiotic irrigation for acute osteomyelitis of the right femur. Which intervention will be included in the plan of care? A. Immobilization of the right leg B. Frequent weight-bearing exercise C. Avoiding administration of nonsteroidal anti-inflammatory drugs (NSAIDs) D. Support of the right leg in a flexed position

a (The air entrainment ports regulate the oxygen percentage delivered to the patient, so they must be unobstructed. A high oxygen flow rate is needed when giving oxygen by partial rebreather or non-rebreather masks. The use of an adaptor can improve humidification but not oxygen delivery. Draining oxygen tubing is necessary when caring for a patient receiving mechanical ventilation.)

A patient is receiving 35% oxygen via a Venturi mask. To ensure the correct amount of oxygen delivery, it is most important that the nurse A. keep the air entrainment ports clean and unobstructed. B. give a high enough flow rate to keep the bag from collapsing. C. use an appropriate adaptor to ensure adequate oxygen delivery. D. drain moisture condensation from the oxygen tubing every hour.

b (A shorter limb after a fall indicates a possible dislocation, which is an orthopedic emergency. Bruising, pain, and decreased range of motion also should be reported, but these do not indicate that emergent treatment is needed to preserve function.)

A patient is seen at the urgent care center after falling on the right arm and shoulder. Which finding is most important for the nurse to communicate to the health care provider? A. There is bruising at the shoulder area. B. The right arm appears shorter than the left. C. There is decreased range of motion of the shoulder. D. The patient is complaining of arm and shoulder pain.

d (The patient needs to sleep in a position that prevents excessive internal rotation or flexion of the hip. The other patient statements indicate that the patient has understood the teaching.)

A patient is to be discharged from the hospital 4 days after insertion of a femoral head prosthesis using a posterior approach. A statement by the patient that indicates a need for additional discharge instructions is A. "I should not cross my legs while sitting." B. "I will use a toilet elevator on the toilet seat." C. "I will have someone else put on my shoes and socks." D. "I can sleep in any position that is comfortable for me."

c (Secondary Raynaud's phenomenon may occur in conjunction with autoimmune diseases such as rheumatoid arthritis, and patients should be screened for autoimmune disorders. Raynaud's phenomenon is not associated with hyperlipidemia, hypertension, or coronary artery disease.)

A patient tells the health care provider about experiencing cold, numb fingers when running during the winter and is diagnosed with Raynaud's phenomenon. The nurse will anticipate teaching the patient about tests for A. hypertension. B. hyperlipidemia. C. autoimmune disorders. D. coronary artery disease.

b (The nurse should monitor for hemorrhage after the surgery. The prosthesis will not be removed. To avoid flexion contracture of the hip, the leg will not be elevated on a pillow. The patient is placed in a prone position after amputation to prevent hip flexion, but this would not be done during the immediate postoperative period.)

A patient undergoes a right above-the-knee amputation with an immediate prosthetic fitting. When the patient first arrives on the orthopedic unit after surgery, the nurse should A. place the patient in a prone position. B. check the surgical site for hemorrhage. C. remove the prosthesis and wrap the site. D. keep the residual leg elevated on a pillow.

a (Skin breakdown on the residual limb can prevent the use of a prosthesis, and the limb should be inspected every day for signs of irritation or pressure areas. No substances except water and mild soap should be used on the residual limb, and ROM exercises are not necessary when the patient is using a prosthesis. A residual limb shrinker is an elastic stocking that is used to mold the limb in preparation for prosthesis use, but a cotton residual preparation for prosthesis use, but a cotton residual limb sock is worn with the prosthesis)

A patient who had a below-the-knee amputation is to be fitted with a temporary prosthesis. It is most important for the nurse to teach the patient to: A. inspect the residual limb daily for irritation B. apply elastic shrinker before applying the prosthesis C. perform ROM exercises to the affected leg four times a day D. apply alcohol to the residual limb every morning and evening to toughen the skin

b (Bone healing starts immediately after the injury, but since ossification does not begin until 3 weeks postinjury, the cast will need to be worn for at least 3 weeks. Complete union may take up to a year. Resolution of swelling does not indicate bone healing.)

A patient who has a cast in place after fracturing the radius asks when the cast can be removed. The nurse will instruct the patient that the cast will need to remain in place A. for several months. B. for at least 3 weeks. C. until swelling of the wrist has resolved. D. until x-rays show complete bony union.

c (The axilla can become excoriated when a sling is used to support the arm, and the nurse should check the axilla and apply absorbent dressings to prevent this. A patient with a sling would not have traction applied by hanging. The patient will be encouraged to move the fingers on the injured arm to maintain function and to help decrease swelling. The patient will do active ROM on the uninjured side.)

A patient who has a proximal humerus fracture that is immobilized with a left-sided long-arm cast and a sling is admitted to the medical-surgical unit. Which nursing intervention will be included in the plan of care? A. Use surgical net dressing to hang the arm from an IV pole. B. Immobilize the fingers on the left hand with gauze dressings. C. Assess the left axilla and change absorbent dressings as needed. D. Assist the patient in passive range of motion (ROM) for the right arm

b (The patient's clinical manifestations and history are consistent with a pulmonary embolus, and the nurse's first action should be to ensure adequate oxygenation. The nurse should offer reassurance to the patient, but meeting the physiologic need for oxygen is a higher priority. The health care provider should be notified after the oxygen is started and pulse oximetry and assessment for fat embolus or venous thromboembolism, VTE, are obtained.)

A patient who has been hospitalized for 3 days with a hip fracture has sudden onset shortness of breath and tachypnea. The patient tells the nurse, "I feel like I am going to die!" Which action should the nurse take first? A. Stay with the patient and offer reassurance. B. Administer the prescribed PRN oxygen at 4 L/min. C. Check the patient's legs for swelling or tenderness. D. Notify the health care provider about the symptoms.

b (The patient should avoid sitting for long periods because of the increased stress on the suture linecaused by leg edema and because of the risk for venous thromboembolism (VTE). The otheractions by the LPN/LVN are appropriate)

A patient who has had a femoral-popliteal bypass graft to the right leg is being cared for on the surgical unit. Which action by an LPN/LVN caring for the patient requires the RN to intervene? A. The LPN/LVN places the patient in a Fowler's position for meals. B. The LPN/LVN has the patient sit in a bedside chair for 90 minutes. C. The LPN/LVN assists the patient to ambulate 40 feet in the hallway. D. The LPN/LVN administers the ordered aspirin 160 mg after breakfast.

a (The patient's clinical manifestations suggest compartment syndrome and delay in diagnosis and treatment may lead to severe functional impairment. The data do not suggest problems with blood pressure or infection. Elevation of the leg will decrease arterial flow and further reduce perfusion.)

A patient who has had an open reduction and internal fixation (ORIF) of left lower leg fractures complains of constant severe pain in the leg, which is unrelieved by the prescribed morphine. Pulses are faintly palpable and the foot is cool. Which action should the nurse take next? A. Notify the health care provider. B. Assess the incision for redness. C. Reposition the left leg on pillows. D. Check the patient's blood pressure.

c (Rheumatoid nodules can break down or become infected. They are not associated with changes in rheumatoid factor and injection is not needed. Rheumatoid nodules are usually not removed surgically because of a high probability of recurrence.)

A patient who has rheumatoid arthritis is seen in the outpatient clinic and the nurse notes that rheumatoid nodules are present on the patient's elbows. Which action will the nurse take? A. Draw blood for rheumatoid factor analysis. B. Teach the patient about injection of the nodule. C. Assess the nodules for skin breakdown or infection. D. Discuss the need for surgical removal of the nodule.

a (Assessment of the patient's breath sounds will help determine how effectively the patient is ventilating and whether rapid intubation may be necessary. The length of time the attack has persisted is not as important as determining the patient's status at present. Most patients having an acute attack will be unable to cooperate with an FEV measurement. It is important to know about the medications the patient is using but not as important as assessing the breath sounds.)

A patient who is experiencing an acute asthma attack is admitted to the emergency department. The nurse's first action should be to A. listen to the patient's breath sounds. B. ask about inhaled corticosteroid use. C. determine when the dyspnea started. D. obtain the forced expiratory volume (FEV) flow rate.

d (The color of the mucus and the patient's history of CF suggest Pseudomonas infection; TOBI is the standard of care for treatment of Pseudomonas in patients with CF. Oral corticosteroids and inhaled bronchodilators will not be effective in treating the respiratory infection; the effectiveness of bronchodilators has not been established for CF. Pseudomonas infections are usually responsive, not resistant, to TOBI.)

A patient who is hospitalized with cystic fibrosis (CF) coughs up large quantities of thick green mucus. The nurse will plan to teach the patient about A. antibiotic resistance. B. inhaled bronchodilators. C. oral corticosteroid therapy. D. aerosolized tobramycin (TOBI).

b (The disruption in body image caused by an amputation often causes a patient to go through psychological stages of grieving, and the patient should be allowed to go through a period of depression as a normal consequence of the amputation. The grieving process is not ineffective coping or impaired adjustment but a normal process of adjusting to loss)

A patient who suffered a traumatic below-the-elbow amputation in a boating accident is withdrawn, does not look at the arm, and asks to be left alone. An appropriate nursing diagnosis of the patient includes: A. impaired adjustment B. disturbed body image C. impaired social interaction D. ineffective individual coping

a (Exercises can help to strengthen the muscles that support the back. Flexion of the hips and knees places less strain on the back. Modifications in the way the patient lifts boxes are needed, but sitting for prolonged periods can aggravate back pain. The patient should not lift above the level of the elbows.)

A patient whose work involves lifting has a history of chronic back pain. After the nurse has taught the patient about correct body mechanics, which patient statement indicates that the teaching has been effective? A. "I plan to start doing exercises to strengthen the muscles of my back." B. "I will try to sleep with my hips and knees extended to prevent back strain." C. "I can tell my boss that I need to change to a job where I can work at a desk." D. "I will keep my back straight when I need to lift anything higher than my waist."

a, b, e (Warm legs and feet increase circulation. The lower extremities should be assessed at a regular interval for changes. Walking exercise increases oxygen extraction in the legs and improves skeletal muscle metabolism. The patient with PAD should walk at least 30min a day, preferably twice a day. Exercise should be stopped when pain occurs and resumed when pain subsides. Nicotine in all forms causes vasoconstriction and must be eliminated.)

A patient with PAD has a nursing diagnosis of ineffective peripheral tissue perfusion. Appropriate teaching for the patient includes instructions to (select all that apply). a) keep legs and feet warm b) walk at least 30 min/day to the point of discomfort c) apply cold compresses when the legs become swollen d) use nicotine replacement therapy as a substitute for smoking e) inspect lower extremities for pulses, temperature, and any swelling

b (Raynaud's phenomenon is an episodic vasospastic disorder of small cutaneous arteries, most frequently involving the fingers and toes. Diltiazem, Cardizem, is a calcium channel blocker that will relax smooth muscles of the arterioles by blocking the influx of calcium into the cells, thus reducing the frequency and severity of vasospastic attacks. There will be improved perfusion to the fingertips and a reduction of the vasospastic attacks.)

A patient with Raynaud's phenomenon is prescribed diltiazem (Cardizem). To evaluate the patient's response to this medication, what is most important for the nurse to assess in this patient? A. Increased prothrombin time (PT) B. Improved perfusion to distal fingers C. Increased mean arterial pressure D. Increased capillary refill time

d (exercise programs for patients recovering from venous thromboembolism, VTE, should emphasize swimming, which is particularly beneficial because of the gentle, even pressure of the water. Coumadin will not blacken stools, if this occurs, it could be a sign of gastrointestinal bleeding. Dark green and leafy vegetables have high amounts of vitamin K and should not be increased during Coumadin therapy, but they do not need to be restricted. The legs must not be massaged because of the risk of dislodging any clots that may be present)

A patient with VTE is to be discharged on long-term warfarin therapy and is taught about prevention and continuing treatment of VTE. The nurse determines that discharge teaching for the patient has been effective when the patient states... A. "I should expect the Coumadin will cause my stools to be somewhat black." B. "I should avoid all dark green and leafy vegetables while I am taking Coumadin." C. "Massaging my legs several times a day will help increase my venous circulation." D. "Swimming is a good activity to include in my exercise program to increase my circulation."

d (The patient can lift the buttocks off the bed by using the left leg without changing the right-leg alignment. Turning the patient will tend to move the leg out of alignment. Disconnecting the traction will interrupt the weight needed to immobilize and align the fracture.)

A patient with a comminuted fracture of the right femur has Buck's traction in place while waiting for surgery. To assess for pressure areas on the patient's back and sacral area and to provide skin care, the nurse should A. loosen the traction and have the patient turn onto the unaffected side. B. place a pillow between the patient's legs and turn gently to each side. C. turn the patient partially to each side with the assistance of another nurse. D. have the patient lift the buttocks by bending and pushing with the left leg.

c (Airway clearance devices assist with moving mucus into larger airways where it can more easily be expectorated. The other actions may be appropriate for some patients with COPD, but they are not indicated for this patient's problem of thick mucous secretions.)

A patient with chronic obstructive pulmonary disease (COPD) has rhonchi throughout the lung fields and a chronic, nonproductive cough. Which nursing action will be most effective? A. Change the oxygen flow rate to the highest prescribed rate. B. Reinforce the ongoing use of pursed lip breathing techniques. C. Educate the patient to use the Flutter airway clearance device. D. Teach the patient about consistent use of inhaled corticosteroids.

a (Although most initial treatment for aortic dissection involves a period of lowering the BP and myocardial contractility to diminish the pulsatile forces in the aorta, immediate surgery is indicated when complications, such as occlusion of the carotid arteries, occur. Anticoagulants would prolong and intensify the bleeding, and blood is administered only if the dissection ruptures)

A patient with a dissection of the arch of the aorta has a decreased LOC and weak carotid pulses. The nurse anticipates that initial treatment of the patient will include... A. immediate surgery to replace the torn area with a graft B. administration of anticoagulants to prevent embolization C. administration of packed red blood cells (RBCs) to replace blood loss D. administration of antihypertensives to maintain a mean arterial pressure of 70-80 mmHG

c (The spine should be kept in correct alignment after laminectomy. The other positions will create misalignment of the spine.)

A patient with a herniated intravertebral disk undergoes a laminectomy and discectomy. Following the surgery, the nurse should position the patient on the side by A. instructing the patient to move the legs before turning the rest of the body. B. having the patient turn by grasping the side rails and pulling the shoulders over. C. placing a pillow between the patient's legs and turning the entire body as a unit. D. turning the patient's head and shoulders first, followed by the hips, legs, and feet.

a (Since the patient appears to be experiencing aortic dissection, the nurse's first action should be to determine the hemodynamic status by assessing blood pressure. The other actions also may be done, but they will not provide information that will determine what interventions are needed immediately for this patient.)

A patient with a history of an abdominal aortic aneurysm is admitted to the emergency department (ED) with severe back pain and absent pedal pulses. Which action should the nurse take first? A. Obtain the blood pressure. B. Ask the patient about tobacco use. C. Draw blood for ordered laboratory testing. D. Assess for the presence of an abdominal bruit.

b (Control hypertension with prescribed therapy, because Increased systolic BP continually puts pressure on the diseased area of the artery, promoting its expansion. Small aneurysms can be treated by decreasing BP, modifying atherosclerosis risk factors, and monitoring the size of the aneurysm.)

A patient with a small AAA is not a good surgical candidate. The nurse teaches the patient that one of the best ways to prevent expansion of the lesion is to A. Avoid strenous physical activity B. Control hypertension with prescribed therapy C. Comply with prescribed anticoagulant therapy D. Maintain a low-calcium diet to prevent calcification fo the vessel

c (Low molecular weight heparin, LMWH, is used because of the immediate effect on coagulationand discontinued once the international normalized ratio, INR, value indicates that the warfarin has reached a therapeutic level. LMWH has no thrombolytic properties. The use of two anticoagulants is not related to the risk for pulmonary embolism, and two are not necessary to reduce the risk for another VTE.)

A patient with a venous thromboembolism (VTE) is started on enoxaparin (Lovenox) and warfarin (Coumadin). The patient asks the nurse why two medications are necessary. Which response by the nurse is accurate? A. "Administration of two anticoagulants reduces the risk for recurrent venous thrombosis." B. "Lovenox will start to dissolve the clot, and Coumadin will prevent any more clots from occurring." C. "The Lovenox will work immediately, but the Coumadin takes several days to have an effect on coagulation." D. "Because of the potential for a pulmonary embolism, it is important for you to have more than one anticoagulant."

d (The nurse should notify the public health department if drug compliance is questionable so that follow-up of patients can be made by directly observed treatment, DOT, by a public health nurse or a responsible family member. A patient who cannot remember to take the medication usually will not remember to come to the clinic daily or will find it too inconvenient. Additional teaching, or support from others, is not usually effective for this type of patient.)

A patient with active TB continues to have positive sputum cultures after 6 months of treatment because she says she cannot remember to take the medication all the time. The best action by the nurse is to: a)schedule the patient to come to the clinic every day to take the medication b)have a patient who has recovered from TB tell the patient about his successful treatment c)schedule more teaching sessions so the patient will understand the risks of noncompliance d)arrange for directly observed therapy by a responsible family member or a public health nurse

d (Aspirin interferes with the effectiveness of probenecid and should not be taken when the patient is taking probenecid. The patient's sleep pattern will not affect gout management. Drinking 3 quarts of water and eating beef only once or twice a week are appropriate for the patient with gout.)

A patient with an acute attack of gout in the left great toe has a new prescription for probenecid (Benemid). Which information about the patient's home routine indicates a need for teaching regarding gout management? A. The patient sleeps about 8 to 10 hours every night. B. The patient usually eats beef once or twice a week. C. The patient generally drinks about 3 quarts of juice and water daily. D. The patient takes one aspirin a day prophylactically to prevent angina.

a (Colchicine produces pain relief in 24 to 48 hours by decreasing inflammation. The recommended increase in fluid intake of 2 to 3 L/day would increase urine output but would not indicate the effectiveness of colchicine. Elevated uric acid levels would result in increased symptoms. The WBC count might decrease with decreased inflammation, but this would not be as useful in determining the effectiveness of colchicine as a decrease in pain.)

A patient with an acute attack of gout is treated with colchicine. The nurse determines that the drug is effective upon finding A. relief of joint pain. B. increased urine output. C. elevated serum uric acid. D. decreased white blood cells (WBC).

a (Hyperglycemia is a side effect of prednisone. Corticosteroids increase appetite and lead to weight gain. An elevated ESR and no improvement in symptoms would indicate that the prednisone was not effective but would not be side effects of the medication.)

A patient with an exacerbation of rheumatoid arthritis (RA) is taking prednisone (Deltasone) 40 mg daily. Which of these assessment data obtained by the nurse indicate that the patient is experiencing a side effect of the medication? A. The patient's blood glucose is 165 mg/dL. B. The patient has no improvement in symptoms. C. The patient has experienced a recent 5-pound weight loss. D. The patient's erythrocyte sedimentation rate (ESR) has increased.

A nurse is caring for a diabetic patient with a diagnosis of nephropathy. What would the nurse expect the urinalysis report to indicate? a) White blood cells b) Bacteria c) Red blood cells d) Albumin

A) Albumin

c (Early in an asthma attack an increased respiratory rate and hyperventilation creates a respiratory alkalosis with increase pH and decrease PaCO2, accompanied by a hypoxemia. As the attack progresses, pH shifts to normal, then decreases with ABGs that reflect respiratory acidosis with hypoxemia. During the attack, high-flow oxygen should be provided; breathing in a paper bag, although used to treat some types of hyperventilation, would increase the hypoxemia)

A patient with asthma has the following arterial blood gas (ABG) results early in an acute asthma attack: pH 7.48, PaCO2 30mm Hg, PaO2 78 mm Hg. The most appropriate action by the nurse is to: A. Prepare the patient for mechanical ventilation B. Have the patient breath in a paper bag to raise the PaCo2 C. Document the findings and monitor the ABGs for a trend toward acidosis D. Reduce the patient's oxygen flow rate to keep the PaO2 at the current level

b (The patient's peak flow is 70% of normal, indicating a need for immediate use of short-acting b2-adrenergic SABA medications. Assessing for correct use of medications or exposure to allergens also is appropriate, but would not address the current decrease in peak flow. Because the patient is currently in the yellow zone, hospitalization is not needed.)

A patient with asthma who has a baseline peak flow reading of 600 mL calls the nurse, stating that the current peak flow is 420 mL. Which action should the nurse take first? A. Tell the patient to go to the hospital emergency department. B. Instruct the patient to use the prescribed albuterol (Proventil). C. Ask about recent exposure to any new allergens or asthma triggers. D. Question the patient about use of the prescribed inhaled corticosteroids.

a (Coughing is less painful and more likely to be effective when the patient splints the chest during coughing. Fluids should be encouraged to help liquefy secretions. Nasal oxygen will improve gas exchange, but will not improve airway clearance. Pursed lip breathing is used to improve gas exchange in patients with COPD, but will not improve airway clearance.)

A patient with bacterial pneumonia has rhonchi and thick sputum. Which action will the nurse use to promote airway clearance? A. Assist the patient to splint the chest when coughing. B. Educate the patient about the need for fluid restrictions. C. Encourage the patient to wear the nasal oxygen cannula. D. Instruct the patient on the pursed lip breathing technique.

a (Rationale: Adequate hydration helps to liquefy secretions and thus make it easier to remove them. Unless there are contraindications, the nurse should instruct the patient to drink at least 3 liters of fluid daily. Although nutrition, breathing exercises, and medications may be indicated, these interventions will not liquefy or thin secretions.)

A patient with bronchiectasis has copious thick respiratory secretions. Which intervention should the nurse add to the plan of care for this patient? a. Increase intake to 12 to 13 eight ounce glasses of fluids every 24 hours. b. Provide nutritional supplements that are high in protein and carbohydrates. c. Use the incentive spirometer and practice deep breathing exercises every 2 hours. d. Administer prescribed antibiotics and antitussives on a scheduled basis.

c

A patient with chronic arterial disease has a brachial SBP of 132 mm Hg and an ankle SBP of 102 mm Hg. The ankle-brachial index is ______ and indicates _______ (mild/moderate/severe) arterial disease. A. intermittent claudication B. non-healing ischemic ulcers and gangrene C. 0.77; mild D. rest

d (The patient's history and clinical manifestations are consistent with acute arterial occlusion, and resting the leg will decrease the oxygen demand of the tissues and minimize ischemic damage until circulation can be restored. Elevating the leg or applying an elastic wrap will further compromise blood flow to the leg. Exercise will increase oxygen demand for the tissues of the leg.)

A patient with chronic atrial fibrillation develops sudden severe pain, pulselessness, pallor, and coolness in the left leg. The nurse should notify the health care provider and A. elevate the left leg on a pillow. B. apply an elastic wrap to the leg. C. assist the patient in gently exercising the leg. D. keep the patient in bed in the supine position.

d (Pursed lip breathing techniques assist in prolonging the expiratory phase of respiration and decrease air trapping. There is no indication that the patient requires oxygen therapy or an improved diet. Sedative medications should be avoided because they decrease respiratory drive.)

A patient with chronic bronchitis has a nursing diagnosis of impaired breathing pattern related to anxiety. Which nursing action is most appropriate to include in the plan of care? A. Titrate oxygen to keep saturation at least 90%. B. Discuss a high-protein, high-calorie diet with the patient. C. Suggest the use of over-the-counter sedative medications. D. Teach the patient how to effectively use pursed lip breathing.

a (Salmeterol is a long-acting bronchodilator, and fluticasone is a corticosteroid. They work together to prevent asthma attacks. Neither medication is an antihistamine. Advair is not used during an acute attack because the medications do not work rapidly.)

A patient with chronic bronchitis who has a new prescription for Advair Diskus (combined fluticasone and salmeterol) asks the nurse the purpose of using two drugs. The nurse explains that A. one drug decreases inflammation, and the other is a bronchodilator. B. Advair is a combination of long-acting and slow-acting bronchodilators. C. the combination of two drugs works more quickly in an acute asthma attack. D. the two drugs work together to block the effects of histamine on the bronchioles.

d (Eating small amounts more frequently, as occurs with snacking, will increase caloric intake by decreasing the fatigue and feelings of fullness associated with large meals. Patients with COPD should rest before meals. Foods that have a lot of texture may take more energy to eat and lead to decreased intake. Although fruits and juices are not contraindicated, foods high in protein are a better choice.)

A patient with chronic obstructive pulmonary disease (COPD) has a nursing diagnosis of imbalanced nutrition: less than body requirements. An appropriate intervention for this problem is to A. increase the patient's intake of fruits and fruit juices. B. have the patient exercise for 10 minutes before meals. C. assist the patient in choosing foods with a lot of texture. D. offer high calorie snacks between meals and at bedtime.

b (Patients with COPD improve the mechanics of breathing by sitting up in the "tripod" position. Resting in bed with the head elevated would be an alternative position if the patient was confined to bed, but sitting in a chair allows better ventilation. The Trendelenburg position or sitting upright in bed with the knees flexed would decrease the patient's ability to ventilate well.)

A patient with chronic obstructive pulmonary disease (COPD) is admitted to the hospital. How can the nurse best position the patient to improve gas exchange? A. Resting in bed with the head elevated to 45 to 60 degrees B. Sitting up at the bedside in a chair and leaning slightly forward C. Resting in bed in a high-Fowler's position with the knees flexed D. In the Trendelenburg position with several pillows behind the head

b (The glucose levels indicate that the patient has developed CF-related diabetes; insulin therapy will be required. Since the etiology of diabetes in CF is inadequate insulin production, oral hypoglycemic agents are not effective. Patients with CF need a high-calorie diet. Inappropriate use of pancreatic enzymes would not be a cause of hyperglycemia in a patient with CF.)

A patient with cystic fibrosis (CF) has blood glucose levels that are consistently 200 to 250 mg/dL. Which nursing action will the nurse plan to implement? A. Discuss the role of diet in blood glucose control. B. Educate the patient about administration of insulin. C. Give oral hypoglycemic medications before meals. D. Evaluate the patient's home use of pancreatic enzymes.

b (Losartan, an angiotensin II receptor antagonist, will lower blood pressure. It does not affect blood glucose, red blood cell count, RBC, or lymphocytes.)

A patient with gout tells the nurse that he takes losartan (Cozaar) for control of the condition. The nurse will plan to monitor A. blood glucose. B. blood pressure. C. erythrocyte count. D. lymphocyte count.

c (Rationale: The patient has potentially developed acute arterial ischemia, sudden interruption in the arterial blood supply to the extremity, caused by an embolism from a cardiac thrombus that occurred as a complication of infective endocarditis. Clinical manifestations of acute arterial ischemia include pain, pallor, paralysis, pulselessness, paresthesia, and poikilothermia. Without immediate intervention, ischemia may progress quickly to tissue necrosis and gangrene within a few hours. If the nurse detects these signs, the physician should be notified immediately.)

A patient with infective endocarditis develops sudden sudden left leg pain with pallor, paresthesia, and a loss of peripheral pulses. The nurse's initial action should be to: A. elevate the leg to promote venous return B. start anticoagulant therapy with IV heparin C. notify the physician of the change in peripheral perfusion D. place the bed in reverse Trandelelbrug to promote perfusion

a (Pin insertion sites should be cleaned daily to decrease the risk for infection at the site. An external fixator allows the patient to be out of bed and avoid the risks of prolonged immobility. The device is surgically placed and is not removed until the bone is stable. Prophylactic antibiotics are not routinely given when an external fixator is used.)

A patient with lower leg fracture has an external fixation device in place and is scheduled for discharge. Which information will the nurse include in the discharge teaching? A. "You will need to assess and clean the pin insertion sites daily." B. "The external fixator can be removed during the bath or shower." C. "You will need to remain on bed rest until bone healing is complete." D. "Prophylactic antibiotics are used until the external fixator is removed."

1 (Rationale: Peripheral neuropathy is caused by diminished perfusion to neurons and results in loss of both pressure and deep pain sensations. The patient may not notice lower extremity injuries. Neuropathy increases susceptibility to traumatic injury and results in delay in seeking treatment.)

A patient with peripheral vascular disease has marked peripheral neuropathy. An appropriate nursing diagnosis for the patient is 1. risk for injury related to decreased sensation. 2. impaired skin integrity related to decreased peripheral circulation. 3. ineffective peripheral tissue perfusion related to decreased arterial blood flow. 4. activity intolerance related to imbalance between oxygen supply and demand.

a (Secretions are liquefied and more easily removed by coughing when fluid intake is at least 3L/d. positioning and oxygen administration may help ineffective breathing patterns and impaired oxygen exchange but are not indicated for retained secretions. Deep breaths are necessary to move mucus from distal airways.)

A patient with pneumonia has a nursing diagnosis of ineffective airway clearance related to pain, fatigue, and thick secretions. An appropriate nursing intervention for the patient is to: a)encourage a fluid intake of at least 3L/day b)administer oxygen as prescribed to maintain SpO₂ of 95% c)place the patient in semi-Fowler's position to maximize lung expansion d)teach the patient to take three or four shallow breaths before coughing to minimize pain

d (The patient's statement about not being able to do anything for himself or herself supports this diagnosis. Although deficient knowledge, complicated grieving, and ineffective coping also may be appropriate diagnoses for patients with COPD, the data for this patient do not support these diagnoses.)

A patient with severe chronic obstructive pulmonary disease (COPD) tells the nurse, "I wish I were dead! I cannot do anything for myself anymore." Based on this information, which nursing diagnosis is most appropriate? A. Complicated grieving related to expectation of death B. Ineffective coping related to unknown outcome of illness C. Deficient knowledge related to lack of education about COPD D. Chronic low self-esteem related to increased physical dependence

d (The patient's statement about not going anywhere because of hating the way he or she looks supports the diagnosis of social isolation because of embarrassment about the effects of the SLE. Activity intolerance is a possible problem for patients with SLE, but the information about this patient does not support this as a diagnosis. The rash with SLE is nonpruritic. There is no evidence of lack of social skills for this patient.)

A patient with systemic lupus erythematosus (SLE) who has a facial rash and alopecia tells the nurse, "I hate the way I look! I never go anywhere except here to the health clinic." An appropriate nursing diagnosis for the patient is A. activity intolerance related to fatigue and inactivity. B. impaired social interaction related to lack of social skills. C. impaired skin integrity related to itching and skin sloughing. D. social isolation related to embarrassment about the effects of

a (The goal of hand surgery in RA is to restore function, not to correct for cosmetic deformity or treat the underlying process. Hand exercises will be prescribed after the surgery.)

A patient with ulnar drift caused by rheumatoid arthritis (RA) is scheduled for an arthroplasty of the hand. Which patient statement to the nurse indicates realistic expectation for the surgery? A. "I will be able to use my fingers to grasp objects better." B. "I will not have to do as many hand exercises after the surgery." C. "This procedure will prevent further deformity in my hands and fingers." D. "My fingers will appear more normal in size and shape after this surgery."

b

A pouch-like bulge of an artery? A. Pseudoaneurysm B. Saccular Aneurysm C. Fusiform aneurysm

A 52-year-old male patient has received a bolus dose and an infusion of alteplase (Activase) for an ST-segment elevation myocardial infarction (STEMI). To determine the effectiveness of this medication, the nurse should assess the patient for the

A presence of chest pain.

a (The nurse is considered to have a latent TB infection and should be treated with INH daily for 6 to 9 months. The four-drug therapy would be appropriate if the nurse had active TB. TB skin testing is not done for individuals who have already had a positive skin test. BCG vaccine is not used in the United States and would not be helpful for this individual, who already has a TB infection.)

A staff nurse has a tuberculosis (TB) skin test of 16-mm induration. A chest radiograph is negative, and the nurse has no symptoms of TB. The occupational health nurse will plan on teaching the staff nurse about the A. use and side effects of isoniazid (INH). B. standard four-drug therapy for TB. C. need for annual repeat TB skin testing. D. bacille Calmette-Guérin (BCG) vaccine.

C (the presence of a bruit in the periumbilical area, although most abdominal aortic aneurysms, AAA, are asymptomatic, on physical exam a pulsatile mass in the periumbilical area slightly to the left of the midline may be detected, and bruits may be audible with a stethoscope placed over the aneurysm. Hoarseness and dysphagia may occur with aneurysms of the ascending aorta and the aortic arch. Severe back pain with flank ecchymosis is usually present on rupture of an AAA, and neurologic loss in the lower extremities may occur from the pressure of a thoracic aneurysm.)

A surgical repair is planned for a patient who has a 5-cm abdominal aortic aneurysm (AAA). On physical assessment of the patient, the nurse would expect to find... A) hoarseness and dysphagia B) sever back pain with flank ecchymosis C) the presence of a bruit in the periumbilical area D) weakness in the lower extremities progressing to paraplegia

d

A synthetic bifurcation graft is used in aneurysm repair when an AAA extends into the ________ arteries. A. saccular B. synthetic graft C. all D. iliac E. endovascular graft f. renal

d (CT scan, The most accurate test to determine the diameter of the aneurysm and whether a thrombus is present. The other tests may also be used, but the CT yields the most descriptive results.)

A thoracic aortic aneurysm is found when a patient has a routine chest radiograph. The nurse anticipates that additional diagnostic testing to determine the size and structure of the aneurysm will include... A) angiography B) Ultrasonography C) echocardiography D) CT scan

When assessing the patient experiencing the onset of symptoms of type 1 diabetes, which question should the nurse ask? a. ""Have you lost any weight lately?"" b. ""Do you crave fluids containing sugar?"" c. ""How long have you felt anorexic?"" d. ""Is your urine unusually dark-colored?"

A) CORRECT lost any weight?"a. Weight loss occurs because the body is no longer able to absorb glucose and starts to break down protein and fat for energy. b. The patient is thirsty but does not necessarily crave sugar- containing fluids. c. Increased appetite is a classic symptom of type 1 diabetes. d. With the classic symptom of polyuria, urine will be very dilute.

For a patient with suspected pneumonia. which is the highest priority action? A) Obtain baseline vital signs and oxygen saturation B) Obtain a sputum culture C) Obtain a complete history of the client D) Ascertain if the patient received a pnemovas this year

A) Obtain baseline vital signs and oxygen saturation

The client diagnosed with Type I diabetes is found lying unconscious on the floor of the bathroom. Which interventions should the nurse implement first? A. Administer 50% dextrose IVP. B. Notify the health-care provider. C. Move the client to ICD. D. Check the serum glucose level.

A) admin 50% dextrose IVPThe nurse should assume the client is hypoglycemic and administer IVP dextrose, which will rouse the client immediately. If the collapse is the result of hyperglycemia, this additional dextrose will not further injure the client.

The nurse is encouraging a sedentary patient with major risk for CAD to perform physical exercise on a regular basis. In addition to decreasing the risk factor of physical inactivity, the nurse tells the pt that exercise will also directly contribute to reducing which risk factors? A) hyperlipidemia and obesity B) DM and hypertension C) Elevated serum lipids and stressful lifestyle D) Hypertension and elevated serum homocystine

A) hyperlipidemia and obesity

Integrated Process: Nursing Process (Assessment) 2. What information about nutrition does the nurse teach a client with chronic obstructive pulmonary disease (COPD)? (Select all that apply.) a. "Avoid drinking fluids just before and during meals." b. "Rest before meals if you have dyspnea." c. "Have about six small meals a day." d. "Practice diaphragmatic breathing against resistance four times daily." e. "Eat high-fiber foods to promote gastric emptying." f. "Eat dry foods rather than wet foods, which are heavier." g. "Increase carbohydrate intake for energy."

A, B, C Fluids can make a client feel bloated and should be avoided with meals. Resting before the meal will help a client with dyspnea. Six small meals a day also will help to decrease bloating. Dry foods can cause coughing. Fibrous foods can produce gas, which can cause abdominal bloating and can increase shortness of breath. Diaphragmatic breathing will not necessarily help nutrition. DIF: Cognitive Level: Application/Applying or higher REF: N/A TOP: Client Needs Category: Health Promotion and Maintenance (Self-Care)

Safe and Effective Care Environment MULTIPLE RESPONSE 1. Which factors will the nurse consider when calculating the CURB-65 score for a patient with pneumonia (select all that apply)? a. Age b. Blood pressure c. Respiratory rate d. Oxygen saturation e. Presence of confusion f. Blood urea nitrogen (BUN) level

A, B, C, E, F Data collected for the CURB-65 are mental status (confusion), BUN (elevated), blood pressure (decreased), respiratory rate (increased), and age (65 and older). The other information is also essential to assess, but are not used for CURB-65 scoring. DIF: Cognitive Level: Apply (application) REF: 523 TOP: Nursing Process: Assessment MSC:

30. A nurse educator is reviewing the indications for chest drainage systems with a group of medical nurses. What indications should the nurses identify? Select all that apply. A) Post thoracotomy B) Spontaneous pneumothorax C) Need for postural drainage D) Chest trauma resulting in pneumothorax E) Pleurisy

A, B, D Feedback: Chest drainage systems are used in treatment of spontaneous pneumothorax and trauma resulting in pneumothorax. Postural drainage and pleurisy are not criteria for use of a chest drainage system.

Physiological Integrity 2. The nurse is reviewing the medical records for five patients who are scheduled for their yearly physical examinations in September. Which patients should receive the inactivated influenza vaccination (select all that apply)? a. A 76-year-old nursing home resident b. A 36-year-old female patient who is pregnant c. A 42-year-old patient who has a 15 pack-year smoking history d. A 30-year-old patient who takes corticosteroids for rheumatoid arthritis e. A 24-year-old patient who has allergies to penicillin and cephalosporins

A, B, D Current guidelines suggest that healthy individuals between 6 months and age 49 receive intranasal immunization with live, attenuated influenza vaccine. Individuals who are pregnant, residents of nursing homes, or are immunocompromised or who have chronic medical conditions should receive inactivated vaccine by injection. The corticosteroid use by the 30-year-old increases the risk for infection. DIF: Cognitive Level: Apply (application) REF: 504 OBJ: Special Questions: Multiple Patients TOP: Nursing Process: Planning MSC:

Physiological Integrity OTHER 1. The nurse notes new onset confusion in an older patient who is normally alert and oriented. In which order should the nurse take the following actions? (Put a comma and a space between each answer choice [A, B, C, D].) a. Obtain the oxygen saturation. b. Check the patient's pulse rate. c. Document the change in status. d. Notify the health care provider.

A, B, D, C Assessment for physiologic causes of new onset confusion such as pneumonia, infection, or perfusion problems should be the first action by the nurse. Airway and oxygenation should be assessed first, then circulation. After assessing the patient, the nurse should notify the health care provider. Finally, documentation of the assessments and care should be done. DIF: Cognitive Level: Analyze (analysis) REF: 523 OBJ: Special Questions: Prioritization TOP: Nursing Process: Implementation MSC:

Health Promotion and Maintenance OTHER 1. The nurse assumes care of a patient who just returned from surgery for a total laryngectomy and radical neck dissection and notes the following problems. In which order should the nurse address the problems? (Put a comma and a space between each answer choice [A, B, C, D].) a. The patient is in a side-lying position with the head of the bed flat. b. The patient is coughing blood-tinged secretions from the tracheostomy. c. The nasogastric (NG) tube is disconnected from suction and clamped off. d. The wound drain in the neck incision contains 200 mL of bloody drainage.

A, B, D, C The patient should first be placed in a semi-Fowler's position to maintain the airway and reduce incisional swelling. The blood-tinged secretions may obstruct the airway, so suctioning is the next appropriate action. Then the wound drain should be drained because the 200 mL of drainage will decrease the amount of suction in the wound drain and could lead to incisional swelling and poor healing. Finally, the NG tube should be reconnected to suction to prevent gastric dilation, nausea, and vomiting. DIF: Cognitive Level: Analyze (analysis) REF: 516 OBJ: Special Questions: Prioritization TOP: Nursing Process: Implementation MSC:

Selects all of the client at increased risk for developing pneumonia: A) Client with dysphagia B) Client with AIDS C) Client who received pnemovax 6 months ago D) Client who is mechanically ventilated E) Client who had ambulatory surgery

A, B, and D Not E because you are walking right after surgery and then you go home after so you are not in the hospital and prone to getting hospital acquired pnuemonia

Safe and Effective Care Environment MULTIPLE RESPONSE 1. Based on the Joint Commission Core Measures for patients with heart failure, which topics should the nurse include in the discharge teaching plan for a patient who has been hospitalized with chronic heart failure (select all that apply)? a. How to take and record daily weight b. Importance of limiting aerobic exercise c. Date and time of follow-up appointment d. Symptoms indicating worsening heart failure e. Actions and side effects of prescribed medications

A, C, D, E The Joint Commission Core Measures state that patients should be taught about prescribed medications, follow-up appointments, weight monitoring, and actions to take for worsening symptoms. Patients with heart failure are encouraged to begin or continue aerobic exercises such as walking, while self-monitoring to avoid excessive fatigue. DIF: Cognitive Level: Apply (application) REF: 779 TOP: Nursing Process: Planning MSC:

Physiological Integrity MULTIPLE RESPONSE 1. The clinic nurse is teaching a patient with acute sinusitis. Which interventions should the nurse plan to include in the teaching session (select all that apply)? a. Decongestants can be used to relieve swelling. b. Blowing the nose should be avoided to decrease the nosebleed risk. c. Taking a hot shower will increase sinus drainage and decrease pain. d. Saline nasal spray can be made at home and used to wash out secretions. e. You will be more comfortable if you keep your head in an upright position.

A, C, D, E The steam and heat from a shower will help thin secretions and improve drainage. Decongestants can be used to relieve swelling. Patients can use either over-the-counter (OTC) sterile saline solutions or home-prepared saline solutions to thin and remove secretions. Maintaining an upright posture decreases sinus pressure and the resulting pain. Blowing the nose after a hot shower or using the saline spray is recommended to expel secretions. DIF: Cognitive Level: Analyze (analysis) REF: 506 TOP: Nursing Process: Implementation MSC:

Integrated Process: Nursing Process (Planning) 7. The nurse is assessing a client with asthma. Scattered wheezes are noted, and the client's oxygen saturation is 88%. What other assessments are essential for the nurse to perform? (Select all that apply.) a. Assess for accessory muscle use. b. Assess anterior-posterior diameter. c. Assess inspiration/expiration ratios. d. Assess the suprasternal notch. e. Perform a stress test. f. Assess a chest x-ray. g. Assess mucous membranes.

A, C, D, G Accessory muscle use may help the client breathe during an attack. Muscle retraction may be seen at the sternum and at the suprasternal notch. Mucous membranes can also tell the nurse about oxygenation. Inspiration versus expiration can tell the nurse how the client is breathing. The anterior-posterior diameter gives indication of a chronic condition; assessing this during an attack will not help the client. Likewise, performing a stress test and a chest x-ray during an attack would not be beneficial. DIF: Cognitive Level: Application/Applying or higher REF: N/A TOP: Client Needs Category: Physiological Integrity (Physiological Adaptation—Pathophysiology)

34. A nurse is working with a child who is undergoing a diagnostic workup for suspected asthma. What are the signs and symptoms that are consistent with a diagnosis of asthma? Select all that apply. A) Chest tightness B) Crackles C) Bradypnea D) Wheezing E) Cough

A, D, E Feedback: Asthma is a chronic inflammatory disease of the airways that causes airway hyperresponsiveness, mucosal edema, and mucus production. This inflammation ultimately leads to recurrent episodes of asthma symptoms: cough, chest tightness, wheezing, and dyspnea. Crackles and bradypnea are not typical symptoms of asthma.

Integrated Process: Nursing Process (Implementation) 6. The nurse is teaching a client with asthma how to avoid attacks. What information does the nurse give the client? (Select all that apply.) a. "You should not dust your furniture." b. "Stay inside as much as possible." c. "Stay away from people who are sick." d. "Do not go out in the fall." e. "Stay out of the snow." f. "Do not take aspirin."

A, F Dusting the furniture may increase dust in the air and cause an asthma attack. Aspirin may stimulate asthma. Staying inside probably will not help. Staying away from snow probably will not have an effect on the client's attacks; neither will going outside during the fall. DIF: Cognitive Level: Application/Applying or higher REF: N/A TOP: Client Needs Category: Physiological Integrity (Physiological Adaptation—Illness Management)

A patient presents to the emergency room complaining of chest pain. The patient's orders include the following elements. Which order should the nurse complete first? a) 12-lead ECG b) Oxygen 2 liters nasal cannula c) Troponin level d) Aspirin 325 mg orally

A. 12-lead ECG

A nurse is evaluating a client who had a myocardial infarction (MI) 7 days earlier. Which outcome indicates that the client is responding favorably to therapy? a) The client demonstrates ability to tolerate more activity without chest pain. b) The client states that sublingual nitroglycerin usually relieves his chest pain. c) The client verbalizes the intention of making all necessary lifestyle changes except for stopping smoking. d) The client exhibits a heart rate above 100 beats/minute.

A. The client demonstrates ability to tolerate more activity without chest pain

You are caring for a client with coronary artery disease (CAD). What is an appropriate nursing action when evaluating a client with CAD? a) Assess the characteristics of chest pain. b) Assess the skin of the client. c) Assess for any kind of drug abuse. d) Assess the client's mental and emotional status.

A. Assess the characteristics of chest pain

A client comes to the emergency department (ED) complaining of precordial chest pain. In describing the pain, the client describes it as pressure with a sudden onset. What disease process would you suspect in this client? a) Coronary artery disease b) Venous occlusive disease c) Cardiogenic shock d) Raynaud's disease

A. Coronary Artery disease

Which are symptoms of hypoglycemia? A. irritability, B. drowsiness c. Abdominal pain D. nausea and vomiting

A. Irritability: signs of hypoglycemia include irritability, shaky feeling, hunger, headache, dizziness. Other symptoms are hyperglycemia.

The risk factors for type 1 diabetes include all of the following except: "a. Diet b. Genetic c. Autoimmune d. Environmental"

A. Type 1 diabetes is a primary failure of pancreatic beta cells to produce insulin. It primarily affects children and young adults and is unrelated to diet. Diet is a risk factor for Type I diabetes

The nurse is administering a calcium channel blocker to a patient who has symptomatic sinus tachycardia at a rate of 132 bpm. What is the anticipated action of the drug for this patient? a) Decreases the sinoatrial node automaticity b) Increases the atrioventricular node conduction c) Increases the heart rate d) Creates a positive inotropic effect

A. decreases the sinoatrial node automaticity

A nurse is teaching a client how to take nitroglycerin to treat angina pectoris. The client verbalizes an understanding of the need to take up to three sublingual nitroglycerin (Nitrostat) tablets at 5-minute intervals, if necessary, and to notify the physician immediately if chest pain doesn't subside within 15 minutes. The nurse tells the client that, after taking the nitroglycerin, he may experience: a) headache, hypotension, dizziness, and flushing. b) flushing, dizziness, headache, and pedal edema. c) sedation, nausea, vomiting, constipation, and respiratory depression. d) nausea, vomiting, depression, fatigue, and impotence.

A. headache, hypotension, dizziness, and flushing

How do you calculate the ABI?

ABI = BP of ankles / Brachial BP Normal: 0.91 - 1.3 mild PAD: 0.71-0.9 moderate PAD: 0.41 - 0.7 severe PAD: <0.4

The nurse obtains a blood pressure of 176/83 mm Hg for a patient. What is the patient's mean arterial pressure (MAP)?

ANS: 114 mm Hg MAP = (SBP + 2 DBP)/3

Which action will the nurse include in the plan of care for a 72-year-old woman admitted with multiple myeloma? a. Monitor fluid intake and output. b. Administer calcium supplements. c. Assess lymph nodes for enlargement. d. Limit weight bearing and ambulation.

ANS: A A high fluid intake and urine output helps prevent the complications of kidney stones caused by hypercalcemia and renal failure caused by deposition of Bence-Jones protein in the renal tubules. Weight bearing and ambulation are encouraged to help bone retain calcium. Lymph nodes are not enlarged with multiple myeloma. Calcium supplements will further increase the patient's calcium level and are not used.

29. When caring for a patient who has just arrived on the medical-surgical unit after having cardiac catheterization, which nursing intervention should the nurse delegate to a licensed practical/vocational nurse (LPN/LVN)? a. Give the scheduled aspirin and lipid-lowering medication. b. Perform the initial assessment of the catheter insertion site. c. Teach the patient about the usual postprocedure plan of care. d. Titrate the heparin infusion according to the agency protocol.

ANS: A Administration of oral medications is within the scope of practice for LPNs/LVNs. The initial assessment of the patient, patient teaching, and titration of IV anticoagulant medications should be done by the registered nurse (RN).

Propranolol (Inderal) is prescribed for a patient diagnosed with hypertension. The nurse should consult with the health care provider before giving this medication when the patient reveals a history of a. asthma. b. daily alcohol use. c. peptic ulcer disease. d. myocardial infarction (MI).

ANS: A Nonselective b-blockers block b1- and b2-adrenergic receptors and can cause bronchospasm, especially in patients with a history of asthma. b-Blockers will have no effect on the patient's peptic ulcer disease or alcohol use. b-Blocker therapy is recommended after MI

The nurse has just finished teaching a hypertensive patient about the newly prescribed ramipril (Altace). Which patient statement indicates that more teaching is needed? a. "A little swelling around my lips and face is okay." b. "The medication may not work as well if I take any aspirin." c. "The doctor may order a blood potassium level occasionally." d. "I will call the doctor if I notice that I have a frequent cough."

ANS: A Angioedema occurring with angiotensin-converting enzyme (ACE) inhibitor therapy is an indication that the ACE inhibitor should be discontinued. The patient should be taught that if any swelling of the face or oral mucosa occurs, the health care provider should be immediately notified because this could be life threatening. The other patient statements indicate that the patient has an accurate understanding of ACE inhibitor therapy

A patient with acute dyspnea is scheduled for a spiral computed tomography (CT) scan. Which information obtained by the nurse is a priority to communicate to the health care provider before the CT? a. Allergy to shellfish b. Apical pulse of 104 c. Respiratory rate of 30 d. Oxygen saturation of 90%

ANS: A Because iodine-based contrast media is used during a spiral CT, the patient may need to have the CT scan without contrast or be premedicated before injection of the contrast media. The increased pulse, low oxygen saturation, and tachypnea all indicate a need for further assessment or intervention but do not indicate a need to modify the CT procedure.

An appropriate nursing intervention for a patient with non-Hodgkin's lymphoma whose platelet count drops to 18,000/µL during chemotherapy is to a. check all stools for occult blood. b. encourage fluids to 3000 mL/day. c. provide oral hygiene every 2 hours. d. check the temperature every 4 hours.

ANS: A Because the patient is at risk for spontaneous bleeding, the nurse should check stools for occult blood. A low platelet count does not require an increased fluid intake. Oral hygiene is important, but it is not necessary to provide oral care every 2 hours. The low platelet count does not increase risk for infection, so frequent temperature monitoring is not indicated.

34. Which information about a patient who has been receiving thrombolytic therapy for an acute myocardial infarction (AMI) is most important for the nurse to communicate to the health care provider? a. No change in the patient's chest pain b. An increase in troponin levels from baseline c. A large bruise at the patient's IV insertion site d. A decrease in ST-segment elevation on the electrocardiogram

ANS: A Continued chest pain suggests that the thrombolytic therapy is not effective and that other interventions such as percutaneous coronary intervention (PCI) may be needed. Bruising is a possible side effect of thrombolytic therapy, but it is not an indication that therapy should be discontinued. The decrease of the ST-segment elevation indicates that thrombolysis is occurring and perfusion is returning to the injured myocardium. An increase in troponin levels is expected with reperfusion and is related to the washout of cardiac markers into the circulation as the blocked vessel is opened.

On auscultation of a patient's lungs, the nurse hears low-pitched, bubbling sounds during inhalation in the lower third of both lungs. How should the nurse document this finding? a. Inspiratory crackles at the bases b. Expiratory wheezes in both lungs c. Abnormal lung sounds in the apices of both lungs d. Pleural friction rub in the right and left lower lobes

ANS: A Crackles are low-pitched, bubbling sounds usually heard on inspiration. Wheezes are high-pitched sounds. They can be heard during the expiratory or inspiratory phase of the respiratory cycle. The lower third of both lungs are the bases, not apices. Pleural friction rubs are grating sounds that are usually heard during both inspiration and expiration.

25. The nurse is caring for a patient who was admitted to the coronary care unit following an acute myocardial infarction (AMI) and percutaneous coronary intervention the previous day. Teaching for this patient would include a. when cardiac rehabilitation will begin. b. the typical emotional responses to AMI. c. information regarding discharge medications. d. the pathophysiology of coronary artery disease.

ANS: A Early after an AMI, the patient will want to know when resumption of usual activities can be expected. At this time, the patient's anxiety level or denial will interfere with good understanding of complex information such as the pathophysiology of coronary artery disease (CAD). Teaching about discharge medications should be done closer to discharge. The nurse should support the patient by decreasing anxiety rather than discussing the typical emotional responses to myocardial infarction (MI).

Which menu choice indicates that the patient understands the nurse's teaching about best dietary choices for iron-deficiency anemia? a. Omelet and whole wheat toast b. Cantaloupe and cottage cheese c. Strawberry and banana fruit plate d. Cornmeal muffin and orange juice

ANS: A Eggs and whole grain breads are high in iron. The other choices are appropriate for other nutritional deficiencies but are not the best choice for a patient with iron-deficiency anemia.

Which information obtained by the nurse assessing a patient admitted with multiple myeloma is most important to report to the health care provider? a. Serum calcium level is 15 mg/dL. b. Patient reports no stool for 5 days. c. Urine sample has Bence-Jones protein. d. Patient is complaining of severe back pain.

ANS: A Hypercalcemia may lead to complications such as dysrhythmias or seizures, and should be addressed quickly. The other patient findings will also be discussed with the health care provider, but are not life threatening.

Which patient statement to the nurse indicates a need for additional instruction about taking oral ferrous sulfate? a. "I will call my health care provider if my stools turn black." b. "I will take a stool softener if I feel constipated occasionally." c. "I should take the iron with orange juice about an hour before eating." d. "I should increase my fluid and fiber intake while I am taking iron tablets."

ANS: A It is normal for the stools to appear black when a patient is taking iron, and the patient should not call the doctor about this. The other patient statements are correct.

26. A patient who has recently started taking pravastatin (Pravachol) and niacin (Nicobid) reports the following symptoms to the nurse. Which is most important to communicate to the health care provider? a. Generalized muscle aches and pains b. Dizziness when changing positions quickly c. Nausea when taking the drugs before eating d. Flushing and pruritus after taking the medications

ANS: A Muscle aches and pains may indicate myopathy and rhabdomyolysis, which have caused acute kidney injury and death in some patients who have taken the statin medications. These symptoms indicate that the pravastatin may need to be discontinued. The other symptoms are common side effects when taking niacin, and although the nurse should follow-up with the health care provider, they do not indicate that a change in medication is needed.

A 19-year-old woman with immune thrombocytopenic purpura (ITP) has an order for a platelet transfusion. Which information indicates that the nurse should consult with the health care provider before obtaining and administering platelets? a. The platelet count is 42,000/μL. b. Petechiae are present on the chest. c. Blood pressure (BP) is 94/56 mm Hg. d. Blood is oozing from the venipuncture site.

ANS: A Platelet transfusions are not usually indicated until the platelet count is below 10,000 to 20,000/μL unless the patient is actively bleeding. Therefore the nurse should clarify the order with the health care provider before giving the transfusion. The other data all indicate that bleeding caused by ITP may be occurring and that the platelet transfusion is appropriate.

A patient is admitted to the emergency department complaining of sudden onset shortness of breath and is diagnosed with a possible pulmonary embolus. How should the nurse prepare the patient for diagnostic testing to confirm the diagnosis? a. Start an IV so contrast media may be given. b. Ensure that the patient has been NPO for at least 6 hours. c. Inform radiology that radioactive glucose preparation is needed. d. Instruct the patient to undress to the waist and remove any metal objects.

ANS: A Spiral computed tomography (CT) scans are the most commonly used test to diagnose pulmonary emboli, and contrast media may be given IV. A chest x-ray may be ordered but will not be diagnostic for a pulmonary embolus. Preparation for a chest x-ray includes undressing and removing any metal. Bronchoscopy is used to detect changes in the bronchial tree, not to assess for vascular changes, and the patient should be NPO 6 to 12 hours before the procedure. Positron emission tomography (PET) scans are most useful in determining the presence of malignancy, and a radioactive glucose preparation is used.

The charge nurse observes a new registered nurse (RN) doing discharge teaching for a patient with hypertension who has a new prescription for enalapril (Vasotec). The charge nurse will need to intervene if the new RN tells the patient to a. increase the dietary intake of high-potassium foods. b. make an appointment with the dietitian for teaching. c. check the blood pressure (BP) with a home BP monitor at least once a day. d. move slowly when moving from lying to sitting to standing.

ANS: A The ACE inhibitors cause retention of potassium by the kidney, so hyperkalemia is a possible adverse effect. The other teaching by the new RN is appropriate for a patient with newly diagnosed hypertension who has just started therapy with enalapril

The nurse is reviewing the laboratory test results for a patient who has recently been diagnosed with hypertension. Which result is most important to communicate to the health care provider? a. Serum creatinine of 2.8 mg/dL b. Serum potassium of 4.5 mEq/L c. Serum hemoglobin of 14.7 g/dL d. Blood glucose level of 96 mg/dL

ANS: A The elevated creatinine indicates renal damage caused by the hypertension. The other laboratory results are normal

The nurse assesses a patient with chronic obstructive pulmonary disease (COPD) who has been admitted with increasing dyspnea over the last 3 days. Which finding is most important for the nurse to report to the health care provider? a. Respirations are 36 breaths/minute. b. Anterior-posterior chest ratio is 1:1. c. Lung expansion is decreased bilaterally. d. Hyperresonance to percussion is present.

ANS: A The increase in respiratory rate indicates respiratory distress and a need for rapid interventions such as administration of oxygen or medications. The other findings are common chronic changes occurring in patients with COPD.

The nurse caring for a patient with type A hemophilia being admitted to the hospital with severe pain and swelling in the right knee will a. immobilize the joint. b. apply heat to the knee. c. assist the patient with light weight bearing. d. perform passive range of motion to the knee.

ANS: A The initial action should be total rest of the knee to minimize bleeding. Ice packs are used to decrease bleeding. Range of motion (ROM) and weight-bearing exercise are contraindicated initially, but after the bleeding stops, ROM and physical therapy are started.

Which nursing action should the nurse take first in order to assist a patient with newly diagnosed stage 1 hypertension in making needed dietary changes? a. Collect a detailed diet history. b. Provide a list of low-sodium foods. c. Help the patient make an appointment with a dietitian. d. Teach the patient about foods that are high in potassium.

ANS: A The initial nursing action should be assessment of the patient's baseline dietary intake through a thorough diet history. The other actions may be appropriate, but assessment of the patient's baseline should occur first

The nurse has obtained the health history, physical assessment data, and laboratory results shown in the accompanying figure for a patient admitted with aplastic anemia. Which information is most important to communicate to the health care provider? a. Neutropenia b. Increasing fatigue c. Thrombocytopenia d. Frequent constipation

ANS: A The low white blood cell count indicates that the patient is at high risk for infection and needs immediate actions to diagnose and treat the cause of the leucopenia. The other information may require further assessment or treatment, but does not place the patient at immediate risk for complications.

27. A patient who is being admitted to the emergency department with intermittent chest pain gives the following list of medications to the nurse. Which medication has the most immediate implications for the patient's care? a. Sildenafil (Viagra) b. Furosemide (Lasix) c. Captopril (Capoten) d. Warfarin (Coumadin)

ANS: A The nurse will need to avoid giving nitrates to the patient because nitrate administration is contraindicated in patients who are using sildenafil because of the risk of severe hypotension caused by vasodilation. The other home medications also should be documented and reported to the health care provider but do not have as immediate an impact on decisions about the patient's treatment.

19. Three days after experiencing a myocardial infarction (MI), a patient who is scheduled for discharge asks for assistance with hygiene activities, saying, "I am too nervous to take care of myself." Based on this information, which nursing diagnosis is appropriate? a. Ineffective coping related to anxiety b. Activity intolerance related to weakness c. Denial related to lack of acceptance of the MI d. Disturbed personal identity related to understanding of illness

ANS: A The patient data indicate that ineffective coping after the MI caused by anxiety about the impact of the MI is a concern. The other nursing diagnoses may be appropriate for some patients after an MI, but the data for this patient do not support denial, activity intolerance, or disturbed personal identity.

Potassium-Sparing Diuretics

spironolactone (Aldactone), amiloride (Midamore), triamterene (Dyrenium)- increase potassium retention- Cushing disease- avoid prolong exposure to Sunlight.

The nurse on the intermediate care unit received change-of-shift report on four patients with hypertension. Which patient should the nurse assess first? a. 43-year-old with a (blood pressure (BP) of 160/92 who is complaining of chest pain b. 52-year-old with a BP of 212/90 who has intermittent claudication c. 50-year-old with a BP of 190/104 who has a creatinine of 1.7 mg/dL d. 48-year-old with a BP of 172/98 whose urine shows microalbuminuria

ANS: A The patient with chest pain may be experiencing acute myocardial infarction, and rapid assessment and intervention are needed. The symptoms of the other patients also show target organ damage but are not indicative of acute processes

21. A few days after experiencing a myocardial infarction (MI) and successful percutaneous coronary intervention, the patient states, "I just had a little chest pain. As soon as I get out of here, I'm going for my vacation as planned." Which reply would be most appropriate for the nurse to make? a. "What do you think caused your chest pain?" b. "Where are you planning to go for your vacation?" c. "Sometimes plans need to change after a heart attack." d. "Recovery from a heart attack takes at least a few weeks."

ANS: A When the patient is experiencing denial, the nurse should assist the patient in testing reality until the patient has progressed beyond this step of the emotional adjustment to MI. Asking the patient about vacation plans reinforces the patient's plan, which is not appropriate in the immediate post-MI period. Reminding the patient in denial about the MI is likely to make the patient angry and lead to distrust of the nursing staff.

The nurse is educating a pregnant client who has gestational diabetes. Which of the following statements should the nurse make to the client? Select all that apply. "a. Cakes, candies, cookies, and regular soft drinks should be avoided. b. Gestational diabetes increases the risk that the mother will develop diabetes later in life. c. Gestational diabetes usually resolves after the baby is born. d. Insulin injections may be necessary. e. The baby will likely be born with diabetes f. The mother should strive to gain no more weight during the pregnancy.

ANS: A, B, C, D Gestational diabetes can occur between the 16th and 28th week of pregnancy. If not responsive to diet and exercise, insulin injections may be necessary. Concentrated sugars should be avoided. Weight gain should continue, but not in excessive amounts. Usually, gestational diabetes disappears after the infant is born. However, diabetes can develop 5 to 10 years after the pregnancy"

The nurse is educating a pregnant client who has gestational diabetes. Which of the following statements should the nurse make to the client? Select all that apply. a. Cakes, candies, cookies, and regular soft drinks should be avoided. b. Gestational diabetes increases the risk that the mother will develop diabetes later in life. c. Gestational diabetes usually resolves after the baby is born. d. Insulin injections may be necessary. e. The baby will likely be born with diabetes f. The mother should strive to gain no more weight during the pregnancy.

ANS: A, B, C, D Gestational diabetes can occur between the 16th and 28th week of pregnancy. If not responsive to diet and exercise, insulin injections may be necessary. Concentrated sugars should be avoided. Weight gain should continue, but not in excessive amounts. Usually, gestational diabetes disappears after the infant is born. However, diabetes can develop 5 to 10 years after the pregnancy.

Which patient requires the most rapid assessment and care by the emergency department nurse? a. The patient with hemochromatosis who reports abdominal pain b. The patient with neutropenia who has a temperature of 101.8° F c. The patient with sickle cell anemia who has had nausea and diarrhea for 24 hours d. The patient with thrombocytopenia who has oozing after having a tooth extracted

ANS: B A neutropenic patient with a fever is assumed to have an infection and is at risk for rapidly developing sepsis. Rapid assessment, cultures, and initiation of antibiotic therapy are needed. The other patients also require rapid assessment and care but not as urgently as the neutropenic patient.

After receiving change-of-shift report for several patients with neutropenia, which patient should the nurse assess first? a. 56-year-old with frequent explosive diarrhea b. 33-year-old with a fever of 100.8° F (38.2° C) c. 66-year-old who has white pharyngeal lesions d. 23-year old who is complaining of severe fatigue

ANS: B Any fever in a neutropenic patient indicates infection and can quickly lead to sepsis and septic shock. Rapid assessment and (if prescribed) initiation of antibiotic therapy within 1 hour are needed. The other patients also need to be assessed but do not exhibit symptoms of potentially life-threatening problems.

33. When admitting a patient with a non-ST-segment-elevation myocardial infarction (NSTEMI) to the intensive care unit, which action should the nurse perform first? a. Obtain the blood pressure. b. Attach the cardiac monitor. c. Assess the peripheral pulses. d. Auscultate the breath sounds.

ANS: B Because dysrhythmias are the most common complication of myocardial infarction (MI), the first action should be to place the patient on a cardiac monitor. The other actions also are important and should be accomplished as quickly as possible.

A patient with hypertension who has just started taking atenolol (Tenormin) returns to the health clinic after 2 weeks for a follow-up visit. The blood pressure (BP) is unchanged from the previous visit. Which action should the nurse take first? a. Inform the patient about the reasons for a possible change in drug dosage. b. Question the patient about whether the medication is actually being taken. c. Inform the patient that multiple drugs are often needed to treat hypertension. d. Question the patient regarding any lifestyle changes made to help control BP.

ANS: B Because noncompliance with antihypertensive therapy is common, the nurse's initial action should be to determine whether the patient is taking the atenolol as prescribed. The other actions also may be implemented, but these would be done after assessing patient compliance with the prescribed therapy

39. To improve the physical activity level for a mildly obese 71-year-old patient, which action should the nurse plan to take? a. Stress that weight loss is a major benefit of increased exercise. b. Determine what kind of physical activities the patient usually enjoys. c. Tell the patient that older adults should exercise for no more than 20 minutes at a time. d. Teach the patient to include a short warm-up period at the beginning of physical activity.

ANS: B Because patients are more likely to continue physical activities that they already enjoy, the nurse will plan to ask the patient about preferred activities. The goal for older adults is 30 minutes of moderate activity on most days. Older adults should plan for a longer warm-up period. Benefits of exercises, such as improved activity tolerance, should be emphasized rather than aiming for significant weight loss in older mildly obese adults.

After the nurse has received change-of-shift report, which patient should the nurse assess first? a. A patient with pneumonia who has crackles in the right lung base b. A patient with possible lung cancer who has just returned after bronchoscopy c. A patient with hemoptysis and a 16-mm induration with tuberculin skin testing d. A patient with chronic obstructive pulmonary disease (COPD) and pulmonary function testing (PFT) that indicates low forced vital capacity

ANS: B Because the cough and gag are decreased after bronchoscopy, this patient should be assessed for airway patency. The other patients do not have clinical manifestations or procedures that require immediate assessment by the nurse.

Which collaborative problem will the nurse include in a care plan for a patient admitted to the hospital with idiopathic aplastic anemia? a. Potential complication: seizures b. Potential complication: infection c. Potential complication: neurogenic shock d. Potential complication: pulmonary edema

ANS: B Because the patient with aplastic anemia has pancytopenia, the patient is at risk for infection and bleeding. There is no increased risk for seizures, neurogenic shock, or pulmonary edema.

A routine complete blood count indicates that an active 80-year-old man may have myelodysplastic syndrome. The nurse will plan to teach the patient about a. blood transfusion b. bone marrow biopsy. c. filgrastim (Neupogen) administration. d. erythropoietin (Epogen) administration.

ANS: B Bone marrow biopsy is needed to make the diagnosis and determine the specific type of myelodysplastic syndrome. The other treatments may be necessary if there is progression of the myelodysplastic syndrome, but the initial action for this asymptomatic patient will be a bone marrow biopsy.

3. Which assessment data collected by the nurse who is admitting a patient with chest pain suggest that the pain is caused by an acute myocardial infarction (AMI)? a. The pain increases with deep breathing. b. The pain has lasted longer than 30 minutes. c. The pain is relieved after the patient takes nitroglycerin. d. The pain is reproducible when the patient raises the arms.

ANS: B Chest pain that lasts for 20 minutes or more is characteristic of AMI. Changes in pain that occur with raising the arms or with deep breathing are more typical of musculoskeletal pain or pericarditis. Stable angina is usually relieved when the patient takes nitroglycerin.

20. When caring for a patient who is recovering from a sudden cardiac death (SCD) event and has no evidence of an acute myocardial infarction (AMI), the nurse will anticipate teaching the patient that a. sudden cardiac death events rarely reoccur. b. additional diagnostic testing will be required. c. long-term anticoagulation therapy will be needed. d. limited physical activity after discharge will be needed to prevent future events.

ANS: B Diagnostic testing (e.g., stress test, Holter monitor, electrophysiologic studies, cardiac catheterization) is used to determine the possible cause of the SCD and treatment options. SCD is likely to recur. Anticoagulation therapy will not have any effect on the incidence of SCD, and SCD can occur even when the patient is resting.

Which information obtained by the nurse caring for a patient with thrombocytopenia should be immediately communicated to the health care provider? a. The platelet count is 52,000/µL. b. The patient is difficult to arouse. c. There are purpura on the oral mucosa. d. There are large bruises on the patient's back.

ANS: B Difficulty in arousing the patient may indicate a cerebral hemorrhage, which is life threatening and requires immediate action. The other information should be documented and reported but would not be unusual in a patient with thrombocytopenia.

Which assessment finding for a patient who is receiving IV furosemide (Lasix) to treat stage 2 hypertension is most important to report to the health care provider? a. Blood glucose level of 175 mg/dL b. Blood potassium level of 3.0 mEq/L c. Most recent blood pressure (BP) reading of 168/94 mm Hg d. Orthostatic systolic BP decrease of 12 mm Hg

ANS: B Hypokalemia is a frequent adverse effect of the loop diuretics and can cause life-threatening dysrhythmias. The health care provider should be notified of the potassium level immediately and administration of potassium supplements initiated. The elevated blood glucose and BP also indicate a need for collaborative interventions but will not require action as urgently as the hypokalemia. An orthostatic drop of 12 mm Hg is common and will require intervention only if the patient is symptomatic

Which intervention will be included in the nursing care plan for a patient with immune thrombocytopenic purpura (ITP)? a. Assign the patient to a private room. b. Avoid intramuscular (IM) injections. c. Use rinses rather than a soft toothbrush for oral care. d. Restrict activity to passive and active range of motion.

ANS: B IM or subcutaneous injections should be avoided because of the risk for bleeding. A soft toothbrush can be used for oral care. There is no need to restrict activity or place the patient in a private room.

The nurse notes scleral jaundice in a patient being admitted with hemolytic anemia. The nurse will plan to check the laboratory results for the a. Schilling test. b. bilirubin level. c. stool occult blood test. d. gastric analysis testing.

ANS: B Jaundice is caused by the elevation of bilirubin level associated with red blood cell (RBC) hemolysis. The other tests would not be helpful in monitoring or treating a hemolytic anemia.

A diabetic patient's arterial blood gas (ABG) results are pH 7.28; PaCO2 34 mm Hg; PaO2 85 mm Hg; HCO3- 18 mEq/L. The nurse would expect which finding? a. Intercostal retractions b. Kussmaul respirations c. Low oxygen saturation (SpO2) d. Decreased venous O2 pressure

ANS: B Kussmaul (deep and rapid) respirations are a compensatory mechanism for metabolic acidosis. The low pH and low bicarbonate result indicate metabolic acidosis. Intercostal retractions, a low oxygen saturation rate, and a decrease in venous O2 pressure would not be caused by acidosis.

Which action is appropriate for the nurse to delegate to unlicensed assistive personnel (UAP)? a. Listen to a patient's lung sounds for wheezes or rhonchi. b. Label specimens obtained during percutaneous lung biopsy. c. Instruct a patient about how to use home spirometry testing. d. Measure induration at the site of a patient's intradermal skin test.

ANS: B Labeling of specimens is within the scope of practice of UAP. The other actions require nursing judgment and should be done by licensed nursing personnel.

Which problem reported by a patient with hemophilia is most important for the nurse to communicate to the physician? a. Leg bruises b. Tarry stools c. Skin abrasions d. Bleeding gums

ANS: B Melena is a sign of gastrointestinal bleeding and requires collaborative actions such as checking hemoglobin and hematocrit and administration of coagulation factors. The other problems indicate a need for patient teaching about how to avoid injury, but are not indicators of possible serious blood loss.

A patient who is receiving methotrexate for severe rheumatoid arthritis develops a megaloblastic anemia. The nurse will anticipate teaching the patient about increasing oral intake of a. iron. b. folic acid. c. cobalamin (vitamin B12). d. ascorbic acid (vitamin C).

ANS: B Methotrexate use can lead to folic acid deficiency. Supplementation with oral folic acid supplements is the usual treatment. The other nutrients would not correct folic acid deficiency, although they would be used to treat other types of anemia.

An appropriate nursing intervention for a hospitalized patient with severe hemolytic anemia is to a. provide a diet high in vitamin K. b. alternate periods of rest and activity. c. teach the patient how to avoid injury. d. place the patient on protective isolation.

ANS: B Nursing care for patients with anemia should alternate periods of rest and activity to encourage activity without causing undue fatigue. There is no indication that the patient has a bleeding disorder, so a diet high in vitamin K or teaching about how to avoid injury is not needed. Protective isolation might be used for a patient with aplastic anemia, but it is not indicated for hemolytic anemia.

It is important for the nurse providing care for a patient with sickle cell crisis to a. limit the patient's intake of oral and IV fluids. b. evaluate the effectiveness of opioid analgesics. c. encourage the patient to ambulate as much as tolerated. d. teach the patient about high-protein, high-calorie foods.

ANS: B Pain is the most common clinical manifestation of a crisis and usually requires large doses of continuous opioids for control. Fluid intake should be increased to reduce blood viscosity and improve perfusion. Rest is usually ordered to decrease metabolic requirements. Patients are instructed about the need for dietary folic acid, but high-protein, high-calorie diets are not emphasized.

10. The nurse will suspect that the patient with stable angina is experiencing a side effect of the prescribed metoprolol (Lopressor) if the a. patient is restless and agitated. b. blood pressure is 90/54 mm Hg. c. patient complains about feeling anxious. d. cardiac monitor shows a heart rate of 61 beats/minute.

ANS: B Patients taking β-adrenergic blockers should be monitored for hypotension and bradycardia. Because this class of medication inhibits the sympathetic nervous system, restlessness, agitation, hypertension, and anxiety will not be side effects.

13. When titrating IV nitroglycerin (Tridil) for a patient with a myocardial infarction (MI), which action will the nurse take to evaluate the effectiveness of the medication? a. Monitor heart rate. b. Ask about chest pain. c. Check blood pressure. d. Observe for dysrhythmias.

ANS: B The goal of IV nitroglycerin administration in MI is relief of chest pain by improving the balance between myocardial oxygen supply and demand. The nurse also will monitor heart rate and blood pressure (BP) and observe for dysrhythmias, but these parameters will not indicate whether the medication is effective.

7. After the nurse teaches the patient about the use of carvedilol (Coreg) in preventing anginal episodes, which statement by a patient indicates that the teaching has been effective? a. "Carvedilol will help my heart muscle work harder." b. "It is important not to suddenly stop taking the carvedilol." c. "I can expect to feel short of breath when taking carvedilol." d. "Carvedilol will increase the blood flow to my heart muscle."

ANS: B Patients who have been taking β-adrenergic blockers can develop intense and frequent angina if the medication is suddenly discontinued. Carvedilol (Coreg) decreases myocardial contractility. Shortness of breath that occurs when taking β-adrenergic blockers for angina may be due to bronchospasm and should be reported to the health care provider. Carvedilol works by decreasing myocardial oxygen demand, not by increasing blood flow to the coronary arteries.

9. Diltiazem (Cardizem) is ordered for a patient with newly diagnosed Prinzmetal's (variant) angina. When teaching the patient, the nurse will include the information that diltiazem will a. reduce heart palpitations. b. decrease spasm of the coronary arteries. c. increase the force of the heart contractions. d. help prevent plaque from forming in the coronary arteries.

ANS: B Prinzmetal's angina is caused by coronary artery spasm. Calcium channel blockers (e.g., diltiazem, amlodipine [Norvasc

A patient with a chronic cough has a bronchoscopy. After the procedure, which intervention by the nurse is most appropriate? a. Elevate the head of the bed to 80 to 90 degrees. b. Keep the patient NPO until the gag reflex returns. c. Place on bed rest for at least 4 hours after bronchoscopy. d. Notify the health care provider about blood-tinged mucus.

ANS: B Risk for aspiration and maintaining an open airway is the priority. Because a local anesthetic is used to suppress the gag/cough reflexes during bronchoscopy, the nurse should monitor for the return of these reflexes before allowing the patient to take oral fluids or food. Blood-tinged mucus is not uncommon after bronchoscopy. The patient does not need to be on bed rest, and the head of the bed does not need to be in the high-Fowler's position.

23. A patient who is recovering from an acute myocardial infarction (AMI) asks the nurse about when sexual intercourse can be resumed. Which response by the nurse is best? a. "Most patients are able to enjoy intercourse without any complications." b. "Sexual activity uses about as much energy as climbing two flights of stairs." c. "The doctor will provide sexual guidelines when your heart is strong enough." d. "Holding and cuddling are good ways to maintain intimacy after a heart attack."

ANS: B Sexual activity places about as much physical stress on the cardiovascular system as most moderate-energy activities such as climbing two flights of stairs. The other responses do not directly address the patient's question or may not be accurate for this patient.

A patient with a history of hypertension treated with a diuretic and an angiotensin-converting enzyme (ACE) inhibitor arrives in the emergency department complaining of a severe headache and nausea and has a blood pressure (BP) of 238/118 mm Hg. Which question should the nurse ask first? a. "Did you take any acetaminophen (Tylenol) today?" b. "Have you been consistently taking your medications?" c. "Have there been any recent stressful events in your life?" d. "Have you recently taken any antihistamine medications?"

ANS: B Sudden withdrawal of antihypertensive medications can cause rebound hypertension and hypertensive crisis. Although many over-the-counter medications can cause hypertension, antihistamines and acetaminophen do not increase BP. Stressful events will increase BP but not usually to the level seen in this patient

A patient with a history of a transfusion-related acute lung injury (TRALI) is to receive a transfusion of packed red blood cells (PRBCs). Which action by the nurse will decrease the risk for TRALI for this patient? a. Infuse the PRBCs slowly over 4 hours. b. Transfuse only leukocyte-reduced PRBCs. c. Administer the scheduled diuretic before the transfusion. d. Give the PRN dose of antihistamine before the transfusion.

ANS: B TRALI is caused by a reaction between the donor and the patient leukocytes that causes pulmonary inflammation and capillary leaking. The other actions may help prevent respiratory problems caused by circulatory overload or by allergic reactions, but they will not prevent TRALI.

A 28-year-old man with von Willebrand disease is admitted to the hospital for minor knee surgery. The nurse will review the coagulation survey to check the a. platelet count. b. bleeding time. c. thrombin time. d. prothrombin time.

ANS: B The bleeding time is affected by von Willebrand disease. Platelet count, prothrombin time, and thrombin time are normal in von Willebrand disease.

28. Which assessment finding by the nurse caring for a patient who has had coronary artery bypass grafting using a right radial artery graft is most important to communicate to the health care provider? a. Complaints of incisional chest pain b. Pallor and weakness of the right hand c. Fine crackles heard at both lung bases d. Redness on both sides of the sternal incision

ANS: B The changes in the right hand indicate compromised blood flow, which requires immediate evaluation and actions such as prescribed calcium channel blockers or surgery. The other changes are expected and/or require nursing interventions.

Which action will the admitting nurse include in the care plan for a 30-year old woman who is neutropenic? a. Avoid any injections. b. Check temperature every 4 hours. c. Omit fruits or vegetables from the diet. d. Place a "No Visitors" sign on the door.

ANS: B The earliest sign of infection in a neutropenic patient is an elevation in temperature. Although unpeeled fresh fruits and vegetables should be avoided, fruits and vegetables that are peeled or cooked are acceptable. Injections may be required for administration of medications such as filgrastim (Neupogen). The number of visitors may be limited and visitors with communicable diseases should be avoided, but a "no visitors" policy is not needed.

Which blood pressure (BP) finding by the nurse indicates that no changes in therapy are needed for a patient with stage 1 hypertension who has a history of diabetes mellitus? a. 102/60 mm Hg b. 128/76 mm Hg c. 139/90 mm Hg d. 136/82 mm Hg

ANS: B The goal for antihypertensive therapy for a patient with hypertension and diabetes mellitus is a BP <130/80 mm Hg. The BP of 102/60 may indicate overtreatment of the hypertension and an increased risk for adverse drug effects. The other two blood pressures indicate a need for modifications in the patient's treatment

40. Which patient at the cardiovascular clinic requires the most immediate action by the nurse? a. Patient with type 2 diabetes whose current blood glucose level is 145 mg/dL b. Patient with stable angina whose chest pain has recently increased in frequency c. Patient with familial hypercholesterolemia and a total cholesterol of 465 mg/dL d. Patient with chronic hypertension whose blood pressure today is 172/98 mm Hg

ANS: B The history of more frequent chest pain suggests that the patient may have unstable angina, which is part of the acute coronary syndrome spectrum. This will require rapid implementation of actions such as cardiac catheterization and possible percutaneous coronary intervention. The data about the other patients suggest that their conditions are stable.

A 30-year-old man with acute myelogenous leukemia develops an absolute neutrophil count of 850/µL while receiving outpatient chemotherapy. Which action by the outpatient clinic nurse is most appropriate? a. Discuss the need for hospital admission to treat the neutropenia. b. Teach the patient to administer filgrastim (Neupogen) injections. c. Plan to discontinue the chemotherapy until the neutropenia resolves. d. Order a high-efficiency particulate air (HEPA) filter for the patient's home.

ANS: B The patient may be taught to self-administer filgrastim injections. Although chemotherapy may be stopped with severe neutropenia (neutrophil count less than 500/µL), administration of filgrastim usually allows the chemotherapy to continue. Patients with neutropenia are at higher risk for infection when exposed to other patients in the hospital. HEPA filters are expensive and are used in the hospital, where the number of pathogens is much higher than in the patient's home environment.

Which action will the nurse in the hypertension clinic take in order to obtain an accurate baseline blood pressure (BP) for a new patient? a. Deflate the BP cuff at a rate of 5 to 10 mm Hg per second. b. Have the patient sit in a chair with the feet flat on the floor. c. Assist the patient to the supine position for BP measurements. d. Obtain two BP readings in the dominant arm and average the results.

ANS: B The patient should be seated with the feet flat on the floor. The BP is obtained in both arms, and the results of the two arms are not averaged. The patient does not need to be in the supine position. The cuff should be deflated at 2 to 3 mm Hg per second

Several patients call the outpatient clinic and ask to make an appointment as soon as possible. Which patient should the nurse schedule to be seen first? a. 44-year-old with sickle cell anemia who says "my eyes always look sort of yellow" b. 23-year-old with no previous health problems who has a nontender lump in the axilla c. 50-year-old with early-stage chronic lymphocytic leukemia who reports chronic fatigue d. 19-year-old with hemophilia who wants to learn to self-administer factor VII replacement

ANS: B The patient's age and presence of a nontender axillary lump suggest possible lymphoma, which needs rapid diagnosis and treatment. The other patients have questions about treatment or symptoms that are consistent with their diagnosis but do not need to be seen urgently.

The nurse is assessing a patient who has been admitted to the intensive care unit (ICU) with a hypertensive emergency. Which finding is most important to report to the health care provider? a. Urine output over 8 hours is 250 mL less than the fluid intake. b. The patient cannot move the left arm and leg when asked to do so. c. Tremors are noted in the fingers when the patient extends the arms. d. The patient complains of a headache with pain at level 8/10 (0 to 10 scale).

ANS: B The patient's inability to move the left arm and leg indicates that a hemorrhagic stroke may be occurring and will require immediate action to prevent further neurologic damage. The other clinical manifestations are also likely caused by the hypertension and will require rapid nursing actions, but they do not require action as urgently as the neurologic changes

A patient with septicemia develops prolonged bleeding from venipuncture sites and blood in the stools. Which action is most important for the nurse to take? a. Avoid venipunctures. b. Notify the patient's physician. c. Apply sterile dressings to the sites. d. Give prescribed proton-pump inhibitors.

ANS: B The patient's new onset of bleeding and diagnosis of sepsis suggest that disseminated intravascular coagulation (DIC) may have developed, which will require collaborative actions such as diagnostic testing, blood product administration, and heparin administration. The other actions also are appropriate, but the most important action should be to notify the physician so that DIC treatment can be initiated rapidly.

The nurse obtains the following information from a patient newly diagnosed with prehypertension. Which finding is most important to address with the patient? a. Low dietary fiber intake b. No regular aerobic exercise c. Weight 5 pounds above ideal weight d. Drinks a beer with dinner on most nights

ANS: B The recommendations for preventing hypertension include exercising aerobically for 30 minutes most days of the week. A weight that is 5 pounds over the ideal body weight is not a risk factor for hypertension. The Dietary Approaches to Stop Hypertension (DASH) diet is high in fiber, but increasing fiber alone will not prevent hypertension from developing. The patient's alcohol intake is within guidelines and will not increase the hypertension risk

A 68-year-old woman with acute myelogenous leukemia (AML) asks the nurse whether the planned chemotherapy will be worth undergoing. Which response by the nurse is appropriate? a. "If you do not want to have chemotherapy, other treatment options include stem cell transplantation." b. "The side effects of chemotherapy are difficult, but AML frequently goes into remission with chemotherapy." c. "The decision about treatment is one that you and the doctor need to make rather than asking what I would do." d. "You don't need to make a decision about treatment right now because leukemias in adults tend to progress quite slowly."

ANS: B This response uses therapeutic communication by addressing the patient's question and giving accurate information. The other responses either give inaccurate information or fail to address the patient's question, which will discourage the patient from asking the nurse for information.

The nurse is planning to administer a transfusion of packed red blood cells (PRBCs) to a patient with blood loss from gastrointestinal hemorrhage. Which action can the nurse delegate to unlicensed assistive personnel (UAP)? a. Verify the patient identification (ID) according to hospital policy. b. Obtain the temperature, blood pressure, and pulse before the transfusion. c. Double-check the product numbers on the PRBCs with the patient ID band. d. Monitor the patient for shortness of breath or chest pain during the transfusion.

ANS: B UAP education includes measurement of vital signs. UAP would report the vital signs to the registered nurse (RN). The other actions require more education and a larger scope of practice and should be done by licensed nursing staff members.

A patient with acute shortness of breath is admitted to the hospital. Which action should the nurse take during the initial assessment of the patient? a. Ask the patient to lie down to complete a full physical assessment. b. Briefly ask specific questions about this episode of respiratory distress. c. Complete the admission database to check for allergies before treatment. d. Delay the physical assessment to first complete pulmonary function tests.

ANS: B When a patient has severe respiratory distress, only information pertinent to the current episode is obtained, and a more thorough assessment is deferred until later. Obtaining a comprehensive health history or full physical examination is unnecessary until the acute distress has resolved. Brief questioning and a focused physical assessment should be done rapidly to help determine the cause of the distress and suggest treatment. Checking for allergies is important, but it is not appropriate to complete the entire admission database at this time. The initial respiratory assessment must be completed before any diagnostic tests or interventions can be ordered.

22. When evaluating the effectiveness of preoperative teaching with a patient scheduled for coronary artery bypass graft (CABG) surgery using the internal mammary artery, the nurse determines that additional teaching is needed when the patient says which of the following? a. "They will circulate my blood with a machine during the surgery." b. "I will have small incisions in my leg where they will remove the vein." c. "They will use an artery near my heart to go around the area that is blocked." d. "I will need to take an aspirin every day after the surgery to keep the graft open."

ANS: B When the internal mammary artery is used there is no need to have a saphenous vein removed from the leg. The other statements by the patient are accurate and indicate that the teaching has been effective.

A patient is scheduled for a computed tomography (CT) of the chest with contrast media. Which assessment findings should the nurse immediately report to the health care provider (select all that apply)? a. Patient is claustrophobic. b. Patient is allergic to shellfish. c. Patient recently used a bronchodilator inhaler. d. Patient is not able to remove a wedding band. e. Blood urea nitrogen (BUN) and serum creatinine levels are elevated.

ANS: B, E Because the contrast media is iodine-based and may cause dehydration and decreased renal blood flow, asking about iodine allergies (such as allergy to shellfish) and monitoring renal function before the CT scan are necessary. The other actions are not contraindications for CT of the chest, although they may be for other diagnostic tests, such as magnetic resonance imaging (MRI) or pulmonary function testing (PFT).

15. Following an acute myocardial infarction (AMI), a patient ambulates in the hospital hallway. When the nurse is evaluating the patient's response to the activity, which assessment data would indicate that the exercise level should be decreased? a. Blood pressure (BP) changes from 118/60 to 126/68 mm Hg. b. Oxygen saturation drops from 99% to 95%. c. Heart rate increases from 66 to 92 beats/minute. d. Respiratory rate goes from 14 to 20 breaths/minute.

ANS: C A change in heart rate of more than 20 beats over the resting heart rate indicates that the patient should stop and rest. The increases in BP and respiratory rate, and the slight decrease in oxygen saturation, are normal responses to exercise.

Which action for a patient with neutropenia is appropriate for the registered nurse (RN) to delegate to a licensed practical/vocational nurse (LPN/LVN)? a. Assessing the patient for signs and symptoms of infection b. Teaching the patient the purpose of neutropenic precautions c. Administering subcutaneous filgrastim (Neupogen) injection d. Developing a discharge teaching plan for the patient and family

ANS: C Administration of subcutaneous medications is included in LPN/LVN education and scope of practice. Patient education, assessment, and developing the plan of care require RN level education and scope of practice.

32. A patient admitted to the coronary care unit (CCU) with an ST-segment-elevation myocardial infarction (STEMI) is restless and anxious. The blood pressure is 86/40 and heart rate is 123. Based on this information, which nursing diagnosis is a priority for the patient? a. Acute pain related to myocardial infarction b. Anxiety related to perceived threat of death c. Stress overload related to acute change in health d. Decreased cardiac output related to cardiogenic shock

ANS: C All the nursing diagnoses may be appropriate for this patient, but the hypotension and tachycardia indicate decreased cardiac output and shock from the damaged myocardium. This will result in decreased perfusion to all vital organs (e.g., brain, kidney, heart) and is a priority.

A patient in metabolic alkalosis is admitted to the emergency department, and pulse oximetry (SpO2) indicates that the O2 saturation is 94%. Which action should the nurse take next? a. Administer bicarbonate. b. Complete a head-to-toe assessment. c. Place the patient on high-flow oxygen. d. Obtain repeat arterial blood gases (ABGs).

ANS: C Although the O2 saturation is adequate, the left shift in the oxyhemoglobin dissociation curve will decrease the amount of oxygen delivered to tissues, so high oxygen concentrations should be given. Bicarbonate would worsen the patient's condition. A head-to-toe assessment and repeat ABGs may be implemented. However, the priority intervention is to give high-flow oxygen.

When a patient with splenomegaly is scheduled for splenectomy, which action will the nurse include in the preoperative plan of care? a. Discourage deep breathing to reduce risk for splenic rupture. b. Teach the patient to use ibuprofen (Advil) for left upper quadrant pain. c. Schedule immunization with the pneumococcal vaccine (Pneumovax). d. Avoid the use of acetaminophen (Tylenol) for 2 weeks prior to surgery.

ANS: C Asplenic patients are at high risk for infection with Pneumococcus and immunization reduces this risk. There is no need to avoid acetaminophen use before surgery, but nonsteroidal antiinflammatory drugs (NSAIDs) may increase bleeding risk and should be avoided. The enlarged spleen may decrease respiratory depth and the patient should be encouraged to take deep breaths.

A 52-year-old patient has a new diagnosis of pernicious anemia. The nurse determines that the patient understands the teaching about the disorder when the patient states, "I a. need to start eating more red meat and liver." b. will stop having a glass of wine with dinner." c. could choose nasal spray rather than injections of vitamin B12." d. will need to take a proton pump inhibitor like omeprazole (Prilosec)."

ANS: C Because pernicious anemia prevents the absorption of vitamin B12, this patient requires injections or intranasal administration of cobalamin. Alcohol use does not cause cobalamin deficiency. Proton pump inhibitors decrease the absorption of vitamin B12. Eating more foods rich in vitamin B12 is not helpful because the lack of intrinsic factor prevents absorption of the vitamin.

Which instruction will the nurse plan to include in discharge teaching for the patient admitted with a sickle cell crisis? a. Take a daily multivitamin with iron. b. Limit fluids to 2 to 3 quarts per day. c. Avoid exposure to crowds when possible. d. Drink only two caffeinated beverages daily.

ANS: C Exposure to crowds increases the patient's risk for infection, the most common cause of sickle cell crisis. There is no restriction on caffeine use. Iron supplementation is generally not recommended. A high-fluid intake is recommended.

The nurse completes a shift assessment on a patient admitted in the early phase of heart failure. When auscultating the patient's lungs, which finding would the nurse most likely hear? a. Continuous rumbling, snoring, or rattling sounds mainly on expiration b. Continuous high-pitched musical sounds on inspiration and expiration c. Discontinuous, high-pitched sounds of short duration heard on inspiration d. A series of long-duration, discontinuous, low-pitched sounds during inspiration

ANS: C Fine crackles are likely to be heard in the early phase of heart failure. Fine crackles are discontinuous, high-pitched sounds of short duration heard on inspiration. Rhonchi are continuous rumbling, snoring, or rattling sounds mainly on expiration. Course crackles are a series of long-duration, discontinuous, low-pitched sounds during inspiration. Wheezes are continuous high-pitched musical sounds on inspiration and expiration.

The nurse teaches a patient about pulmonary function testing (PFT). Which statement, if made by the patient, indicates teaching was effective? a. "I will use my inhaler right before the test." b. "I won't eat or drink anything 8 hours before the test." c. "I should inhale deeply and blow out as hard as I can during the test." d. "My blood pressure and pulse will be checked every 15 minutes after the test."

ANS: C For PFT, the patient should inhale deeply and exhale as long, hard, and fast as possible. The other actions are not needed with PFT. The administration of inhaled bronchodilators should be avoided 6 hours before the procedure.

1. When developing a teaching plan for a 61-year-old man with the following risk factors for coronary artery disease (CAD), the nurse should focus on the a. family history of coronary artery disease. b. increased risk associated with the patient's gender. c. increased risk of cardiovascular disease as people age. d. elevation of the patient's low-density lipoprotein (LDL) level.

ANS: D Because family history, gender, and age are nonmodifiable risk factors, the nurse should focus on the patient's LDL level. Decreases in LDL will help reduce the patient's risk for developing CAD.

An older patient has been diagnosed with possible white coat hypertension. Which action will the nurse plan to take next? a. Schedule the patient for regular blood pressure (BP) checks in the clinic. b. Instruct the patient about the need to decrease stress levels. c. Tell the patient how to self-monitor and record BPs at home. d. Inform the patient that ambulatory blood pressure monitoring will be needed.

ANS: C Having the patient self-monitor BPs at home will provide a reliable indication about whether the patient has hypertension. Regular BP checks in the clinic are likely to be high in a patient with white coat hypertension. Ambulatory blood pressure monitoring may be used if the data from self-monitoring are unclear. Although elevated stress levels may contribute to hypertension, instructing the patient about this is unlikely to reduce BP

12. Heparin is ordered for a patient with a non-ST-segment-elevation myocardial infarction (NSTEMI). What is the purpose of the heparin? a. Heparin enhances platelet aggregation. b. Heparin decreases coronary artery plaque size. c. Heparin prevents the development of new clots in the coronary arteries. d. Heparin dissolves clots that are blocking blood flow in the coronary arteries.

ANS: C Heparin helps prevent the conversion of fibrinogen to fibrin and decreases coronary artery thrombosis. It does not change coronary artery plaque, dissolve already formed clots, or enhance platelet aggregation.

The registered nurse (RN) is caring for a patient with a hypertensive crisis who is receiving sodium nitroprusside (Nipride). Which nursing action can the nurse delegate to an experienced licensed practical/vocational nurse (LPN/LVN)? a. Titrate nitroprusside to decrease mean arterial pressure (MAP) to 115 mm Hg. b. Evaluate effectiveness of nitroprusside therapy on blood pressure (BP). c. Set up the automatic blood pressure machine to take BP every 15 minutes. d. Assess the patient's environment for adverse stimuli that might increase BP.

ANS: C LPN/LVN education and scope of practice include the correct use of common equipment such as automatic blood pressure machines. The other actions require advanced nursing judgment and education, and should be done by RNs

Which action should the nurse take when administering the initial dose of oral labetalol (Normodyne) to a patient with hypertension? a. Encourage the use of hard candy to prevent dry mouth. b. Instruct the patient to ask for help if heart palpitations occur. c. Ask the patient to request assistance when getting out of bed. d. Teach the patient that headaches may occur with this medication.

ANS: C Labetalol decreases sympathetic nervous system activity by blocking both á- and b-adrenergic receptors, leading to vasodilation and a decrease in heart rate, which can cause severe orthostatic hypotension. Heart palpitations, dry mouth, dehydration, and headaches are possible side effects of other antihypertensives

2. Which nursing intervention will be most effective when assisting the patient with coronary artery disease (CAD) to make appropriate dietary changes? a. Give the patient a list of low-sodium, low-cholesterol foods that should be included in the diet. b. Emphasize the increased risk for heart problems unless the patient makes the dietary changes. c. Help the patient modify favorite high-fat recipes by using monosaturated oils when possible. d. Inform the patient that a diet containing no saturated fat and minimal salt will be necessary.

ANS: C Lifestyle changes are more likely to be successful when consideration is given to the patient's values and preferences. The highest percentage of calories from fat should come from monosaturated fats. Although low-sodium and low-cholesterol foods are appropriate, providing the patient with a list alone is not likely to be successful in making dietary changes. Completely removing saturated fat from the diet is not a realistic expectation. Up to 7% of calories in the therapeutic lifestyle changes (TLC) diet can come from saturated fat. Telling the patient about the increased risk without assisting further with strategies for dietary change is unlikely to be successful.

The nurse observes a student who is listening to a patient's lungs who is having no problems with breathing. Which action by the student indicates a need to review respiratory assessment skills? a. The student starts at the apices of the lungs and moves to the bases. b. The student compares breath sounds from side to side avoiding bony areas. c. The student places the stethoscope over the posterior chest and listens during inspiration. d. The student instructs the patient to breathe slowly and a little more deeply than normal through the mouth.

ANS: C Listening only during inspiration indicates the student needs a review of respiratory assessment skills. At each placement of the stethoscope, listen to at least one cycle of inspiration and expiration. During chest auscultation, instruct the patient to breathe slowly and a little deeper than normal through the mouth. Auscultation should proceed from the lung apices to the bases, comparing opposite areas of the chest, unless the patient is in respiratory distress or will tire easily. If so, start at the bases (see Fig. 26-7). Place the stethoscope over lung tissue, not over bony prominences.

A 54-year-old woman with acute myelogenous leukemia (AML) is considering treatment with a hematopoietic stem cell transplant (HSCT). The best approach for the nurse to assist the patient with a treatment decision is to a. emphasize the positive outcomes of a bone marrow transplant. b. discuss the need for adequate insurance to cover post-HSCT care. c. ask the patient whether there are any questions or concerns about HSCT. d. explain that a cure is not possible with any other treatment except HSCT.

ANS: C Offering the patient an opportunity to ask questions or discuss concerns about HSCT will encourage the patient to voice concerns about this treatment and also will allow the nurse to assess whether the patient needs more information about the procedure. Treatment of AML using chemotherapy is another option for the patient. It is not appropriate for the nurse to ask the patient to consider insurance needs in making this decision.

Which finding about a patient with polycythemia vera is most important for the nurse to report to the health care provider? a. Hematocrit 55% b. Presence of plethora c. Calf swelling and pain d. Platelet count 450,000/μL

ANS: C The calf swelling and pain suggest that the patient may have developed a deep vein thrombosis, which will require diagnosis and treatment to avoid complications such as pulmonary embolus. The other findings will also be reported to the health care provider but are expected in a patient with this diagnosis.

16. During the administration of the thrombolytic agent to a patient with an acute myocardial infarction (AMI), the nurse should stop the drug infusion if the patient experiences a. bleeding from the gums. b. increase in blood pressure. c. a decrease in level of consciousness. d. a nonsustained episode of ventricular tachycardia.

ANS: C The change in level of consciousness indicates that the patient may be experiencing intracranial bleeding, a possible complication of thrombolytic therapy. Some bleeding of the gums is an expected side effect of the therapy but not an indication to stop infusion of the thrombolytic medication. A decrease in blood pressure could indicate internal bleeding. A nonsustained episode of ventricular tachycardia is a common reperfusion dysrhythmia and may indicate that the therapy is effective.

35. The nurse obtains the following data when assessing a patient who experienced an ST-segment-elevation myocardial infarction (STEMI) 2 days previously. Which information is most important to report to the health care provider? a. The troponin level is elevated. b. The patient denies ever having a heart attack. c. Bilateral crackles are auscultated in the mid-lower lobes. d. The patient has occasional premature atrial contractions (PACs).

ANS: C The crackles indicate that the patient may be developing heart failure, a possible complication of myocardial infarction (MI). The health care provider may need to order medications such as diuretics or angiotensin-converting enzyme (ACE) inhibitors for the patient. Elevation in troponin level at this time is expected. PACs are not life-threatening dysrhythmias. Denial is a common response in the immediate period after the MI.

While caring for a patient with respiratory disease, the nurse observes that the patient's SpO2 drops from 93% to 88% while the patient is ambulating in the hallway. What is the priority action of the nurse? a. Notify the health care provider. b. Document the response to exercise. c. Administer the PRN supplemental O2. d. Encourage the patient to pace activity.

ANS: C The drop in SpO2 to 85% indicates that the patient is hypoxemic and needs supplemental oxygen when exercising. The other actions are also important, but the first action should be to correct the hypoxemia

5. After the nurse has finished teaching a patient about the use of sublingual nitroglycerin (Nitrostat), which patient statement indicates that the teaching has been effective? a. "I can expect some nausea as a side effect of nitroglycerin." b. "I should only take the nitroglycerin if I start to have chest pain." c. "I will call an ambulance if I still have pain after taking 3 nitroglycerin 5 minutes apart." d. "Nitroglycerin helps prevent a clot from forming and blocking blood flow to my heart."

ANS: C The emergency medical services (EMS) system should be activated when chest pain or other symptoms are not completely relieved after 3 sublingual nitroglycerin tablets taken 5 minutes apart. Nitroglycerin can be taken to prevent chest pain or other symptoms from developing (e.g., before intercourse). Gastric upset (e.g., nausea) is not an expected side effect of nitroglycerin. Nitroglycerin does not impact the underlying pathophysiology of coronary artery atherosclerosis.

Which action will the nurse include in the plan of care for a patient who has thalassemia major? a. Teach the patient to use iron supplements. b. Avoid the use of intramuscular injections. c. Administer iron chelation therapy as needed. d. Notify health care provider of hemoglobin 11g/dL.

ANS: C The frequent transfusions used to treat thalassemia major lead to iron toxicity in patients unless iron chelation therapy is consistently used. Iron supplementation is avoided in patients with thalassemia. There is no need to avoid intramuscular injections. The goal for patients with thalassemia major is to maintain a hemoglobin of 10 g/dL or greater.

During change-of-shift report, the nurse obtains the following information about a hypertensive patient who received the first dose of nadolol (Corgard) during the previous shift. Which information indicates that the patient needs immediate intervention? a. The patient's most recent blood pressure (BP) reading is 158/91 mm Hg. b. The patient's pulse has dropped from 68 to 57 beats/minute. c. The patient has developed wheezes throughout the lung fields. d. The patient complains that the fingers and toes feel quite cold.

ANS: C The most urgent concern for this patient is the wheezes, which indicate that bronchospasm (a common adverse effect of the noncardioselective b-blockers) is occurring. The nurse should immediately obtain an oxygen saturation measurement, apply supplemental oxygen, and notify the health care provider. The mild decrease in heart rate and complaint of cold fingers and toes are associated with b-receptor blockade but do not require any change in therapy. The BP reading may indicate that a change in medication type or dose may be indicated. However, this is not as urgently needed as addressing the bronchospasm

42. After reviewing information shown in the accompanying figure from the medical records of a 43-year-old, which risk factor modification for coronary artery disease should the nurse include in patient teaching? a. Importance of daily physical activity b. Effect of weight loss on blood pressure c. Dietary changes to improve lipid levels d. Ongoing cardiac risk associated with history of tobacco use

ANS: C The patient has an elevated low-density lipoprotein (LDL) cholesterol and low high-density lipoprotein (HDL) cholesterol, which will increase the risk of coronary artery disease. Although the blood pressure is in the prehypertensive range, the patient's waist circumference and body mass index (BMI) indicate an appropriate body weight. The risk for coronary artery disease a year after quitting smoking is the same as a nonsmoker. The patient's occupation indicates that daily activity is at the levels suggested by national guidelines.

30. Which electrocardiographic (ECG) change is most important for the nurse to report to the health care provider when caring for a patient with chest pain? a. Inverted P wave b. Sinus tachycardia c. ST-segment elevation d. First-degree atrioventricular block

ANS: C The patient is likely to be experiencing an ST-segment-elevation myocardial infarction (STEMI). Immediate therapy with percutaneous coronary intervention (PCI) or thrombolytic medication is indicated to minimize myocardial damage. The other ECG changes may also suggest a need for therapy, but not as rapidly.

Following successful treatment of Hodgkin's lymphoma for a 55-year-old woman, which topic will the nurse include in patient teaching? a. Potential impact of chemotherapy treatment on fertility b. Application of soothing lotions to treat residual pruritus c. Use of maintenance chemotherapy to maintain remission d. Need for follow-up appointments to screen for malignancy

ANS: D The chemotherapy used in treating Hodgkin's lymphoma results in a high incidence of secondary malignancies; follow-up screening is needed. The fertility of a 55-year-old woman will not be impacted by chemotherapy. Maintenance chemotherapy is not used for Hodgkin's lymphoma. Pruritus is a clinical manifestation of lymphoma, but should not be a concern after treatment.

18. In preparation for discharge, the nurse teaches a patient with chronic stable angina how to use the prescribed short-acting and long-acting nitrates. Which patient statement indicates that the teaching has been effective? a. "I will check my pulse rate before I take any nitroglycerin tablets." b. "I will put the nitroglycerin patch on as soon as I get any chest pain." c. "I will stop what I am doing and sit down before I put the nitroglycerin under my tongue." d. "I will be sure to remove the nitroglycerin patch before taking any sublingual nitroglycerin."

ANS: C The patient should sit down before taking the nitroglycerin to decrease cardiac workload and prevent orthostatic hypotension. Transdermal nitrates are used prophylactically rather than to treat acute pain and can be used concurrently with sublingual nitroglycerin. Although the nurse should check blood pressure before giving nitroglycerin, patients do not need to check the pulse rate before taking nitrates.

A patient with possible disseminated intravascular coagulation arrives in the emergency department with a blood pressure of 82/40, temperature 102° F (38.9° C), and severe back pain. Which physician order will the nurse implement first? a. Administer morphine sulfate 4 mg IV. b. Give acetaminophen (Tylenol) 650 mg. c. Infuse normal saline 500 mL over 30 minutes. d. Schedule complete blood count and coagulation studies.

ANS: C The patient's blood pressure indicates hypovolemia caused by blood loss and should be addressed immediately to improve perfusion to vital organs. The other actions also are appropriate and should be rapidly implemented, but improving perfusion is the priority for this patient.

A postoperative patient receiving a transfusion of packed red blood cells develops chills, fever, headache, and anxiety 35 minutes after the transfusion is started. After stopping the transfusion, what action should the nurse take? a. Draw blood for a new crossmatch. b. Send a urine specimen to the laboratory. c. Administer PRN acetaminophen (Tylenol). d. Give the PRN diphenhydramine (Benadryl).

ANS: C The patient's clinical manifestations are consistent with a febrile, nonhemolytic transfusion reaction. The transfusion should be stopped and antipyretics administered for the fever as ordered. A urine specimen is needed if an acute hemolytic reaction is suspected. Diphenhydramine (Benadryl) is used for allergic reactions. This type of reaction does not indicate incorrect crossmatching.

17. A patient is recovering from a myocardial infarction (MI) and develops chest pain on day 3 that increases when taking a deep breath and is relieved by leaning forward. Which action should the nurse take next? a. Assess the feet for pedal edema. b. Palpate the radial pulses bilaterally. c. Auscultate for a pericardial friction rub. d. Check the heart monitor for dysrhythmias.

ANS: C The patient's symptoms are consistent with the development of pericarditis, a possible complication of MI. The other assessments listed are not consistent with the description of the patient's symptoms.

37. A patient who has chest pain is admitted to the emergency department (ED) and all of the following are ordered. Which one should the nurse arrange to be completed first? a. Chest x-ray b. Troponin level c. Electrocardiogram (ECG) d. Insertion of a peripheral IV

ANS: C The priority for the patient is to determine whether an acute myocardial infarction (AMI) is occurring so that reperfusion therapy can begin as quickly as possible. ECG changes occur very rapidly after coronary artery occlusion, and an ECG should be obtained as soon as possible. Troponin levels will increase after about 3 hours. Data from the chest x-ray may impact the patient's care but are not helpful in determining whether the patient is experiencing a myocardial infarction (MI). Peripheral access will be needed but not before the ECG.

41. A patient with diabetes mellitus and chronic stable angina has a new order for captopril (Capoten). The nurse should teach the patient that the primary purpose of captopril is to a. lower heart rate. b. control blood glucose levels. c. prevent changes in heart muscle. d. reduce the frequency of chest pain.

ANS: C The purpose for angiotensin-converting enzyme (ACE) inhibitors in patients with chronic stable angina who are at high risk for a cardiac event is to decrease ventricular remodeling. ACE inhibitors do not directly impact angina frequency, blood glucose, or heart rate.

14. A patient with ST-segment elevation in three contiguous electrocardiographic (ECG) leads is admitted to the emergency department (ED) and diagnosed as having an ST-segment-elevation myocardial infarction (STEMI). Which question should the nurse ask to determine whether the patient is a candidate for thrombolytic therapy? a. "Do you have any allergies?" b. "Do you take aspirin on a daily basis?" c. "What time did your chest pain begin?" d. "Can you rate your chest pain using a 0 to 10 scale?"

ANS: C Thrombolytic therapy should be started within 6 hours of the onset of the myocardial infarction (MI), so the time at which the chest pain started is a major determinant of the appropriateness of this treatment. The other information will also be needed, but it will not be a factor in the decision about thrombolytic therapy.

A 56-year-old patient who has no previous history of hypertension or other health problems suddenly develops a blood pressure (BP) of 198/110 mm Hg. After reconfirming the BP, it is appropriate for the nurse to tell the patient that a. a BP recheck should be scheduled in a few weeks. b. dietary sodium and fat content should be decreased. c. there is an immediate danger of a stroke and hospitalization will be required. d. diagnosis of a possible cause, treatment, and ongoing monitoring will be needed.

ANS: D A sudden increase in BP in a patient over age 50 with no previous hypertension history or risk factors indicates that the hypertension may be secondary to some other problem. The BP will need treatment and ongoing monitoring. If the patient has no other risk factors, a stroke in the immediate future is unlikely. There is no indication that dietary salt or fat intake have contributed to this sudden increase in BP, and reducing intake of salt and fat alone will not be adequate to reduce this BP to an acceptable level

A patient who has been receiving a heparin infusion and warfarin (Coumadin) for a deep vein thrombosis (DVT) is diagnosed with heparin-induced thrombocytopenia (HIT) when her platelet level drops to 110,000/µL. Which action will the nurse include in the plan of care? a. Use low-molecular-weight heparin (LMWH) only. b. Administer the warfarin (Coumadin) at the scheduled time. c. Teach the patient about the purpose of platelet transfusions. d. Discontinue heparin and flush intermittent IV lines using normal saline.

ANS: D All heparin is discontinued when the HIT is diagnosed. The patient should be instructed to never receive heparin or LMWH. Warfarin is usually not given until the platelet count has returned to 150,000/µL. The platelet count does not drop low enough in HIT for a platelet transfusion, and platelet transfusions increase the risk for thrombosis.

The nurse analyzes the results of a patient's arterial blood gases (ABGs). Which finding would require immediate action? a. The bicarbonate level (HCO3-) is 31 mEq/L. b. The arterial oxygen saturation (SaO2) is 92%. c. The partial pressure of CO2 in arterial blood (PaCO2) is 31 mm Hg. d. The partial pressure of oxygen in arterial blood (PaO2) is 59 mm Hg.

ANS: D All the values are abnormal, but the low PaO2 indicates that the patient is at the point on the oxyhemoglobin dissociation curve where a small change in the PaO2 will cause a large drop in the O2 saturation and a decrease in tissue oxygenation. The nurse should intervene immediately to improve the patient's oxygenation.

6. Which statement made by a patient with coronary artery disease after the nurse has completed teaching about therapeutic lifestyle changes (TLC) diet indicates that further teaching is needed? a. "I will switch from whole milk to 1% milk." b. "I like salmon and I will plan to eat it more often." c. "I can have a glass of wine with dinner if I want one." d. "I will miss being able to eat peanut butter sandwiches."

ANS: D Although only 30% of the daily calories should come from fats, most of the fat in the TLC diet should come from monosaturated fats such as are found in nuts, olive oil, and canola oil. The patient can include peanut butter sandwiches as part of the TLC diet. The other patient comments indicate a good understanding of the TLC diet.

Which statement by a patient indicates good understanding of the nurse's teaching about prevention of sickle cell crisis? a. "Home oxygen therapy is frequently used to decrease sickling." b. "There are no effective medications that can help prevent sickling." c. "Routine continuous dosage narcotics are prescribed to prevent a crisis." d. "Risk for a crisis is decreased by having an annual influenza vaccination."

ANS: D Because infection is the most common cause of a sickle cell crisis, influenza, Haemophilus influenzae, pneumococcal pneumonia, and hepatitis immunizations should be administered. Although continuous dose opioids and oxygen may be administered during a crisis, patients do not receive these therapies to prevent crisis. Hydroxyurea (Hydrea) is a medication used to decrease the number of sickle cell crises.

Which patient information is most important for the nurse to monitor when evaluating the effectiveness of deferoxamine (Desferal) for a patient with hemochromatosis? a. Skin color b. Hematocrit c. Liver function d. Serum iron level

ANS: D Because iron chelating agents are used to lower serum iron levels, the most useful information will be the patient's iron level. The other parameters will also be monitored, but are not the most important to monitor when determining the effectiveness of deferoxamine.

11. Nadolol (Corgard) is prescribed for a patient with chronic stable angina and left ventricular dysfunction. To determine whether the drug is effective, the nurse will monitor for a. decreased blood pressure and heart rate. b. fewer complaints of having cold hands and feet. c. improvement in the strength of the distal pulses. d. the ability to do daily activities without chest pain.

ANS: D Because the medication is ordered to improve the patient's angina, effectiveness is indicated if the patient is able to accomplish daily activities without chest pain. Blood pressure and heart rate may decrease, but these data do not indicate that the goal of decreased angina has been met. The noncardioselective β-adrenergic blockers can cause peripheral vasoconstriction, so the nurse would not expect an improvement in distal pulse quality or skin temperature.

The nurse is caring for a 70-year-old who uses hydrochlorothiazide (HydroDIURIL) and enalapril (Norvasc), but whose self-monitored blood pressure (BP) continues to be elevated. Which patient information may indicate a need for a change? a. Patient takes a daily multivitamin tablet. b. Patient checks BP daily just after getting up. c. Patient drinks wine three to four times a week. d. Patient uses ibuprofen (Motrin) daily to treat osteoarthritis.

ANS: D Because use of nonsteroidal antiinflammatory drugs (NSAIDs) can prevent adequate BP control, the patient may need to avoid the use of ibuprofen. A multivitamin tablet will help supply vitamin D, which may help lower BP. BP decreases while sleeping, so self-monitoring early in the morning will result in obtaining pressures that are at their lowest. The patient's alcohol intake is not excessive

4. Which information given by a patient admitted with chronic stable angina will help the nurse confirm this diagnosis? a. The patient states that the pain "wakes me up at night." b. The patient rates the pain at a level 3 to 5 (0 to 10 scale). c. The patient states that the pain has increased in frequency over the last week. d. The patient states that the pain "goes away" with one sublingual nitroglycerin tablet.

ANS: D Chronic stable angina is typically relieved by rest or nitroglycerin administration. The level of pain is not a consistent indicator of the type of angina. Pain occurring at rest or with increased frequency is typical of unstable angina.

When assessing the respiratory system of an older patient, which finding indicates that the nurse should take immediate action? a. Weak cough effort b. Barrel-shaped chest c. Dry mucous membranes d. Bilateral crackles at lung bases

ANS: D Crackles in the lower half of the lungs indicate that the patient may have an acute problem such as heart failure. The nurse should immediately accomplish further assessments, such as oxygen saturation, and notify the health care provider. A barrel-shaped chest, hyperresonance to percussion, and a weak cough effort are associated with aging. Further evaluation may be needed, but immediate action is not indicated. An older patient has a less forceful cough and fewer and less functional cilia. Mucous membranes tend to be drier.

Which laboratory test will the nurse use to determine whether filgrastim (Neupogen) is effective for a patient with acute lymphocytic leukemia who is receiving chemotherapy? a. Platelet count b. Reticulocyte count c. Total lymphocyte count d. Absolute neutrophil count

ANS: D Filgrastim increases the neutrophil count and function in neutropenic patients. Although total lymphocyte, platelet, and reticulocyte counts also are important to monitor in this patient, the absolute neutrophil count is used to evaluate the effects of filgrastim.

After the nurse teaches the patient with stage 1 hypertension about diet modifications that should be implemented, which diet choice indicates that the teaching has been effective? a. The patient avoids eating nuts or nut butters. b. The patient restricts intake of chicken and fish. c. The patient has two cups of coffee in the morning. d. The patient has a glass of low-fat milk with each meal.

ANS: D For the prevention of hypertension, the Dietary Approaches to Stop Hypertension (DASH) recommendations include increasing the intake of calcium-rich foods. Caffeine restriction and decreased protein intake are not included in the recommendations. Nuts are high in beneficial nutrients and 4 to 5 servings weekly are recommended in the DASH diet.

Which action will be included in the plan of care when the nurse is caring for a patient who is receiving nicardipine (Cardene) to treat a hypertensive emergency? a. Keep the patient NPO to prevent aspiration caused by nausea and possible vomiting. b. Organize nursing activities so that the patient has undisturbed sleep for 6 to 8 hours at night. c. Assist the patient up in the chair for meals to avoid complications associated with immobility. d. Use an automated noninvasive blood pressure machine to obtain frequent blood pressure (BP) measurements.

ANS: D Frequent monitoring of BP is needed when the patient is receiving rapid-acting IV antihypertensive medications. This can be most easily accomplished with an automated BP machine or arterial line. The patient will require frequent assessments, so allowing 6 to 8 hours of undisturbed sleep is not appropriate. When patients are receiving IV vasodilators, bed rest is maintained to prevent decreased cerebral perfusion and fainting. There is no indication that this patient is nauseated or at risk for aspiration, so an NPO status is unnecessary

a, f

Budesonide inhaler is associated with A. Long-term control B. Quick-relief C. B2-adrenergic agonist D. Mast-cell stabilizer E. Leukotriene inhibitor F. Steroid antiinflammatory G. Methylxanthine bronchodilator H. Anticholinergic I. Anti-IgE

Which information should the nurse include when teaching a patient with newly diagnosed hypertension? a. Increasing physical activity will control blood pressure (BP) for most patients. b. Most patients are able to control BP through dietary changes. c. Annual BP checks are needed to monitor treatment effectiveness. d. Hypertension is usually asymptomatic until target organ damage occurs.

ANS: D Hypertension is usually asymptomatic until target organ damage has occurred. Lifestyle changes (e.g., physical activity, dietary changes) are used to help manage blood pressure, but drugs are needed for most patients. Home BP monitoring should be taught to the patient and findings checked by the health care provider frequently when starting treatment for hypertension and then every 3 months once stable

A patient who has a history of chronic obstructive pulmonary disease (COPD) was hospitalized for increasing shortness of breath and chronic hypoxemia (SaO2 levels of 89% to 90%). In planning for discharge, which action by the nurse will be most effective in improving compliance with discharge teaching? a. Start giving the patient discharge teaching on the day of admission. b. Have the patient repeat the instructions immediately after teaching. c. Accomplish the patient teaching just before the scheduled discharge. d. Arrange for the patient's caregiver to be present during the teaching.

ANS: D Hypoxemia interferes with the patient's ability to learn and retain information, so having the patient's caregiver present will increase the likelihood that discharge instructions will be followed. Having the patient repeat the instructions will indicate that the information is understood at the time, but it does not guarantee retention of the information. Because the patient is likely to be distracted just before discharge, giving discharge instructions just before discharge is not ideal. The patient is likely to be anxious and even more hypoxemic than usual on the day of admission, so teaching about discharge should be postponed.

36. A patient had a non-ST-segment-elevation myocardial infarction (NSTEMI) 3 days ago. Which nursing intervention included in the plan of care is most appropriate for the registered nurse (RN) to delegate to an experienced licensed practical/vocational nurse (LPN/LVN)? a. Evaluation of the patient's response to walking in the hallway b. Completion of the referral form for a home health nurse follow-up c. Education of the patient about the pathophysiology of heart disease d. Reinforcement of teaching about the purpose of prescribed medications

ANS: D LPN/LVN education and scope of practice include reinforcing education that has previously been done by the RN. Evaluating the patient response to exercise after a NSTEMI requires more education and should be done by the RN. Teaching and discharge planning/ documentation are higher level skills that require RN education and scope of practice.

A patient who has non-Hodgkin's lymphoma is receiving combination treatment with rituximab (Rituxan) and chemotherapy. Which patient assessment finding requires the most rapid action by the nurse? a. Anorexia b. Vomiting c. Oral ulcers d. Lip swelling

ANS: D Lip swelling in angioedema may indicate a hypersensitivity reaction to the rituximab. The nurse should stop the infusion and further assess for anaphylaxis. The other findings may occur with chemotherapy, but are not immediately life threatening.

A critical action by the nurse caring for a patient with an acute exacerbation of polycythemia vera is to a. place the patient on bed rest. b. administer iron supplements. c. avoid use of aspirin products. d. monitor fluid intake and output.

ANS: D Monitoring hydration status is important during an acute exacerbation because the patient is at risk for fluid overload or underhydration. Aspirin therapy is used to decrease risk for thrombosis. The patient should be encouraged to ambulate to prevent deep vein thrombosis (DVT). Iron is contraindicated in patients with polycythemia vera.

Which laboratory result will the nurse expect to show a decreased value if a patient develops heparin-induced thrombocytopenia (HIT)? a. Prothrombin time b. Erythrocyte count c. Fibrinogen degradation products d. Activated partial thromboplastin time

ANS: D Platelet aggregation in HIT causes neutralization of heparin, so that the activated partial thromboplastin time will be shorter and more heparin will be needed to maintain therapeutic levels. The other data will not be affected by HIT.

43. After reviewing a patient's history, vital signs, physical assessment, and laboratory data, which information shown in the accompanying figure is most important for the nurse to communicate to the health care provider? a. Q waves on ECG b. Elevated troponin levels c. Fever and hyperglycemia d. Tachypnea and crackles in lungs

ANS: D Pulmonary congestion and tachypnea suggest that the patient may be developing heart failure, a complication of myocardial infarction (MI). Mild fever and hyperglycemia are common after MI because of the inflammatory process that occurs with tissue necrosis. Troponin levels will be elevated for several days after MI. Q waves often develop with ST-segment-elevation MI.

A patient has just been diagnosed with hypertension and has been started on captopril (Capoten). Which information is important to include when teaching the patient about this medication? a. Check blood pressure (BP) in both arms before taking the medication. b. Increase fluid intake if dryness of the mouth is a problem. c. Include high-potassium foods such as bananas in the diet. d. Change position slowly to help prevent dizziness and falls.

ANS: D The angiotensin-converting enzyme (ACE) inhibitors frequently cause orthostatic hypotension, and patients should be taught to change position slowly to allow the vascular system time to compensate for the position change. Increasing fluid intake may counteract the effect of the medication, and the patient is taught to use gum or hard candy to relieve dry mouth. The BP should be taken in the nondominant arm by newly diagnosed patients in the morning, before taking the medication, and in the evening. Because ACE inhibitors cause potassium retention, increased intake of high-potassium foods is inappropriate

24. A patient with hyperlipidemia has a new order for colesevelam (Welchol). Which nursing action is most appropriate when giving the medication? a. Have the patient take this medication with an aspirin. b. Administer the medication at the patient's usual bedtime. c. Have the patient take the colesevelam with a sip of water. d. Give the patient's other medications 2 hours after the colesevelam.

ANS: D The bile acid sequestrants interfere with the absorption of many other drugs, and giving other medications at the same time should be avoided. Taking an aspirin concurrently with the colesevelam may increase the incidence of gastrointestinal side effects such as heartburn. An increased fluid intake is encouraged for patients taking the bile acid sequestrants to reduce the risk for constipation. For maximum effect, colesevelam should be administered with meals.

The nurse admits a patient who has a diagnosis of an acute asthma attack. Which statement indicates that the patient may need teaching regarding medication use? a. "I have not had any acute asthma attacks during the last year." b. "I became short of breath an hour before coming to the hospital." c. "I've been taking Tylenol 650 mg every 6 hours for chest-wall pain." d. "I've been using my albuterol inhaler more frequently over the last 4 days."

ANS: D The increased need for a rapid-acting bronchodilator should alert the patient that an acute attack may be imminent and that a change in therapy may be needed. The patient should be taught to contact a health care provider if this occurs. The other data do not indicate any need for additional teaching.

31. When caring for a patient with acute coronary syndrome who has returned to the coronary care unit after having angioplasty with stent placement, the nurse obtains the following assessment data. Which data indicate the need for immediate action by the nurse? a. Heart rate 102 beats/min b. Pedal pulses 1+ bilaterally c. Blood pressure 103/54 mm Hg d. Chest pain level 7 on a 0 to 10 point scale

ANS: D The patient's chest pain indicates that restenosis of the coronary artery may be occurring and requires immediate actions, such as administration of oxygen and nitroglycerin, by the nurse. The other information indicates a need for ongoing assessments by the nurse.

A 62-year old man with chronic anemia is experiencing increased fatigue and occasional palpitations at rest. The nurse would expect the patient's laboratory findings to include a. a hematocrit (Hct) of 38%. b. an RBC count of 4,500,000/μL. c. normal red blood cell (RBC) indices. d. a hemoglobin (Hgb) of 8.6 g/dL (86 g/L).

ANS: D The patient's clinical manifestations indicate moderate anemia, which is consistent with a Hgb of 6 to 10 g/dL. The other values are all within the range of normal.

A patient in the emergency department complains of back pain and difficulty breathing 15 minutes after a transfusion of packed red blood cells is started. The nurse's first action should be to a. administer oxygen therapy at a high flow rate. b. obtain a urine specimen to send to the laboratory. c. notify the health care provider about the symptoms. d. disconnect the transfusion and infuse normal saline.

ANS: D The patient's symptoms indicate a possible acute hemolytic reaction caused by the transfusion. The first action should be to disconnect the transfusion and infuse normal saline. The other actions also are needed but are not the highest priority.

Using the illustrated technique, the nurse is assessing for which finding in a patient with chronic obstructive pulmonary disease (COPD)? a. Hyperresonance b. Tripod positioning c. Accessory muscle use d. Reduced chest expansion

ANS: D The technique for palpation for chest expansion is shown in the illustrated technique. Reduced chest movement would be noted on palpation of a patient's chest with COPD. Hyperresonance would be assessed through percussion. Accessory muscle use and tripod positioning would be assessed by inspection.

The nurse prepares a patient with a left-sided pleural effusion for a thoracentesis. How should the nurse position the patient? a. Supine with the head of the bed elevated 30 degrees b. In a high-Fowler's position with the left arm extended c. On the right side with the left arm extended above the head d. Sitting upright with the arms supported on an over bed table

ANS: D The upright position with the arms supported increases lung expansion, allows fluid to collect at the lung bases, and expands the intercostal space so that access to the pleural space is easier. The other positions would increase the work of breathing for the patient and make it more difficult for the health care provider performing the thoracentesis.

The laboratory has just called with the arterial blood gas (ABG) results on four patients. Which result is most important for the nurse to report immediately to the health care provider? a. pH 7.34, PaO2 82 mm Hg, PaCO2 40 mm Hg, and O2 sat 97% b. pH 7.35, PaO2 85 mm Hg, PaCO2 45 mm Hg, and O2 sat 95% c. pH 7.46, PaO2 90 mm Hg, PaCO2 32 mm Hg, and O2 sat 98% d. pH 7.31, PaO2 91 mm Hg, PaCO2 50 mm Hg, and O2 sat 96%

ANS: D These ABGs indicate uncompensated respiratory acidosis and should be reported to the health care provider. The other values are normal or close to normal.

38. After receiving change-of-shift report about the following four patients, which patient should the nurse assess first? a. 39-year-old with pericarditis who is complaining of sharp, stabbing chest pain b. 56-year-old with variant angina who is to receive a dose of nifedipine (Procardia) c. 65-year-old who had a myocardial infarction (MI) 4 days ago and is anxious about the planned discharge d. 59-year-old with unstable angina who has just returned to the unit after having a percutaneous coronary intervention (PCI)

ANS: D This patient is at risk for bleeding from the arterial access site for the PCI, so the nurse should assess the patient's blood pressure, pulse, and the access site immediately. The other patients should also be assessed as quickly as possible, but assessment of this patient has the highest priority.

The nurse palpates the posterior chest while the patient says "99" and notes absent fremitus. Which action should the nurse take next? a. Palpate the anterior chest and observe for barrel chest. b. Encourage the patient to turn, cough, and deep breathe. c. Review the chest x-ray report for evidence of pneumonia. d. Auscultate anterior and posterior breath sounds bilaterally.

ANS: D To assess for tactile fremitus, the nurse should use the palms of the hands to assess for vibration when the patient repeats a word or phrase such as "99." After noting absent fremitus, the nurse should then auscultate the lungs to assess for the presence or absence of breath sounds. Absent fremitus may be noted with pneumothorax or atelectasis. The vibration is increased in conditions such as pneumonia, lung tumors, thick bronchial secretions, and pleural effusion. Turning, coughing, and deep breathing is an appropriate intervention for atelectasis, but the nurse needs to first assess breath sounds. Fremitus is decreased if the hand is farther from the lung or the lung is hyperinflated (barrel chest).The anterior of the chest is more difficult to palpate for fremitus because of the presence of large muscles and breast tissue.

8. A patient who has had chest pain for several hours is admitted with a diagnosis of rule out acute myocardial infarction (AMI). Which laboratory test should the nurse monitor to help determine whether the patient has had an AMI? a. Myoglobin b. Homocysteine c. C-reactive protein d. Cardiac-specific troponin

ANS: D Troponin levels increase about 4 to 6 hours after the onset of myocardial infarction (MI) and are highly specific indicators for MI. Myoglobin is released within 2 hours of MI, but it lacks specificity and its use is limited. The other laboratory data are useful in determining the patient's risk for developing coronary artery disease (CAD) but are not helpful in determining whether an acute MI is in progress.

a (The first action should be to determine whether the patient has been compliant with drug therapy because negative sputum smears would be expected if the TB bacillus is susceptible to the medications and if the medications have been taken correctly. Depending on whether the patient has been compliant or not, different medications or directly observed therapy may be indicated. A two-drug regimen will be used only if the sputum smears are negative for AFB.)

After 2 months of tuberculosis (TB) treatment with a standard four-drug regimen, a patient continues to have positive sputum smears for acid-fast bacilli (AFB). Which action should the nurse take next? A. Ask the patient whether medications have been taken as directed. B. Discuss the need to use some different medications to treat the TB. C. Schedule the patient for directly observed therapy three times weekly. D. Educate about using a 2-drug regimen for the last 4 months of treatment.

c (Ice application for the first 24 hours after a fracture will help reduce swelling and can be placed over the cast. Plaster casts should not get wet. The patient should be encouraged to move the joints above and below the cast. Patients should not insert objects inside the cast.)

After a patient has a short-arm plaster cast applied in the emergency department, which statement by the patient indicates a good understanding of the nurse's discharge teaching? A. "I can get the cast wet as long as I dry it right away with a hair dryer." B. "I should avoid moving my fingers and elbow until the cast is removed." C. "I will apply an ice pack to the cast over the fracture site for the next 24 hours." D. "I can use a cotton-tipped applicator to rub lotion on any dry areas under the cast."

c (Assessment of bowel tones, abdominal pain, and nausea and vomiting will detect the development of cast syndrome. To avoid breakage, the support bar should not be used for repositioning. After the cast dries, the patient can begin ambulating with the assistance of physical therapy personnel and may be turned to the prone position.)

After a patient with a left femur fracture has a hip spica cast applied, which nursing intervention will be included in the plan of care? A. Avoid placing the patient in the prone position. B. Use the cast support bar to reposition the patient. C. Ask the patient about any abdominal discomfort or nausea. D. Discuss the reasons for remaining on bed rest for several weeks.

c (The normal WBC count indicates that the antibiotics have been effective. All the other data suggest that a change in treatment is needed.)

After a patient with right lower-lobe pneumonia has been treated with intravenous (IV) antibiotics for 2 days, which assessment data obtained by the nurse indicates that the treatment has been effective? A. Bronchial breath sounds are heard at the right base. B. The patient coughs up small amounts of green mucus. C. The patient's white blood cell (WBC) count is 9000/µl. D. Increased tactile fremitus is palpable over the right chest.

a (Phantom limb pain sensation is a real experience with no know cause or cure. The pain must be acknowledged and interventions to relieve the discomfort explored)

After an amputation of a limb, a client begins to experience extreme discomfort in the area where the limb once was. The nurse's greatest concern at this time is: A. Addressing the pain B. Reversing feelings of hopelessness C. Promoting mobility in the residual limb D. Acknowledge the grieving for the lost limb

b (Methotrexate is teratogenic, and the patient should be taking contraceptives during methotrexate therapy. The other information will not impact the choice of methotrexate as therapy.)

After obtaining the health history from a 28-year-old woman who is taking methotrexate (Rheumatrex) to treat rheumatoid arthritis, which information about the patient is most important for the nurse to report to the health care provider? A. The patient had a history of infectious mononucleosis as a teenager. B. The patient is trying to have a baby before her disease becomes more severe. C. The patient has a family history of age-related macular degeneration of the retina. D. The patient has been using large doses of vitamins and health foods to treat the RA.

b (Rationale: Patients should avoid temperature extremes by exercising indoors when it is cold. To avoid burn injuries, the patient should use warm, rather than hot, water to warm the hands. Pseudoephedrine is a vasoconstrictor and should be avoided. There is no reason to avoid taking aspirin and NSAIDs with Raynaud's phenomenon.)

After teaching a patient with newly diagnosed Raynaud's phenomenon about how to manage the condition, which behavior by the patient indicates that the teaching has been effective? A. The patient avoids the use of aspirin and nonsteroidal anti-inflammatory drugs (NSAIDs). B. The patient exercises indoors during the winter months. C. The patient places the hands in hot water when they turn pale. D. The patient takes pseudoephedrine (Sudafed) for cold symptoms.

c (The initial nursing action should be to assess the patient's knowledge level and feelings about the options available. Discussion about the patient's option to not have the procedure, the seriousness of the condition, or rehabilitation after the procedure may be appropriate after the nurse knows more about the patient's current level of knowledge and emotional state.)

After the health care provider has recommended an amputation for a patient who has ischemic foot ulcers, the patient tells the nurse, "If they want to cut off my foot, they should just shoot me instead." Which response by the nurse is best? A. "Many people are able to function normally with a foot prosthesis." B. "I understand that you are upset, but you may lose the foot anyway." C. "Tell me what you know about what your options for treatment are." D. "If you do not want the surgery, you do not have to have an amputation."

c (A sputum specimen for Gram stain and culture should be done before initiating antibiotic therapy in a hospitalized patient with suspected pneumonia, and then antibiotics should be started without delay. Chest x-rays and blood cell tests will not be altered significantly by delaying the tests until after the first dose of antibiotics.)

After the health care provider sees a patient hospitalized with a stroke who developed a fever and adventitious lung sounds, the following orders are written. Which will the nurse implement first? a)anterior/posterior and lateral chest x-rays b)start IV levofloxacin (Levaquin) 500mg q24h c)sputum specimen for gram stain and culture and sensitivity d)CBC with WBC count and differential

b (High-calorie foods like ice cream are an appropriate snack for patients with COPD. Fluid intake of 3 L/day is recommended, but fluids should be taken between meals rather than with meals to improve oral intake of solid foods. The patient should avoid exercise for an hour before meals to prevent fatigue while eating. Meat and dairy products are high in protein and are good choices for the patient with COPD.)

After the nurse has completed diet teaching for a patient with chronic obstructive pulmonary disease (COPD) who has a body mass index (BMI) of 20, which patient statement indicates that the teaching has been effective? A. "I will drink lots of fluids with my meals." B. "I will have ice cream as a snack every day." C. "I will exercise for 15 minutes before meals." D. "I will decrease my intake of meat or poultry."

b (The patient should relax the facial muscles without puffing the cheeks while doing pursed lip breathing. The other actions by the patient indicate a good understanding of pursed lip breathing.)

After the nurse has finished teaching a patient about pursed lip breathing, which patient action indicates that more teaching is needed? A. The patient inhales slowly through the nose. B. The patient puffs up the cheeks while exhaling. C. The patient practices by blowing through a straw. D. The patient's ratio of inhalation to exhalation is 1:3

b (No more than 4 g of acetaminophen should be taken daily to avoid liver damage. The other patient statements are correct and indicate good understanding of OA management.)

After the nurse has finished teaching a patient with osteoarthritis (OA) of the left hip and knee about how to manage the OA, which patient statement indicates a need for more education? A. "I can take glucosamine to help decrease my knee pain." B. "I will take 1 g of acetaminophen (Tylenol) every 4 hours." C. "I will take a shower in the morning to help relieve stiffness." D. "I can use a cane to decrease the pressure and pain in my hip."

b, c

Albuterol nebulizer is associated with A. Long-term control B. Quick-relief C. B2-adrenergic agonist D. Mast-cell stabilizer E. Leukotriene inhibitor F. Steroid antiinflammatory G. Methylxanthine bronchodilator H. Anticholinergic I. Anti-IgE

d (Directly observed therapy is the most effective means for ensuring compliance with the treatment regimen, and arranging a daily meal will help to ensure that the patient is available to receive the medication. The other nursing interventions may be appropriate for some patients, but are not likely to be as helpful with this patient.)

An alcoholic and homeless patient is diagnosed with active tuberculosis (TB). Which intervention by the nurse will be most effective in ensuring adherence with the treatment regimen? A. Educating the patient about the long-term impact of TB on health B. Giving the patient written instructions about how to take the medications C. Teaching the patient about the high risk for infecting others unless treatment is followed D. Arranging for a daily noontime meal at a community center and giving the medication then

a (Osteoporosis occurring in the vertebrae produces a gradual loss of height. Bowed legs are associated with osteomalacia. Low intake of dairy products is a risk factor for osteoporosis, but it does not indicate that osteoporosis is present. Frequent falls increase the risk for fractures but are not an indicator of osteoporosis.)

An assessment finding that alerts the nurse to the presence of osteoporosis in a middle-aged patient is A. measurable loss of height. B. the presence of bowed legs. C. an aversion to dairy products. D. statements about frequent falls.

b (because the device covers the residual limb, the surgical site cannot be directly seen, and postoperative hemorrhage is not apparent on dressings, requiring vigilant assessment of vital signs for signs of bleeding. Elevation of the residual limb with an immediate prosthetic fitting is not necessary because the device itself prevents edema formation. Exercises to the leg are not performed in the immediate postoperative period so as to avoid disruption of ligatures and the suture line)

An immediate prosthetic fitting during surgery is used for a patient with a traumatic below-the-knee amputation. During the immediate postoperative period, a priority nursing intervention is to: A. assess the site for hemorrhage B. monitor the patient's vital signs C. reduce edema in the residual limb D. relieve pressure on the incision site

When taking a health history, the nurse screens for manifestations suggestive of diabetes type I. Which of the following manifestations are considered the primary manifestations of diabetes type I and would be most suggestive of diabetes type I and require follow-up investigation? "a. Excessive intake of calories, rapid weight gain, and difficulty losing weight b. Poor circulation, wound healing, and leg ulcers, c. Lack of energy, weight gain, and depression d. An increase in three areas: thirst, intake of fluids, and hunger

An increase in three areas: thirst, intake of fluids, and hunger "The primary manifestations of diabetes type I are polyuria (increased urine output), polydipsia (increased thirst), polyphagia (increased hunger). Excessive calorie intake, weight gain, and difficulty losing weight are common risk factors for type 2 diabetes. Poor circulation, wound healing and leg ulcers are signs of chronic diabetes. Lack of energy, weight gain and depression are not necessarily indicative of any type of diabetes.

d (Rationale: Confusion or stupor, related to hypoxia, may be the only clinical manifestation of pneumonia in an older adult patient. An elevated temperature, coarse crackles, and pleuritic chest pain may occur with pneumonia, but these symptoms do not indicate hypoxia.)

An older adult patient is admitted to the hospital with a diagnosis of pneumococcal pneumonia. Which clinical manifestation, if observed by the nurse, indicates that the patient is hypoxic? a. Temperature is 102.3o F (orally) b. Presence of pleuritic chest pain c. Coarse crackles in lung bases d. Sudden onset of confusion

a (Anakinra is administered by subcutaneous injection. GI bleeding is not a side effect of this medication. Because the medication is injected, instructions to take it with 8 oz of fluid would not be appropriate. The patient is likely to be concurrently taking aspirin or NSAIDs, and these should not be discontinued.)

Anakinra (Kineret) is prescribed for a patient who has rheumatoid arthritis (RA). When teaching the patient about this drug, the nurse will include information about A. self-administration of subcutaneous injections. B. taking the medication with at least 8 oz of fluid. C. avoiding concurrently taking aspirin or nonsteroidal antiinflammatory drugs (NSAIDs). D. symptoms of gastrointestinal (GI) irritation or bleeding.

A patient with newly diagnosed type 2 diabetes mellitus asks the nurse what "type 2" means in relation to diabetes. The nurse explains to the patient that type 2 diabetes differs from type 1 diabetes primarily in that with type 2 diabetes a. the pt is totally dependent on an outside source of insulin b. there is a decreased insulin secretion and cellular resistance to insulin that is produced c. the immune system destroys the pancreatic insulin-producing cells d. the insulin precurosr that is secreted by the pancreas is not activated by the liver

Answer B - Rationale: In type 2 diabetes, the pancreas produces insulin, but the insulin is insufficient for the body's needs or the cells do not respond to the insulin appropriately. The other information describes the physiology of type 1 diabetes

A nurse is caring for a male client hospitalized with acute exacerbation of chronic obstructive pulmonary disease. Which of the following would the nurse expect to note on assessment of this client? a. Hypocapnia b. A hyperinflated chest noted on the chest x-ray c. Increase oxygen saturation with exercise d. A widened diaphragm noted on the chest x-ray

Answer B. Clinical manifestations of chronic obstructive pulmonary disease (COPD) include hypoxemia, - hypercapnia, - dyspnea on exertion and at rest - oxygen desaturation with exercise - and the use of accessory muscles of respiration. Chest x-rays reveal a hyperinflated chest and a flattened diaphragm if the disease is advanced.

The nurse caring for a 54-year-old patient hospitalized with diabetes mellitus would look for which of the following laboratory test results to obtain information on the patient's past glucose control? a. prealbumin level b. urine ketone level c. fasting glucose level d. glycosylated hemoglobin level

Answer d: A glycosylated hemoglobin level detects the amount of glucose that is bound to red blood cells (RBCs). When circulating glucose levels are high, glucose attaches to the RBCs and remains there for the life of the blood cell, which is approximately 120 days. Thus the test can give an indication of glycemic control over approximately 2 to 3 months.

Presence of overdistended and non-functional alveoli is a condition called: a. Bronchitis b. Emphysema c. Empyema d. Atelectasis

Answer: B. An overdistended and non-functional alveoli is a condition called emphysema. Atelectasis is the collapse of a part or the whole lung. Empyema is the presence of pus in the lung.

HF etiology

Any interference with normal regulating mechanisms for CO. Changes lead to decreased ventricular function. 1) preload 2) afterload 3) myocardial contractility 4) heart rate

v

Arterial or Venous disease? Brown pigmentation of the legs

a

Arterial or Venous disease? Decreased peripheral pulses

v

Arterial or Venous disease? Dull ache in calf or thigh

v

Arterial or Venous disease? Edema around the ankles

v

Arterial or Venous disease? Heavy ulcer drainage

a

Arterial or Venous disease? Pallor on elevation of the legs

a

Arterial or Venous disease? Paresthesia

v

Arterial or Venous disease? Pruritis

a

Arterial or Venous disease? Thickened, brittle nails

v

Arterial or Venous disease? Ulceration around the medial malleolus

a

Arterial or Venous disease? Ulcers over bony prominences on toes and feet

25. The nurse is caring for a patient with a severe nosebleed. The physician inserts a nasal sponge and tells the patient it may have to remain in place up to 6 days before it is removed. The nurse should identify that this patient is at increased risk for what? A) Viral sinusitis B) Toxic shock syndrome C) Pharyngitis D) Adenoiditis

B Feedback: A compressed nasal sponge may be used. Once the sponge becomes saturated with blood or is moistened with a small amount of saline, it will expand and produce tamponade to halt the bleeding. The packing may remain in place for 48 hours or up to 5 or 6 days if necessary to control bleeding. Antibiotics may be prescribed because of the risk of iatrogenic sinusitis and toxic shock syndrome.

3. A nurse is caring for a young adult patient whose medical history includes an alpha1-antitrypsin deficiency. This deficiency predisposes the patient to what health problem? A) Pulmonary edema B) Lobular emphysema C) Cystic fibrosis (CF) D) Empyema

B Feedback: A host risk factor for COPD is a deficiency of alpha1-antitrypsin, an enzyme inhibitor that protects the lung parenchyma from injury. This deficiency predisposes young patients to rapid development of lobular emphysema even in the absence of smoking. This deficiency does not influence the patient's risk of pulmonary edema, CF, or empyema.

28. A nurse is developing a teaching plan for a patient with COPD. What should the nurse include as the most important area of teaching? A) Avoiding extremes of heat and cold B) Setting and accepting realistic short- and long-range goals C) Adopting a lifestyle of moderate activity D) Avoiding emotional disturbances and stressful situations

B Feedback: A major area of teaching involves setting and accepting realistic short-term and long-range goals. The other options should also be included in the teaching plan, but they are not areas that are as high a priority as setting and accepting realistic goals.

20. The nurse is caring for a patient who is experiencing mild shortness of breath during the immediate postoperative period, with oxygen saturation readings between 89% and 91%. What method of oxygen delivery is most appropriate for the patient's needs? A) Non-rebreathing mask B) Nasal cannula C) Simple mask D) Partial-rebreathing mask

B Feedback: A nasal cannula is used when the patient requires a low to medium concentration of oxygen for which precise accuracy is not essential. The Venturi mask is used primarily for patients with COPD because it can accurately provide an appropriate level of supplemental oxygen, thus avoiding the risk of suppressing the hypoxic drive. The patient's respiratory status does not require a partial- or non-rebreathing mask.

3. What would the critical care nurse recognize as a condition that may indicate a patient's need to have a tracheostomy? A) A patient has a respiratory rate of 10 breaths per minute. B) A patient requires permanent ventilation. C) A patient exhibits symptoms of dyspnea. D) A patient has respiratory acidosis.

B Feedback: A tracheostomy permits long-term use of mechanical ventilation to prevent aspiration of oral and gastric secretions in the unconscious or paralyzed patient. Indications for a tracheostomy do not include a respiratory rate of 10 breaths per minute, symptoms of dyspnea, or respiratory acidosis.

33. A mother calls the clinic asking for a prescription for Amoxicillin for her 2-year-old son who has what the nurse suspects to be viral rhinitis. What should the nurse explain to this mother? A) "I will relay your request promptly to the doctor, but I suspect that she won't get back to you if it's a cold." B) "I'll certainly inform the doctor, but if it is a cold, antibiotics won't be used because they do not affect the virus." C) "I'll phone in the prescription for you since it can be prescribed by the pharmacist." D) "Amoxicillin is not likely the best antibiotic, but I'll call in the right prescription for you."

B Feedback: Antimicrobial agents (antibiotics) should not be used because they do not affect the virus or reduce the incidence of bacterial complications. In addition, their inappropriate use has been implicated in development of organisms resistant to therapy. It would be inappropriate to tell the patient that the physician will not respond to her request.

33. A patient is being admitted to the preoperative holding area for a thoracotomy. Preoperative teaching includes what? A) Correct use of a ventilator B) Correct use of incentive spirometry C) Correct use of a mini-nebulizer D) Correct technique for rhythmic breathing

B Feedback: Instruction in the use of incentive spirometry begins before surgery to familiarize the patient with its correct use. You do not teach a patient the use of a ventilator; you explain that he may be on a ventilator to help him breathe. Rhythmic breathing and mini-nebulizers are unnecessary.

38. An asthma nurse educator is working with a group of adolescent asthma patients. What intervention is most likely to prevent asthma exacerbations among these patients? A) Encouraging patients to carry a corticosteroid rescue inhaler at all times B) Educating patients about recognizing and avoiding asthma triggers C) Teaching patients to utilize alternative therapies in asthma management D) Ensuring that patients keep their immunizations up to date

B Feedback: Asthma exacerbations are best managed by early treatment and education, including the use of written action plans as part of any overall effort to educate patients about self-management techniques, especially those with moderate or severe persistent asthma or with a history of severe exacerbations. Corticosteroids are not used as rescue inhalers. Alternative therapies are not normally a high priority, though their use may be appropriate in some cases. Immunizations should be kept up to date, but this does not necessarily prevent asthma exacerbations.

35. A nurse is caring for a patient who has been hospitalized with an acute asthma exacerbation. What drugs should the nurse expect to be ordered for this patient to gain underlying control of persistent asthma? A) Rescue inhalers B) Anti-inflammatory drugs C) Antibiotics D) Antitussives

B Feedback: Because the underlying pathology of asthma is inflammation, control of persistent asthma is accomplished primarily with regular use of anti-inflammatory medications. Rescue inhalers, antibiotics, and antitussives do not aid in the first-line control of persistent asthma.

21. The nurse is performing the health interview of a patient with chronic rhinosinusitis who experiences frequent nose bleeds. The nurse asks the patient about her current medication regimen. Which medication would put the patient at a higher risk for recurrent epistaxis? A) Afrin B) Beconase C) Sinustop Pro D) Singulair

B Feedback: Beconase should be avoided in patients with recurrent epistaxis, glaucoma, and cataracts. Sinustop Pro and Afrin are pseudoephedrine and do not have a side effect of epistaxis. Singulair is a bronchodilator and does not have epistaxis as a side effect.

10. The acute medical nurse is preparing to wean a patient from the ventilator. Which assessment parameter is most important for the nurse to assess? A) Fluid intake for the last 24 hours B) Baseline arterial blood gas (ABG) levels C) Prior outcomes of weaning D) Electrocardiogram (ECG) results

B Feedback: Before weaning a patient from mechanical ventilation, it is most important to have baseline ABG levels. During the weaning process, ABG levels will be checked to assess how the patient is tolerating the procedure. Other assessment parameters are relevant, but less critical. Measuring fluid volume intake and output is always important when a patient is being mechanically ventilated. Prior attempts at weaning and ECG results are documented on the patient's record, and the nurse can refer to them before the weaning process begins.

28. A patient states that her family has had several colds during this winter and spring despite their commitment to handwashing. The high communicability of the common cold is attributable to what factor? A) Cold viruses are increasingly resistant to common antibiotics. B) The virus is shed for 2 days prior to the emergence of symptoms. C) A genetic predisposition to viral rhinitis has recently been identified. D) Overuse of OTC cold remedies creates a "rebound" susceptibility to future colds.

B Feedback: Colds are highly contagious because virus is shed for about 2 days before the symptoms appear and during the first part of the symptomatic phase. Antibiotic resistance is not relevant to viral illnesses and OTC medications do not have a "rebound" effect. Genetic factors do not exist.

27. The nurse is caring for a client with an endotracheal tube who is on a ventilator. When assessing the client, the nurse knows to maintain what cuff pressure to maintain appropriate pressure on the tracheal wall? A) Between 10 and 15 mm Hg B) Between 15 and 20 mm Hg C) Between 20 and 25 mm Hg D) Between 25 and 30 mm Hg

B Feedback: Complications can occur from pressure exerted by the cuff on the tracheal wall. Cuff pressures should be maintained between 15 and 20 mm Hg.

13. The nurse is conducting a presurgical interview for a patient with laryngeal cancer. The patient states that he drinks approximately six to eight shots of vodka per day. It is imperative that the nurse inform the surgical team so the patient can be assessed for what? A) Increased risk for infection B) Delirium tremens C) Depression D) Nonadherence to postoperative care

B Feedback: Considering the known risk factors for cancer of the larynx, it is essential to assess the patient's history of alcohol intake. Infection is a risk in the postoperative period, but not an appropriate answer based on the patient's history. Depression and nonadherence are risks in the postoperative phase, but would not be critical short-term assessments.

31. The nurse recognizes that aspiration is a potential complication of a laryngectomy. How should the nurse best manage this risk? A) Facilitate total parenteral nutrition (TPN). B) Keep a complete suction setup at the bedside. C) Feed the patient several small meals daily. D) Refer the patient for occupational therapy.

B Feedback: Due to the risk for aspiration, the nurse keeps a suction setup available in the hospital and instructs the family to do so at home for use if needed. TPN is not indicated and small meals do not necessarily reduce the risk of aspiration. Physical therapists do not address swallowing ability.

13. A nurse is developing the teaching portion of a care plan for a patient with COPD. What would be the most important component for the nurse to emphasize? A) Smoking up to one-half of a pack of cigarettes weekly is allowable. B) Chronic inhalation of indoor toxins can cause lung damage. C) Minor respiratory infections are considered to be self-limited and are not treated. D) Activities of daily living (ADLs) should be clustered in the early morning hours.

B Feedback: Environmental risk factors for COPD include prolonged and intense exposure to occupational dusts and chemicals, indoor air pollution, and outdoor air pollution. Smoking cessation should be taught to all patients who are currently smoking. Minor respiratory infections that are of no consequence to the person with normal lungs can produce fatal disturbances in the lungs of the person with emphysema. ADLs should be paced throughout the day to permit patients to perform these without excessive distress.

17. The perioperative nurse has admitted a patient who has just underwent a tonsillectomy. The nurse's postoperative assessment should prioritize which of the following potential complications of this surgery? A) Difficulty ambulating B) Hemorrhage C) Infrequent swallowing D) Bradycardia

B Feedback: Hemorrhage is a potential complication of a tonsillectomy. Increased pulse, fever, and restlessness may indicate a postoperative hemorrhage. Difficulty ambulating and bradycardia are not common complications in a patient after a tonsillectomy. Infrequent swallowing does not indicate hemorrhage; frequent swallowing does.

15. While caring for a patient with an endotracheal tube, the nurses recognizes that suctioning is required how often? A) Every 2 hours when the patient is awake B) When adventitious breath sounds are auscultated C) When there is a need to prevent the patient from coughing D) When the nurse needs to stimulate the cough reflex

B Feedback: It is usually necessary to suction the patient's secretions because of the decreased effectiveness of the cough mechanism. Tracheal suctioning is performed when adventitious breath sounds are detected or whenever secretions are present. Unnecessary suctioning, such as scheduling every 2 hours, can initiate bronchospasm and cause trauma to the tracheal mucosa.

1. The nurse is caring for a patient with chronic obstructive pulmonary disease (COPD). The patient has been receiving high-flow oxygen therapy for an extended time. What symptoms should the nurse anticipate if the patient were experiencing oxygen toxicity? A) Bradycardia and frontal headache B) Dyspnea and substernal pain C) Peripheral cyanosis and restlessness D) Hypotension and tachycardia

B Feedback: Oxygen toxicity can occur when patients receive too high a concentration of oxygen for an extended period. Symptoms of oxygen toxicity include dyspnea, substernal pain, restlessness, fatigue, and progressive respiratory difficulty. Bradycardia, frontal headache, cyanosis, hypotension, and tachycardia are not symptoms of oxygen toxicity.

39. An asthma educator is teaching a patient newly diagnosed with asthma and her family about the use of a peak flow meter. The educator should teach the patient that a peak flow meter measures what value? A) Highest airflow during a forced inspiration B) Highest airflow during a forced expiration C) Airflow during a normal inspiration D) Airflow during a normal expiration

B Feedback: Peak flow meters measure the highest airflow during a forced expiration.

13. The physician has ordered continuous positive airway pressure (CPAP) with the delivery of a patient's high-flow oxygen therapy. The patient asks the nurse what the benefit of CPAP is. What would be the nurse's best response? A) CPAP allows a higher percentage of oxygen to be safely used. B) CPAP allows a lower percentage of oxygen to be used with a similar effect. C) CPAP allows for greater humidification of the oxygen that is administered. D) CPAP allows for the elimination of bacterial growth in oxygen delivery systems.

B Feedback: Prevention of oxygen toxicity is achieved by using oxygen only as prescribed. Often, positive end-expiratory pressure (PEEP) or CPAP is used with oxygen therapy to reverse or prevent microatelectasis, thus allowing a lower percentage of oxygen to be used. Oxygen is moistened by passing through a humidification system. Changing the tubing on the oxygen therapy equipment is the best technique for controlling bacterial growth.

5. The nurse is caring for a patient who is scheduled to have a thoracotomy. When planning preoperative teaching, what information should the nurse communicate to the patient? A) How to milk the chest tubing B) How to splint the incision when coughing C) How to take prophylactic antibiotics correctly D) How to manage the need for fluid restriction

B Feedback: Prior to thoracotomy, the nurse educates the patient about how to splint the incision with the hands, a pillow, or a folded towel. The patient is not taught how to milk the chest tubing because this is performed by the nurse. Prophylactic antibiotics are not normally used and fluid restriction is not indicated following thoracotomy.

22. An older adult patient has been diagnosed with COPD. What characteristic of the patient's current health status would preclude the safe and effective use of a metered-dose inhaler (MDI)? A) The patient has not yet quit smoking. B) The patient has severe arthritis in her hands. C) The patient requires both corticosteroids and beta2-agonists. D) The patient has cataracts.

B Feedback: Safe and effective MDI use requires the patient to be able to manipulate the device independently, which may be difficult if the patient has arthritis. Smoking does not preclude MDI use. A modest loss of vision does not preclude the use of an MDI and a patient can safely use more than one MDI.

21. A nurse is reviewing the pathophysiology of cystic fibrosis (CF) in anticipation of a new admission. The nurse should identify what characteristic aspects of CF? A) Alveolar mucus plugging, infection, and eventual bronchiectasis B) Bronchial mucus plugging, inflammation, and eventual bronchiectasis C) Atelectasis, infection, and eventual COPD D) Bronchial mucus plugging, infection, and eventual COPD

B Feedback: The hallmark pathology of CF is bronchial mucus plugging, inflammation, and eventual bronchiectasis. Commonly, the bronchiectasis begins in the upper lobes and progresses to involve all lobes. Infection, atelectasis, and COPD are not hallmark pathologies of CF.

38. A patient has just been diagnosed with squamous cell carcinoma of the neck. While the nurse is doing health education, the patient asks, "Does this kind of cancer tend to spread to other parts of the body?" What is the nurse's best response? A) "In many cases, this type of cancer spreads to other parts of the body." B) "This cancer usually does not spread to distant sites in the body." C) "You will have to speak to your oncologist about that." D) "Squamous cell carcinoma is nothing to be concerned about, so try to focus on your health."

B Feedback: The incidence of distant metastasis with squamous cell carcinoma of the head and neck (including larynx cancer) is relatively low. The patient's prognosis is determined by the oncologist, but the patient has asked a general question and it would be inappropriate to refuse a response. The nurse must not downplay the patient's concerns.

30. The nurse is doing discharge teaching in the ED with a patient who had a nosebleed. What should the nurse include in the discharge teaching of this patient? A) Avoid blowing the nose for the next 45 minutes. B) In case of recurrence, apply direct pressure for 15 minutes. C) Do not take aspirin for the next 2 weeks. D) Seek immediate medical attention if the nosebleed recurs.

B Feedback: The nurse explains how to apply direct pressure to the nose with the thumb and the index finger for 15 minutes in case of a recurrent nosebleed. If recurrent bleeding cannot be stopped, the patient is instructed to seek additional medical attention. ASA is not contraindicated in most cases and the patient should avoiding blowing the nose for an extended period of time, not just 45 minutes.

34. The nurse is providing care for a patient who has just been admitted to the postsurgical unit following a laryngectomy. What assessment should the nurse prioritize? A) The patient's swallowing ability B) The patient's airway patency C) The patient's carotid pulses D) Signs and symptoms of infection

B Feedback: The patient with a laryngectomy is a risk for airway occlusion and respiratory distress. As in all nursing situations, assessment of the airway is a priority over other potential complications and assessment parameters.

29. The nurse has explained to the patient that after his thoracotomy, it will be important to adhere to a coughing schedule. The patient is concerned about being in too much pain to be able to cough. What would be an appropriate nursing intervention for this client? A) Teach him postural drainage. B) Teach him how to perform huffing. C) Teach him to use a mini-nebulizer. D) Teach him how to use a metered dose inhaler.

B Feedback: The technique of "huffing" may be helpful for the patient with diminished expiratory flow rates or for the patient who refuses to cough because of severe pain. Huffing is the expulsion of air through an open glottis. Inhalers, nebulizers, and postural drainage are not substitutes for performing coughing exercises.

24. An admitting nurse is assessing a patient with COPD. The nurse auscultates diminished breath sounds, which signify changes in the airway. These changes indicate to the nurse to monitor the patient for what? A) Kyphosis and clubbing of the fingers B) Dyspnea and hypoxemia C) Sepsis and pneumothorax D) Bradypnea and pursed lip breathing

B Feedback: These changes in the airway require that the nurse monitor the patient for dyspnea and hypoxemia. Kyphosis is a musculoskeletal problem. Sepsis and pneumothorax are atypical complications. Tachypnea is much more likely than bradypnea. Pursed lip breathing can relieve dyspnea.

24. The nurse is caring for a patient who has just been diagnosed with chronic rhinosinusitis. While being admitted to the clinic, the patient asks, "Will this chronic infection hurt my new kidney?" What should the nurse know about chronic rhinosinusitis in patients who have had a transplant? A) The patient will have exaggerated symptoms of rhinosinusitis due to immunosuppression. B) Taking immunosuppressive drugs can contribute to chronic rhinosinusitis. C) Chronic rhinosinusitis can damage the transplanted organ. D) Immunosuppressive drugs can cause organ rejection.

B Feedback: URIs, specifically chronic rhinosinusitis and recurrent acute rhinosinusitis, may be linked to primary or secondary immune deficiency or treatment with immunosuppressive therapy (i.e., for cancer or organ transplantation). Typical symptoms may be blunted or absent due to immunosuppression. No evidence indicates damage to the transplanted organ due to chronic rhinosinusitis. Immunosuppressive drugs do not cause organ rejection.

26. A nurse's assessment reveals that a client with COPD may be experiencing bronchospasm. What assessment finding would suggest that the patient is experiencing bronchospasm? A) Fine or coarse crackles on auscultation B) Wheezes or diminished breath sounds on auscultation C) Reduced respiratory rate or lethargy D) Slow, deliberate respirations

B Feedback: Wheezing and diminished breath sounds are consistent with bronchospasm. Crackles are usually attributable to other respiratory or cardiac pathologies. Bronchospasm usually results in rapid, inefficient breathing and agitation.

Physiological Integrity 10. When auscultating over the patient's abdominal aorta, the nurse hears a humming sound. The nurse documents this finding as a a. thrill. b. bruit. c. murmur. d. normal finding.

B A bruit is the sound created by turbulent blood flow in an artery. Thrills are palpable vibrations felt when there is turbulent blood flow through the heart or in a blood vessel. A murmur is the sound caused by turbulent blood flow through the heart. Auscultating a bruit in an artery is not normal and indicates pathology. DIF: Cognitive Level: Understand (comprehension) REF: 695 TOP: Nursing Process: Assessment MSC:

Health Promotion and Maintenance 15. When caring for a patient who is hospitalized with active tuberculosis (TB), the nurse observes a student nurse who is assigned to take care of a patient. Which action, if performed by the student nurse, would require an intervention by the nurse? a. The patient is offered a tissue from the box at the bedside. b. A surgical face mask is applied before visiting the patient. c. A snack is brought to the patient from the unit refrigerator. d. Hand washing is performed before entering the patient's room.

B A high-efficiency particulate-absorbing (HEPA) mask, rather than a standard surgical mask, should be used when entering the patient's room because the HEPA mask can filter out 100% of small airborne particles. Hand washing before entering the patient's room is appropriate. Because anorexia and weight loss are frequent problems in patients with TB, bringing food to the patient is appropriate. The student nurse should perform hand washing after handling a tissue that the patient has used, but no precautions are necessary when giving the patient an unused tissue. DIF: Cognitive Level: Apply (application) REF: 533 TOP: Nursing Process: Implementation MSC:

Physiological Integrity 42. The nurse receives a change-of-shift report on the following patients with chronic obstructive pulmonary disease (COPD). Which patient should the nurse assess first? a. A patient with loud expiratory wheezes b. A patient with a respiratory rate of 38/minute c. A patient who has a cough productive of thick, green mucus d. A patient with jugular venous distention and peripheral edema

B A respiratory rate of 38/minute indicates severe respiratory distress, and the patient needs immediate assessment and intervention to prevent possible respiratory arrest. The other patients also need assessment as soon as possible, but they do not need to be assessed as urgently as the tachypneic patient. DIF: Cognitive Level: Analyze (analysis) REF: 576 OBJ: Special Questions: Prioritization; Multiple Patients TOP: Nursing Process: Assessment MSC:

Physiological Integrity 7. A patient is scheduled for a cardiac catheterization with coronary angiography. Before the test, the nurse informs the patient that a. it will be important to lie completely still during the procedure. b. a flushed feeling may be noted when the contrast dye is injected. c. monitored anesthesia care will be provided during the procedure. d. arterial pressure monitoring will be required for 24 hours after the test.

B A sensation of warmth or flushing is common when the contrast material is injected, which can be anxiety-producing unless it has been discussed with the patient. The patient may receive a sedative drug before the procedure, but monitored anesthesia care is not used. Arterial pressure monitoring is not routinely used after the procedure to monitor blood pressure. The patient is not immobile during cardiac catheterization and may be asked to cough or take deep breaths. DIF: Cognitive Level: Apply (application) REF: 706 TOP: Nursing Process: Implementation MSC:

Physiological Integrity 30. A patient has just been admitted with probable bacterial pneumonia and sepsis. Which order should the nurse implement first? a. Chest x-ray via stretcher b. Blood cultures from two sites c. Ciprofloxacin (Cipro) 400 mg IV d. Acetaminophen (Tylenol) rectal suppository

B Initiating antibiotic therapy rapidly is essential, but it is important that the cultures be obtained before antibiotic administration. The chest x-ray and acetaminophen administration can be done last. DIF: Cognitive Level: Apply (application) REF: 525 OBJ: Special Questions: Prioritization TOP: Nursing Process: Implementation MSC:

Health Promotion and Maintenance 27. A patient with chronic obstructive pulmonary disease (COPD) has rhonchi throughout the lung fields and a chronic, nonproductive cough. Which nursing intervention will be most effective? a. Change the oxygen flow rate to the highest prescribed rate. b. Teach the patient to use the Flutter airway clearance device. c. Reinforce the ongoing use of pursed lip breathing techniques. d. Teach the patient about consistent use of inhaled corticosteroids.

B Airway clearance devices assist with moving mucus into larger airways where it can more easily be expectorated. The other actions may be appropriate for some patients with COPD, but they are not indicated for this patient's problem of thick mucus secretions. DIF: Cognitive Level: Apply (application) REF: 595 TOP: Nursing Process: Implementation MSC:

Integrated Process: Nursing Process (Assessment) 5. The nurse is caring for an older adult who reports experiencing frequent asthma attacks and severe arthritic pain. What action by the nurse is most appropriate? a. Review pulmonary function test results. b. Assess use of medication for arthritis. c. Assess frequency of bronchodilator use. d. Review arterial blood gas results.

B Aspirin and other NSAIDs can trigger asthma in some people. This results from increased production of leukotriene when aspirin or NSAIDs suppress other inflammatory pathways and is a high priority given the client's history. Reviewing pulmonary function test results will not address the immediate problem of frequent asthma attacks. This is a good intervention for reviewing response to bronchodilators. Questioning the client about the use of bronchodilators will address interventions for the attacks but not their cause. Reviewing arterial blood gas results would not be of use in a client between attacks because many clients are asymptomatic when not having attacks. DIF: Cognitive Level: Application/Applying or higher REF: N/A TOP: Client Needs Category: Physiological Integrity (Pharmacological and Parenteral Therapies—Adverse Effects/Contraindications/Interactions/Side Effects)

Physiological Integrity 27. A patient with idiopathic pulmonary arterial hypertension (IPAH) is receiving nifedipine (Procardia). Which assessment would best indicate to the nurse that the patient's condition is improving? a. Blood pressure (BP) is less than 140/90 mm Hg. b. Patient reports decreased exertional dyspnea. c. Heart rate is between 60 and 100 beats/minute. d. Patient's chest x-ray indicates clear lung fields.

B Because a major symptom of IPAH is exertional dyspnea, an improvement in this symptom would indicate that the medication was effective. Nifedipine will affect BP and heart rate, but these parameters would not be used to monitor the effectiveness of therapy for a patient with IPAH. The chest x-ray will show clear lung fields even if the therapy is not effective. DIF: Cognitive Level: Apply (application) REF: 554 TOP: Nursing Process: Evaluation MSC:

Physiological Integrity 33. When admitting a patient with a non-ST-segment-elevation myocardial infarction (NSTEMI) to the intensive care unit, which action should the nurse perform first? a. Obtain the blood pressure. b. Attach the cardiac monitor. c. Assess the peripheral pulses. d. Auscultate the breath sounds.

B Because dysrhythmias are the most common complication of myocardial infarction (MI), the first action should be to place the patient on a cardiac monitor. The other actions also are important and should be accomplished as quickly as possible. DIF: Cognitive Level: Apply (application) REF: 756 OBJ: Special Questions: Prioritization TOP: Nursing Process: Implementation MSC:

Physiological Integrity 39. To improve the physical activity level for a mildly obese 71-year-old patient, which action should the nurse plan to take? a. Stress that weight loss is a major benefit of increased exercise. b. Determine what kind of physical activities the patient usually enjoys. c. Tell the patient that older adults should exercise for no more than 20 minutes at a time. d. Teach the patient to include a short warm-up period at the beginning of physical activity.

B Because patients are more likely to continue physical activities that they already enjoy, the nurse will plan to ask the patient about preferred activities. The goal for older adults is 30 minutes of moderate activity on most days. Older adults should plan for a longer warm-up period. Benefits of exercises, such as improved activity tolerance, should be emphasized rather than aiming for significant weight loss in older mildly obese adults. DIF: Cognitive Level: Apply (application) REF: 761 TOP: Nursing Process: Planning MSC:

Physiological Integrity 18. After the nurse has received change-of-shift report, which patient should the nurse assess first? a. A patient with pneumonia who has crackles in the right lung base b. A patient with possible lung cancer who has just returned after bronchoscopy c. A patient with hemoptysis and a 16-mm induration with tuberculin skin testing d. A patient with chronic obstructive pulmonary disease (COPD) and pulmonary function testing (PFT) that indicates low forced vital capacity

B Because the cough and gag are decreased after bronchoscopy, this patient should be assessed for airway patency. The other patients do not have clinical manifestations or procedures that require immediate assessment by the nurse. DIF: Cognitive Level: Apply (application) REF: 492 OBJ: Special Questions: Prioritization; Multiple Patients TOP: Nursing Process: Assessment MSC:

Physiological Integrity 3. Which assessment data collected by the nurse who is admitting a patient with chest pain suggest that the pain is caused by an acute myocardial infarction (AMI)? a. The pain increases with deep breathing. b. The pain has lasted longer than 30 minutes. c. The pain is relieved after the patient takes nitroglycerin. d. The pain is reproducible when the patient raises the arms.

B Chest pain that lasts for 20 minutes or more is characteristic of AMI. Changes in pain that occur with raising the arms or with deep breathing are more typical of musculoskeletal pain or pericarditis. Stable angina is usually relieved when the patient takes nitroglycerin. DIF: Cognitive Level: Apply (application) REF: 748 TOP: Nursing Process: Assessment MSC:

Physiological Integrity 3. A diabetic patient's arterial blood gas (ABG) results are pH 7.28; PaCO2 34 mm Hg; PaO2 85 mm Hg; HCO3- 18 mEq/L. The nurse would expect which finding? a. Intercostal retractions b. Kussmaul respirations c. Low oxygen saturation (SpO2) d. Decreased venous O2 pressure

B Kussmaul (deep and rapid) respirations are a compensatory mechanism for metabolic acidosis. The low pH and low bicarbonate result indicate metabolic acidosis. Intercostal retractions, a low oxygen saturation rate, and a decrease in venous O2 pressure would not be caused by acidosis. DIF: Cognitive Level: Apply (application) REF: 479 TOP: Nursing Process: Assessment MSC:

Integrated Process: Nursing Process (Evaluation) 8. Which statement indicates that a client understands teaching about the correct use of a corticosteroid medication? a. "This drug can reverse my symptoms during an asthma attack." b. "This drug is effective in decreasing the frequency of my asthma attacks." c. "This drug can be used most effectively as a rescue agent." d. "This drug can be used safely on a long-term basis for multiple applications daily."

B Corticosteroids decrease inflammatory and immune responses in many ways, including preventing the synthesis of mediators. Both inhaled corticosteroids and those taken orally are preventive; they are not effective in reversing symptoms during an asthma attack and should not be used as rescue drugs. Systemic corticosteroids, because of severe side effects, are avoided for mild to moderate intermittent asthma and are used on a short-term basis for moderate asthma. DIF: Cognitive Level: Application/Applying or higher REF: N/A TOP: Client Needs Category: Physiological Integrity (Pharmacological and Parenteral Therapies—Expected Actions/Outcomes)

Psychosocial Integrity 20. When caring for a patient who is recovering from a sudden cardiac death (SCD) event and has no evidence of an acute myocardial infarction (AMI), the nurse will anticipate teaching the patient that a. sudden cardiac death events rarely reoccur. b. additional diagnostic testing will be required. c. long-term anticoagulation therapy will be needed. d. limited physical activity after discharge will be needed to prevent future events.

B Diagnostic testing (e.g., stress test, Holter monitor, electrophysiologic studies, cardiac catheterization) is used to determine the possible cause of the SCD and treatment options. SCD is likely to recur. Anticoagulation therapy will not have any effect on the incidence of SCD, and SCD can occur even when the patient is resting. DIF: Cognitive Level: Apply (application) REF: 762 TOP: Nursing Process: Planning MSC:

Physiological Integrity 37. Which assessment finding in a patient who has received omalizumab (Xolair) is most important to report immediately to the health care provider? a. Pain at injection site b. Flushing and dizziness c. Peak flow reading 75% of normal d. Respiratory rate 22 breaths/minute

B Flushing and dizziness may indicate that the patient is experiencing an anaphylactic reaction, and immediate intervention is needed. The other information should also be reported, but do not indicate possibly life-threatening complications of omalizumab therapy. DIF: Cognitive Level: Apply (application) REF: 572 OBJ: Special Questions: Prioritization TOP: Nursing Process: Assessment MSC:

Physiological Integrity 14. Which finding by the nurse for a patient with a nursing diagnosis of impaired gas exchange will be most useful in evaluating the effectiveness of treatment? a. Even, unlabored respirations b. Pulse oximetry reading of 92% c. Respiratory rate of 18 breaths/minute d. Absence of wheezes, rhonchi, or crackles

B For the nursing diagnosis of impaired gas exchange, the best data for evaluation are arterial blood gases (ABGs) or pulse oximetry. The other data may indicate either improvement or impending respiratory failure caused by fatigue. DIF: Cognitive Level: Apply (application) REF: 598 TOP: Nursing Process: Evaluation MSC:

Health Promotion and Maintenance 45. An experienced nurse instructs a new nurse about how to care for a patient with dyspnea caused by a pulmonary fungal infection. Which action by the new nurse indicates a need for further teaching? a. Listening to the patient's lung sounds several times during the shift b. Placing the patient on droplet precautions and in a private hospital room c. Increasing the oxygen flow rate to keep the oxygen saturation above 90% d. Monitoring patient serology results to identify the specific infecting organism

B Fungal infections are not transmitted from person to person. Therefore no isolation procedures are necessary. The other actions by the new nurse are appropriate. DIF: Cognitive Level: Apply (application) REF: 534 TOP: Nursing Process: Evaluation MSC:

Physiological Integrity 28. The nurse provides dietary teaching for a patient with chronic obstructive pulmonary disease (COPD) who has a low body mass index (BMI). Which patient statement indicates that the teaching has been effective? a. "I will drink lots of fluids with my meals." b. "I can have ice cream as a snack every day." c. "I will exercise for 15 minutes before meals." d. "I will decrease my intake of meat and poultry."

B High-calorie foods like ice cream are an appropriate snack for patients with COPD. Fluid intake of 3 L/day is recommended, but fluids should be taken between meals rather than with meals to improve oral intake of solid foods. The patient should avoid exercise for an hour before meals to prevent fatigue while eating. Meat and dairy products are high in protein and are good choices for the patient with COPD. DIF: Cognitive Level: Apply (application) REF: 595-596 TOP: Nursing Process: Evaluation MSC:

Integrated Process: Teaching/Learning 32. A client's chest tube is accidentally dislodged. What action by the nurse is best? a. No action is necessary because the area will reseal itself. b. Cover the insertion site with a sterile gauze and tape three sides. c. Obtain a suture kit and prepare for the physician to suture the site. d. Cover the area with an occlusive dressing.

B Immediately covering the insertion site helps prevent air from entering the pleural space and causing a pneumothorax. The area will not reseal quickly enough to prevent air from entering the chest. The nurse should not leave the client to obtain a suture kit. An occlusive dressing may cause a tension pneumothorax. DIF: Cognitive Level: Application/Applying or higher REF: N/A TOP: Client Needs Category: Physiological Integrity (Physiological Adaptation—Medical Emergencies)

Physiological Integrity 12. A 53-year-old patient with Stage D heart failure and type 2 diabetes asks the nurse whether heart transplant is a possible therapy. Which response by the nurse is most appropriate? a. "Because you have diabetes, you would not be a candidate for a heart transplant." b. "The choice of a patient for a heart transplant depends on many different factors." c. "Your heart failure has not reached the stage in which heart transplants are needed." d. "People who have heart transplants are at risk for multiple complications after surgery."

B Indications for a heart transplant include end-stage heart failure (Stage D), but other factors such as coping skills, family support, and patient motivation to follow the rigorous posttransplant regimen are also considered. Diabetic patients who have well-controlled blood glucose levels may be candidates for heart transplant. Although heart transplants can be associated with many complications, this response does not address the patient's question. DIF: Cognitive Level: Apply (application) REF: 783 TOP: Nursing Process: Implementation MSC:

Physiological Integrity 22. Which action is appropriate for the nurse to delegate to unlicensed assistive personnel (UAP)? a. Listen to a patient's lung sounds for wheezes or rhonchi. b. Label specimens obtained during percutaneous lung biopsy. c. Instruct a patient about how to use home spirometry testing. d. Measure induration at the site of a patient's intradermal skin test.

B Labeling of specimens is within the scope of practice of UAP. The other actions require nursing judgment and should be done by licensed nursing personnel. DIF: Cognitive Level: Apply (application) REF: 15 OBJ: Special Questions: Delegation TOP: Nursing Process: Assessment MSC:

Integrated Process: Nursing Process (Evaluation) 21. The nurse is teaching a client about different medications for asthma. Which medication does the nurse teach the client to administer to control the prolonged inflammatory response? a. Diphenhydramine (Benadryl) b. Montelukast (Singulair) c. Aspirin d. Bitolterol (Tornalate)

B Leukotriene and eotaxin cause later, prolonged inflammatory responses in asthma, which can be blocked by drugs like montelukast (Singulair), zafirlukast (Accolate), and zileuton (Zyflo). No evidence suggests that aspirin helps this inflammatory response. Histamine starts an immediate inflammatory response, which can be blocked by drugs like diphenhydramine (Benadryl). Bitolterol (Tornalate) is a short-acting beta agonist that will enhance bronchodilation during an asthma attack, but it will not assist in controlling late inflammation. DIF: Cognitive Level: Comprehension/Understanding REF: Chart 32-6, p. 606 TOP: Client Needs Category: Physiological Integrity (Pharmacological and Parenteral Therapies—Expected Actions/Outcomes)

Physiological Integrity 12. When assessing a newly admitted patient, the nurse notes a murmur along the left sternal border. To document more information about the murmur, which action will the nurse take next? a. Find the point of maximal impulse. b. Determine the timing of the murmur. c. Compare the apical and radial pulse rates. d. Palpate the quality of the peripheral pulses.

B Murmurs are caused by turbulent blood flow, such as occurs when blood flows through a damaged valve. Relevant information includes the position in which the murmur is heard best (e.g., sitting and leaning forward), the timing of the murmur in relation to the cardiac cycle (e.g., systole, diastole), and where on the thorax the murmur is heard best. The other information is also important in the cardiac assessment but will not provide information that is relevant to the murmur. DIF: Cognitive Level: Apply (application) REF: 697 TOP: Nursing Process: Assessment MSC:

Physiological Integrity 37. The nurse receives change-of-shift report on the following four patients. Which patient should the nurse assess first? a. A 23-year-old patient with cystic fibrosis who has pulmonary function testing scheduled b. A 46-year-old patient on bed rest who is complaining of sudden onset of shortness of breath c. A 77-year-old patient with tuberculosis (TB) who has four antitubercular medications due in 15 minutes d. A 35-year-old patient who was admitted the previous day with pneumonia and has a temperature of 100.2° F (37.8° C)

B Patients on bed rest who are immobile are at high risk for deep vein thrombosis (DVT). Sudden onset of shortness of breath in a patient with a DVT suggests a pulmonary embolism and requires immediate assessment and action such as oxygen administration. The other patients should also be assessed as soon as possible, but there is no indication that they may need immediate action to prevent clinical deterioration. DIF: Cognitive Level: Analyze (analysis) REF: 551-553 OBJ: Special Questions: Prioritization; Multiple Patients TOP: Nursing Process: Planning MSC:

Physiological Integrity 4. The nurse provides discharge instructions to a patient who was hospitalized for pneumonia. Which statement, if made by the patient, indicates a good understanding of the instructions? a. "I will call the doctor if I still feel tired after a week." b. "I will continue to do the deep breathing and coughing exercises at home." c. "I will schedule two appointments for the pneumonia and influenza vaccines." d. "I'll cancel my chest x-ray appointment if I'm feeling better in a couple weeks."

B Patients should continue to cough and deep breathe after discharge. Fatigue is expected for several weeks. The Pneumovax and influenza vaccines can be given at the same time in different arms. Explain that a follow-up chest x-ray needs to be done in 6 to 8 weeks to evaluate resolution of pneumonia. DIF: Cognitive Level: Apply (application) REF: 528 TOP: Nursing Process: Evaluation MSC:

Physiological Integrity 10. The nurse will suspect that the patient with stable angina is experiencing a side effect of the prescribed metoprolol (Lopressor) if the a. patient is restless and agitated. b. blood pressure is 90/54 mm Hg. c. patient complains about feeling anxious. d. cardiac monitor shows a heart rate of 61 beats/minute.

B Patients taking -adrenergic blockers should be monitored for hypotension and bradycardia. Because this class of medication inhibits the sympathetic nervous system, restlessness, agitation, hypertension, and anxiety will not be side effects. DIF: Cognitive Level: Apply (application) REF: 745 TOP: Nursing Process: Evaluation MSC:

Physiological Integrity 7. After the nurse teaches the patient about the use of carvedilol (Coreg) in preventing anginal episodes, which statement by a patient indicates that the teaching has been effective? a. "Carvedilol will help my heart muscle work harder." b. "It is important not to suddenly stop taking the carvedilol." c. "I can expect to feel short of breath when taking carvedilol." d. "Carvedilol will increase the blood flow to my heart muscle."

B Patients who have been taking -adrenergic blockers can develop intense and frequent angina if the medication is suddenly discontinued. Carvedilol (Coreg) decreases myocardial contractility. Shortness of breath that occurs when taking -adrenergic blockers for angina may be due to bronchospasm and should be reported to the health care provider. Carvedilol works by decreasing myocardial oxygen demand, not by increasing blood flow to the coronary arteries. DIF: Cognitive Level: Apply (application) REF: 745 TOP: Nursing Process: Evaluation MSC:

Physiological Integrity 9. Diltiazem (Cardizem) is ordered for a patient with newly diagnosed Prinzmetal's (variant) angina. When teaching the patient, the nurse will include the information that diltiazem will a. reduce heart palpitations. b. decrease spasm of the coronary arteries. c. increase the force of the heart contractions. d. help prevent plaque from forming in the coronary arteries.

B Prinzmetal's angina is caused by coronary artery spasm. Calcium channel blockers (e.g., diltiazem, amlodipine [Norvasc]) are a first-line therapy for this type of angina. Lipid-lowering drugs help reduce atherosclerosis (i.e., plaque formation), and -adrenergic blockers decrease sympathetic stimulation of the heart (i.e., palpitations). Medications or activities that increase myocardial contractility will increase the incidence of angina by increasing oxygen demand. DIF: Cognitive Level: Apply (application) REF: 742 | 744 TOP: Nursing Process: Implementation MSC:

Physiological Integrity 39. When taking an admission history of a patient with possible asthma who has new-onset wheezing and shortness of breath, the nurse will be most concerned about which information? a. The patient has a history of pneumonia 2 years ago. b. The patient takes propranolol (Inderal) for hypertension. c. The patient uses acetaminophen (Tylenol) for headaches. d. The patient has chronic inflammatory bowel disease.

B Rationale: -blockers such as propranolol can cause bronchospasm in some patients. The other information will be documented in the health history but does not indicate a need for a change in therapy. Cognitive Level: Application Text Reference: p. 610 Nursing Process: Assessment

Physiological Integrity 12. When auscultating over the patient's abdominal aorta, the nurse hears a humming sound. The nurse documents this finding as a a. thrill. b. bruit. c. heave. d. murmur.

B Rationale: A bruit is the sound created by turbulent blood flow in an artery. Thrills are palpable vibrations felt when there is turbulent blood flow through the heart or in a blood vessel. Heaves are sustained lifts over the precordium that can be observed or palpated. A murmur is the sound caused by turbulent blood flow through the heart. Cognitive Level: Comprehension Text Reference: pp. 748, 750 Nursing Process: Assessment

Health Promotion and Maintenance 14. The nurse identifies the collaborative problem of potential complication: pulmonary edema for a patient in ADHF. When assessing the patient, the nurse will be most concerned about a. an apical pulse rate of 106 beats/min. b. an oxygen saturation of 88% on room air. c. weight gain of 1 kg (2.2 lb) over 24 hours. d. decreased hourly patient urinary output.

B Rationale: A decrease in oxygen saturation to less than 92% indicates hypoxemia. The nurse should administer supplemental oxygen immediately to the patient. An increase in apical pulse rate, 1-kg weight gain, and decreases in urine output also indicate worsening heart failure and require rapid nursing actions, but the low oxygen saturation rate requires the most immediate nursing action. Cognitive Level: Analysis Text Reference: pp. 829-830 Nursing Process: Assessment

Physiological Integrity 22. When caring for a patient who is hospitalized with active TB, the nurse observes a family member who is visiting the patient. The nurse will need to intervene if the family member a. washes the hands before entering the patient's room. b. puts on a surgical face mask before visiting the patient. c. brings food from a "fast-food" restaurant to the patient. d. hands the patient a tissue from the box at the bedside.

B Rationale: A high-efficiency particulate-absorbing (HEPA) mask, rather than a standard surgical mask, should be used when entering the patient's room because the HEPA mask can filter out 100% of small airborne particles. Handwashing before visiting the patient is not necessary, but there is no reason for the nurse to stop the family member from doing this. Because anorexia and weight loss are frequent problems in patients with TB, bringing food from outside the hospital is appropriate. The family member should wash the hands after handling a tissue that the patient has used, but no precautions are necessary when giving the patient an unused tissue. Cognitive Level: Application Text Reference: p. 574 Nursing Process: Implementation

Physiological Integrity 32. The nurse identifies a nursing diagnosis of ineffective airway clearance for a patient who has incisional pain, a poor cough effort, and scattered rhonchi after having a pneumonectomy. To promote airway clearance, the nurse's first action should be to a. have the patient use the incentive spirometer. b. medicate the patient with the ordered morphine. c. splint the patient's chest during coughing. d. assist the patient to sit up at the bedside.

B Rationale: A major reason for atelectasis and poor airway clearance in patients after chest surgery is incisional pain (which increases with deep breathing and coughing). The first action by the nurse should be to medicate the patient to minimize incisional pain. The other actions are all appropriate ways to improve airway clearance but should be done after the morphine is given. Cognitive Level: Application Text Reference: pp. 591, 594 Nursing Process: Implementation

Safe and Effective Care Environment 2. A hypothermic patient is admitted to the emergency department, and pulse oximetry (SpO2) indicates that the O2 saturation is 95%. Which action should the nurse take next? a. Complete a head-to-toe assessment. b. Place the patient on high-flow oxygen. c. Start rewarming the patient. d. Obtain arterial blood gases (ABG).

B Rationale: Although the O2 saturation is adequate, the left shift in the oxyhemoglobin dissociation curve will decrease the amount of oxygen delivered to tissues, so high oxygen concentrations should be given until the patient is normothermic. The other actions are also appropriate, but the initial action should be to administer oxygen. Cognitive Level: Analysis Text Reference: pp. 513, 515 Nursing Process: Assessment

Physiological Integrity 22. A patient who is receiving dobutamine (Dobutrex) for the treatment of ADHF has all of the following nursing actions included in the plan of care. Which action will be best for the RN to delegate to an experienced LPN/LVN? a. Teach the patient the reasons for remaining on bed rest. b. Monitor the patient's BP every hour. c. Adjust the drip rate to keep the systolic BP >90 mm Hg. d. Call the health care provider about a decrease in urine output.

B Rationale: An experienced LPN/LVN would be able to monitor BP and would know to report significant changes to the RN. Teaching patients and making adjustments to the drip rate for vasoactive medications are RN-level skills. Because the health care provider may order changes in therapy based on the decrease in urine output, the RN should call the health care provider about the decreased urine output. Cognitive Level: Application Text Reference: pp. 827-829 Nursing Process: Planning

Physiological Integrity 21. During postoperative teaching with a patient who had a mitral valve replacement with a mechanical valve, the nurse instructs the patient regarding a. the need to avoid high-voltage electrical fields. b. how to monitor anticoagulation therapy. c. the need for valve replacement in 7 to 10 years. d. how to check the radial pulse.

B Rationale: Anticoagulation with warfarin (Coumadin) is needed for a patient with mechanical valves to prevent clotting on the valve. There is no need to avoid high-voltage electrical fields. Mechanical valves are durable and would last longer than 7 to 10 years. Monitoring of radial pulse is not necessary after valve replacement. Cognitive Level: Comprehension Text Reference: p. 882 Nursing Process: Implementation

2. A nurse is creating a health promotion intervention focused on chronic obstructive pulmonary disease (COPD). What should the nurse identify as a complication of COPD? A) Lung cancer B) Cystic fibrosis C) Respiratory failure D) Hemothorax

C Feedback: Complications of COPD include respiratory failure, pneumothorax, atelectasis, pneumonia, and pulmonary hypertension (corpulmonale). Lung cancer, cystic fibrosis, and hemothorax are not common complications.

Physiological Integrity 34. For a patient who has been admitted the previous day to the coronary care unit with an AMI, the nurse will anticipate teaching the patient about a. the pathophysiology of coronary artery disease. b. when patient cardiac rehabilitation will begin. c. home-discharge drugs such as aspirin and -blockers. d. typical emotional responses to MI.

B Rationale: At this time, the patient's anxiety level or denial will prevent good understanding of complex information such as CAD pathophysiology. Teaching about discharge medications should be done when the time for discharge is closer. The nurse should support the patient by decreasing anxiety rather than discussing the typical emotional response to MI. Cognitive Level: Application Text Reference: p. 813 Nursing Process: Planning

Physiological Integrity 14. Nadolol (Corgard) is prescribed for a patient with angina. In evaluating the effectiveness of the drug, the nurse will monitor for a. improvement in the quality of the peripheral pulses. b. ability to do daily activities without chest discomfort. c. decreased BP and apical pulse rate. d. fewer complaints of having cold hands and feet.

B Rationale: Because the medication is ordered to improve the patient's angina, so effectiveness is indicated if the patient is able to accomplish daily activities without chest pain. BP and apical pulse rate may decrease, but these data do not indicate that the goal of decreased angina has been met. The non-cardioselective -blockers can cause peripheral vasoconstriction, so the nurse would not expect an improvement in peripheral pulse quality or skin temperature. Cognitive Level: Application Text Reference: p. 810 Nursing Process: Evaluation

Safe and Effective Care Environment 26. When analyzing the waveforms of a patient's ECG, the nurse will need to investigate further upon finding a a. PR interval of 0.18 second. b. QRS interval of 0.14 second. c. T wave of 0.16 second. d. QT interval of 0.34 second.

B Rationale: Because the normal QRS interval is 0.04 to 0.10 seconds, the patient's QRS interval of 0.14 seconds indicates that the conduction through the ventricular conduction system is prolonged. The PR interval and QT interval are within the normal range. T-wave intervals are not measured when monitoring ECGs. Cognitive Level: Application Text Reference: p. 847 Nursing Process: Assessment

Health Promotion and Maintenance 33. The nurse caring for a patient with CF recognizes that the manifestations of the disease are caused by the pathophysiologic processes of a. inflammation and fibrosis of lung tissue. b. altered function of exocrine glands. c. failure of the mucus-producing goblet cells. d. thickening and fibrosis of the pleural linings.

B Rationale: CF is characterized by abnormal secretions of exocrine glands, mainly of the lungs, pancreas, and sweat glands. Damage to lung tissue develops late in the disease. The goblet cells continue to produce mucus. Cognitive Level: Comprehension Text Reference: pp. 655-656 Nursing Process: Assessment

Health Promotion and Maintenance 16. Which assessment information obtained by the nurse for a patient with aortic stenosis would be most important to report to the health care provider? a. A loud systolic murmur is audible along the right sternal border. b. The patient complains of chest pain associated with ambulation. c. The point of maximum impulse (PMI) is at the left midclavicular line. d. A thrill is palpable at the 2nd intercostal space, right sternal border.

B Rationale: Chest pain occurring with aortic stenosis is caused by cardiac ischemia, and reporting this information would be a priority. A systolic murmur and thrill would not be unusual for a patient with aortic stenosis. A PMI at the left midclavicular line is normal. Cognitive Level: Application Text Reference: pp. 879, 880 Nursing Process: Assessment

Physiological Integrity 35. A 68-year-old man has a long history of COPD and is admitted to the hospital with cor pulmonale. Which clinical manifestation noted by the nurse is consistent with the cor pulmonale diagnosis? a. Audible crackles at both lung bases b. 3+ edema in the lower extremities c. Loud murmur at the mitral area d. High systemic BP

B Rationale: Cor pulmonale is right ventricular failure caused by pulmonary hypertension, so clinical manifestations of right ventricular failure such as peripheral edema, jugular vein distension, and right upper-quadrant abdominal tenderness would be expected. Lung crackles, a murmur, and numbness and tingling are not caused by cor pulmonale. Cognitive Level: Application Text Reference: p. 602 Nursing Process: Assessment

Physiological Integrity 24. Which information obtained by the nurse when assessing a patient admitted with mitral valve stenosis should be communicated to the health care provider immediately? a. The patient has a loud diastolic murmur all across the precordium. b. The patient has crackles audible to the lung apices. c. The patient has a palpable thrill felt over the left anterior chest. d. The patient has 4+ peripheral edema in both legs.

B Rationale: Crackles that are audible throughout the lungs indicate that the patient is experiencing severe left ventricular failure and needs immediate interventions such as diuretics. A diastolic murmur and palpable thrill would be expected in a patient with mitral stenosis. Although 4+ peripheral edema indicates a need for a change in therapy, it does not need to be addressed urgently. Cognitive Level: Application Text Reference: p. 884 Nursing Process: Assessment

Physiological Integrity 8. A patient who has mild persistent asthma uses an albuterol (Proventil) inhaler for chest tightness and wheezing has a new prescription for cromolyn (Intal). To increase the patient's management and control of the asthma, the nurse should teach the patient to a. use the cromolyn when the albuterol does not relieve symptoms. b. use the cromolyn to prevent inflammatory airway changes. c. administer the cromolyn first for chest tightness or wheezing. d. administer the albuterol regularly to prevent airway inflammation.

B Rationale: Cromolyn is prescribed to reduce airway inflammation. It takes several weeks for maximal effect and is not used to treat acute asthma symptoms Albuterol is used as a rescue medication in mild persistent asthma and will not decrease inflammation. Cognitive Level: Application Text Reference: p. 620 Nursing Process: Implementation

Physiological Integrity 10. A patient with an acute attack of asthma comes to the emergency department, where ABGs are drawn. The nurse determines the patient is in the early phase of the attack, based on the ABG results of a. pH 7.0, PaCO2 50 mm Hg, and PaO2 74 mm Hg. b. pH 7.4, PaCO2 32 mm Hg, and PaO2 70 mm Hg. c. pH 7.36, PaCO2 40 mm Hg, and PaO2 80 mm Hg. d. pH 7.32, PaCO2 58 mm Hg, and PaO2 60 mm Hg.

B Rationale: The initial response to hypoxemia caused by airway narrowing in a patient having an acute asthma attack is an increase in respiratory rate, which causes a drop in PaCO2. The other PaCO2 levels are normal or elevated, which would indicate that the attack was progressing and that the patient is decompensating. Cognitive Level: Application Text Reference: pp. 614, 626 Nursing Process: Assessment

Physiological Integrity 28. A few days after experiencing an MI, the patient states, "I just had a little chest pain. As soon as I get out of here, I'm going for my vacation as planned." Which nursing intervention is appropriate to include in the nursing care plan? a. Have the family members encourage the patient to continue planning for the vacation. b. Allow the use of denial as a coping mechanism until the patient begins asking questions about the MI. c. Implement reality orientation by reminding the patient several times a day about the MI. d. Begin teaching the patient about the normal functions of the heart to improve understanding of the MI.

B Rationale: Denial is a normal coping mechanism after an acute episode like an MI; waiting until the patient asks questions will improve the patient's ability to take in needed information. The patient should not be encouraged to leave for a vacation during the MI recovery period. Reminding the patient about the MI is likely to make the patient angry and lead to distrust of the nursing staff. The patient in denial will not be interested in learning about the normal functions of the heart. Cognitive Level: Application Text Reference: p. 814 Nursing Process: Planning

Physiological Integrity 3. A patient has a 5-cm thoracic aortic aneurysm that was discovered during a routine chest x-ray. When obtaining a nursing history from the patient, the nurse will ask the patient about a. abdominal tenderness. b. difficulty swallowing. c. changes in bowel habits. d. dizziness or weakness.

B Rationale: Difficulty swallowing may occur with a thoracic aneurysm because of pressure on the esophagus. Abdominal tenderness or changes in bowel habits are consistent with an abdominal aneurysm. Dizziness or weakness may occur if there is blood loss from the aneurysm, but this aneurysm was discovered accidentally, not because the patient was symptomatic. Cognitive Level: Application Text Reference: p. 895 Nursing Process: Assessment

Physiological Integrity 16. The nurse identifies the nursing diagnosis of imbalanced nutrition: less than body requirements for a patient with COPD. An appropriate intervention for this problem is to a. have the patient exercise for 10 minutes before meals. b. offer high calorie snacks between meals and at bedtime. c. assist the patient in choosing foods with a lot of texture. d. increase the patient's intake of fruits and fruit juices.

B Rationale: Eating small amounts more frequently (as occurs with snacking) will increase caloric intake by decreasing the fatigue and feelings of fullness associated with large meals. Patients with COPD should rest before meals. Foods that have a lot of texture may take more energy to eat and lead to decreased intake. Although fruits and juices are not contraindicated, foods high in protein are a better choice. Cognitive Level: Application Text Reference: pp. 649,652 Nursing Process: Planning

Physiological Integrity 8. A patient has a normal cardiac rhythm strip except that the PR interval is 0.34 seconds. The appropriate intervention by the nurse is to a. prepare the patient for temporary pacemaker insertion. b. document the finding and continue to monitor the patient. c. notify the health care provider immediately. d. administer atropine per protocol.

B Rationale: First-degree atrioventricular (AV) block is asymptomatic and requires ongoing monitoring because it may progress to more serious forms of heart block. The rate is normal, so there is no indication that atropine or a pacemaker is needed. Immediate notification of the health care provider about an asymptomatic rhythm is not necessary. Cognitive Level: Application Text Reference: p. 853 Nursing Process: Implementation

Physiological Integrity 35. Which statement by the COPD patient indicates that the nurse's teaching about nutrition has been effective? a. "I will drink lots of fluids with my meals." b. "I will have ice cream as a snack every day." c. "I should exercise for 15 minutes before meals." d. "I should avoid much meat or dairy products."

B Rationale: High-calorie foods like ice cream are an appropriate snack for patients with COPD. Fluid intake of 3 L/day is recommended, but fluids should be taken between meals rather than with meals to improve oral intake of solid foods. The patient should avoid exercise for an hour before meals to prevent fatigue while eating. Meat and dairy products are high in protein and are good choices for the patient with COPD. Cognitive Level: Application Text Reference: p. 649 Nursing Process: Evaluation

Physiological Integrity 27. When caring for a patient who has survived a sudden cardiac death (SCD) event and has no evidence of an AMI, the nurse will anticipate teaching the patient a. that sudden cardiac death events rarely reoccur. b. about the purpose of outpatient Holter monitoring. c. how to self-administer low-molecular-weight heparin. d. to limit activities after discharge to prevent future events.

B Rationale: Holter monitoring is used to determine whether the patient is experiencing dysrhythmias such as ventricular tachycardia during normal daily activities. SCD is likely to recur. Heparin will not have any effect on the incidence of SCD, which can occur even when the patient is resting. Cognitive Level: Application Text Reference: p. 818 Nursing Process: Planning

Physiological Integrity 5. When the nurse is developing a teaching plan to prevent the development of heart failure in a patient with stage 1 hypertension, the information that is most likely to improve compliance with antihypertensive therapy is that a. hypertensive crisis may lead to development of acute heart failure in some patients. b. hypertension eventually will lead to heart failure by overworking the heart muscle. c. high BP increases risk for rheumatic heart disease. d. high systemic pressure precipitates papillary muscle rupture.

B Rationale: Hypertension is a primary cause of heart failure because the increase in ventricular afterload leads to ventricular hypertrophy and dilation. Hypertensive crisis may precipitate acute heart failure is some patients, but this patient with stage 1 hypertension may not be concerned about a crisis that happens only to some patients. Hypertension does not directly cause rheumatic heart disease (which is precipitated by infection with group A -hemolytic streptococcus) or papillary muscle rupture (which is caused by myocardial infarction/necrosis of the papillary muscle). Cognitive Level: Application Text Reference: p. 822 Nursing Process: Planning

Physiological Integrity 8. On auscultation of a patient's lungs, the nurse hears short, high-pitched sounds just before the end of inspiration in the right and left lower lobes. The nurse records this finding as a. abnormal lung sounds in the bases of both lungs. b. inspiratory wheezes in both lungs. c. crackles in the right and left lower lobes. d. pleural friction rub in the right and left lower lobes.

B Rationale: Wheezes are high-pitched sounds; in this case, they are heard during the inspiratory phase of the respiratory cycle. Abnormal breath sounds are bronchial or bronchovesicular sounds heard in the peripheral lung fields. Crackles are low-pitched, "bubbling' sounds. Pleural friction rubs are grating sounds that are usually heard during both inspiration and expiration. Cognitive Level: Comprehension Text Reference: p. 525 Nursing Process: Assessment

Physiological Integrity 21. A patient with chronic hypoxemia (SaO2 levels of 89%-90%) caused by COPD has just been admitted with increasing shortness of breath. In planning for discharge, which of these actions by the nurse will be most effective in improving compliance with discharge teaching? a. Have the patient repeat the instructions immediately after the teaching. b. Arrange for the patient's spouse to be present during the teaching. c. Accomplish the patient teaching just before the scheduled discharge. d. Start giving the patient discharge teaching on the day of admission.

B Rationale: Hypoxemia interferes with the patient's ability to learn and retain information, so having the patient's spouse present will increase the likelihood that discharge instructions will be followed. Having the patient repeat the instructions will indicate that the information is understood at the time, but it does not guarantee retention of the information. Because the patient is likely to be distracted just before discharge, giving discharge instructions just before discharge is not ideal. The patient is likely to be anxious and even more hypoxemic than usual on the day of admission, so teaching about discharge should be postponed. Cognitive Level: Application Text Reference: p. 520 Nursing Process: Planning

Physiological Integrity 7. A nursing action that is indicated for the collaborative problem of potential complication: cardiac dysrhythmia in a patient who has had a repair of a descending thoracic aortic aneurysm is to a. assess level of consciousness and orientation hourly. b. titrate oxygen to keep O2 saturation greater than 90%. c. turn the patient every 1 to 2 hours while on bed rest. d. monitor hourly fluid intake and urine output levels.

B Rationale: Hypoxemia may precipitate dysrhythmias in patients after aneurysm repair. Neurologic assessments, turning the patient, and monitoring intake and output are also appropriate nursing actions after aneurysm repair but will not have an effect on dysrhythmias. Cognitive Level: Application Text Reference: p. 897 Nursing Process: Implementation

Physiological Integrity 19. A patient who has a history of sudden cardiac death has an ICD inserted. When performing discharge teaching with the patient, it is important for the nurse to instruct the patient and family that a. medications will no longer be needed to control dysrhythmias. b. if the ICD fires and the patient loses consciousness, 911 should be called. c. CPR may displace the ICD leads and should not be performed. d. the ICD rarely triggers airport security alarms and travel without restrictions is allowed.

B Rationale: If the ICD fires and the patient continues to have symptoms of cardiac arrest, activation of the emergency response system is indicated. The patient is likely to continue on medications to control dysrhythmias. If the patient experiences cardiac arrest, CPR should be performed. ICDs do trigger airport security alarms, and the patient will need to notify airport personnel about the presence of the device. Cognitive Level: Application Text Reference: p. 858 Nursing Process: Implementation

Physiological Integrity 23. A 21-year-old college student arrives at the student health center at the end of the quarter complaining, "My heart is skipping beats." The nurse obtains an ECG and notes the presence of occasional PVCs. What action should the nurse take first? a. Ask the patient about any history of coronary artery disease. b. Question the patient about current stress level and coffee use. c. Have the patient transported to the hospital ED. d. Administer O2 to the patient at 2 to 3 L/min using nasal prongs.

B Rationale: In a patient with a normal heart, occasional PVCs are a benign finding. The timing of the PVCs suggests stress or caffeine as possible etiologic factors. It is unlikely that the patient has coronary artery disease, and this should not be the first question the nurse asks. There is no indication that the patient needs to be seen in the ED or that oxygen needs to be administered. Cognitive Level: Application Text Reference: pp. 847, 854 Nursing Process: Implementation

Physiological Integrity 24. A 19-year-old student has a mandatory ECG before participating on a college swim team and is found to have sinus bradycardia, rate 52. BP is 114/54, and the student denies any health problems. What action by the nurse is appropriate? a. Refer the student to a cardiologist for further assessment. b. Allow the student to participate on the swim team. c. Obtain more detailed information about the student's health history. d. Tell the student to stop swimming immediately if any dyspnea occurs.

B Rationale: In an aerobically trained individual, sinus bradycardia is normal. The student's normal BP and negative health history indicate that there is no need for a cardiology referral or for more detailed information about the health history. Dyspnea during an aerobic activity such as swimming is normal. Cognitive Level: Application Text Reference: p. 849 Nursing Process: Implementation

Safe and Effective Care Environment 38. Which information given by an asthmatic patient during the admission assessment will be of most concern to the nurse? a. The patient says that the asthma symptoms are worse every spring. b. The patient's only asthma medications are albuterol (Proventil) and salmeterol (Serevent). c. The patient uses cromolyn (Intal) before any aerobic exercise. d. The patient's heart rate increases after using the albuterol (Proventil) inhaler.

B Rationale: Long-acting 2-agonists should be used only in patients who are also using another medication for long-term control (typically an inhaled corticosteroid). Salmeterol should not be used as the first-line therapy for long-term control. The other information given by the patient requires further assessment by the nurse but is not unusual for a patient with asthma. Cognitive Level: Application Text Reference: pp. 615, 621 Nursing Process: Assessment

Health Promotion and Maintenance 25. A lobectomy is scheduled for a patient with stage I non-small cell lung cancer. The patient tells the nurse, "I would rather have radiation than surgery." Which response by the nurse is most appropriate? a. "Are you afraid that the surgery will be very painful?" b. "Tell me what you know about the various treatments available." c. "Surgery is the treatment of choice for stage I lung cancer." d. "Did you have bad experiences with previous surgeries?"

B Rationale: More assessment of the patient's concerns about surgery is indicated; an open-ended response will elicit the most information from the patient. The answer beginning, "Surgery is the treatment of choice" is accurate, but it discourages the patient from sharing concerns about surgery. The remaining two answers indicate that the nurse has jumped to conclusions about the patient's reasons for not wanting surgery. Cognitive Level: Application Text Reference: pp. 583-584 Nursing Process: Implementation

Physiological Integrity 4. The health care provider has prescribed triamcinolone (Azmacort) metered-dose inhaler (MDI) two puffs every 8 hours and pirbuterol (Maxair) MDI 2 puffs four times a day for a patient with asthma. In teaching the patient about the use of the inhalers, the best instruction by the nurse is a. "Use the Maxair inhaler first, wait a few minutes, then use the Azmacort inhaler." b. "Using a spacer with the MDIs will improve the inhalation of the medications." c. "To avoid side effects, the inhalers should not be used within 1 hour of each other." d. "To maximize the effectiveness of the drugs, inhale quickly when using the inhalers."

B Rationale: More medication reaches the bronchioles when a spacer is used along with an MDI. There is no evidence that using a bronchodilator before a corticosteroid inhaler is helpful. The medications can be used at the same time. The patient should inhale slowly when using an MDI. Cognitive Level: Application Text Reference: p. 621 Nursing Process: Implementation

Physiological Integrity 7. The nurse notes new-onset confusion in an 89-year-old patient in a long-term-care facility; the patient is normally alert and oriented. Which action should the nurse take next? a. Check the patient's pulse rate. b. Obtain an oxygen saturation. c. Notify the health care provider. d. Document the change.

B Rationale: New-onset confusion caused by hypoxia may be the first sign of pneumonia in older patients. The other actions are also appropriate in this order: check the pulse, notify the health care provider, and document the change in status. Cognitive Level: Application Text Reference: p. 565 Nursing Process: Implementation

Physiological Integrity 8. A patient admitted to the hospital with an exacerbation of chronic heart failure tells the nurse, "I felt fine when I went to bed, but I woke up in the middle of the night feeling like I was suffocating!" The nurse can best document this assessment information as a. pulsus alternans. b. paroxysmal nocturnal dyspnea. c. two-pillow orthopnea. d. acute bilateral pleural effusion.

B Rationale: Paroxysmal nocturnal dyspnea is caused by the reabsorption of fluid from dependent body areas when the patient is sleeping and is characterized by waking up suddenly with the feeling of suffocation. Pulsus alternans is the alternation of strong and weak peripheral pulses during palpation. Orthopnea indicates that the patient is unable to lie flat because of dyspnea. Pleural effusions develop over a longer time period. Cognitive Level: Comprehension Text Reference: p. 825 Nursing Process: Assessment

Physiological Integrity 16. After teaching a patient with newly diagnosed Raynaud's phenomenon about how to manage the condition, which behavior by the patient indicates that the teaching has been effective? a. The patient places the hands in hot water when they turn pale. b. The patient exercises indoors during the winter months. c. The patient takes pseudoephedrine (Sudafed) for cold symptoms. d. The patient avoids the use of aspirin and the NSAIDs.

B Rationale: Patients should avoid temperature extremes by exercising indoors when it is cold. To avoid burn injuries, the patient should use warm, rather than hot, water to warm the hands. Pseudoephedrine is a vasoconstrictor and should be avoided. There is no reason to avoid taking aspirin and NSAIDs with Raynaud's phenomenon. Cognitive Level: Application Text Reference: p. 909 Nursing Process: Implementation

Physiological Integrity 21. The nurse has initiated discharge teaching for a patient who is to be maintained on warfarin (Coumadin) following hospitalization for DVT. The nurse determines that additional teaching is needed when the patient says, a. "I should wear a Medic Alert bracelet to indicate I am on anticoagulant therapy." b. "I should change my diet to include more green, leafy vegetables." c. "I will check with my health care provider before I begin or stop any medication." d. "I will need to have blood tests routinely to monitor the effects of the Coumadin."

B Rationale: Patients taking Coumadin are taught to follow a consistent diet with regard to foods that are high in vitamin K, such as green leafy vegetables. The other patient statements are accurate. Cognitive Level: Application Text Reference: p. 917 Nursing Process: Evaluation

Physiological Integrity 3. During assessment of the chest in a patient with pneumococcal pneumonia, the nurse would expect to find a. hyperresonance on percussion. b. increased vocal fremitus on palpation. c. fine crackles in all lobes on auscultation. d. asymmetric chest expansion on inspection.

B Rationale: Pneumonias caused by Streptococcus pneumoniae are typically lobar or segmental. The nurse would expect to find increased vocal fremitus over the affected area of the lungs. The area would be dull to percussion. Fine crackles in all lobes would indicate a diffuse infection, which is more typical of viral pneumonias. Asymmetric chest expansion is not typical with pneumonia. Cognitive Level: Application Text Reference: p. 565 Nursing Process: Assessment

Health Promotion and Maintenance 11. When developing a plan to decrease preload in the patient with heart failure, the nurse will include actions such as a. administering sedatives to promote rest and decrease myocardial oxygen demand. b. positioning the patient in a high-Fowler's position with the feet horizontal in the bed. c. administering oxygen per mask or nasal cannula. d. encouraging leg exercises to improve venous return.

B Rationale: Positioning the patient in a high-Fowler's position with the legs dependent will reduce preload by decreasing venous return to the right atrium. The other interventions may also be appropriate for patients with heart failure but will not help in decreasing preload. Cognitive Level: Application Text Reference: pp. 827-828 Nursing Process: Planning

Physiological Integrity 12. Nifedipine (Procardia) is ordered for a patient with newly diagnosed Prinzmetal's (variant) angina. When teaching the patient, the nurse will include the information that Procardia will a. help to prevent clotting in the coronary arteries. b. decrease spasm of the coronary arteries. c. increase the force of myocardial contraction. d. reduce the "fight or flight" response.

B Rationale: Prinzmetal's angina is caused by coronary artery spasm. Calcium-channel blockers (e.g., nifedipine) are a first-line therapy for this type of angina. Platelet inhibitors, such as aspirin, help to prevent coronary artery thrombosis, and -blockers decrease sympathetic stimulation of the heart. Medications or activities that increase myocardial contractility will increase the incidence of angina by increasing oxygen demand. Cognitive Level: Application Text Reference: p. 801 Nursing Process: Implementation

Safe and Effective Care Environment 26. Which statement by a patient who is being discharged 5 days after an abdominal aortic aneurysm repair and graft indicates that the discharge teaching has been effective? a. "I will call the doctor if my temperature is higher than 101° F." b. "I will tell my dentist about this surgery the next time I have an appointment." c. "I should not need to take anything but acetaminophen (Tylenol) for my pain." d. "I am eager to get home so that I can pick up my 6-year-old granddaughter."

B Rationale: Prophylactic antibiotics may be ordered to prevent graft infection when the patient has any invasive procedures, including dental procedures. The patient is instructed to call if the temperature is higher than 100° F. After abdominal surgery, patients may need to use opioid pain medications on a PRN basis. Because heavy lifting is avoided for at least 4 to 6 weeks after surgery, the patient should not pick up a 6-year-old child. Cognitive Level: Application Text Reference: p. 898 Nursing Process: Evaluation

Physiological Integrity 5. A patient who has a 30-pack-year history of smoking asks the nurse, "How does smoking really harm my lungs?" The nurse's response will be based on the effect of smoking on a. cough and gag reflexes. b. mucociliary clearance. c. reflex bronchoconstriction. d. the filtration of inspired air.

B Rationale: Smoking decreases ciliary action and the ability of the mucociliary clearance system to trap particles and move them out of the lung. The cough/gag reflexes, reflex bronchoconstriction, and filtration of air by the nasal hairs are not affected by smoking. Cognitive Level: Comprehension Text Reference: p. 516 Nursing Process: Assessment

Physiological Integrity 7. Intravenous sodium nitroprusside (Nipride) is ordered for a patient with acute pulmonary edema. During the first hours of administration, the nurse will need to adjust the Nipride rate if the patient develops a. a drop in heart rate to 54 beats/min. b. a systolic BP <90 mm Hg. c. any symptoms indicating cyanide toxicity. d. an increased amount of ventricular ectopy.

B Rationale: Sodium nitroprusside is a potent vasodilator, and the major adverse effect is severe hypotension. After 48 hours of continuous use, cyanide toxicity is a possible (though rare) adverse effect. Reflex tachycardia (not bradycardia) is another adverse effect of this medication. Nitroprusside does not cause increased ventricular ectopy. Cognitive Level: Application Text Reference: p. 828 Nursing Process: Evaluation

Physiological Integrity 22. A patient is admitted to the emergency department complaining of sudden onset shortness of breath and diagnosed with a possible pulmonary embolus. To confirm the diagnosis, the nurse will anticipate preparing the patient for a a. chest x-ray. b. spiral CT scan. c. bronchoscopy. d. PET scan.

B Rationale: Spiral CT scans are the most commonly used test to diagnose pulmonary emboli. A chest x-ray may be ordered but will not be diagnostic for a pulmonary embolus. Bronchoscopy is used to inspect for changes in the bronchial tree, not to assess for vascular changes. PET scans are most useful in determining the presence of malignancy. Cognitive Level: Application Text Reference: p. 526 Nursing Process: Planning

Physiological Integrity 12. A patient who is restricted to bed rest asks the nurse the purpose of the deep breathing exercises. Which reply by the nurse is correct? a. Deep breathing enhances ciliary activity and promotes bronchial clearance. b. Deep breathing stretches the alveoli and stimulates the production of surfactant. c. Deep breathing increases the diaphragmatic strength improving respiratory effort. d. Deep breathing stimulates the Hering-Breuer reflex to increase respiratory rate.

B Rationale: Taking deep breaths or sighs usually occurs every five to six breaths and (through alveolar stretching and improved surfactant production) decreases the risk for atelectasis. Ciliary activity, diaphragmatic strength, and the respiratory rate are not changed by deep breathing. Cognitive Level: Application Text Reference: p. 511 Nursing Process: Implementation

Physiological Integrity 10. The nurse working in the heart failure clinic will know that teaching for a 74-year-old patient with newly diagnosed heart failure has been effective when the patient a. says that the nitroglycerin patch will be used for any chest pain that develops. b. calls when the weight increases from 124 to 130 pounds in a week. c. tells the home care nurse that furosemide (Lasix) is taken daily at bedtime. d. makes an appointment to see the doctor at least once yearly.

B Rationale: Teaching for a patient with heart failure includes information about the need to weigh daily and notify the health care provider about an increase of 3 pounds in 2 days or 5 pounds in a week. Nitroglycerin patches are used primarily to reduce preload (not to prevent chest pain) in patients with heart failure and should be used daily, not on an "as necessary" basis. Diuretics should be taken earlier in the day to avoid nocturia and sleep disturbance. Heart failure is a chronic condition that will require frequent follow-up rather than an annual health care provider examination. Cognitive Level: Application Text Reference: pp. 826, 833-834, 838 Nursing Process: Evaluation

Physiological Integrity 9. Which of these assessment data obtained by the nurse when assessing a patient with acute pericarditis should be reported immediately to the health care provider? a. Blood pressure (BP) of 166/96 b. Jugular vein distension (JVD) to the level of the jaw c. Pulsus paradoxus 8 mm Hg d. Level 6/10 chest pain with deep inspiration

B Rationale: The JVD indicates that the patient may have developed cardiac tamponade and may need rapid intervention to maintain adequate cardiac output. Hypertension would not be associated with complications of pericarditis, and the BP is not high enough to indicate that there is any immediate need to call the health care provider. A pulsus paradoxus of 8 mm Hg is normal. Level 6/10 chest pain should be treated but is not unusual with pericarditis. Cognitive Level: Application Text Reference: p. 872 Nursing Process: Assessment

Physiological Integrity 5. During a physical examination of a patient, the nurse palpates the PMI in the sixth intercostal space lateral to the midclavicular line. The most appropriate action for the nurse to take next will be to a. document that the PMI is in the normal location. b. assess the patient for symptoms of left ventricular hypertrophy. c. ask the patient about risk factors for coronary artery disease. d. auscultate both the carotid arteries for a bruit.

B Rationale: The PMI should be felt at the intersection of the 5th intercostal space and the midclavicular line. A PMI located outside these landmarks indicates possible cardiac enlargement, such as with left ventricular hypertrophy. Cardiac enlargement is not necessarily associated with coronary or carotid artery disease. Cognitive Level: Application Text Reference: pp. 750-751 Nursing Process: Assessment

Physiological Integrity Lewis: Medical-Surgical Nursing, 7th Edition Test Bank Chapter 36: Nursing Management: Dysrhythmias MULTIPLE CHOICE 1. In analyzing a patient's electrocardiographic (ECG) rhythm strip, the nurse uses the knowledge that the time of the conduction of an impulse through the Purkinje fibers is represented by the a. P wave. b. PR interval. c. QRS complex. d. QT interval.

B Rationale: The PR interval represents depolarization of the atria, AV node, bundle of His, bundle branches, and the Purkinje fibers, up to the point of depolarization of the ventricular cells. The P wave represents atrial depolarization. The QRS represents ventricular depolarization. The QT interval represents depolarization of the depolarization and repolarization of the entire conduction system. Cognitive Level: Comprehension Text Reference: pp. 846-847 Nursing Process: Assessment

Physiological Integrity 9. A finding indicating to the nurse that a 22-year-old patient with respiratory distress is in acute respiratory failure includes a a. shallow breathing pattern. b. partial pressure of arterial oxygen (PaO2) of 45 mm Hg. c. partial pressure of carbon dioxide in arterial gas (PaCO2) of 34 mm Hg. d. respiratory rate of 32/min.

B Rationale: The PaO2 indicates severe hypoxemia and that the nurse should take immediate action to correct this problem. Shallow breathing, rapid respiratory rate, and low PaCO2 can be caused by other factors, such as anxiety or pain. Cognitive Level: Application Text Reference: p. 1806 Nursing Process: Assessment

Physiological Integrity 10. The nurse reviews data from the cardiac monitor indicating that a patient with a myocardial infarction experienced a 50-second episode of ventricular tachycardia before a sinus rhythm and a heart rate of 98 were re-established. The most appropriate initial action by the nurse is to a. notify the health care provider. b. administer IV antidysrhythmic drugs per protocol. c. defibrillate the patient. d. document the rhythm and continue to monitor the patient.

B Rationale: The burst of sustained ventricular tachycardia indicates that the patient has significant ventricular irritability, and antidysrhythmic medication administration is needed to prevent further episodes. The nurse should notify the health care provider after the medications are administered. Defibrillation is not indicated given that the patient is currently in a sinus rhythm. Documentation and continued monitoring are not adequate responses to this situation. Cognitive Level: Application Text Reference: p. 855 Nursing Process: Implementation

Physiological Integrity 17. The RN is observing a student nurse who is doing a physical assessment on a patient. The RN will need to intervene immediately if the student nurse a. presses on the skin over the tibia for 10 seconds to check for edema. b. palpates both carotid arteries simultaneously to compare pulse quality. c. places the patient in the left lateral position to check for the PMI. d. uses the palm of the hand to assess extremity skin temperature.

B Rationale: The carotid pulses should never be palpated at the same time to avoid vagal stimulation, dysrhythmias, and decreased cerebral blood flow. The other assessment techniques also need to be corrected; however, they are not dangerous to the patient. Cognitive Level: Application Text Reference: p. 748 Nursing Process: Assessment

Physiological Integrity 15. While admitting an 80-year-old patient with heart failure to the medical unit, the nurse obtains the information that the patient lives alone and sometimes confuses the "water pill" with the "heart pill." The nurse makes a note that discharge planning for the patient will need to include a. transfer to a dementia care service. b. referral to a home health care agency. c. placement in a long-term-care facility. d. arrangements for around-the-clock care.

B Rationale: The data about the patient suggest that assistance in developing a system for taking medications correctly at home is needed. A home health nurse will assess the patient's home situation and help the patient to develop a method for taking the two medications as directed. There is no evidence that the patient requires services such as dementia care, long-term-care, or around-the-clock home care. Cognitive Level: Application Text Reference: pp. 836-837 Nursing Process: Assessment

Physiological Integrity 13. When auscultating a patient's chest while the patient takes a deep breath, the nurse hears loud, high-pitched, "blowing" sounds at both lung bases. The nurse will document these as a. adventitious sounds. b. abnormal sounds. c. vesicular sounds. d. normal sounds.

B Rationale: The description indicates that the nurse hears bronchial breath sounds that are abnormal when heard at the lung base. Adventitious sounds are crackles, wheezes, rhonchi, and friction rubs. Vesicular sounds are low-pitched, soft sounds heard over all lung areas except the major bronchi. Cognitive Level: Application Text Reference: p. 524 Nursing Process: Assessment

Physiological Integrity 25. When developing a teaching plan to help increase activity tolerance at home for a 70-year-old with severe COPD, the nurse should teach the patient that an appropriate exercise goal is to a. exercise until shortness of breath occurs. b. walk for a total of 20 minutes daily. c. limit exercise to activities of daily living (ADLs). d. walk until pulse rate exceeds 150.

B Rationale: The goal for exercise programs for patients with COPD is to increase exercise time gradually to a total of 20 minutes daily. Shortness of breath is normal with exercise and not an indication that the patient should stop. Limiting exercise to ADLs will not improve the patient's exercise tolerance. A 70-year-old patient should have a pulse rate of 120 or less with exercise (80% of the maximal heart rate of 150). Cognitive Level: Application Text Reference: p. 653 Nursing Process: Planning

Physiological Integrity 36. The nurse is caring for a patient with primary pulmonary hypertension who is receiving epoprostenol (Flolan). Which assessment information requires the most immediate action? a. The international normalized ratio (INR) is prolonged. b. The central line is disconnected. c. The oxygen saturation is 90%. d. The BP is 88/56.

B Rationale: The half-life of this drug is 6 minutes, so the nurse will need to restart the infusion as soon as possible to prevent rapid clinical deterioration. The other data also indicate a need for ongoing monitoring or intervention, but the priority action is to reconnect the infusion. Cognitive Level: Application Text Reference: p. 601 Nursing Process: Assessment

Physiological Integrity 29. The health carre provider inserts two chest tubes connected with a Y-connecter in a patient with a hemopneumothorax. When monitoring the patient after the chest tube placement, the nurse will be most concerned about a. a large air leak in the water-seal chamber. b. 400 ml of blood in the collection chamber. c. severe pain with each deep patient inspiration. d. subcutaneous emphysema at the insertion site.

B Rationale: The large amount of blood may indicate that the patient is in danger of developing hypovolemic shock. A large air leak would be expected immediately after chest tube placement for pneumothorax. The severe pain should be treated but is not as urgent a concern as the possibility of continued hemorrhage. Subcutaneous emphysema should be monitored but is not unusual in a patient with pneumothorax. Cognitive Level: Application Text Reference: p. 586 Nursing Process: Assessment

Physiological Integrity 42. All of the following information is obtained by the nurse who is caring for a patient receiving subcutaneous heparin injections to treat a pulmonary embolus. Which assessment data is most important to communicate to the health care provider? a. The patient has many abdominal bruises. b. The patient's BP is 90/46. c. The activated partial thromboplastin time is 2 times the patient baseline. d. The patient's stool is dark green and liquid.

B Rationale: The low BP may indicate that the patient is experiencing bleeding, a possible adverse effect of heparin therapy. Subcutaneous heparin administration is given into the subcutaneous tissue of the abdomen and abdominal bruising is not unusual. An aPTT 2 times the baseline indicates a therapeutic heparin level. The patient should be monitored for gastrointestinal bleeding, which would be indicated by black or red stools. Cognitive Level: Application Text Reference: p. 600 Nursing Process: Assessment

Physiological Integrity 27. A patient with newly diagnosed lung cancer tells the nurse, "I think I am going to die pretty soon, maybe this week." The best response by the nurse is a. "Are you afraid that the treatment for your cancer will not be effective?" b. "Can you tell me what it is that makes you think you will die so soon?" c. "Would you like to talk to the hospital chaplain about your feelings?" d. "Do you think that taking an antidepressant medication would be helpful?"

B Rationale: The nurse's initial response should be to collect more assessment data about the patient's statement. The answer beginning "Can you tell me what it is" is the most open-ended question and will offer the best opportunity for obtaining more data. The answer beginning, "Are you afraid" implies that the patient thinks that the cancer will be immediately fatal, although the patient's statement may not be related to the cancer diagnosis. The remaining two answers offer interventions that may be helpful to the patient, but more assessment is needed to determine whether these interventions are appropriate. Cognitive Level: Application Text Reference: pp. 583-584 Nursing Process: Implementation

Physiological Integrity 23. A patient who has had recent cardiac surgery develops pericarditis and complains of severe chest pain with deep breathing. Which of these ordered PRN medications should the nurse administer? a. Oral acetaminophen (Tylenol) 650 mg b. Oral ibuprofen (Motrin) 800 mg c. IV morphine sulfate 6 mg d. Fentanyl 2 mg IV

B Rationale: The pain associated with pericarditis is caused by inflammation, so nonsteroidal antiinflammatory medications like ibuprofen are most effective. The patient does not need opioid pain medication. Cognitive Level: Application Text Reference: p. 873 Nursing Process: Implementation

Physiological Integrity 8. When the nurse is caring for a patient on the first postoperative day after an abdominal aortic aneurysm repair, the information that is most significant when the nurse is assessing for the return of peristalsis is a. absence of abdominal distention. b. passing of flatus with ambulation. c. dark brown nasogastric (NG) tube drainage. d. moderate abdominal tenderness.

B Rationale: The passing of flatus is the best indicator of returning bowel function. Because the patient usually has an NG tube in place, the absence of abdominal distention is not indicative of gastrointestinal (GI) function. The quality of NG tube drainage may indicate the presence of complications such as GI bleeding but does not provide useful information about GI function. Moderate abdominal tenderness is expected after abdominal surgery. Cognitive Level: Application Text Reference: p. 898 Nursing Process: Assessment

Physiological Integrity 13. A patient who has active TB has just been started on drug therapy for TB. The nurse informs the patient that the disease can be transmitted to others until a. the chest x-ray shows resolution of the tuberculosis. b. three sputum smears for acid-fast bacilli are negative. c. TB medications have been taken for 6 months. d. sputum cultures on 3 consecutive days are negative.

B Rationale: The patient is considered infectious until three sputum smears are negative for acid-fast bacilli. Chest x-rays help to determine the presence of active TB but are not utilized to monitor the effectiveness of treatment. Taking the medications for 6 months is necessary, but the multidrug-resistant forms of the disease might not be eradicated after 6 months of therapy. Sputum cultures are used to diagnose the presence of active TB, but sputum smears are usually done to establish that treatment has been effective. Cognitive Level: Application Text Reference: p. 574 Nursing Process: Implementation

Physiological Integrity 22. A patient develops sinus bradycardia at a rate of 32 beats/min, has a BP of 80/36 mm Hg, and is complaining of feeling faint. Which action should the nurse take? a. Continue to monitor the rhythm and BP. b. Obtain and apply the transcutaneous pacemaker (TCP). c. Give the scheduled dose of diltiazem (Cardizem). d. Have the patient perform the Valsalva maneuver.

B Rationale: The patient is experiencing symptomatic bradycardia and treatment with TCP is appropriate. Continued monitoring of the rhythm and BP is an inadequate response. Calcium-channel blockers will further decrease the heart rate, and the diltiazem should be held. The Valsalva maneuver will further decrease the rate. Cognitive Level: Application Text Reference: p. 859 Nursing Process: Implementation

Physiological Integrity 14. A patient who has had a femoral-popliteal bypass graft to the right leg is being cared for on the surgical unit. Which action by an LPN/LVN caring for the patient requires the RN to intervene? a. The LPN/LVN administers the ordered aspirin 160 mg after breakfast. b. The LPN/LVN has the patient sit in a bedside chair for 90 minutes. c. The LPN/LVN assists the patient to ambulate 40 feet in the hallway. d. The LPN/LVN places the patient in a Fowler's position for meals.

B Rationale: The patient should avoid sitting for long periods because of the increased stress on the suture line caused by leg edema and because of the risk for DVT. The other actions by the LPN/LVN are appropriate. Cognitive Level: Application Text Reference: p. 906 Nursing Process: Implementation

Physiological Integrity 26. A patient who has developed acute pulmonary edema is hospitalized and diagnosed with dilated cardiomyopathy. Which information will the nurse plan to include when teaching the patient about management of this disorder? a. Careful compliance with diet and medications will control the patient's symptoms. b. Notify the doctor about any symptoms of heart failure such as shortness of breath. c. No more than one or two alcoholic drinks daily are permitted. d. Elevating the legs above the heart will help relieve angina.

B Rationale: The patient should be instructed to notify the health care provider about any worsening of heart failure symptoms. Because dilated cardiomyopathy does not respond well to therapy, even patients with good compliance with therapy may have recurrent episodes of heart failure. The patient is instructed to avoid alcoholic beverages. Elevation of the legs above the heart will worsen symptoms (although this approach is appropriate for a patient with hypertrophic cardiomyopathy). Cognitive Level: Application Text Reference: pp. 886, 888 Nursing Process: Planning

Physiological Integrity 13. When the nurse is caring for an obese patient with left lower-lobe pneumonia, gas exchange will be best when the patient is positioned a. on the left side. b. on the right side. c. in the high-Fowler's position. d. in the tripod position.

B Rationale: The patient should be positioned with the "good" lung in the dependent position to improve the match between ventilation and perfusion. The obese patient's abdomen will limit respiratory excursion when sitting in the high-Fowler's or tripod positions. Cognitive Level: Comprehension Text Reference: pp. 1809-1810 Nursing Process: Implementation

Physiological Integrity 7. When caring for a patient with ACS who has returned to the coronary care unit after having a PCI, the nurse obtains these assessment data. Which data indicate the need for immediate intervention by the nurse? a. Heart rate 100 beats/min b. Chest pain level 8 on a 10-point scale c. Blood pressure (BP) 104/56 mm Hg d. Pedal pulses 2+

B Rationale: The patient's chest pain may indicate that restenosis of the coronary artery is occurring and requires immediate actions, such as administration of oxygen and nitroglycerin, by the nurse. The other information indicates a need for ongoing assessments by the nurse. Cognitive Level: Application Text Reference: p. 807 Nursing Process: Assessment

Health Promotion and Maintenance 22. A few days after an acute MI, a patient complains of stabbing chest pain that increases with deep breathing. Which action will the nurse take first? a. Notify the patient's health care provider. b. Auscultate the heart sounds. c. Check the patient's oral temperature. d. Give the ordered acetaminophen (Tylenol).

B Rationale: The patient's clinical manifestations and history are consistent with pericarditis, and the first action by the nurse should be to listen for a pericardial friction rub. Checking the temperature, giving acetaminophen (Tylenol), and notifying the health care provider are also appropriate actions but would not be done before listening for a rub. Cognitive Level: Application Text Reference: p. 872 Nursing Process: Implementation

Physiological Integrity 2. A 69-year old patient is admitted to the hospital for elective repair of an abdominal aortic aneurysm. The history includes hypertension for 25 years, hyperlipidemia for 15 years, and smoking for 50 years. The patient asks the nurse what caused the aneurysm. The nurse's best response includes the information that a. congenital weakness of arterial walls eventually results in an aneurysm. b. atherosclerotic plaques damage the artery and may lead to aneurysms. c. chronic infections of blood vessel walls may have contributed to the aneurysm. d. uncontrolled hypertension, hyperlipidemia, and smoking caused the aneurysm.

B Rationale: The patient's gender, age, and risk factor history indicate that the aneurysm was likely caused by atherosclerosis rather than a congenital weakness or chronic infection. Although the patient's BP, elevated lipids, and smoking undoubtedly have contributed to the atherosclerotic process, it is not appropriate for the nurse to imply that the patient is to blame. Cognitive Level: Application Text Reference: p. 894 Nursing Process: Implementation

Psychosocial Integrity 23. The nurse obtains the vital signs for a patient admitted 2 days ago with gram-negative sepsis: temperature 101.2° F, blood pressure 90/56 mm Hg, pulse 92, respirations 34. Which action should the nurse take next? a. Notify the health care provider of the patient's vital signs. b. Obtain oxygen saturation using pulse oximetry. c. Document the vital signs and continue to monitor. d. Administer PRN acetaminophen (Tylenol) 650 mg.

B Rationale: The patient's increased respiratory rate in combination with the admission diagnosis of gram-negative sepsis indicates that acute respiratory distress syndrome (ARDS) may be developing; the nurse should check for hypoxemia, a hallmark of ARDS. The health care provider should be notified after further assessment of the patient. Documentation and continued monitoring of the vital signs are needed but do not constitute an adequate response to the patient situation. Tylenol administration is appropriate but not the highest priority for this patient. Cognitive Level: Application Text Reference: pp. 1813-1814 Nursing Process: Implementation

Physiological Integrity 11. A patient with hypercapnic respiratory failure has a respiratory rate of 8 and an SpO2 of 89%. The patient is increasingly lethargic. Which collaborative intervention will the nurse anticipate? a. Administration of 100% oxygen by non-rebreather mask b. Endotracheal intubation and positive pressure ventilation c. Insertion of a mini-tracheostomy with frequent suctioning d. Initiation of bilevel positive pressure ventilation (BiPAP)

B Rationale: The patient's lethargy, low respiratory rate, and SpO2 indicate the need for mechanical ventilation with ventilator-controlled respiratory rate. Administration of high flow oxygen will not be helpful because the patient's respiratory rate is so low. Insertion of a mini-tracheostomy will facilitate removal of secretions, but it will not improve the patient's respiratory rate or oxygenation. BiPAP requires that the patient initiate an adequate respiratory rate to allow adequate gas exchange. Cognitive Level: Application Text Reference: pp. 1807-1808, 1810 Nursing Process: Planning

Physiological Integrity 28. Which assessment finding by the nurse caring for a patient who has had coronary artery bypass grafting using a right radial artery graft is most important to communicate to the health care provider? a. Complaints of incisional chest pain b. Pallor and weakness of the right hand c. Fine crackles heard at both lung bases d. Redness on both sides of the sternal incision

B The changes in the right hand indicate compromised blood flow, which requires immediate evaluation and actions such as prescribed calcium channel blockers or surgery. The other changes are expected and/or require nursing interventions. DIF: Cognitive Level: Apply (application) REF: 752 OBJ: Special Questions: Prioritization TOP: Nursing Process: Assessment MSC:

Physiological Integrity 25. Two days after having an MI, a patient tells the nurse, "I wish I had died when I had this heart attack. I won't be able to do anything now." The most appropriate nursing diagnosis is a. ineffective coping related to depression and anxiety. b. situational low self-esteem related to perceived role changes. c. impaired adjustment related to unwillingness to alter lifestyle. d. ineffective health maintenance related to lack of knowledge.

B Rationale: The patient's statements indicate that the perceived change in role is the major concern. The patient is experiencing progression through the normal stages of loss and grief that often occur after an MI, so ineffective coping is not an appropriate diagnosis. There is no evidence to support an unwillingness to alter lifestyle or ineffective health maintenance. Cognitive Level: Application Text Reference: pp. 813-814 Nursing Process: Diagnosis

Physiological Integrity 17. When caring for a patient with mitral valve stenosis, it is most important that the nurse assess for a. angina and syncope. b. dyspnea and hemoptysis. c. JVD and peripheral edema. d. hypotension and paroxysmal nocturnal dyspnea (PND).

B Rationale: The pressure gradient changes in mitral stenosis lead to fluid backup into the lungs, resulting in dyspnea and hemoptysis. Angina and syncope are associated with aortic stenosis. JVD and peripheral edema are more common in right-sided valvular disorders. Hypotension and PND suggest aortic regurgitation. Cognitive Level: Application Text Reference: p. 879 Nursing Process: Assessment

Physiological Integrity 4. When reviewing the 12-lead ECG for a healthy 86-year-old patient who is having an annual physical examination, which of these observations will be of most concern to the nurse? a. The PR interval is 0.21 seconds. b. The HR is 43 beats/min. c. There is a right bundle-branch block. d. There is a QRS duration of 0.13 seconds.

B Rationale: The resting HR does not change with aging, so the decrease in HR requires further investigation. Bundle-branch block and slight increases in PR interval or QRS duration are common in older individuals because of increases in conduction time through the AV node, the bundle of His, and the bundle branches. Cognitive Level: Application Text Reference: p. 744 Nursing Process: Assessment

Health Promotion and Maintenance 9. To protect susceptible patients in the hospital from aspiration pneumonia, the nurse will plan to a. turn and reposition immobile patients at least every 2 hours. b. position patients with altered consciousness in lateral positions. c. monitor frequently for respiratory symptoms in patients who are immunosuppressed. d. provide for continuous subglottic aspiration in patients receiving enteral feedings.

B Rationale: The risk for aspiration is decreased when patients with a decreased level of consciousness are placed in a side-lying or upright position. Frequent turning prevents pooling of secretions in immobilized patients but will not decrease the risk for aspiration in patients at risk. Monitoring of parameters such as breath sounds and oxygen saturation will help detect pneumonias in immune compromised patients, but it will not decrease the risk for aspiration. Continuous subglottic suction is recommended for intubated patients but not for all patients receiving enteral feedings. Cognitive Level: Application Text Reference: p. 567 Nursing Process: Planning

Physiological Integrity 3. The health care provider performs a thoracentesis on a patient with a right pleural effusion. In preparing the patient for the procedure, the nurse positions the patient a. supine with the head of the bed elevated 45 degrees. b. sitting upright with the arms supported on an overbed table. c. on the left side with the right arm extended above the head. d. in Trendelenburg's position with both arms extended.

B Rationale: The upright position with the arms supported increases lung expansion, allows fluid to collect at the lung bases, and expands the intercostal space so that access to the pleural space is easier. The other positions would increase the work of breathing for the patient and make it more difficult for the health care provider performing the thoracentesis. Cognitive Level: Application Text Reference: pp. 528, 530 Nursing Process: Implementation

Physiological Integrity 13. When teaching a patient with chronic obstructive pulmonary disease (COPD) about reasons to quit smoking, the nurse will explain that long-term exposure to tobacco smoke leads to a a. weakening of the smooth muscle lining the airways. b. decrease in the area available for oxygen absorption. c. lesser number of red blood cells for oxygen delivery. d. decreased production of protective respiratory secretions.

B Rationale: Tobacco smoke leads to an increase in proteolytic enzymes, which break down alveolar walls and lead to less alveolar surface area for gas exchange. Bronchial smooth muscle is not weakened by chronic smoking. Polycythemia is a common compensatory mechanism for patients with COPD. The quantity of respiratory secretions increases as a result of smoking. Cognitive Level: Application Text Reference: p. 633 Nursing Process: Implementation

Physiological Integrity 36. When teaching the patient with COPD about exercise, which information should the nurse include? a. "Stop exercising if you start to feel short of breath." b. "Use the bronchodilator before you start to exercise." c. "Breathe in and out through the mouth while you exercise." d. "Upper body exercise should be avoided to prevent dyspnea."

B Rationale: Use of a bronchodilator before exercise improves airflow for some patients and is recommended. Shortness of breath is normal with exercise and not a reason to stop. Patients should be taught to breathe in through the nose and out through the mouth (using a pursed-lip technique). Upper-body exercise can improve the mechanics of breathing in patients with COPD. Cognitive Level: Application Text Reference: p. 653 Nursing Process: Implementation

Physiological Integrity 7. The nurse determines that a patient has ventricular bigeminy when the rhythm strip indicates that a. there are pairs of wide and distorted QRS complexes. b. every other QRS complex is wide and starts prematurely. c. all QRS complexes are wide and the rate is 150 to 250 beats/min. d. there are premature QRS complexes with two different shapes.

B Rationale: Ventricular bigeminy describes a rhythm in which every other QRS complex is wide and bizarre looking. Pairs of wide QRS complexes are described as couplets. Wide QRS complexes at a rate of 150 to 250 indicate ventricular tachycardia. Wide QRS complexes with different shapes are described as multifocal premature ventricular contractions (PVCs). Cognitive Level: Comprehension Text Reference: p. 854 Nursing Process: Assessment

Psychosocial Integrity 29. When evaluating the outcomes of preoperative teaching with a patient scheduled for a coronary artery bypass graft (CABG) using the internal mammary artery, the nurse determines that additional teaching is needed when the patient says, a. "I will need to take an aspirin a day after the surgery to keep the graft open." b. "I will have incisions in my leg where they will remove the vein." c. "They will stop my heart and circulate my blood with a machine during the surgery." d. "They will use an artery near my heart to bypass the area that is obstructed."

B Rationale: When the internal mammary artery is used, there will be no need to have a saphenous vein removed from the leg. The other statements by the patient are accurate and indicate that the teaching has been effective. Cognitive Level: Application Text Reference: p. 809 Nursing Process: Evaluation

Physiological Integrity 6. A patient with a chronic cough has a bronchoscopy. After the procedure, which intervention by the nurse is most appropriate? a. Elevate the head of the bed to 80 to 90 degrees. b. Keep the patient NPO until the gag reflex returns. c. Place on bed rest for at least 4 hours after bronchoscopy. d. Notify the health care provider about blood-tinged mucus.

B Risk for aspiration and maintaining an open airway is the priority. Because a local anesthetic is used to suppress the gag/cough reflexes during bronchoscopy, the nurse should monitor for the return of these reflexes before allowing the patient to take oral fluids or food. Blood-tinged mucus is not uncommon after bronchoscopy. The patient does not need to be on bed rest, and the head of the bed does not need to be in the high-Fowler's position. DIF: Cognitive Level: Apply (application) REF: 492 TOP: Nursing Process: Planning MSC:

Physiological Integrity 23. A patient who is recovering from an acute myocardial infarction (AMI) asks the nurse about when sexual intercourse can be resumed. Which response by the nurse is best? a. "Most patients are able to enjoy intercourse without any complications." b. "Sexual activity uses about as much energy as climbing two flights of stairs." c. "The doctor will provide sexual guidelines when your heart is strong enough." d. "Holding and cuddling are good ways to maintain intimacy after a heart attack."

B Sexual activity places about as much physical stress on the cardiovascular system as most moderate-energy activities such as climbing two flights of stairs. The other responses do not directly address the patient's question or may not be accurate for this patient. DIF: Cognitive Level: Apply (application) REF: 761 TOP: Nursing Process: Implementation MSC:

Physiological Integrity 17. Which nursing action could the registered nurse (RN) working in a skilled care hospital unit delegate to an experienced licensed practical/vocational nurse (LPN/LVN) caring for a patient with a permanent tracheostomy? a. Assess the patient's risk for aspiration. b. Suction the tracheostomy when needed. c. Teach the patient about self-care of the tracheostomy. d. Determine the need for replacement of the tracheostomy tube.

B Suctioning of a stable patient can be delegated to LPNs/LVNs. Patient assessment and patient teaching should be done by the RN. DIF: Cognitive Level: Apply (application) REF: 518 OBJ: Special Questions: Delegation TOP: Nursing Process: Planning MSC:

Physiological Integrity 9. The nurse teaches a patient about the transmission of pulmonary tuberculosis (TB). Which statement, if made by the patient, indicates that teaching was effective? a. "I will avoid being outdoors whenever possible." b. "My husband will be sleeping in the guest bedroom." c. "I will take the bus instead of driving to visit my friends." d. "I will keep the windows closed at home to contain the germs."

B Teach the patient how to minimize exposure to close contacts and household members. Homes should be well ventilated, especially the areas where the infected person spends a lot of time. While still infectious, the patient should sleep alone, spend as much time as possible outdoors, and minimize time in congregate settings or on public transportation. DIF: Cognitive Level: Apply (application) REF: 533 TOP: Nursing Process: Evaluation MSC:

Physiological Integrity 18. A patient is receiving 35% oxygen via a Venturi mask. To ensure the correct amount of oxygen delivery, which action by the nurse is most important? a. Teach the patient to keep mask on at all times. b. Keep the air entrainment ports clean and unobstructed. c. Give a high enough flow rate to keep the bag from collapsing. d. Drain moisture condensation from the oxygen tubing every hour.

B The air entrainment ports regulate the oxygen percentage delivered to the patient, so they must be unobstructed. A high oxygen flow rate is needed when giving oxygen by partial rebreather or non-rebreather masks. Draining oxygen tubing is necessary when caring for a patient receiving mechanical ventilation. The mask is uncomfortable and can be removed when the patient eats. DIF: Cognitive Level: Apply (application) REF: 591 TOP: Nursing Process: Implementation MSC:

Physiological Integrity 16. The nurse is admitting a patient diagnosed with an acute exacerbation of chronic obstructive pulmonary disease (COPD).What is the best way for the nurse to determine the appropriate oxygen flow rate? a. Minimize oxygen use to avoid oxygen dependency. b. Maintain the pulse oximetry level at 90% or greater. c. Administer oxygen according to the patient's level of dyspnea. d. Avoid administration of oxygen at a rate of more than 2 L/minute.

B The best way to determine the appropriate oxygen flow rate is by monitoring the patient's oxygenation either by arterial blood gases (ABGs) or pulse oximetry. An oxygen saturation of 90% indicates adequate blood oxygen level without the danger of suppressing the respiratory drive. For patients with an exacerbation of COPD, an oxygen flow rate of 2 L/min may not be adequate. Because oxygen use improves survival rate in patients with COPD, there is no concern about oxygen dependency. The patient's perceived dyspnea level may be affected by other factors (such as anxiety) besides blood oxygen level. DIF: Cognitive Level: Apply (application) REF: 589 TOP: Nursing Process: Implementation MSC:

Physiological Integrity 14. A registered nurse (RN) is observing a student nurse who is doing a physical assessment on a patient. The RN will need to intervene immediately if the student nurse a. presses on the skin over the tibia for 10 seconds to check for edema. b. palpates both carotid arteries simultaneously to compare pulse quality. c. documents a murmur heard along the right sternal border as a pulmonic murmur. d. places the patient in the left lateral position to check for the point of maximal impulse.

B The carotid pulses should never be palpated at the same time to avoid vagal stimulation, dysrhythmias, and decreased cerebral blood flow. The other assessment techniques also need to be corrected. However, they are not dangerous to the patient. DIF: Cognitive Level: Apply (application) REF: 694-695 TOP: Nursing Process: Assessment MSC:

Physiological Integrity 19. When admitting a patient for a cardiac catheterization and coronary angiogram, which information about the patient is most important for the nurse to communicate to the health care provider? a. The patient's pedal pulses are +1. b. The patient is allergic to shellfish. c. The patient had a heart attack a year ago. d. The patient has not eaten anything today.

B The contrast dye used for the procedure is iodine based, so patients who have shellfish allergies will require treatment with medications such as corticosteroids and antihistamines before the angiogram. The other information is also communicated to the health care provider but will not require a change in the usual precardiac catheterization orders or medications. DIF: Cognitive Level: Apply (application) REF: 703 OBJ: Special Questions: Prioritization TOP: Nursing Process: Assessment MSC:

Physiological Integrity 16. The nurse obtains the following assessment data on an older patient who has influenza. Which information will be most important for the nurse to communicate to the health care provider? a. Fever of 100.4° F (38° C) b. Diffuse crackles in the lungs c. Sore throat and frequent cough d. Myalgia and persistent headache

B The crackles indicate that the patient may be developing pneumonia, a common complication of influenza, which would require aggressive treatment. Myalgia, headache, mild temperature elevation, and sore throat with cough are typical manifestations of influenza and are treated with supportive care measures such as over-the-counter (OTC) pain relievers and increased fluid intake. DIF: Cognitive Level: Apply (application) REF: 503 OBJ: Special Questions: Prioritization TOP: Nursing Process: Assessment MSC:

Physiological Integrity 39. A patient is admitted to the emergency department with an open stab wound to the left chest. What is the first action that the nurse should take? a. Position the patient so that the left chest is dependent. b. Tape a nonporous dressing on three sides over the chest wound. c. Cover the sucking chest wound firmly with an occlusive dressing. d. Keep the head of the patient's bed at no more than 30 degrees elevation.

B The dressing taped on three sides will allow air to escape when intrapleural pressure increases during expiration, but it will prevent air from moving into the pleural space during inspiration. Placing the patient on the left side or covering the chest wound with an occlusive dressing will allow trapped air in the pleural space and cause tension pneumothorax. The head of the bed should be elevated to 30 to 45 degrees to facilitate breathing. DIF: Cognitive Level: Apply (application) REF: 541 OBJ: Special Questions: Prioritization TOP: Nursing Process: Implementation MSC:

Physiological Integrity 14. Following a laryngectomy a patient coughs violently during suctioning and dislodges the tracheostomy tube. Which action should the nurse take first? a. Cover stoma with sterile gauze and ventilate through stoma. b. Attempt to reinsert the tracheostomy tube with the obturator in place. c. Assess the patient's oxygen saturation and notify the health care provider. d. Ventilate the patient with a manual bag and face mask until the health care provider arrives.

B The first action should be to attempt to reinsert the tracheostomy tube to maintain the patient's airway. Assessing the patient's oxygenation is an important action, but it is not the most appropriate first action in this situation. Covering the stoma with a dressing and manually ventilating the patient may be an appropriate action if the nurse is unable to reinsert the tracheostomy tube. Ventilating with a facemask is not appropriate for a patient with a total laryngectomy because there is a complete separation between the upper airway and the trachea. DIF: Cognitive Level: Apply (application) REF: 509 OBJ: Special Questions: Prioritization TOP: Nursing Process: Implementation MSC:

Physiological Integrity 13. After 2 months of tuberculosis (TB) treatment with isoniazid (INH), rifampin (Rifadin), pyrazinamide (PZA), and ethambutol, a patient continues to have positive sputum smears for acid-fast bacilli (AFB). Which action should the nurse take next? a. Teach about treatment for drug-resistant TB treatment. b. Ask the patient whether medications have been taken as directed. c. Schedule the patient for directly observed therapy three times weekly. d. Discuss with the health care provider the need for the patient to use an injectable antibiotic.

B The first action should be to determine whether the patient has been compliant with drug therapy because negative sputum smears would be expected if the TB bacillus is susceptible to the medications and if the medications have been taken correctly. Assessment is the first step in the nursing process. Depending on whether the patient has been compliant or not, different medications or directly observed therapy may be indicated. The other options are interventions based on assumptions until an assessment has been completed. DIF: Cognitive Level: Apply (application) REF: 531 OBJ: Special Questions: Prioritization TOP: Nursing Process: Implementation MSC:

31. An interdisciplinary team is planning the care of a patient with bronchiectasis. What aspects of care should the nurse anticipate? Select all that apply. A) Occupational therapy B) Antimicrobial therapy C) Positive pressure isolation D) Chest physiotherapy E) Smoking cessation

B, D, E Feedback: Chest physiotherapy, antibiotics, and smoking cessation are cornerstones of the care of patients with bronchiectasis. Occupational therapy and isolation are not normally indicated.

Which of the following is inconsistent as a condition related to metabolic syndrome? a) Abdominal obesity b) Hypotension c) Insulin resistance d) Dyslipidemia

B. hypotension

Physiological Integrity 33. A patient with cystic fibrosis (CF) has blood glucose levels that are consistently between 180 to 250 mg/dL. Which nursing action will the nurse plan to implement? a. Discuss the role of diet in blood glucose control. b. Teach the patient about administration of insulin. c. Give oral hypoglycemic medications before meals. d. Evaluate the patient's home use of pancreatic enzymes.

B The glucose levels indicate that the patient has developed CF-related diabetes, and insulin therapy is required. Because the etiology of diabetes in CF is inadequate insulin production, oral hypoglycemic agents are not effective. Patients with CF need a high-calorie diet. Inappropriate use of pancreatic enzymes would not be a cause of hyperglycemia in a patient with CF. DIF: Cognitive Level: Apply (application) REF: 603 TOP: Nursing Process: Planning MSC:

Physiological Integrity 5. The emergency department nurse is evaluating the effectiveness of therapy for a patient who has received treatment during an asthma attack. Which assessment finding is the best indicator that the therapy has been effective? a. No wheezes are audible. b. Oxygen saturation is >90%. c. Accessory muscle use has decreased. d. Respiratory rate is 16 breaths/minute.

B The goal for treatment of an asthma attack is to keep the oxygen saturation >90%. The other patient data may occur when the patient is too fatigued to continue with the increased work of breathing required in an asthma attack. DIF: Cognitive Level: Apply (application) REF: 569 TOP: Nursing Process: Evaluation MSC:

Physiological Integrity 13. When titrating IV nitroglycerin (Tridil) for a patient with a myocardial infarction (MI), which action will the nurse take to evaluate the effectiveness of the medication? a. Monitor heart rate. b. Ask about chest pain. c. Check blood pressure. d. Observe for dysrhythmias.

B The goal of IV nitroglycerin administration in MI is relief of chest pain by improving the balance between myocardial oxygen supply and demand. The nurse also will monitor heart rate and blood pressure (BP) and observe for dysrhythmias, but these parameters will not indicate whether the medication is effective. DIF: Cognitive Level: Apply (application) REF: 753 TOP: Nursing Process: Evaluation MSC:

Physiological Integrity 40. Which patient at the cardiovascular clinic requires the most immediate action by the nurse? a. Patient with type 2 diabetes whose current blood glucose level is 145 mg/dL b. Patient with stable angina whose chest pain has recently increased in frequency c. Patient with familial hypercholesterolemia and a total cholesterol of 465 mg/dL d. Patient with chronic hypertension whose blood pressure today is 172/98 mm Hg

B The history of more frequent chest pain suggests that the patient may have unstable angina, which is part of the acute coronary syndrome spectrum. This will require rapid implementation of actions such as cardiac catheterization and possible percutaneous coronary intervention. The data about the other patients suggest that their conditions are stable. DIF: Cognitive Level: Analyze (analysis) REF: 743 OBJ: Special Questions: Prioritization; Multiple Patients TOP: Nursing Process: Planning MSC:

Psychosocial Integrity 21. The nurse monitors a patient after chest tube placement for a hemopneumothorax. The nurse is most concerned if which assessment finding is observed? a. A large air leak in the water-seal chamber b. 400 mL of blood in the collection chamber c. Complaint of pain with each deep inspiration d. Subcutaneous emphysema at the insertion site

B The large amount of blood may indicate that the patient is in danger of developing hypovolemic shock. An air leak would be expected immediately after chest tube placement for a pneumothorax. Initially, brisk bubbling of air occurs in this chamber when a pneumothorax is evacuated. The pain should be treated but is not as urgent a concern as the possibility of continued hemorrhage. Subcutaneous emphysema should be monitored but is not unusual in a patient with pneumothorax. A small amount of subcutaneous air is harmless and will be reabsorbed. DIF: Cognitive Level: Apply (application) REF: 547 TOP: Nursing Process: Assessment MSC:

Physiological Integrity OTHER 1. While listening at the mitral area, the nurse notes abnormal heart sounds at the patient's fifth intercostal space, midclavicular line. After listening to the audio clip, describe how the nurse will document the assessment finding. Click here to listen to the audio clip a. S3 gallop heard at the aortic area b. Systolic murmur noted at mitral area c. Diastolic murmur noted at tricuspid area d. Pericardial friction rub heard at the apex

B The mitral area location is at the intersection of the fifth intercostal space and the midclavicular line. The murmur is a pansystolic murmur. DIF: Cognitive Level: Apply (application) REF: 691 TOP: Nursing Process: Assessment MSC:

Integrated Process: Nursing Process (Implementation) 15. What is the best instruction for a client who has step II (mild persistent) asthma? a. "Avoid participating in aerobic exercise." b. "You will need daily inhaled low-dose steroids." c. "You need to evaluate your diet for asthma triggers." d. "Make sure you use a rescue inhaler three times per day."

B The most important information for clients with step II (mild persistent) asthma is that they need daily preventive anti-inflammatory medication. Low-dose inhaled steroids are necessary. The client should exercise as tolerated; however, using a rescue inhaler frequently is not recommended and, if this is needed, it should be reported to the health care provider because a change in therapy is likely needed. DIF: Cognitive Level: Comprehension/Understanding REF: Chart 32-2, p. 603 TOP: Client Needs Category: Physiological Integrity (Physiological Adaptation—Illness Management)

Integrated Process: Nursing Process (Assessment) 25. A client with lung cancer is lying flat in bed and reports shortness of breath. What action does the nurse take first? a. Notify the health care provider. b. Elevate the head of the bed. c. Assess oxygen saturation. d. Have the client take deep breaths.

B The nurse's first action should be to elevate the head of the bed. Next, assessing oxygen saturation will help the nurse determine the client's status. If the oxygen is low, the nurse would increase oxygen flow and have the client take deep breaths. The provider could be notified after the nurse performs the interventions. DIF: Cognitive Level: Application/Applying or higher REF: N/A TOP: Client Needs Category: Physiological Integrity (Physiological Adaptation—Alterations in Body Systems)

Physiological Integrity 26. A young adult female patient with cystic fibrosis (CF) tells the nurse that she is not sure about getting married and having children some day. Which initial response by the nurse is best? a. "Are you aware of the normal lifespan for patients with CF?" b. "Do you need any information to help you with that decision?" c. "Many women with CF do not have difficulty conceiving children." d. "You will need to have genetic counseling before making a decision."

B The nurse's initial response should be to assess the patient's knowledge level and need for information. Although the lifespan for patients with CF is likely to be shorter than normal, it would not be appropriate for the nurse to address this as the initial response to the patient's comments. The other responses have accurate information, but the nurse should first assess the patient's understanding about the issues surrounding pregnancy. DIF: Cognitive Level: Apply (application) REF: 605-606 TOP: Nursing Process: Implementation MSC:

Physiological Integrity 20. A patient with newly diagnosed lung cancer tells the nurse, "I don't think I'm going to live to see my next birthday." Which response by the nurse is best? a. "Would you like to talk to the hospital chaplain about your feelings?" b. "Can you tell me what it is that makes you think you will die so soon?" c. "Are you afraid that the treatment for your cancer will not be effective?" d. "Do you think that taking an antidepressant medication would be helpful?"

B The nurse's initial response should be to collect more assessment data about the patient's statement. The answer beginning "Can you tell me what it is" is the most open-ended question and will offer the best opportunity for obtaining more data. The answer beginning, "Are you afraid" implies that the patient thinks that the cancer will be immediately fatal, although the patient's statement may not be related to the cancer diagnosis. The remaining two answers offer interventions that may be helpful to the patient, but more assessment is needed to determine whether these interventions are appropriate. DIF: Cognitive Level: Apply (application) REF: 539-540 TOP: Nursing Process: Implementation MSC:

Physiological Integrity 13. The nurse teaches a patient about pursed lip breathing. Which action by the patient would indicate to the nurse that further teaching is needed? a. The patient inhales slowly through the nose. b. The patient puffs up the cheeks while exhaling. c. The patient practices by blowing through a straw. d. The patient's ratio of inhalation to exhalation is 1:3.

B The patient should relax the facial muscles without puffing the cheeks while doing pursed lip breathing. The other actions by the patient indicate a good understanding of pursed lip breathing. DIF: Cognitive Level: Apply (application) REF: 579 TOP: Nursing Process: Evaluation MSC:

Physiological Integrity 20. A transesophageal echocardiogram (TEE) is ordered for a patient with possible endocarditis. Which action included in the standard TEE orders will the nurse need to accomplish first? a. Start an IV line. b. Place the patient on NPO status. c. Administer O2 per nasal cannula. d. Give lorazepam (Ativan) 1 mg IV.

B The patient will need to be NPO for 6 hours preceding the TEE, so the nurse should place the patient on NPO status as soon as the order is received. The other actions also will need to be accomplished but not until just before or during the procedure. DIF: Cognitive Level: Apply (application) REF: 701 OBJ: Special Questions: Prioritization TOP: Nursing Process: Implementation MSC:

Physiological Integrity 22. When assessing a patient with a sore throat, the nurse notes anterior cervical lymph node swelling, a temperature of 101.6° F (38.7° C), and yellow patches on the tonsils. Which action will the nurse anticipate taking? a. Teach the patient about the use of expectorants. b. Use a swab to obtain a sample for a rapid strep antigen test. c. Discuss the need to rinse the mouth out after using any inhalers. d. Teach the patient to avoid use of nonsteroidal antiinflammatory drugs (NSAIDs).

B The patient's clinical manifestations are consistent with streptococcal pharyngitis and the nurse will anticipate the need for a rapid strep antigen test and/or cultures. Because patients with streptococcal pharyngitis usually do not have a cough, use of expectorants will not be anticipated. Rinsing the mouth out after inhaler use may prevent fungal oral infections, but the patient's assessment data are not consistent with a fungal infection. NSAIDs are frequently prescribed for pain and fever relief with pharyngitis. DIF: Cognitive Level: Apply (application) REF: 506 TOP: Nursing Process: Planning MSC:

Safe and Effective Care Environment 25. After receiving change-of-shift report on a heart failure unit, which patient should the nurse assess first? a. Patient who is taking carvedilol (Coreg) and has a heart rate of 58 b. Patient who is taking digoxin and has a potassium level of 3.1 mEq/L c. Patient who is taking isosorbide dinitrate/hydralazine (BiDil) and has a headache d. Patient who is taking captopril (Capoten) and has a frequent nonproductive cough

B The patient's low potassium level increases the risk for digoxin toxicity and potentially fatal dysrhythmias. The nurse should assess the patient for other signs of digoxin toxicity and then notify the health care provider about the potassium level. The other patients also have side effects of their medications, but their symptoms do not indicate potentially life-threatening complications. DIF: Cognitive Level: Analyze (analysis) REF: 777 OBJ: Special Questions: Prioritization; Multiple Patients TOP: Nursing Process: Assessment MSC:

Safe and Effective Care Environment 40. The clinic nurse makes a follow-up telephone call to a patient with asthma. The patient reports having a baseline peak flow reading of 600 L/minute and the current peak flow is 420 L/minute. Which action should the nurse take first? a. Tell the patient to go to the hospital emergency department. b. Instruct the patient to use the prescribed albuterol (Proventil). c. Ask about recent exposure to any new allergens or asthma triggers. d. Question the patient about use of the prescribed inhaled corticosteroids.

B The patient's peak flow is 70% of normal, indicating a need for immediate use of short-acting 2-adrenergic SABA medications. Assessing for correct use of medications or exposure to allergens also is appropriate, but would not address the current decrease in peak flow. Because the patient is currently in the yellow zone, hospitalization is not needed. DIF: Cognitive Level: Analyze (analysis) REF: 580 OBJ: Special Questions: Prioritization TOP: Nursing Process: Implementation MSC:

Hypervolemia

S/S: Edema, weight gain, jugular vein distention, crackles, SOB, bounding pulse (Hypervolemia/hypovolemia)

Physiological Integrity 5. The nurse develops a plan of care to prevent aspiration in a high-risk patient. Which nursing action will be most effective? a. Turn and reposition immobile patients at least every 2 hours. b. Place patients with altered consciousness in side-lying positions. c. Monitor for respiratory symptoms in patients who are immunosuppressed. d. Insert nasogastric tube for feedings for patients with swallowing problems.

B The risk for aspiration is decreased when patients with a decreased level of consciousness are placed in a side-lying or upright position. Frequent turning prevents pooling of secretions in immobilized patients but will not decrease the risk for aspiration in patients at risk. Monitoring of parameters such as breath sounds and oxygen saturation will help detect pneumonia in immunocompromised patients, but it will not decrease the risk for aspiration. Conditions that increase the risk of aspiration include decreased level of consciousness (e.g., seizure, anesthesia, head injury, stroke, alcohol intake), difficulty swallowing, and nasogastric intubation with or without tube feeding. With loss of consciousness, the gag and cough reflexes are depressed, and aspiration is more likely to occur. Other high-risk groups are those who are seriously ill, have poor dentition, or are receiving acid-reducing medications. DIF: Cognitive Level: Apply (application) REF: 527 TOP: Nursing Process: Implementation MSC:

Integrated Process: Nursing Process (Implementation) 10. A client is demonstrating diaphragmatic breathing for the nurse. Which action by the client shows adequate understanding of this breathing technique? a. Lying on his or her side with knees bent b. Having his or her hands on the abdomen c. Having his or her hands over the head d. Lying in the prone position

B To perform diaphragmatic breathing correctly, the client should put the hands on his or her abdomen to create resistance. This type of breathing cannot be performed effectively while lying on the side or with hands over the head. This type of breathing would not be as effective lying prone. DIF: Cognitive Level: Application/Applying or higher REF: N/A TOP: Client Needs Category: Physiological Integrity (Physiological Adaptation—Alterations in Body Systems)

Physiological Integrity 21. The nurse and unlicensed assistive personnel (UAP) on the telemetry unit are caring for four patients. Which nursing action can be delegated to the UAP? a. Teaching a patient scheduled for exercise electrocardiography about the procedure b. Placing electrodes in the correct position for a patient who is to receive ECG monitoring c. Checking the catheter insertion site for a patient who is recovering from a coronary angiogram d. Monitoring a patient who has just returned to the unit after a transesophageal echocardiogram

B UAP can be educated in standardized lead placement for ECG monitoring. Assessment of patients who have had procedures where airway maintenance (transesophageal echocardiography) or bleeding (coronary angiogram) is a concern must be done by the registered nurse (RN). Patient teaching requires RN level education and scope of practice. DIF: Cognitive Level: Analyze (analysis) REF: 15-16 OBJ: Special Questions: Delegation TOP: Nursing Process: Planning MSC:

Physiological Integrity 29. Which instruction should the nurse include in an exercise teaching plan for a patient with chronic obstructive pulmonary disease (COPD)? a. "Stop exercising if you start to feel short of breath." b. "Use the bronchodilator before you start to exercise." c. "Breathe in and out through the mouth while you exercise." d. "Upper body exercise should be avoided to prevent dyspnea."

B Use of a bronchodilator before exercise improves airflow for some patients and is recommended. Shortness of breath is normal with exercise and not a reason to stop. Patients should be taught to breathe in through the nose and out through the mouth (using a pursed lip technique). Upper-body exercise can improve the mechanics of breathing in patients with COPD. DIF: Cognitive Level: Apply (application) REF: 572 TOP: Nursing Process: Implementation MSC:

1. A patient with acute shortness of breath is admitted to the hospital. Which action should the nurse take during the initial assessment of the patient? a. Ask the patient to lie down to complete a full physical assessment. b. Briefly ask specific questions about this episode of respiratory distress. c. Complete the admission database to check for allergies before treatment. d. Delay the physical assessment to first complete pulmonary function tests.

B When a patient has severe respiratory distress, only information pertinent to the current episode is obtained, and a more thorough assessment is deferred until later. Obtaining a comprehensive health history or full physical examination is unnecessary until the acute distress has resolved. Brief questioning and a focused physical assessment should be done rapidly to help determine the cause of the distress and suggest treatment. Checking for allergies is important, but it is not appropriate to complete the entire admission database at this time. The initial respiratory assessment must be completed before any diagnostic tests or interventions can be ordered. DIF: Cognitive Level: Apply (application) REF: 482 TOP: Nursing Process: Assessment MSC:

Psychosocial Integrity 22. When evaluating the effectiveness of preoperative teaching with a patient scheduled for coronary artery bypass graft (CABG) surgery using the internal mammary artery, the nurse determines that additional teaching is needed when the patient says which of the following? a. "They will circulate my blood with a machine during the surgery." b. "I will have small incisions in my leg where they will remove the vein." c. "They will use an artery near my heart to go around the area that is blocked." d. "I will need to take an aspirin every day after the surgery to keep the graft open."

B When the internal mammary artery is used there is no need to have a saphenous vein removed from the leg. The other statements by the patient are accurate and indicate that the teaching has been effective. DIF: Cognitive Level: Apply (application) REF: 752 TOP: Nursing Process: Evaluation MSC:

Physiological Integrity 28. A patient with a pleural effusion is scheduled for a thoracentesis. Which action should the nurse take to prepare the patient for the procedure? a. Start a peripheral IV line to administer the necessary sedative drugs. b. Position the patient sitting upright on the edge of the bed and leaning forward. c. Obtain a large collection device to hold 2 to 3 liters of pleural fluid at one time. d. Remove the water pitcher and remind the patient not to eat or drink anything for 6 hours.

B When the patient is sitting up, fluid accumulates in the pleural space at the lung bases and can more easily be located and removed. The patient does not usually require sedation for the procedure, and there are no restrictions on oral intake because the patient is not sedated or unconscious. Usually only 1000 to 1200 mL of pleural fluid is removed at one time. Rapid removal of a large volume can result in hypotension, hypoxemia, or pulmonary edema. DIF: Cognitive Level: Apply (application) REF: 550 TOP: Nursing Process: Planning MSC:

After teaching a patient with chronic stable angina about nitroglycerin, the nurse recognizes the need for further teaching when the patient makes which statement?

B "I can take up to five tablets every 3 minutes for relief of my chest pain."

Which patient is at greatest risk for sudden cardiac death?

B A 52-year-old African American man with left ventricular failure

Which individuals would the nurse identify as having the highest risk for CAD?

B A 60-year-old male with below normal homocysteine levels

The patient comes to the ED with severe, prolonged angina that is not immediately reversible. The nurse knows that if the patient once had angina related to a stable atherosclerotic plaque and the plaque ruptures, there may be occlusion of a coronary vessel and this type of pain. How will the nurse document this situation related to pathophysiology, presentation, diagnosis, prognosis, and interventions for this disorder?

B Acute coronary syndrome (ACS)

The nurse would assess a patient with complaints of chest pain for which clinical manifestations associated with a myocardial infarction (MI) (select all that apply)?

B Ashen skin C Diaphoresis D Nausea and vomiting E S3 or S4 heart sounds

The nurse is providing teaching to a patient recovering from an MI. How should resumption of sexual activity be discussed?

B Discussed along with other physical activities

The nurse prepares a discharge teaching plan for a 44-year-old male patient who has recently been diagnosed with coronary artery disease. Which risk factor should the nurse plan to focus on during the teaching session?

B Elevated serum lipids

Which antilipemic medications should the nurse question for a patient with cirrhosis of the liver (select all that apply)?

B Ezetimibe (Zetia) D Atorvastatin (Lipitor)

The nurse is examining the ECG of a patient who has just been admitted with a suspected MI. Which ECG change is most indicative of prolonged or complete coronary occlusion?

B Pathologic Q wave

An agitated, confused client arrives in the emergency department. The client's history includes type 1 diabetes, hypertension, and angina pectoris. Assessment reveals pallor, diaphoresis, headache, and intense hunger. A stat blood glucose sample measures 42 mg/dl, and the client is treated for an acute hypoglycemic reaction. After recovery, the nurse teaches the client to treat hypoglycemia by ingesting: a) 2 to 5 g of a simple carbohydrate. b) 10 to 15 g of a simple carbohydrate. c) 18 to 20 g of a simple carbohydrate. d) 25 to 30 g of a simple carbohydrate.

B) 10 to 15 g of a simple carbohydrate

A client with type 1 diabetes presents with a decreased level of consciousness and a fingerstick glucose level of 39 mg/dl. His family reports that he has been skipping meals in an effort to lose weight. Which nursing intervention is most appropriate? a) Administering a 500-ml bolus of normal saline solution b) Administering 1 ampule of 50% dextrose solution, per physician's order c) Observing the client for 1 hour, then rechecking the fingerstick glucose level d) Inserting a feeding tube and providing tube feedings

B) Administering 1 ampule of 50% dextrose solution, per physician's order

29. A nurse is assessing a patient who is suspected of having bronchiectasis. The nurse should consider which of the following potential causes? Select all that apply. A) Pulmonary hypertension B) Airway obstruction C) Pulmonary infections D) Genetic disorders E) Atelectasis

B, C, D Feedback: Bronchiectasis is a chronic, irreversible dilation of the bronchi and bronchioles. Under the new definition of COPD, it is considered a disease process separate from COPD. Bronchiectasis may be caused by a variety of conditions, including airway obstruction, diffuse airway injury, pulmonary infections and obstruction of the bronchus or complications of long-term pulmonary infections, or genetic disorders such as cystic fibrosis. Bronchiectasis is not caused by pulmonary hypertension or atelectasis.

22. The nurse is performing an assessment on a patient who has been diagnosed with cancer of the larynx. Part of the nurse's assessment addresses the patient's general state of nutrition. Which laboratory values would be assessed when determining the nutritional status of the patient? Select all that apply. A) White blood cell count B) Protein level C) Albumin level D) Platelet count E) Glucose level

B, C, E Feedback: The nurse also assesses the patient's general state of nutrition, including height and weight and body mass index, and reviews laboratory values that assist in determining the patient's nutritional status (albumin, protein, glucose, and electrolyte levels). The white blood cell count and the platelet count would not normally assist in determining the patient's nutritional status.

Integrated Process: Teaching/Learning 3. The nurse is assessing a client with chronic obstructive pulmonary disease (COPD) to determine activity tolerance. Which questions elicit the most important information? (Select all that apply.) a. "What color is your sputum?" b. "Do you have any difficulty sleeping?" c. "How long does it take to perform your morning routine?" d. "Do you walk upstairs every day?" e. "Have you lost any weight lately?"

B, C, E Difficulty sleeping could indicate worsening breathlessness, as could taking longer to perform activities of daily living. Weight loss could mean increased dyspnea as the client becomes too fatigued to eat. The color of the client's sputum would not assist in determining activity tolerance. Asking whether the client walks upstairs every day is not as pertinent as determining if the client becomes short of breath on walking upstairs, or if the client goes upstairs less often than previously. DIF: Cognitive Level: Application/Applying or higher REF: N/A TOP: Client Needs Category: Physiological Integrity (Physiological Adaptation—Pathophysiology)

Physiological Integrity OTHER 1. When a patient requires defibrillation, in which order will the nurse accomplish the following steps? a. Place the paddles on the patient's chest. b. Turn the defibrillator on. c. Check the location of other personnel and call out "all clear." d. Select the appropriate energy level. e. Deliver the electrical charge.

B, D, A, C, E Rationale: This order will result in rapid defibrillation without endangering hospital personnel. Cognitive Level: Application Text Reference: p. 857 Nursing Process: Implementation

hyperkalemia

S/S: arrhythmias, weakness, paresthesia, ECG changed such as tented T wave, nausea, vomiting (hyperkalemia, hypokalmeia)

Integrated Process: Nursing Process (Assessment) MULTIPLE RESPONSE 1. A client has a mediastinal chest tube. Which symptoms require the nurse's immediate intervention? (Select all that apply.) a. Production of pink sputum b. Tracheal deviation c. Oxygen saturation greater than 95% d. Sudden onset of shortness of breath e. Drainage greater than 70 mL/hr f. Pain at insertion site g. Disconnection at Y site

B, D, E, G Immediate intervention is warranted if the client has tracheal deviation because this could indicate a tension pneumothorax; sudden shortness of breath because this could indicate dislodgment of the tube, occlusion of the tube, or pneumothorax; or drainage greater than 70 mL/hr because this could indicate hemorrhage. Disconnection at the Y site could result in air entering the tubing. Production of pink sputum, oxygen saturation less than 95%, and pain at the insertion site are not signs/symptoms that would require immediate intervention. DIF: Cognitive Level: Application/Applying or higher REF: N/A TOP: Client Needs Category: Physiological Integrity (Physiological Adaptation—Illness Management)

Safe and Effective Care Environment MULTIPLE RESPONSE 1. A patient is scheduled for a computed tomography (CT) of the chest with contrast media. Which assessment findings should the nurse immediately report to the health care provider (select all that apply)? a. Patient is claustrophobic. b. Patient is allergic to shellfish. c. Patient recently used a bronchodilator inhaler. d. Patient is not able to remove a wedding band. e. Blood urea nitrogen (BUN) and serum creatinine levels are elevated.

B, E Because the contrast media is iodine-based and may cause dehydration and decreased renal blood flow, asking about iodine allergies (such as allergy to shellfish) and monitoring renal function before the CT scan are necessary. The other actions are not contraindications for CT of the chest, although they may be for other diagnostic tests, such as magnetic resonance imaging (MRI) or pulmonary function testing (PFT). DIF: Cognitive Level: Analyze (analysis) REF: 492 TOP: Nursing Process: Implementation MSC:

Integrated Process: Nursing Process (Assessment) 4. A client has a chest tube. What assessment findings require immediate intervention from the nurse? (Select all that apply.) a. Intermittent bubbling in the water seal chamber in the client with a pneumothorax b. "Silent chest" in the client with a pneumothorax c. Tidaling in the water seal chamber in a client with a pneumothorax d. Bloody drainage in the tubing of a client with a hemothorax e. Tracheal deviation in a client after chest trauma f. No drainage in the chest tube of a client with a pneumothorax g. Constant bubbling in the water seal chamber in a client post chest surgery

B, E, G The client with a silent chest could have a mucous plug, the client with tracheal deviation could have a collapsed lung or tension pneumothorax, and the client with constant bubbling in the water seal could have an air leak. All of these assessments require intervention. The others are normal for the condition stated. DIF: Cognitive Level: Application/Applying or higher REF: N/A TOP: Client Needs Category: Physiological Integrity (Physiological Adaptation—Illness Management)

A client with Raynaud's disease complains of cold and numbness in the fingers. Which of the following would the nurse identify as an early sign of vasoconstriction? a) Gangrene b) Pallor c) Cyanosis d) Clubbing of the fingers

B. Pallor

A client is receiving nitroglycerin ointment (Nitro-Dur) to treat angina pectoris. The nurse evaluates the therapeutic effectiveness of this drug by assessing the client's response and checking for adverse effects. Which vital sign ismost likely to reflect an adverse effect of nitroglycerin? a) Temperature of 100.2° F (37.9° C) b) Blood pressure 84/52 mm Hg c) Pulse rate of 84 beats/minute d) Respiration 26 breaths/minute

B. blood pressure 84/52 mm Hg

A client is taking Humulin NPH insulin daily every morning. The nurse instructs the client that the mostlikely time for a hypoglycemic reaction to occur is: A) 2-4 hours after administration B) 4-12 hours after administration C) 16-18 hours after administration D) 18-24 hours after administration

B: Rationale: Humulin is an intermediate acting insulin. The onset of action is 1.5 hours, it peaks in 4-12 hours, and its duration is 24 hours. Hypoglycemic reactions to insulin are most likely to occur during the peak time.

Infective Endocarditis

Bacterial infection cause heart damage and deterioration of heart. S/S: SOB, fatigue, chest pain. Give bed rest, antibiotics, oxygen therapy,

Aneurysms

Ballooning of an Artery. Risks for rupture and hemorrhage. S/S: "Heart" beating in abdomen/back. Ultrasound/Xray is done. Monitor renal function and pedal pulse. Do not palate mass.

a, f

Beclomethasone inhaler is associated with A. Long-term control B. Quick-relief C. B2-adrenergic agonist D. Mast-cell stabilizer E. Leukotriene inhibitor F. Steroid antiinflammatory G. Methylxanthine bronchodilator H. Anticholinergic I. Anti-IgE

a (The patient should be adequately medicated for pain before any attempt to ambulate. Instructions about the benefits of ambulation may increase the patient's willingness to ambulate, but decreasing pain with ambulation is more important. The presence of an incisional drain or timing of dressing change will not affect ambulation.)

Before assisting a patient with ambulation on the day after a total hip replacement, which action is most important for the nurse to take? A. Administer the ordered oral opioid pain medication. B. Instruct the patient about the benefits of ambulation. C. Ensure that the incisional drain has been discontinued. D. Change the hip dressing and document the wound appearance.

11. While assessing the patient, the nurse observes constant bubbling in the water-seal chamber of the patient's closed chest-drainage system. What should the nurse conclude? A) The system is functioning normally. B) The patient has a pneumothorax. C) The system has an air leak. D) The chest tube is obstructed.

C Feedback: Constant bubbling in the chamber often indicates an air leak and requires immediate assessment and intervention. The patient with a pneumothorax will have intermittent bubbling in the water-seal chamber. If the tube is obstructed, the nurse should notice that the fluid has stopped fluctuating in the water-seal chamber.

What happens to LEs w/ PAD when they are elevated and then lowered?

Blanching w/ rebound hyperemia.

Left ventricular thrombus

Blood pooling in LV Decreased ejection fraction -complication of HF

Physiological Integrity 15. A patient with an acute exacerbation of COPD has the following ABG analysis: pH 7.32, PaO2 58 mm Hg, PaCO2 55 mm Hg, and SaO2 86%. The nurse recognizes these values as evidence of a. normal acid-base balance with hypoxemia. b. normal acid-base balance with hypercapnia. c. respiratory acidosis. d. respiratory alkalosis.

C Rationale: The elevated PaCO2 and low pH indicate respiratory acidosis. The patient is hypoxemic and hypercapnic, but the pH indicates acidosis, not a normal acid-base balance. Cognitive Level: Comprehension Text Reference: p. 650 Nursing Process: Assessment

Postoperative care of a patient undergoing coronary artery bypass graft (CABG) surgery includes monitoring for what common complication?

C Atrial dysrhythmias

Physiological Integrity 2. A patient with chronic heart failure who has been following a low-sodium diet tells the nurse at the clinic about a 5-pound weight gain in the last 3 days. The nurse's first action will be to a. ask the patient to recall the dietary intake for the last 3 days because there may be hidden sources of sodium in the patient's diet. b. instruct the patient in a low-calorie, low-fat diet because the weight gain has likely been caused by excessive intake of inappropriate foods. c. assess the patient for clinical manifestations of acute heart failure because an exacerbation of the chronic heart failure may be occurring. d. educate the patient about the use of diuretic therapy because it is likely that the patient will need medications to reduce the hypervolemia.

C Rationale: The 5-pound weight gain over 3 days indicates that the patient's chronic heart failure may be worsening; it is important that the patient be immediately assessed for other clinical manifestations of decompensation, such as lung crackles. A dietary recall to detect hidden sodium in the diet and teaching about diuretic therapy are appropriate interventions but are not the first nursing actions indicated. There is no evidence that the patient's weight gain is caused by excessive dietary intake of fat or calories, so the answer beginning "instruct the patient in a low-calorie, low-fat diet" describes an inappropriate action. Cognitive Level: Application Text Reference: p. 826 Nursing Process: Assessment

37. A patient is being treated for bacterial pharyngitis. Which of the following should the nurse recommend when promoting the patient's nutrition during treatment? A) A 1.5 L/day fluid restriction B) A high-potassium, low-sodium diet C) A liquid or soft diet D) A high-protein diet

C Feedback: A liquid or soft diet is provided during the acute stage of the disease, depending on the patient's appetite and the degree of discomfort that occurs with swallowing. The patient is encouraged to drink as much fluid as possible (at least 2 to 3 L/day). There is no need for increased potassium or protein intake.

8. A student nurse is developing a teaching plan for an adult patient with asthma. Which teaching point should have the highest priority in the plan of care that the student is developing? A) Gradually increase levels of physical exertion. B) Change filters on heaters and air conditioners frequently. C) Take prescribed medications as scheduled. D) Avoid goose-down pillows.

C Feedback: Although all of the measures are appropriate for a client with asthma, taking prescribed medications on time is the most important measure in preventing asthma attacks.

22. The nurse is preparing to suction a patient with an endotracheal tube. What should be the nurse's first step in the suctioning process? A) Explain the suctioning procedure to the patient and reposition the patient. B) Turn on suction source at a pressure not exceeding 120 mm Hg. C) Assess the patient's lung sounds and SAO2 via pulse oximeter. D) Perform hand hygiene and don nonsterile gloves, goggles, gown, and mask.

C Feedback: Assessment data indicate the need for suctioning and allow the nurse to monitor the effect of suction on the patient's level of oxygenation. Explaining the procedure would be the second step; performing hand hygiene is the third step, and turning on the suction source is the fourth step.

7. A school nurse is caring for a 10-year-old girl who is having an asthma attack. What is the preferred intervention to alleviate this client's airflow obstruction? A) Administer corticosteroids by metered dose inhaler B) Administer inhaled anticholinergics C) Administer an inhaled beta-adrenergic agonist D) Utilize a peak flow monitoring device

C Feedback: Asthma exacerbations are best managed by early treatment and education of the patient. Quick-acting beta-adrenergic medications are the first used for prompt relief of airflow obstruction. Systemic corticosteroids may be necessary to decrease airway inflammation in patients who fail to respond to inhaled beta-adrenergic medication. A peak flow device will not resolve short-term shortness of breath.

36. A nurse is teaching a patient with asthma about Azmacort, an inhaled corticosteroid. Which adverse effects should the nurse be sure to address in patient teaching? A) Dyspnea and increased respiratory secretions B) Nausea and vomiting C) Cough and oral thrush D) Fatigue and decreased level of consciousness

C Feedback: Azmacort has possible adverse effects of cough, dysphonia, oral thrush (candidiasis), and headache. In high doses, systemic effects may occur (e.g., adrenal suppression, osteoporosis, skin thinning, and easy bruising). The other listed adverse effects are not associated with this drug.

9. A student nurse is preparing to care for a patient with bronchiectasis. The student nurse should recognize that this patient is likely to experience respiratory difficulties related to what pathophysiologic process? A) Intermittent episodes of acute bronchospasm B) Alveolar distention and impaired diffusion C) Dilation of bronchi and bronchioles D) Excessive gas exchange in the bronchioles

C Feedback: Bronchiectasis is a chronic, irreversible dilation of the bronchi and bronchioles that results from destruction of muscles and elastic connective tissue. It is not characterized by acute bronchospasm, alveolar distention, or excessive gas exchange.

Hirudin derivatives,Thrombin inhibitor

Match the following anticoagulant drugs with their characteristics. Can choose unfractionated heparin, LMWH, Hirudin derivatives, Warfarin (Coumadin), Thrombin inhibitor (argatroban (Acova)) a) Is only administered IV

9. The critical care nurse is precepting a new nurse on the unit. Together they are caring for a patient who has a tracheostomy tube and is receiving mechanical ventilation. What action should the critical care nurse recommend when caring for the cuff? A) Deflate the cuff overnight to prevent tracheal tissue trauma. B) Inflate the cuff to the highest possible pressure in order to prevent aspiration. C) Monitor the pressure in the cuff at least every 8 hours D) Keep the tracheostomy tube plugged at all times.

C Feedback: Cuff pressure must be monitored by the respiratory therapist or nurse at least every 8 hours by attaching a handheld pressure gauge to the pilot balloon of the tube or by using the minimal leak volume or minimal occlusion volume technique. Plugging is only used when weaning the patient from tracheal support. Deflating the cuff overnight would be unsafe and inappropriate. High cuff pressure can cause tissue trauma.

29. It is cold season and the school nurse been asked to provide an educational event for the parent teacher organization of the local elementary school. What should the nurse include in teaching about the treatment of pharyngitis? A) Pharyngitis is more common in children whose immunizations are not up to date. B) There are no effective, evidence-based treatments for pharyngitis. C) Use of warm saline gargles or throat irrigations can relieve symptoms. D) Heat may increase the spasms in pharyngeal muscles.

C Feedback: Depending on the severity of the pharyngitis and the degree of pain, warm saline gargles or throat irrigations are used. The benefits of this treatment depend on the degree of heat that is applied. The nurse teaches about these procedures and about the recommended temperature of the solution: high enough to be effective and as warm as the patient can tolerate, usually 105ºF to 110ºF (40.6ºC to 43.3ºC). Irrigating the throat may reduce spasm in the pharyngeal muscles and relieve soreness of the throat. You would not tell the parent teacher organization that there is no real treatment of pharyngitis.

19. The nurse is caring for a patient in the ED for epistaxis. What information should the nurse include in patient discharge teaching as a way to prevent epistaxis? A) Keep nasal passages clear. B) Use decongestants regularly. C) Humidify the indoor environment. D) Use a tissue when blowing the nose.

C Feedback: Discharge teaching for prevention of epistaxis should include the following: avoid forceful nose bleeding, straining, high altitudes, and nasal trauma (nose picking). Adequate humidification may prevent drying of the nasal passages. Keeping nasal passages clear and using a tissue when blowing the nose are not included in discharge teaching for the prevention of epistaxis. Decongestants are not indicated.

5. The nurse is planning the care of a patient who is scheduled for a laryngectomy. The nurse should assign the highest priority to which postoperative nursing diagnosis? A) Anxiety related to diagnosis of cancer B) Altered nutrition related to swallowing difficulties C) Ineffective airway clearance related to airway alterations D) Impaired verbal communication related to removal of the larynx

C Feedback: Each of the listed diagnoses is valid, but ineffective airway clearance is the priority nursing diagnosis for all conditions.

34. A patient in the ICU has had an endotracheal tube in place for 3 weeks. The physician has ordered that a tracheostomy tube be placed. The patient's family wants to know why the endotracheal tube cannot be left in place. What would be the nurse's best response? A) "The physician may feel that mechanical ventilation will have to be used long-term." B) "Long-term use of an endotracheal tube diminishes the normal breathing reflex." C) "When an endotracheal tube is left in too long it can damage the lining of the windpipe." D) "It is much harder to breathe through an endotracheal tube than a tracheostomy."

C Feedback: Endotracheal intubation may be used for no longer than 2 to 3 weeks, by which time a tracheostomy must be considered to decrease irritation of and, trauma to, the tracheal lining, to reduce the incidence of vocal cord paralysis (secondary to laryngeal nerve damage), and to decrease the work of breathing. The need for long-term ventilation would not be the primary rationale for this change in treatment. Endotracheal tubes do not diminish the breathing reflex.

12. The occupational health nurse is obtaining a patient history during a pre-employment physical. During the history, the patient states that he has hereditary angioedema. The nurse should identify what implication of this health condition? A) It will result in increased loss of work days. B) It may cause episodes of weakness due to reduced cardiac output. C) It can cause life-threatening airway obstruction. D) It is unlikely to interfere with the individual's health.

C Feedback: Hereditary angioedema is an inherited condition that is characterized by episodes of life-threatening laryngeal edema. No information supports lost days of work or reduced cardiac function.

36. The nurse is performing nasotracheal suctioning on a medical patient and obtains copious amounts of secretions from the patient's airway, even after inserting and withdrawing the catheter several times. How should the nurse proceed? A) Continue suctioning the patient until no more secretions are obtained. B) Perform chest physiotherapy rather than nasotracheal suctioning. C) Wait several minutes and then repeat suctioning. D) Perform postural drainage and then repeat suctioning.

C Feedback: If additional suctioning is needed, the nurse should withdraw the catheter to the back of the pharynx, reassure the patient, and oxygenate for several minutes before resuming suctioning. Chest physiotherapy and postural drainage are not necessarily indicated.

4. The nurse is assessing a patient whose respiratory disease in characterized by chronic hyperinflation of the lungs. What would the nurse most likely assess in this patient? A) Signs of oxygen toxicity B) Chronic chest pain C) A barrel chest D) Long, thin fingers

C Feedback: In COPD patients with a primary emphysematous component, chronic hyperinflation leads to the barrel chest thorax configuration. The nurse most likely would not assess chest pain or long, thin fingers; these are not characteristic of emphysema. The patient would not show signs of oxygen toxicity unless he or she received excess supplementary oxygen.

39. The nurse is performing preoperative teaching with a patient who has cancer of the larynx. After completing patient teaching, what would be most important for the nurse to do? A) Give the patient his or her cell phone number. B) Refer the patient to a social worker or psychologist. C) Provide the patient with audiovisual materials about the surgery. D) Reassure the patient and family that everything will be alright.

C Feedback: Informational materials (written and audiovisual) about the surgery are given to the patient and family for review and reinforcement. The nurse never gives personal contact information to the patient. Nothing in the scenario indicates that a referral to a social worker or psychologist is necessary. False reassurance must always be avoided.

19. A nurse is providing discharge teaching for a client with COPD. When teaching the client about breathing exercises, what should the nurse include in the teaching? A) Lie supine to facilitate air entry B) Avoid pursed lip breathing C) Use diaphragmatic breathing D) Use chest breathing

C Feedback: Inspiratory muscle training and breathing retraining may help improve breathing patterns in patients with COPD. Training in diaphragmatic breathing reduces the respiratory rate, increases alveolar ventilation, and, sometimes, helps expel as much air as possible during expiration. Pursed-lip breathing helps slow expiration, prevents collapse of small airways, and controls the rate and depth of respiration. Diaphragmatic breathing, not chest breathing, increases lung expansion. Supine positioning does not aid breathing.

32. A patient's severe asthma has necessitated the use of a long-acting beta2-agonist (LABA). Which of the patient's statements suggests a need for further education? A) "I know that these drugs can sometimes make my heart beat faster." B) "I've heard that this drug is particularly good at preventing asthma attacks during exercise." C) "I'll make sure to use this each time I feel an asthma attack coming on." D) "I've heard that this drug sometimes gets less effective over time."

C Feedback: LABAs are not used for management of acute asthma symptoms. Tachycardia is a potential adverse effect and decreased protection against exercise-induced bronchospasm may occur with regular use.

3. The nurse is creating a plan of car for a patient diagnosed with acute laryngitis. What intervention should be included in the patient's plan of care? A) Place warm cloths on the patient's throat, as needed. B) Have the patient inhale warm steam three times daily. C) Encourage the patient to limit speech whenever possible. D) Limit the patient's fluid intake to 1.5 L/day.

C Feedback: Management of acute laryngitis includes resting the voice, avoiding irritants (including smoking), resting, and inhaling cool steam or an aerosol. Fluid intake should be increased. Warm cloths on the throat will not help relieve the symptoms of acute laryngitis.

26. A nursing student is discussing a patient with viral pharyngitis with the preceptor at the walk-in clinic. What should the preceptor tell the student about nursing care for patients with viral pharyngitis? A) Teaching focuses on safe and effective use of antibiotics. B) The patient should be preliminarily screened for surgery. C) Symptom management is the main focus of medical and nursing care. D) The focus of care is resting the voice to prevent chronic hoarseness.

C Feedback: Nursing care for patients with viral pharyngitis focuses on symptomatic management. Antibiotics are not prescribed for viral etiologies. Surgery is not indicated in the treatment of viral pharyngitis. Chronic hoarseness is not a common sequela of viral pharyngitis, so teaching ways to prevent it would be of no use in this instance.

18. A 45-year-old obese man arrives in a clinic with complaints of daytime sleepiness, difficulty going to sleep at night, and snoring. The nurse should recognize the manifestations of what health problem? A) Adenoiditis B) Chronic tonsillitis C) Obstructive sleep apnea D) Laryngeal cancer

C Feedback: Obstructive sleep apnea occurs in men, especially those who are older and overweight. Symptoms include excessive daytime sleepiness, insomnia, and snoring. Daytime sleepiness and difficulty going to sleep at night are not indications of tonsillitis or adenoiditis. This patient's symptoms are not suggestive of laryngeal cancer.

23. The nurse is teaching a patient with allergic rhinitis about the safe and effective use of his medications. What would be the most essential information to give this patient about preventing possible drug interactions? A) Prescription medications can be safely supplemented with OTC medications. B) Use only one pharmacy so the pharmacist can check drug interactions. C) Read drug labels carefully before taking OTC medications. D) Consult the Internet before selecting an OTC medication.

C Feedback: Patient education is essential when assisting the patient in the use of all medications. To prevent possible drug interactions, the patient is cautioned to read drug labels before taking any OTC medications. Some Web sites are reliable and valid information sources, but this is not always the case. Patients do not necessarily need to limit themselves to one pharmacy, though checking for potential interactions is important. Not all OTC medications are safe additions to prescription medication regimens.

11. A patient arrives in the emergency department with an attack of acute bronchiectasis. Chest auscultation reveals the presence of copious secretions. What intervention should the nurse prioritize in this patient's care? A) Oral administration of diuretics B) Intravenous fluids to reduce the viscosity of secretions C) Postural chest drainage D) Pulmonary function testing

C Feedback: Postural drainage is part of all treatment plans for bronchiectasis, because draining of the bronchiectatic areas by gravity reduces the amount of secretions and the degree of infection. Diuretics and IV fluids will not aid in the mobilization of secretions. Lung function testing may be indicated, but this assessment will not relieve the patient's symptoms.

25. The nurse is preparing to discharge a patient after thoracotomy. The patient is going home on oxygen therapy and requires wound care. As a result, he will receive home care nursing. What should the nurse include in discharge teaching for this patient? A) Safe technique for self-suctioning of secretions B) Technique for performing postural drainage C) Correct and safe use of oxygen therapy equipment D) How to provide safe and effective tracheostomy care

C Feedback: Respiratory care and other treatment modalities (oxygen, incentive spirometry, chest physiotherapy [CPT], and oral, inhaled, or IV medications) may be continued at home. Therefore, the nurse needs to instruct the patient and family in their correct and safe use. The scenario does not indicate the patient needs help with suctioning, postural drainage, or tracheostomy care.

19. A patient with a severe exacerbation of COPD requires reliable and precise oxygen delivery. Which mask will the nurse expect the physician to order? A) Non-rebreather air mask B) Tracheostomy collar C) Venturi mask D) Face tent

C Feedback: The Venturi mask provides the most accurate method of oxygen delivery. Other methods of oxygen delivery include the aerosol mask, tracheostomy collar, and face tents, but these do not match the precision of a Venturi mask.

The nurse assesses the right femoral artery puncture site as soon as the patient arrives after having a stent inserted into a coronary artery. The insertion site is not bleeding or discolored. What should the nurse do next to ensure the femoral artery is intact?

C Inspect the patient's right side and back.

35. The nurse has noted the emergence of a significant amount of fresh blood at the drain site of a patient who is postoperative day 1 following total laryngectomy. How should the nurse respond to this development? A) Remove the patient's drain and apply pressure with a sterile gauze. B) Assess the patient, reposition the patient supine, and apply wall suction to the drain. C) Rapidly assess the patient and notify the surgeon about the patient's bleeding. D) Administer a STAT dose of vitamin K to aid coagulation.

C Feedback: The nurse promptly notifies the surgeon of any active bleeding, which can occur at a variety of sites, including the surgical site, drains, and trachea. The drain should not be removed or connected to suction. Supine positioning would exacerbate the bleeding. Vitamin K would not be administered without an order.

37. A nurse has performed tracheal suctioning on a patient who experienced increasing dyspnea prior to a procedure. When applying the nursing process, how can the nurse best evaluate the outcomes of this intervention? A) Determine whether the patient can now perform forced expiratory technique (FET). B) Percuss the patient's lungs and thorax. C) Measure the patient's oxygen saturation. D) Have the patient perform incentive spirometry.

C Feedback: The patient's response to suctioning is usually determined by performing chest auscultation and by measuring the patient's oxygen saturation. FET, incentive spirometry, and percussion are not normally used as evaluative techniques.

12. A nurse is completing a focused respiratory assessment of a child with asthma. What assessment finding is most closely associated with the characteristic signs and symptoms of asthma? A) Shallow respirations B) Increased anterior-posterior (A-P) diameter C) Bilateral wheezes D) Bradypnea

C Feedback: The three most common symptoms of asthma are cough, dyspnea, and wheezing. There may be generalized wheezing (the sound of airflow through narrowed airways), first on expiration and then, possibly, during inspiration as well. Respirations are not usually slow and the child's A-P diameter does not normally change.

14. The home care nurse is assessing a patient who requires home oxygen therapy. What criterion indicates that an oxygen concentrator will best meet the needs of the patient in the home environment? A) The patient desires a low-maintenance oxygen delivery system that delivers oxygen flow rates up to 6 L/min. B) The patient requires a high-flow system for use with a tracheostomy collar. C) The patient desires a portable oxygen delivery system that can deliver 2 L/min. D) The patient's respiratory status requires a system that provides an FiO2 of 65%.

C Feedback: The use of oxygen concentrators is another means of providing varying amounts of oxygen, especially in the home setting. They can deliver oxygen flows from 1 to 10 L/min and provide an FiO2 of about 40%. They require regular maintenance and are not used for high-flow applications. The patient desiring a portable oxygen delivery system of 2L/min will benefit from the use of an oxygen concentrator.

36. The nurse is creating a care plan for a patient who is status post-total laryngectomy. Much of the plan consists of a long-term postoperative communication plan for alaryngeal communication. What form of alaryngeal communication will likely be chosen? A) Esophageal speech B) Electric larynx C) Tracheoesophageal puncture D) American sign language (ASL)

C Feedback: Tracheoesophageal puncture is simple and has few complications. It is associated with high phonation success, good phonation quality, and steady long-term results. As a result, it is preferred over esophageal speech, and electric larynx or ASL.

Physiological Integrity 31. The nurse takes an admission history on a patient with possible asthma who has new-onset wheezing and shortness of breath. Which information may indicate a need for a change in therapy? a. The patient has chronic inflammatory bowel disease. b. The patient has a history of pneumonia 6 months ago. c. The patient takes propranolol (Inderal) for hypertension. d. The patient uses acetaminophen (Tylenol) for headaches.

C -Blockers such as propranolol can cause bronchospasm in some patients with asthma. The other information will be documented in the health history but does not indicate a need for a change in therapy. DIF: Cognitive Level: Apply (application) REF: 576 TOP: Nursing Process: Assessment MSC:

Physiological Integrity 15. Following an acute myocardial infarction (AMI), a patient ambulates in the hospital hallway. When the nurse is evaluating the patient's response to the activity, which assessment data would indicate that the exercise level should be decreased? a. Blood pressure (BP) changes from 118/60 to 126/68 mm Hg. b. Oxygen saturation drops from 99% to 95%. c. Heart rate increases from 66 to 92 beats/minute. d. Respiratory rate goes from 14 to 20 breaths/minute.

C A change in heart rate of more than 20 beats over the resting heart rate indicates that the patient should stop and rest. The increases in BP and respiratory rate, and the slight decrease in oxygen saturation, are normal responses to exercise. DIF: Cognitive Level: Apply (application) REF: 761 TOP: Nursing Process: Evaluation MSC:

A 74-year-old man with a history of prostate cancer and hypertension is admitted to the emergency department with substernal chest pain. Which action will the nurse complete before administering sublingual nitroglycerin?

C Obtain a 12-lead electrocardiogram (ECG).

Physiological Integrity 12. The nurse interviews a patient with a new diagnosis of chronic obstructive pulmonary disease (COPD). Which information is most helpful in confirming a diagnosis of chronic bronchitis? a. The patient tells the nurse about a family history of bronchitis. b. The patient's history indicates a 30 pack-year cigarette history. c. The patient complains about a productive cough every winter for 3 months. d. The patient denies having any respiratory problems until the last 12 months.

C A diagnosis of chronic bronchitis is based on a history of having a productive cough for 3 months for at least 2 consecutive years. There is no family tendency for chronic bronchitis. Although smoking is the major risk factor for chronic bronchitis, a smoking history does not confirm the diagnosis. DIF: Cognitive Level: Apply (application) REF: 579 TOP: Nursing Process: Assessment MSC:

Physiological Integrity 40. The nurse notes that a patient has incisional pain, a poor cough effort, and scattered rhonchi after a thoracotomy. Which action should the nurse take first? a. Assist the patient to sit upright in a chair. b. Splint the patient's chest during coughing. c. Medicate the patient with prescribed morphine. d. Observe the patient use the incentive spirometer.

C A major reason for atelectasis and poor airway clearance in patients after chest surgery is incisional pain (which increases with deep breathing and coughing). The first action by the nurse should be to medicate the patient to minimize incisional pain. The other actions are all appropriate ways to improve airway clearance but should be done after the morphine is given. DIF: Cognitive Level: Apply (application) REF: 549 OBJ: Special Questions: Prioritization TOP: Nursing Process: Implementation MSC:

Physiological Integrity 17. A patient in metabolic alkalosis is admitted to the emergency department, and pulse oximetry (SpO2) indicates that the O2 saturation is 94%. Which action should the nurse take next? a. Administer bicarbonate. b. Complete a head-to-toe assessment. c. Place the patient on high-flow oxygen. d. Obtain repeat arterial blood gases (ABGs).

C Although the O2 saturation is adequate, the left shift in the oxyhemoglobin dissociation curve will decrease the amount of oxygen delivered to tissues, so high oxygen concentrations should be given. Bicarbonate would worsen the patient's condition. A head-to-toe assessment and repeat ABGs may be implemented. However, the priority intervention is to give high-flow oxygen. DIF: Cognitive Level: Apply (application) REF: eTable 26-1 OBJ: Special Questions: Prioritization TOP: Nursing Process: Implementation MSC:

Physiological Integrity 24. A patient who is receiving dobutamine (Dobutrex) for the treatment of acute decompensated heart failure (ADHF) has the following nursing interventions included in the plan of care. Which action will be most appropriate for the registered nurse (RN) to delegate to an experienced licensed practical/vocational nurse (LPN/LVN)? a. Assess the IV insertion site for signs of extravasation. b. Teach the patient the reasons for remaining on bed rest. c. Monitor the patient's blood pressure and heart rate every hour. d. Titrate the rate to keep the systolic blood pressure >90 mm Hg.

C An experienced LPN/LVN would be able to monitor BP and heart rate and would know to report significant changes to the RN. Teaching patients, making adjustments to the drip rate for vasoactive medications, and monitoring for serious complications such as extravasation require RN level education and scope of practice. DIF: Cognitive Level: Apply (application) REF: 15-16 OBJ: Special Questions: Delegation TOP: Nursing Process: Planning MSC:

Physiological Integrity 13. Which diagnostic test will be most useful to the nurse in determining whether a patient admitted with acute shortness of breath has heart failure? a. Serum troponin b. Arterial blood gases c. B-type natriuretic peptide d. 12-lead electrocardiogram

C B-type natriuretic peptide (BNP) is secreted when ventricular pressures increase, as they do with heart failure. Elevated BNP indicates a probable or very probable diagnosis of heart failure. A twelve-lead electrocardiogram, arterial blood gases, and troponin may also be used in determining the causes or effects of heart failure but are not as clearly diagnostic of heart failure as BNP. DIF: Cognitive Level: Apply (application) REF: 772 TOP: Nursing Process: Assessment MSC:

Safe and Effective Care Environment 5. A patient with a tracheostomy has a new order for a fenestrated tracheostomy tube. Which action should the nurse include in the plan of care in collaboration with the speech therapist? a. Leave the tracheostomy inner cannula inserted at all times. b. Place the decannulation cap in the tube before cuff deflation. c. Assess the ability to swallow before using the fenestrated tube. d. Inflate the tracheostomy cuff during use of the fenestrated tube.

C Because the cuff is deflated when using a fenestrated tube, the patient's risk for aspiration should be assessed before changing to a fenestrated tracheostomy tube. The decannulation cap is never inserted before cuff deflation because to do so would obstruct the patient's airway. The cuff is deflated and the inner cannula removed to allow air to flow across the patient's vocal cords when using a fenestrated tube. DIF: Cognitive Level: Apply (application) REF: 511 TOP: Nursing Process: Planning MSC:

Physiological Integrity 3. A patient is scheduled for pulmonary function testing. Which action should the nurse take to prepare the patient for this procedure? a. Give the rescue medication immediately before testing. b. Administer oral corticosteroids 2 hours before the procedure. c. Withhold bronchodilators for 6 to 12 hours before the examination. d. Ensure that the patient has been NPO for several hours before the test.

C Bronchodilators are held before pulmonary function testing (PFT) so that a baseline assessment of airway function can be determined. Testing is repeated after bronchodilator use to determine whether the decrease in lung function is reversible. There is no need for the patient to be NPO. Oral corticosteroids should be held before PFTs. Rescue medications (which are bronchodilators) would not be given until after the baseline pulmonary function was assessed. DIF: Cognitive Level: Apply (application) REF: 566 TOP: Nursing Process: Implementation MSC:

Physiological Integrity 33. A patient is diagnosed with both human immunodeficiency virus (HIV) and active tuberculosis (TB) disease. Which information obtained by the nurse is most important to communicate to the health care provider? a. The Mantoux test had an induration of 7 mm. b. The chest-x-ray showed infiltrates in the lower lobes. c. The patient is being treated with antiretrovirals for HIV infection. d. The patient has a cough that is productive of blood-tinged mucus.

C Drug interactions can occur between the antiretrovirals used to treat HIV infection and the medications used to treat TB. The other data are expected in a patient with HIV and TB. DIF: Cognitive Level: Apply (application) REF: 532 OBJ: Special Questions: Prioritization TOP: Nursing Process: Assessment MSC:

Physiological Integrity 15. A patient with heart failure has a new order for captopril (Capoten) 12.5 mg PO. After administering the first dose and teaching the patient about the drug, which statement by the patient indicates that teaching has been effective? a. "I will be sure to take the medication with food." b. "I will need to eat more potassium-rich foods in my diet." c. "I will call for help when I need to get up to use the bathroom." d. "I will expect to feel more short of breath for the next few days."

C Captopril can cause hypotension, especially after the initial dose, so it is important that the patient not get up out of bed without assistance until the nurse has had a chance to evaluate the effect of the first dose. The angiotensin-converting enzyme (ACE) inhibitors are potassium sparing, and the nurse should not teach the patient to purposely increase sources of dietary potassium. Increased shortness of breath is expected with the initiation of -adrenergic blocker therapy for heart failure, not for ACE inhibitor therapy. ACE inhibitors are best absorbed when taken an hour before eating. DIF: Cognitive Level: Apply (application) REF: 776 TOP: Nursing Process: Evaluation MSC:

Physiological Integrity 11. The nurse has received the laboratory results for a patient who developed chest pain 4 hours ago and may be having a myocardial infarction. The most important laboratory result to review will be a. myoglobin. b. low-density lipoprotein (LDL) cholesterol. c. troponins T and I. d. creatine kinase-MB (CK-MB).

C Cardiac troponins start to elevate 4 to 6 hours after myocardial injury and are highly specific to myocardium. They are the preferred diagnostic marker for myocardial infarction. Myoglobin rises in response to myocardial injury within 30 to 60 minutes. It is rapidly cleared from the body, thus limiting its use in the diagnosis of myocardial infarction. LDL cholesterol is useful in assessing cardiovascular risk but is not helpful in determining whether a patient is having an acute myocardial infarction. Creatine kinase (CK-MB) is specific to myocardial injury and infarction and increases 4 to 6 hours after the infarction occurs. It is often trended with troponin levels. DIF: Cognitive Level: Apply (application) REF: 698 TOP: Nursing Process: Assessment MSC:

Physiological Integrity 9. The nurse is caring for a patient with chronic obstructive pulmonary disease (COPD). Which information obtained from the patient would prompt the nurse to consult with the health care provider before administering the prescribed theophylline? a. The patient reports a recent 15-pound weight gain. b. The patient denies any shortness of breath at present. c. The patient takes cimetidine (Tagamet) 150 mg daily. d. The patient complains about coughing up green mucus.

C Cimetidine interferes with the metabolism of theophylline, and concomitant administration may lead rapidly to theophylline toxicity. The other patient information would not affect whether the theophylline should be administered or not. DIF: Cognitive Level: Apply (application) REF: 571 | 572 TOP: Nursing Process: Assessment MSC:

Integrated Process: Nursing Process (Assessment) 14. A client with lung cancer refuses pain medications because he or she is "afraid of addiction." What is the nurse's best response? a. "I can ask the physician to change your medication to a drug that is less potent." b. "I can use other measures such as music therapy to distract you." c. "It is unlikely you will become addicted from taking medicine for pain." d. "I can just give you aspirin or acetaminophen (Tylenol) if you like."

C Clients should be encouraged to take their pain medications; addiction usually is not an issue with a client in pain. The nurse would not request that the pain medication be changed unless it was not effective. Other methods to decrease pain can be used, in addition to pain medications. DIF: Cognitive Level: Application/Applying or higher REF: N/A TOP: Client Needs Category: Physiological Integrity (Pharmacological and Parenteral Therapies—Pharmacological Pain Management)

Integrated Process: Nursing Process (Implementation) 23. The nurse is teaching a client with cystic fibrosis. What activity does the nurse teach as the priority? a. Taking daily antibiotics b. Having genetic screening c. Maintaining good nutrition d. Exercising daily

C Clients with cystic fibrosis (CF) often are malnourished owing to vitamin deficiency and pancreatic malfunction. Maintaining nutrition is essential. Daily antibiotics and daily exercise are not essential actions. Genetic screening would not help the client manage CF better. DIF: Cognitive Level: Application/Applying or higher REF: N/A TOP: Client Needs Category: Health Promotion and Maintenance (Self-Care)

Physiological Integrity 24. A patient who has a right-sided chest tube following a thoracotomy has continuous bubbling in the suction-control chamber of the collection device. Which action by the nurse is most appropriate? a. Document the presence of a large air leak. b. Notify the surgeon of a possible pneumothorax. c. Take no further action with the collection device. d. Adjust the dial on the wall regulator to decrease suction.

C Continuous bubbling is expected in the suction-control chamber and indicates that the suction-control chamber is connected to suction. An air leak would be detected in the water-seal chamber. There is no evidence of pneumothorax. Increasing or decreasing the vacuum source will not adjust the suction pressure. The amount of suction applied is regulated by the amount of water in this chamber and not by the amount of suction applied to the system. DIF: Cognitive Level: Apply (application) REF: 547 TOP: Nursing Process: Implementation MSC:

Integrated Process: Nursing Process (Implementation) 29. The nurse is teaching a client with bronchiolitis obliterans organizing pneumonia (BOOP) about corticosteroid therapy. What statement is accurate for the nurse to teach the client? a. "You will be on this drug the rest of your life." b. "You will be prone to many long-term side effects of this drug." c. "A short course of therapy will help with acute episodes." d. "This medication cannot be taken with antibiotic therapy."

C Corticosteroids are used for acute episodes and are very effective in decreasing manifestations. The client may never have another relapse after therapy. The client is not on the drug for "life," and therefore is not prone to long-term side effects. Agents can be given with antibiotics. DIF: Cognitive Level: Comprehension/Understanding REF: p. 630 TOP: Client Needs Category: Physiological Integrity (Pharmacological and Parenteral Therapies—Expected Actions/Outcomes)

Physiological Integrity 43. A patient has acute bronchitis with a nonproductive cough and wheezes. Which topic should the nurse plan to include in the teaching plan? a. Purpose of antibiotic therapy b. Ways to limit oral fluid intake c. Appropriate use of cough suppressants d. Safety concerns with home oxygen therapy

C Cough suppressants are frequently prescribed for acute bronchitis. Because most acute bronchitis is viral in origin, antibiotics are not prescribed unless there are systemic symptoms. Fluid intake is encouraged. Home oxygen is not prescribed for acute bronchitis, although it may be used for chronic bronchitis. DIF: Cognitive Level: Apply (application) REF: 522 TOP: Nursing Process: Planning MSC:

Physiological Integrity 11. A patient with chronic obstructive pulmonary disease (COPD) has a nursing diagnosis of imbalanced nutrition: less than body requirements. Which intervention would be most appropriate for the nurse to include in the plan of care? a. Encourage increased intake of whole grains. b. Increase the patient's intake of fruits and fruit juices. c. Offer high-calorie snacks between meals and at bedtime. d. Assist the patient in choosing foods with high vegetable and mineral content.

C Eating small amounts more frequently (as occurs with snacking) will increase caloric intake by decreasing the fatigue and feelings of fullness associated with large meals. Patients with COPD should rest before meals. Foods that have a lot of texture like whole grains may take more energy to eat and get absorbed and lead to decreased intake. Although fruits, juices, and vegetables are not contraindicated, foods high in protein are a better choice. DIF: Cognitive Level: Apply (application) REF: 596 TOP: Nursing Process: Planning MSC:

Physiological Integrity 7. The nurse completes a shift assessment on a patient admitted in the early phase of heart failure. When auscultating the patient's lungs, which finding would the nurse most likely hear? a. Continuous rumbling, snoring, or rattling sounds mainly on expiration b. Continuous high-pitched musical sounds on inspiration and expiration c. Discontinuous, high-pitched sounds of short duration heard on inspiration d. A series of long-duration, discontinuous, low-pitched sounds during inspiration

C Fine crackles are likely to be heard in the early phase of heart failure. Fine crackles are discontinuous, high-pitched sounds of short duration heard on inspiration. Rhonchi are continuous rumbling, snoring, or rattling sounds mainly on expiration. Course crackles are a series of long-duration, discontinuous, low-pitched sounds during inspiration. Wheezes are continuous high-pitched musical sounds on inspiration and expiration. DIF: Cognitive Level: Apply (application) REF: 489 TOP: Nursing Process: Assessment MSC:

Physiological Integrity 9. The nurse teaches a patient about pulmonary function testing (PFT). Which statement, if made by the patient, indicates teaching was effective? a. "I will use my inhaler right before the test." b. "I won't eat or drink anything 8 hours before the test." c. "I should inhale deeply and blow out as hard as I can during the test." d. "My blood pressure and pulse will be checked every 15 minutes after the test."

C For PFT, the patient should inhale deeply and exhale as long, hard, and fast as possible. The other actions are not needed with PFT. The administration of inhaled bronchodilators should be avoided 6 hours before the procedure. DIF: Cognitive Level: Apply (application) REF: 493-495 TOP: Nursing Process: Planning MSC:

Physiological Integrity 25. The nurse provides preoperative instruction for a patient scheduled for a left pneumonectomy for cancer of the lung. Which information should the nurse include about the patient's postoperative care? a. Positioning on the right side b. Bed rest for the first 24 hours c. Frequent use of an incentive spirometer d. Chest tube placement with continuous drainage

C Frequent deep breathing and coughing are needed after chest surgery to prevent atelectasis. To promote gas exchange, patients after pneumonectomy are positioned on the surgical side. Early mobilization decreases the risk for postoperative complications such as pneumonia and deep vein thrombosis. In a pneumonectomy, chest tubes may or may not be placed in the space from which the lung was removed. If a chest tube is used, it is clamped and only released by the surgeon to adjust the volume of serosanguineous fluid that will fill the space vacated by the lung. If the cavity overfills, it could compress the remaining lung and compromise the cardiovascular and pulmonary function. Daily chest x-rays can be used to assess the volume and space. DIF: Cognitive Level: Apply (application) REF: 548 TOP: Nursing Process: Planning MSC:

Physiological Integrity 12. Heparin is ordered for a patient with a non-ST-segment-elevation myocardial infarction (NSTEMI). What is the purpose of the heparin? a. Heparin enhances platelet aggregation. b. Heparin decreases coronary artery plaque size. c. Heparin prevents the development of new clots in the coronary arteries. d. Heparin dissolves clots that are blocking blood flow in the coronary arteries.

C Heparin helps prevent the conversion of fibrinogen to fibrin and decreases coronary artery thrombosis. It does not change coronary artery plaque, dissolve already formed clots, or enhance platelet aggregation. DIF: Cognitive Level: Understand (comprehension) REF: 746 TOP: Nursing Process: Implementation MSC:

Physiological Integrity 22. A patient has recently started on digoxin (Lanoxin) in addition to furosemide (Lasix) and captopril (Capoten) for the management of heart failure. Which assessment finding by the home health nurse is a priority to communicate to the health care provider? a. Presence of 1 to 2+ edema in the feet and ankles b. Palpable liver edge 2 cm below the ribs on the right side c. Serum potassium level 3.0 mEq/L after 1 week of therapy d. Weight increase from 120 pounds to 122 pounds over 3 days

C Hypokalemia can predispose the patient to life-threatening dysrhythmias (e.g., premature ventricular contractions), and potentiate the actions of digoxin and increase the risk for digoxin toxicity, which can also cause life-threatening dysrhythmias. The other data indicate that the patient's heart failure requires more effective therapies, but they do not require nursing action as rapidly as the low serum potassium level. DIF: Cognitive Level: Apply (application) REF: 777 OBJ: Special Questions: Prioritization TOP: Nursing Process: Assessment MSC:

Physiological Integrity 10. A patient who had a total laryngectomy has a nursing diagnosis of hopelessness related to loss of control of personal care. Which information obtained by the nurse is the best indicator that this identified problem is resolving? a. The patient lets the spouse provide tracheostomy care. b. The patient allows the nurse to suction the tracheostomy. c. The patient asks how to clean the tracheostomy stoma and tube. d. The patient uses a communication board to request "No Visitors."

C Independently caring for the laryngectomy tube indicates that the patient has regained control of personal care and hopelessness is at least partially resolved. Letting the nurse and spouse provide care and requesting no visitors may indicate that the patient is still experiencing hopelessness. DIF: Cognitive Level: Apply (application) REF: 518 TOP: Nursing Process: Evaluation MSC:

Physiological Integrity 32. A patient newly diagnosed with asthma is being discharged. The nurse anticipates including which topic in the discharge teaching? a. Use of long-acting -adrenergic medications b. Side effects of sustained-release theophylline c. Self-administration of inhaled corticosteroids d. Complications associated with oxygen therapy

C Inhaled corticosteroids are more effective in improving asthma than any other drug and are indicated for all patients with persistent asthma. The other therapies would not typically be first-line treatments for newly diagnosed asthma. DIF: Cognitive Level: Apply (application) REF: 569 TOP: Nursing Process: Implementation MSC:

Integrated Process: Nursing Process (Implementation) 27. A client with pulmonary fibrosis is being discharged home. What is the highest priority teaching need? a. Dietary modifications b. Determining activity tolerance c. Avoiding infection d. Medication therapy

C It is extremely important to teach the client with pulmonary fibrosis to avoid infection because the disease will quickly become worse as a result of decreased lung function. The client may take longer to recover from an infection, and the ability to recover may be severely limited owing to the progression of the disease. Teaching the client about modifications in diet, how to determine response to activity, and treatment medications would be secondary. DIF: Cognitive Level: Application/Applying or higher REF: N/A TOP: Client Needs Category: Health Promotion and Maintenance (Self-Care)

Health Promotion and Maintenance 2. Which nursing intervention will be most effective when assisting the patient with coronary artery disease (CAD) to make appropriate dietary changes? a. Give the patient a list of low-sodium, low-cholesterol foods that should be included in the diet. b. Emphasize the increased risk for heart problems unless the patient makes the dietary changes. c. Help the patient modify favorite high-fat recipes by using monosaturated oils when possible. d. Inform the patient that a diet containing no saturated fat and minimal salt will be necessary.

C Lifestyle changes are more likely to be successful when consideration is given to the patient's values and preferences. The highest percentage of calories from fat should come from monosaturated fats. Although low-sodium and low-cholesterol foods are appropriate, providing the patient with a list alone is not likely to be successful in making dietary changes. Completely removing saturated fat from the diet is not a realistic expectation. Up to 7% of calories in the therapeutic lifestyle changes (TLC) diet can come from saturated fat. Telling the patient about the increased risk without assisting further with strategies for dietary change is unlikely to be successful. DIF: Cognitive Level: Apply (application) REF: 736-737 TOP: Nursing Process: Implementation MSC:

Physiological Integrity 10. The nurse observes a student who is listening to a patient's lungs who is having no problems with breathing. Which action by the student indicates a need to review respiratory assessment skills? a. The student starts at the apices of the lungs and moves to the bases. b. The student compares breath sounds from side to side avoiding bony areas. c. The student places the stethoscope over the posterior chest and listens during inspiration. d. The student instructs the patient to breathe slowly and a little more deeply than normal through the mouth.

C Listening only during inspiration indicates the student needs a review of respiratory assessment skills. At each placement of the stethoscope, listen to at least one cycle of inspiration and expiration. During chest auscultation, instruct the patient to breathe slowly and a little deeper than normal through the mouth. Auscultation should proceed from the lung apices to the bases, comparing opposite areas of the chest, unless the patient is in respiratory distress or will tire easily. If so, start at the bases (see Fig. 26-7). Place the stethoscope over lung tissue, not over bony prominences. DIF: Cognitive Level: Apply (application) REF: 486 TOP: Nursing Process: Assessment MSC:

Physiological Integrity 16. Which information obtained by the nurse who is admitting the patient for magnetic resonance imaging (MRI) will be most important to report to the health care provider before the MRI? a. The patient has an allergy to shellfish. b. The patient has a history of atherosclerosis. c. The patient has a permanent ventricular pacemaker. d. The patient took all the prescribed cardiac medications today.

C MRI is contraindicated for patients with implanted metallic devices such as pacemakers. The other information also will be reported to the health care provider but does not impact on whether or not the patient can have an MRI. DIF: Cognitive Level: Apply (application) REF: 702 TOP: Nursing Process: Implementation MSC:

Integrated Process: Teaching/Learning 22. A client with chronic obstructive pulmonary disease (COPD) reports social isolation. What does the nurse encourage the client to do? a. Join a support group for people with COPD. b. Ask the client's physician for an antianxiety agent. c. Verbalize his or her thoughts and feelings. d. Participate in community activities.

C Many clients with moderate to severe COPD become socially isolated because they are embarrassed by frequent coughing and mucus production. They also can experience fatigue, which limits their activities. The nurse needs to encourage the client to verbalize thoughts and feelings so that appropriate interventions can be selected. Joining a support group would not decrease feelings of social isolation if the client does not verbalize feelings. Antianxiety agents will not help the client with social isolation. Encouraging a client to participate in activities without verbalizing concerns also would not be an effective strategy for decreasing social isolation. DIF: Cognitive Level: Application/Applying or higher REF: N/A TOP: Client Needs Category: Psychosocial Integrity (Coping Mechanisms)

Physiological Integrity 9. The nurse plans discharge teaching for a patient with chronic heart failure who has prescriptions for digoxin (Lanoxin) and hydrochlorothiazide (HydroDIURIL). Appropriate instructions for the patient include a. limit dietary sources of potassium. b. take the hydrochlorothiazide before bedtime. c. notify the health care provider if nausea develops. d. skip the digoxin if the pulse is below 60 beats/minute.

C Nausea is an indication of digoxin toxicity and should be reported so that the provider can assess the patient for toxicity and adjust the digoxin dose, if necessary. The patient will need to include potassium-containing foods in the diet to avoid hypokalemia. Patients should be taught to check their pulse daily before taking the digoxin and if the pulse is less than 60, to call their provider before taking the digoxin. Diuretics should be taken early in the day to avoid sleep disruption. DIF: Cognitive Level: Apply (application) REF: 777 TOP: Nursing Process: Planning MSC:

Health Promotion and Maintenance 10. A patient who is taking rifampin (Rifadin) for tuberculosis calls the clinic and reports having orange discolored urine and tears. Which is the best response by the nurse? a. Ask if the patient is experiencing shortness of breath, hives, or itching. b. Ask the patient about any visual abnormalities such as red-green color discrimination. c. Explain that orange discolored urine and tears are normal while taking this medication. d. Advise the patient to stop the drug and report the symptoms to the health care provider.

C Orange-colored body secretions are a side effect of rifampin. The patient does not have to stop taking the medication. The findings are not indicative of an allergic reaction. Alterations in red-green color discrimination commonly occurs when taking ethambutol (Myambutol), which is a different TB medication. DIF: Cognitive Level: Apply (application) REF: 531 TOP: Nursing Process: Planning MSC:

Physiological Integrity 5. A patient who has chronic heart failure tells the nurse, "I was fine when I went to bed, but I woke up in the middle of the night feeling like I was suffocating!" The nurse will document this assessment finding as a. orthopnea. b. pulsus alternans. c. paroxysmal nocturnal dyspnea. d. acute bilateral pleural effusion.

C Paroxysmal nocturnal dyspnea is caused by the reabsorption of fluid from dependent body areas when the patient is sleeping and is characterized by waking up suddenly with the feeling of suffocation. Pulsus alternans is the alternation of strong and weak peripheral pulses during palpation. Orthopnea indicates that the patient is unable to lie flat because of dyspnea. Pleural effusions develop over a longer time period. DIF: Cognitive Level: Remember (knowledge) REF: 770-771 TOP: Nursing Process: Assessment MSC:

Psychosocial Integrity 22. A patient with chronic obstructive pulmonary disease (COPD) has poor gas exchange. Which action by the nurse would be most appropriate? a. Have the patient rest in bed with the head elevated to 15 to 20 degrees. b. Ask the patient to rest in bed in a high-Fowler's position with the knees flexed. c. Encourage the patient to sit up at the bedside in a chair and lean slightly forward. d. Place the patient in the Trendelenburg position with several pillows behind the head.

C Patients with COPD improve the mechanics of breathing by sitting up in the "tripod" position. Resting in bed with the head elevated in a semi-Fowler's position would be an alternative position if the patient was confined to bed, but sitting in a chair allows better ventilation. The Trendelenburg position or sitting upright in bed with the knees flexed would decrease the patient's ability to ventilate well. DIF: Cognitive Level: Apply (application) REF: 599 TOP: Nursing Process: Implementation MSC:

Physiological Integrity 16. An occupational health nurse works at a manufacturing plant where there is potential exposure to inhaled dust. Which action, if recommended by the nurse, will be most helpful in reducing the incidence of lung disease? a. Treat workers with pulmonary fibrosis. b. Teach about symptoms of lung disease. c. Require the use of protective equipment. d. Monitor workers for coughing and wheezing.

C Prevention of lung disease requires the use of appropriate protective equipment such as masks. The other actions will help in recognition or early treatment of lung disease but will not be effective in prevention of lung damage. Repeated exposure eventually results in diffuse pulmonary fibrosis. Fibrosis is the result of tissue repair after inflammation. DIF: Cognitive Level: Apply (application) REF: 535 TOP: Nursing Process: Implementation MSC:

Physiological Integrity 9. A patient is scheduled for a cardiac catheterization with coronary angiography. Before the test, the nurse informs the patient that a. a catheter will be inserted into a vein in the arm or leg and advanced to the heart. b. ECG monitoring will be required for 24 hours following the test to detect any dysrhythmias. c. a feeling of warmth may be experienced as the contrast material is injected into the catheter. d. it will be important to lie completely still during the coronary angiography procedure.

C Rationale: A sensation of warmth or flushing is common when the iodine-based contrast material is injected, which can be anxiety-producing unless it has been discussed with the patient. The catheter is inserted in an artery (typically the femoral artery) and advanced to the openings for the coronary arteries at the aortic root. Dysrhythmias may occur during the procedure, but most patients are discharged a few hours after the coronary arteriogram or angiogram is completed. The patient is not required to be completely immobile during the procedure. Cognitive Level: Application Text Reference: pp. 755, 759 Nursing Process: Implementation

Physiological Integrity 27. The nurse is taking a health history from a 24-year-old patient with hypertrophic cardiomyopathy (HC); which information obtained by the nurse is most relevant? a. The patient reports using cocaine once at age 16. b. The patient has a history of a recent upper respiratory infection. c. The patient's 29-year-old brother has had a sudden cardiac arrest. d. The patient has a family history of coronary artery disease (CAD).

C Rationale: About half of all cases of HC have a genetic basis, and HC is the most common cause of sudden cardiac death in otherwise healthy young people; the information about the patient's brother will be helpful in planning care (such as an automatic implantable cardioverter-defibrillator [AICD]) for the patient and in counseling other family members. The patient should be counseled against use of stimulant drugs, but the one-time use indicates that the patient is not at current risk for cocaine use. Viral infections and CAD are risk factors for dilated cardiomyopathy, but not for HC. Cognitive Level: Application Text Reference: pp. 886, 888 Nursing Process: Assessment

Physiological Integrity 5. The nurse identifies a nursing diagnosis of risk for altered peripheral tissue perfusion related to bypass graft thrombosis for a patient following an abdominal aneurysm repair. An appropriate intervention to prevent this problem in the immediate postoperative period is to a. use a cooling blanket to maintain the patient's temperature within a normal range to prevent hypercoagulability. b. place the patient in Trendelenburg position to reduce pressure at the suture line and prevent leaking of blood at the site. c. administer IV fluids at a rate to keep the arterial BP within a normal range. d. perform passive range-of-motion (ROM) exercises to the legs hourly to promote venous return.

C Rationale: Administration of IV fluids to maintain BP within normal range will allow adequate blood flow to prevent thrombosis while preventing the risk of suture damage that can occur with high BP. Coagulation is not usually affected by temperature. The Trendelenburg position compromises respiratory status in postoperative patients. Passive ROM will not improve arterial blood flow through the graft. Cognitive Level: Application Text Reference: p. 897 Nursing Process: Planning

Physiological Integrity 24. Which of these nursing actions included in the care of a mechanically ventilated patient with acute respiratory distress syndrome (ARDS) is most appropriate for the RN to delegate to an experienced LPN/LVN working in the intensive care unit? a. Placing the patient in the prone position b. Assessment of patient breath sounds c. Administration of enteral tube feedings d. Obtaining the pulmonary artery pressures

C Rationale: Administration of tube feedings is included in LPN/LVN education and scope of practice and can be safely delegated to an LPN/LVN who is experienced in caring for critically ill patients. Placing a patient who is on a ventilator in the prone position requires multiple staff and should be supervised by an RN. Assessment of breath sounds and obtaining pulmonary artery pressures require advanced assessment skills and should be done by the RN caring for a critically ill patient. Cognitive Level: Application Text Reference: pp. 1816-1818 Nursing Process: Implementation

Physiological Integrity 37. The nurse has just received change-of-shift report about these four patients. Which patient should the nurse assess first? a. A 38-year-old who has pericarditis and is complaining of sharp, stabbing chest pain b. A 45-year-old who had an MI 4 days ago and is anxious about the planned discharge c. A 51-year-old who has just returned to the unit after a coronary arteriogram and PCI d. A 60-year-old who has a scheduled dose of atenolol (Tenormin) 25 mg PO due

C Rationale: After PCI, the patient is at risk for bleeding from the arterial access site for the PCI, so the nurse should assess the patient immediately. The other patients also should be assessed as quickly as possible, but assessment of this patient has the highest priority. Cognitive Level: Application Text Reference: p. 807 Nursing Process: Assessment

Health Promotion and Maintenance 23. In planning care for a patient with a venous stasis ulcer on the right lower leg, the nurse understands that the most important intervention in promoting healing of the ulcer is a. adequate dietary intake of proteins and vitamins. b. prevention of infection with prophylactic antibiotics. c. application of external compression to the lower leg. d. keeping the ulcer moist with hydrocolloid dressings.

C Rationale: Although all the interventions are used, the most essential is compression of the leg to prevent the ulcer from becoming wider and deeper. Cognitive Level: Application Text Reference: p. 919 Nursing Process: Planning

Physiological Integrity 9. Which of these statements made by a patient after the nurse has completed teaching about the TLC diet indicates that further teaching is needed? a. "I will switch from whole milk to 1% or nonfat milk." b. "I like fresh salmon and I will plan to eat it more often." c. "I will miss being able to eat peanut butter sandwiches." d. "I can have a cup of coffee with breakfast if I want one."

C Rationale: Although only 30% of the daily calories should come from fats, most of the fat in the TLC diet should come from monosaturated fats such as are found in nuts, olive oil, and canola oil. The patient can include peanut butter sandwiches as part of the TLC diet. The other patient comments indicate a good understanding of the TLC diet. Cognitive Level: Application Text Reference: p. 793 Nursing Process: Evaluation

Physiological Integrity 40. A patient who is experiencing an acute asthma attack is admitted to the emergency department. The nurse's first action should be to a. determine when the dyspnea started. b. obtain the forced expiratory flow rate. c. listen to the patient's breath sounds. d. ask about inhaled corticosteroid use.

C Rationale: Assessment of the patient's breath sounds will help to determine how effectively the patient is ventilating and whether rapid intubation may be necessary. The length of time the attack has persisted is not as important as determining the patient's status at present. Most patients having an acute attack will be unable to cooperate with a FEV measurement. It is important to know about the medications the patient is using but not as important as assessing the breath sounds. Cognitive Level: Application Text Reference: p. 626 Nursing Process: Assessment

Physiological Integrity 25. Which of these nursing actions in the care plan for a patient who had a repair of an abdominal aortic aneurysm 5 days previously is most appropriate for the nurse to delegate to an experienced nursing assistant? a. Teach the patient the signs of possible wound infection. b. Monitor the quality and presence of the pedal pulses. c. Assist the patient in using a pillow to splint while coughing. d. Check the lower extremity strength and movement.

C Rationale: Assisting a patient who has already been taught how to cough is part of routine postoperative care and within the education and scope of practice for an experienced nursing assistant. Patient teaching and assessment of essential postoperative functions such as circulation and movement are more appropriate for the RN-level education and scope of practice. Cognitive Level: Application Text Reference: pp. 897-898 Nursing Process: Planning

Analysis 20. An elderly patient with a 40-pack-year history of smoking and a recent myocardial infarction is admitted to the medical unit with acute shortness of breath; the nurse need to rule out pneumonia versus heart failure. The diagnostic test that the nurse will monitor to help in determining whether the patient has heart failure is a. 12-lead electrocardiogram (ECG). b. arterial blood gases (ABGs). c. B-type natriuretic peptide (BNP). d. serum creatine kinase (CK).

C Rationale: BNP is secreted when ventricular pressures increase, as with heart failure, and elevated BNP indicates a probable or very probable diagnosis of heart failure. 12-lead ECGs, ABGs, and CK may also be used in determining the causes or effects of heart failure but are not as clearly diagnostic of heart failure as BNP. Cognitive Level: Application Text Reference: p. 827 Nursing Process: Assessment

Physiological Integrity 11. A patient with a chronic cough with blood-tinged sputum undergoes a bronchoscopy. Following the bronchoscopy, the nurse should a. check vital signs every 15 minutes for 2 hours. b. place the patient on bed rest for at least 4 hours. c. keep the patient NPO until the gag reflex returns. d. elevate the head of the bed to 80 to 90 degrees.

C Rationale: Because a local anesthetic is used to suppress the gag/cough reflexes during bronchoscopy, the nurse should monitor for the return of these reflexes before allowing the patient to take oral fluids or food. Vital signs are monitored immediately after the procedure but should not need to be obtained every 15 minutes for 2 hours. The patient does not need to be on bed rest, and the head of the bed does not need to be in the high-Fowler's position. Cognitive Level: Application Text Reference: p. 528 Nursing Process: Implementation

Physiological Integrity 17. While admitting a patient with an AMI, which action should the nurse carry out first? a. Assess peripheral pulses. b. Check the oxygen saturation. c. Attach the cardiac monitor. d. Obtain the BP.

C Rationale: Because dysrhythmias are the most common complication of MI, the first action should be to place the patient on a cardiac monitor. The other actions are also important and should be accomplished as quickly as possible. Cognitive Level: Application Text Reference: pp. 799, 806 Nursing Process: Implementation

Physiological Integrity 24. A patient is scheduled for a spiral CT scan to rule out a pulmonary embolus. Which information obtained by the nurse is most important to communicate to the health care provider before the examination? a. The apical pulse is irregular. b. The oxygen saturation is 93%. c. The patient is allergic to shellfish. d. The patient is very tachypneic.

C Rationale: Because the contrast solution is iodine-based, the patient may need to have the CT scan without contrast or be premedicated before contrast injection. The irregular pulse, oxygen saturation, and tachypnea all need further assessment or intervention but are not unusual for a patient with a possible pulmonary embolus. Cognitive Level: Application Text Reference: p. 527 Nursing Process: Implementation

Physiological Integrity 24. A patient is admitted to the hospital with a diagnosis of chronic venous insufficiency. Which of these statements by the patient is most consistent with the diagnosis? a. "I have burning leg pains after I walk three blocks." b. "I wake up during the night because my legs hurt." c. "I can't get my shoes on at the end of the day." d. "I can never seem to get my feet warm enough."

C Rationale: Because the edema associated with venous insufficiency increases when the patient has been standing, shoes will feel tighter at the end of the day. The other patient statements are characteristic of PAD. Cognitive Level: Application Text Reference: p. 920 Nursing Process: Implementation

Physiological Integrity 13. The nurse identifies the nursing diagnosis of ineffective peripheral perfusion related to decreased arterial blood flow for a patient with chronic PAD. In evaluating the patient outcomes following patient teaching, the nurse determines a need for further instruction when the patient says, a. "I will have to buy some loose clothing that does not bind across my legs or waist." b. "I will change my position every hour and avoid long periods of sitting with my legs down." c. "I will use a heating pad on my feet at night to increase the circulation and warmth in my feet." d. "I will walk to the point of pain, rest, and walk again until I develop pain for a half hour daily."

C Rationale: Because the patient has impaired circulation and sensation to the feet, the use of a heating pad could lead to burns. The other patient statements are correct and indicate that teaching has been successful. Cognitive Level: Application Text Reference: p. 905 Nursing Process: Evaluation

Physiological Integrity 14. When performing an assessment of a newly admitted patient, the nurse notes a thrill along the left sternal border. To obtain more information about the cause of the thrill, which action will the nurse take next? a. Palpate the quality of the peripheral pulses. b. Compare the apical and radial pulse rates. c. Assess for murmurs. d. Locate the PMI.

C Rationale: Both thrills and murmurs are caused by turbulent blood flow, such as occurs when blood flows through a damaged valve. Relevant information includes the quality of the murmur, where in the cardiac cycle the murmur is heard, and where on the thorax the murmur is heard best. The other information is also important in the cardiac assessment but will not provide information that is relevant to the thrill. Cognitive Level: Application Text Reference: pp. 748-749, 751 Nursing Process: Assessment

Physiological Integrity 5. When preparing a patient with possible asthma for pulmonary function testing, the nurse will teach the patient to a. avoid eating or drinking for 4 hours before the forced expiratory volume in 1 second (FEV1)/forced expiratory volume (FEV) test. b. take oral corticosteroids at least 2 hours before the examination. c. withhold bronchodilators for 6 to 12 hours before the examination. d. use rescue medications immediately before the FEV1/FEV testing.

C Rationale: Bronchodilators are held before pulmonary function testing so that a baseline assessment of airway function can be determined. Testing is repeated after bronchodilator use to determine whether the decrease in lung function is reversible. There is no need for the patient to be NPO. Oral corticosteroids should also be held before the examination and corticosteroids given 2 hours before the examination would be at a high level. Rescue medications (which are bronchodilators) would not be given until after the baseline pulmonary function was assessed. Cognitive Level: Application Text Reference: p. 614 Nursing Process: Planning

Physiological Integrity 12. The nurse performing an assessment with a patient who has chronic peripheral arterial disease (PAD) of the legs would expect to find a. swollen, dry, scaly ankles. b. a positive Homans' sign. c. prolonged capillary refill. d. a draining ulcer on the heel.

C Rationale: Capillary refill is prolonged in PAD because of the slower and decreased blood flow to the periphery. The other listed clinical manifestations are consistent with chronic venous disease. Cognitive Level: Application Text Reference: p. 901 Nursing Process: Assessment

Physiological Integrity 5. Which information given by a patient admitted with chronic stable angina will help the nurse confirm this diagnosis? a. The patient rates the pain at a level 3 to 5 (0-10 scale). b. The patient states that the pain "wakes me up at night." c. The patient indicates that the pain is resolved after taking one sublingual nitroglycerin tablet. d. The patient says that the frequency of the pain has increased over the last few weeks.

C Rationale: Chronic stable angina is typically relieved by rest or nitroglycerin administration. The level of pain is not a consistent indicator of the type of angina. Pain occurring at rest or with increased frequency is typical of unstable angina. Cognitive Level: Application Text Reference: p. 798 Nursing Process: Assessment

Physiological Integrity 28. A patient with COPD tells the nurse, "At home, I only have to use an albuterol (Proventil) inhaler. Why did the doctor add an ipratropium (Atrovent) inhaler while I'm in the hospital? The appropriate response by the nurse is a. "Atrovent will dilate the airways and allow the Proventil to penetrate more deeply." b. "Atrovent is being used to decrease airway inflammation and sputum production." c. "Atrovent works differently to dilate the bronchi, and the two drugs together are more effective." d. "Atrovent is a potent bronchodilator and patients need to be hospitalized when receiving it."

C Rationale: Combining bronchodilators improves effectiveness. Atrovent does not have to be used before Proventil, it does not decrease airway inflammation, and it does not require hospitalization. Cognitive Level: Application Text Reference: p. 640 Nursing Process: Implementation

Physiological Integrity 33. A patient who has a right-sided chest tube following a thoracotomy has continuous bubbling in the suction-control chamber of the collection device. The most appropriate action by the nurse is to a. document the presence of a large air leak. b. obtain and attach a new collection device. c. notify the health care provider of a possible pneumothorax. d. take no further action with the collection device.

C Rationale: Continuous bubbling is expected in the suction-control chamber and indicates that the suction-control chamber is connected to suction. An air leak would be detected in the water-seal chamber. There is no evidence of pneumothorax. A new collection device is needed when the collection chamber is filled. Cognitive Level: Application Text Reference: p. 591 Nursing Process: Implementation

Physiological Integrity 20. When reading the chart for a patient with COPD, the nurse notes that the patient has cor pulmonale. To assess for cor pulmonale, the nurse will monitor the patient for a. elevated temperature. b. complaints of chest pain. c. jugular vein distension. d. clubbing of the fingers.

C Rationale: Cor pulmonale causes clinical manifestations of right ventricular failure, such as jugular vein distension. The other clinical manifestations may occur in the patient with other complications of COPD but are not indicators of cor pulmonale. Cognitive Level: Application Text Reference: pp. 635-636 Nursing Process: Assessment

Physiological Integrity 19. When assessing the respiratory system of a 78-year-old patient, which of these data indicate that the nurse should take immediate action? a. The chest appears barrel shaped. b. The patient has a weak cough effort. c. Crackles are audible in the lower two thirds of the posterior chest. d. Hyperresonance is present across both sides of the chest.

C Rationale: Crackles in the lower two thirds of the lungs indicate that the patient may have an acute problem such as heart failure. The nurse should immediately accomplish further assessments, such as oxygen saturation, and notify the health care provider. A barrel-shaped chest, hyperresonance to percussion, and a weak cough effort are associated with aging. Further evaluation may be needed, but immediate action is not indicated. Cognitive Level: Application Text Reference: p. 524 Nursing Process: Assessment

1. A patient with a history of asthma is admitted to the hospital in acute respiratory distress. During assessment of the patient, the nurse would notify the health care provider immediately about a. a pulse oximetry reading of 90%. b. a peak expiratory flow rate of 240 ml/min. c. decreased breath sounds and wheezing. d. a respiratory rate of 26 breaths/min.

C Rationale: Decreased breath sounds and wheezing would indicate that the patient was experiencing an asthma attack, and immediate bronchodilator treatment would be indicated. The other data indicate that the patient needs ongoing monitoring and assessment but do not indicate a need for immediate treatment. Cognitive Level: Application Text Reference: pp. 608, 612, 614 Nursing Process: Assessment

Physiological Integrity 4. The nurse identifies the nursing diagnosis of decreased cardiac output related to valvular insufficiency for the patient with IE based on the assessment finding of a. petechiae of the buccal mucosa and conjunctiva. b. fever, chills, and diaphoresis. c. urine output less than 30 ml/hr. d. an increase in pulse rate of 15 beats/min with activity.

C Rationale: Decreased renal perfusion caused by inadequate cardiac output will lead to poor urine output. Petechiae, fever, chills, and diaphoresis are symptoms of IE but are not caused by decreased cardiac output. An increase in pulse rate of 15 beats/min is normal with exercise Cognitive Level: Application Text Reference: p. 870 Nursing Process: Diagnosis

Physiological Integrity Lewis: Medical-Surgical Nursing, 7th Edition Test Bank Chapter 37: Nursing Management: Inflammatory and Structural Heart Disorders MULTIPLE CHOICE 1. The nurse obtains a health history from a patient with a prosthetic mitral valve who has symptoms of infective endocarditis. Which question by the nurse is most appropriate? a. "Do you have a history of a heart attack?" b. "Have you any recent immunizations?" c. "Have you been to the dentist lately?" d. "Is there a family history of endocarditis?"

C Rationale: Dental procedures place the patient with a prosthetic mitral valve at risk for infectious endocarditis (IE). Myocardial infarction (MI), immunizations, and a family history of endocarditis are not risk factors for IE. Cognitive Level: Application Text Reference: pp. 867, 869 Nursing Process: Assessment

Physiological Integrity 17. An alcoholic and homeless patient is diagnosed with active TB. Which intervention by the nurse will be most effective in ensuring adherence with the treatment regimen? a. Giving the patient written instructions about how to take the medications b. Teaching the patient about the high risk for infecting others unless treatment is followed c. Arranging for a daily noontime meal at a community center and give the medication then d. Educating the patient about the long-term impact of TB on health

C Rationale: Directly observed therapy is the most effective means for ensuring compliance with the treatment regimen, and arranging a daily meal will help to ensure that the patient is available to receive the medication. The other nursing interventions may be appropriate for some patients, but are not likely to be as helpful with this patient. Cognitive Level: Application Text Reference: pp. 572, 575 Nursing Process: Implementation

Physiological Integrity 46. The nurse has received change-of-shift report about these four patients. Which one will the nurse plan to assess first? a. A 23-year-year-old patient with cystic fibrosis who has pulmonary function testing scheduled in 30 minutes b. A 35-year-old patient who was admitted the previous day with bacterial pneumonia and has a temperature of 100.2° F c. A 46-year-old patient who is complaining of dyspnea after having a thoracentesis an hour previously d. A 77-year-old patient with TB who has four antitubercular medications due in 15 minutes

C Rationale: Dyspnea after a thoracentesis may indicate a pneumothorax or hemothorax and requires immediate evaluation by the nurse. The other patients should also be assessed as soon as possible, but there is no indication that they may need immediate action to prevent clinical deterioration. Cognitive Level: Application Text Reference: p. 596 Nursing Process: Planning

Physiological Integrity 25. When caring for the patient with infective endocarditis of the tricuspid valve, the nurse will plan to monitor the patient for a. flank pain. b. hemiparesis. c. dyspnea. d. splenomegaly.

C Rationale: Embolization from the tricuspid valve would cause symptoms of pulmonary embolus. Flank pain, hemiparesis, and splenomegaly would be associated with embolization from the left-sided valves. Cognitive Level: Application Text Reference: p. 867 Nursing Process: Planning

Physiological Integrity 6. To auscultate for S3 or S4 gallops in the mitral area, the nurse listens with the a. diaphragm of the stethoscope with the patient in a reclining position. b. diaphragm of the stethoscope with the patient lying flat on the left side. c. bell of the stethoscope with the patient in the left lateral position. d. bell of the stethoscope with the patient sitting and leaning forward.

C Rationale: Gallop rhythms generate low-pitched sounds and are most easily heard with the bell of the stethoscope. Sounds associated with the mitral valve are accentuated by turning the patient to the left side, which brings the heart closer to the chest wall. The diaphragm of the stethoscope is best to use for the higher-pitched sounds such as S1 and S2. Cognitive Level: Application Text Reference: pp. 750-751 Nursing Process: Assessment

Physiological Integrity 17. All the following medications are ordered for a mechanically ventilated patient with acute respiratory distress syndrome (ARDS) and acute renal failure. Which medication should the nurse discuss with the health care provider before administration? a. IV ranitidine (Zantac) 50 mg IV b. sucralfate (Carafate) 1 g per nasogastric tube c. IV gentamicin (Garamycin) 60 mg d. IV methylprednisolone (Solu-Medrol) 40 mg

C Rationale: Gentamicin, which is one of the aminoglycoside antibiotics, is potentially nephrotoxic, and the nurse should clarify the drug and dosage with the health care provider before administration. The other medications are appropriate for the patient with ARDS. Cognitive Level: Application Text Reference: p. 1816 Nursing Process: Implementation

Physiological Integrity 16. A patient with a non-ST segment elevation myocardial infarction (NSTEMI) is receiving heparin. What is the purpose of the heparin? a. Heparin will dissolve the clot that is blocking blood flow to the heart. b. Coronary artery plaque size and adherence are decreased with heparin. c. Heparin will prevent the development of clots in the coronary arteries. d. Platelet aggregation is enhanced by IV heparin infusion.

C Rationale: Heparin helps prevent the conversion of fibrinogen to fibrin and decreases coronary artery thrombosis. It does not change coronary artery plaque, dissolve already formed clots, or enhance platelet aggregation. Cognitive Level: Comprehension Text Reference: p. 800 Nursing Process: Implementation

Health Promotion and Maintenance 16. A home health care patient has recently started taking oral digoxin (Lanoxin) and furosemide (Lasix) for control of heart failure. The patient data that will require the most immediate action by the nurse is if the patient's a. weight increases from 120 pounds to 122 pounds over 3 days. b. liver is palpable 2 cm below the ribs on the right side. c. serum potassium level is 3.0 mEq/L after 1 week of therapy. d. has 1 to 2+ edema in the feet and ankles in the morning.

C Rationale: Hypokalemia potentiates the actions of digoxin and increases the risk for digoxin toxicity, which can cause life-threatening dysrhythmias. The other data indicate that the patient's heart failure requires more effective therapies, but they do not require nursing action as rapidly as the low serum potassium level. Cognitive Level: Application Text Reference: pp. 832-833 Nursing Process: Assessment

1. It will be most important for the nurse to check pulse oximetry for which of these patients? a. A patient with emphysema and a respiratory rate of 16 b. A patient with massive obesity who is refusing to get out of bed c. A patient with pneumonia who has just been admitted to the unit d. A patient who has just received morphine sulfate for postoperative pain

C Rationale: Hypoxemia and hypoxemic respiratory failure are caused by disorders that interfere with the transfer of oxygen into the blood, such as pneumonia. The other listed disorders are more likely to cause problems with hypercapnia because of ventilatory failure. Cognitive Level: Application Text Reference: pp. 1799-1800 Nursing Process: Assessment

Physiological Integrity 20. A patient with a DVT is started on IV heparin and oral warfarin (Coumadin). The patient asks the nurse why two medications are necessary. The nurse's best response to the patient is, a. "Heparin will start to dissolve the clot, and Coumadin will prevent any more clots from occurring." b. "Because of the potential for a pulmonary embolism, it is important for you to have more than one anticoagulant." c. "The heparin will work immediately, but the Coumadin takes several days to have an effect on coagulation." d. "Administration of two anticoagulants reduces the risk for recurrent deep vein thrombosis."

C Rationale: IV heparin is used because of the immediate effect on coagulation and discontinued once the international normalized ratio (INR) value indicates that the warfarin has reached a therapeutic level. Heparin is not a thrombolytic drug. The use of two anticoagulants is not related to the risk for pulmonary embolism, and two are not necessary to reduce the risk for another DVT. Cognitive Level: Application Text Reference: pp. 912, 913 Nursing Process: Implementation

Physiological Integrity 3. A patient has a junctional escape rhythm on the monitor. The nurse would expect the patient to have a pulse rate of ____ beats/min. a. 15-20 b. 20-40 c. 40-60 d. 60-100

C Rationale: If the SA node fails to discharge, the junction will automatically discharge at the normal junctional rate of 40 to 60. The slower rates are typical of the bundle of His and the Purkinje system and may be seen with failure of both the SA and AV node to discharge. The normal SA node rate is 60 to 100 beats/min. Cognitive Level: Comprehension Text Reference: pp. 846-847, 852 Nursing Process: Assessment

Physiological Integrity 15. During change-of-shift report, the nurse learns that a patient with a large myocardial infarction has been having frequent PVCs. When monitoring the patient for the effects of PVCs, the nurse will check the patient's a. medications. b. recent electrolyte values. c. apical radial heart rate. d. oxygen saturation.

C Rationale: It is important to assess the patient's apical-radial pulse rate because PVCs often do not generate a sufficient ventricular contraction to result in a peripheral pulse, which can lead to a pulse deficit. Electrolyte imbalances, hypoxia, and certain medications may precipitate PVCs. Cognitive Level: Comprehension Text Reference: p. 854 Nursing Process: Assessment

Physiological Integrity 44. Which assessment information obtained by the nurse when caring for a patient who has just had a thoracentesis is most important to communicate to the health care provider? a. BP is 150/90. b. Pain level is 5/10 with a deep breath. c. Oxygen saturation is 89%. d. Respiratory rate is 24 when lying flat.

C Rationale: Oxygen saturation would be expected to improve after a thoracentesis; a saturation of 89 indicates that a complication such as pneumothorax may be occurring. The other assessment data also indicate a need for ongoing assessment or intervention, but the low oxygen saturation is the priority. Cognitive Level: Application Text Reference: p. 596 Nursing Process: Assessment

Physiological Integrity 13. A patient with chronic stable angina is being treated with metoprolol (Lopressor). The nurse will suspect that the patient is experiencing a side effect of the metoprolol if a. the patient is restless and agitated. b. the BP is 190/110 mm Hg. c. the cardiac monitor shows a heart rate of 45. d. the patient complains about feeling anxious.

C Rationale: Patients taking -blockers should be monitored for bradycardia. Because this category of medication inhibits the sympathetic nervous system, restless, agitation, hypertension, and anxiety will not be side effects. Cognitive Level: Application Text Reference: p. 801 Nursing Process: Evaluation

Physiological Integrity 21. The nurse is performing TB screening in a clinic that has many patients who have immigrated to the United States. Before doing a TB skin test on a patient, which question is most important for the nurse to ask? a. "How long have you lived in the United States?" b. "Is there any family history of TB?" c. "Have you received the BCG vaccine for TB?" d. "Do you take any over-the-counter (OTC) medications?"

C Rationale: Patients who have received the BCG vaccine will have a positive Mantoux test; another method for screening (such as a chest x-ray) will be used in determining whether the patient has a TB infection. The other information may also be valuable but is not as pertinent to the decision about doing TB skin testing. Cognitive Level: Application Text Reference: p. 572 Nursing Process: Assessment

Physiological Integrity 3. When a patient is diagnosed with pulmonary fibrosis, the nurse will teach the patient about the risk for poor oxygenation because of a. too-rapid movement of blood flow through the pulmonary blood vessels. b. incomplete filling of the alveoli with air because of reduced respiratory ability. c. decreased transfer of oxygen into the blood because of thickening of the alveoli. d. mismatch between lung ventilation and blood flow through the blood vessels of the lung.

C Rationale: Pulmonary fibrosis causes the alveolar-capillary interface to become thicker, which increases the amount of time it takes for gas to diffuse across the membrane. Too-rapid pulmonary blood flow is another cause of shunt but does not describe the pathology of pulmonary fibrosis. Decrease in alveolar ventilation will cause hypercapnia. Ventilation and perfusion are matched in pulmonary fibrosis; the problem is with diffusion. Cognitive Level: Application Text Reference: p. 1802 Nursing Process: Implementation

1. While assessing a patient who has just arrived in the emergency department, the nurse notes a pulse deficit. The nurse will anticipate that the patient may require a. hourly blood pressure (BP) checks. b. a coronary arteriogram. c. electrocardiographic (ECG) monitoring. d. a 2-D echocardiogram.

C Rationale: Pulse deficit is a difference between simultaneously obtained apical and radial pulses and indicates that there may be cardiac dysrhythmias that would be detected with ECG monitoring. Frequent BP monitoring, coronary arteriograms, and echocardiograms are used for diagnosis of other cardiovascular disorders but would not be as helpful in determining the reason for the pulse deficit. Cognitive Level: Application Text Reference: pp. 750-751 Nursing Process: Assessment

Physiological Integrity 10. Cardiac tamponade is suspected in a patient who has acute pericarditis. To assess for the presence of pulsus paradoxus, the nurse should a. subtract the diastolic blood pressure (DBP) from the systolic blood pressure (SBP). b. auscultate for a pericardial friction rub that increases in volume during inspiration. c. note when Korotkoff sounds are audible during both inspiration and expiration. d. check the ECG for variations in rate in relation to inspiration and expiration.

C Rationale: Pulsus paradoxus exists when there is a gap of greater than 10 mm Hg between when Korotkoff sounds can be heard during only expiration and when they can be heard throughout the respiratory cycle. The other methods described would not be useful in determining the presence of pulsus paradoxus. Cognitive Level: Comprehension Text Reference: p. 873 Nursing Process: Assessment

Physiological Integrity 18. The nurse teaches a patient with COPD how to perform pursed-lip breathing, explaining that this technique will assist respiration by a. loosening secretions so that they may be coughed up more easily. b. promoting maximal inhalation for better oxygenation of the lungs. c. preventing airway collapse and air trapping in the lungs during expiration. d. decreasing anxiety by giving the patient control of respiratory patterns.

C Rationale: Pursed-lip breathing increases the airway pressure during the expiratory phase and prevents collapse of the airways, allowing for more complete exhalation. Although loosening of secretions, improving inhalation, and decreasing anxiety are desirable outcomes for the patient with COPD, pursed-lip breathing does not directly impact these. Cognitive Level: Comprehension Text Reference: p. 646 Nursing Process: Implementation

Physiological Integrity 3. An asthmatic patient who has a new prescription for Advair Diskus (combined fluticasone and salmeterol) asks the nurse the purpose of using two drugs. The nurse explains that a. Advair is a combination of long-acting and slow-acting bronchodilators. b. the two drugs work together to block the effects of histamine on the bronchioles. c. one drug decreases inflammation, and the other is a bronchodilator. d. the combination of two drugs works more quickly in an acute asthma attack.

C Rationale: Salmeterol is a long-acting bronchodilator, and fluticasone is a corticosteroid; they work together to prevent asthma attacks. Neither medication is an antihistamine. Advair is not used during an acute attack because the medications do not work rapidly. Cognitive Level: Application Text Reference: pp. 621 Nursing Process: Implementation

Physiological Integrity 11. The nurse has identified a nursing diagnosis of acute pain related to inflammatory process for a patient with acute pericarditis. The most appropriate intervention by the nurse for this problem is to a. force fluids to 3000 ml/day to decrease fever and inflammation. b. teach the patient to take deep, slow respirations to control the pain. c. position the patient in Fowler's position, leaning forward on the overbed table. d. remind the patient to ask for the opioid pain medication every four hours.

C Rationale: Sitting upright and leaning forward frequently will decrease the pain associated with pericarditis. Forcing fluids will not decrease the inflammation or pain. Taking deep respirations tends to increase pericardial pain. Opioids are not very effective at controlling pain caused by acute inflammatory conditions and are usually ordered PRN. The patient would receive scheduled doses of a nonsteroidal antiinflammatory drug (NSAID). Cognitive Level: Application Text Reference: p. 874 Nursing Process: Planning

Physiological Integrity 30. When developing a teaching plan for a patient newly diagnosed with PAD, which information should the nurse include? a. "Try to keep your legs elevated whenever you are sitting." b. "Exercise only if you do not experience pain." c. "It is important to try to stop smoking." d. "Put on support hose early in the day before swelling occurs."

C Rationale: Smoking cessation is essential for slowing the progression of PAD to critical limb ischemia and reducing the risk of myocardial infarction and death. Circulation to the legs will decrease if the legs are elevated. Patients with PAD are taught to exercise to feeling pain, rest, and then resume walking. Support hose are not used for patients with PAD. Cognitive Level: Application Text Reference: p. 901 Nursing Process: Planning

Physiological Integrity 12. A hospitalized patient who may have tuberculosis (TB) has an order for a sputum specimen. When will be the best time for the nurse to collect the specimen? a. After the patient rinses the mouth with mouthwash b. As soon as the order is received from the health care provider c. Right after the patient gets up in the morning d. After the skin test is administered

C Rationale: Sputum specimens are ideally collected in the morning because mucus is likely to accumulate during the night. The patient should rinse the mouth with water; mouthwash may inhibit the growth of the bacilli. There is no need to wait until the tuberculin skin test is administered. Cognitive Level: Application Text Reference: p. 572 Nursing Process: Implementation

Physiological Integrity 5. The nurse will anticipate discharge today for which of these patients with community-acquired-pneumonia? a. 24-year-old patient who has had temperatures ranging from 100.6° to 101° F b. 35-year-old patient who has had 600 ml of oral fluids in the last 24 hours c. 50-year-old patient who has an oxygen saturation of 91% on room air d. 72-year-old patient with a pulse of 102 and a blood pressure (BP) of 90/56

C Rationale: The 50-year-old meets the Infectious Diseases Society of America (IDSA) hospital discharge criteria. The other patients do not meet the criteria for discharge. Cognitive Level: Application Text Reference: p. 563 Nursing Process: Planning

Physiological Integrity 8. In developing a teaching plan for a patient who has stable angina and is started on sublingual nitroglycerin (Nitrostat), the nurse identifies an expected patient outcome of a. stating that nitroglycerin is to be taken only if chest pain develops. b. listing the side effects of nitroglycerin as gastric upset and dry mouth. c. identifying the need to call the emergency medical services (EMS) if chest pain persists 5 minutes after taking nitroglycerin. d. recognizes that taking the nitroglycerin is important to decrease the ongoing atherosclerosis of the coronary arteries.

C Rationale: The EMS system should be activated when chest pain or other symptoms are not completely relieved 5 minutes after taking nitroglycerin. Nitroglycerin can be taken to prevent chest pain or other symptoms from developing. Gastric upset and dry mouth are not expected side effects of nitroglycerin. Nitroglycerin does not impact the underlying pathophysiology of coronary artery atherosclerosis. Cognitive Level: Application Text Reference: p. 798 Nursing Process: Planning

Physiological Integrity 6. The nurse obtains a monitor strip on a patient admitted to the coronary care unit with a myocardial infarction and makes the following analysis: P wave not apparent; ventricular rate 162, R-R interval regular; PR interval not measurable; and QRS complex wide and distorted, QRS duration 0.18 second. The nurse interprets the patient's cardiac rhythm as a. sinus tachycardia. b. atrial fibrillation. c. ventricular tachycardia. d. ventricular fibrillation.

C Rationale: The absence of P waves, wide QRS, rate >150, and the regularity of the rhythm indicate ventricular tachycardia. Atrial fibrillation is grossly irregular, has a narrow QRS configuration, and has fibrillatory P waves. Sinus tachycardia has P waves. Ventricular fibrillation is irregular and does not have a consistent QRS duration. Cognitive Level: Application Text Reference: pp. 854-855 Nursing Process: Assessment

Physiological Integrity 6. The nurse identifies the nursing diagnosis of activity intolerance for a patient with asthma. A common etiologic factor for this nursing diagnosis in patients with asthma is a. anxiety about dyspnea. b. side effects of medications. c. work of breathing. d. fear of suffocation.

C Rationale: The activity intolerance patients with asthma experience is related to the increased effort needed to breathe when airways are inflamed and narrowed and interventions are focused on decreasing inflammation and bronchoconstriction. The other listed etiologies are not as appropriate for this diagnosis but would be appropriate for diagnoses seen in patients with asthma, such as social isolation, knowledge deficit, and anxiety. Cognitive Level: Application Text Reference: pp. 612, 624 Nursing Process: Diagnosis

Psychosocial Integrity 35. A patient with hyperlipidemia has a new order for the bile-acid sequestrant medication colesevelam (Welchol). Which nursing action is appropriate when giving the medication? a. Have the patient take this medication with an aspirin. b. Encourage the patient to take it with a sip of water. c. Give the patient's other medications 2 hours after the Welchol. d. Administer the drug at the patient's bedtime.

C Rationale: The bile-acid sequestrants interfere with the absorption of other drugs, and giving other medications at the same time should be avoided. Taking an aspirin concurrently with the Welchol may increase the incidence of gastrointestinal side effects such as heartburn. An increased fluid intake is encouraged for patients taking the bile-acid sequestrants to reduce the risk for constipation. For maximum effect, Welchol should be administered with meals. Cognitive Level: Application Text Reference: pp. 794-795 Nursing Process: Implementation

Physiological Integrity 21. During the administration of the fibrinolytic agent to a patient with an AMI, the nurse should stop the drug infusion if the patient experiences a. bleeding from the gums. b. surface bleeding from the IV site. c. a decrease in level of consciousness. d. a nonsustained episode of ventricular tachycardia.

C Rationale: The change in level of consciousness indicates that the patient may be experiencing intracranial bleeding, a possible complication of fibrinolytic therapy. Bleeding of the gums and prolonged bleeding from IV sites are expected side effects of the therapy. The nurse should address these by avoiding any further injuries, but they are not an indication to stop infusion of the fibrinolytic medication. A nonsustained episode of ventricular tachycardia is a common reperfusion dysrhythmia and may indicate that the therapy is effective. Cognitive Level: Application Text Reference: p. 808 Nursing Process: Evaluation

Physiological Integrity 16. When admitting a patient for a coronary arteriogram and angiogram, the assessment information that will be most important for the nurse to communicate to the health care provider is that the a. patient had an arteriogram a year ago. b. patient has not eaten anything yet today. c. patient is allergic to shellfish. d. patient's pedal pulses are +1.

C Rationale: The contrast dye used for the procedure is iodine based, so patients who have shellfish allergies will require treatment with medications such as corticosteroids and antihistamines before the arteriogram. The other information is also communicated to the health care provider but will not require a change in the usual pre-arteriogram orders or medications. Cognitive Level: Application Text Reference: p. 755 Nursing Process: Assessment

Hirudin derivatives

Match the following anticoagulant drugs with their characteristics. Can choose unfractionated heparin, LMWH, Hirudin derivatives, Warfarin (Coumadin), Thrombin inhibitor (argatroban (Acova)) b) No antidote for anticoagulant effect

Physiological Integrity 14. Which of these is the best goal for the patient admitted with chronic bronchitis who has a nursing diagnosis of ineffective airway clearance? a. Patient denies having dyspnea. b. Patient's mental status is improved. c. Patient has a productive cough. d. Patient's O2 saturation is 90%.

C Rationale: The goal for the nursing diagnosis of ineffective airway clearance is to maintain a clear airway by coughing effectively. The other goals may be appropriate for the patient with COPD, but they do not address the problem of ineffective airway clearance. Cognitive Level: Application Text Reference: p. 660 Nursing Process: Evaluation

Health Promotion and Maintenance 7. The nurse is obtaining a health history for a new patient with possible coronary artery disease. Which question would the nurse use when obtaining subjective data related to the patient's health perception-health management functional health pattern? a. "Do you every have any discomfort or indigestion resulting from exercise or activity?" b. "Have you had any recent episodes of sore throat, fever, or streptococcal infections?" c. "How frequently do you have your cholesterol level and blood pressure checked?" d. "Are there any symptoms that seem to occur when you are feeling very stressed?"

C Rationale: The health perception-health management functional pattern includes information related to what the patient knows about coronary heart disease risk factors and actions the patient is taking to decrease risk. Any patient history of streptococcal infections or sore throat would also be included in this functional pattern, but this patient has possible coronary artery disease, not rheumatic heart disease. Information about discomfort caused by activity would be included in the activity-exercise pattern. The data about symptoms in response to stress would be documented in the coping-stress tolerance functional pattern. Cognitive Level: Application Text Reference: pp. 745-746 Nursing Process: Assessment

Physiological Integrity 3. During assessment of a 72-year-old with ankle swelling, the nurse notes jugular venous distention (JVD) with the head of the patient's bed elevated 45 degrees. The nurse knows this finding indicates a. decreased fluid volume. b. incompetent jugular vein valves. c. elevated right atrial pressure. d. jugular vein atherosclerosis.

C Rationale: The jugular veins empty into the superior vena cava and then into the right atrium, so JVD with the patient sitting at a 45-degree angle reflects elevated right atrial pressure. JVD is an indicator of excessive fluid volume (increased preload), not decreased fluid volume; it is not caused by incompetent jugular vein valves or atherosclerosis. Cognitive Level: Comprehension Text Reference: p. 825 Nursing Process: Assessment

Physiological Integrity 3. The standard orders on the cardiac unit state, "Notify the health care provider for mean arterial pressure (MAP) less than 70 mm Hg." The nurse will need to call the health care provider about a. the patient with left ventricular failure who has a BP of 110/70. b. the patient with a myocardial infarction who has a BP of 114/50. c. the postoperative patient with a BP 116/42. d. the newly admitted patient with a BP of 122/60.

C Rationale: The mean arterial pressure (MAP) is calculated using the formula MAP = (systolic BP + 2 diastolic BP)/3. The MAP for the postoperative patient in answer 3 is 67. The MAP in the other three patients is higher than 70 mm Hg. Cognitive Level: Application Text Reference: p. 744 Nursing Process: Assessment

Physiological Integrity 36. A patient who is being admitted to the emergency department with severe chest pain gives the following list of medications taken at home to the nurse. Which of the medications has the most immediate implications for the patient's care? a. captopril (Capoten) b. furosemide (Lasix) c. sildenafil (Viagra) d. diazepam (Valium)

C Rationale: The nurse will need to avoid giving nitrates to the patient because nitrate administration is contraindicated in patients who are using Viagra because of the risk of sudden death caused by vasodilation. The other home medications should also be documented and reported to the health care provider but do not have as immediate an impact on decisions about the patient's treatment. Cognitive Level: Application Text Reference: p. 817 Nursing Process: Assessment

Physiological Integrity 16. When caring for a patient who developed acute respiratory distress syndrome (ARDS) as a result of a urinary tract infection (UTI), the nurse is asked by the patient's family how a urinary tract infection could cause lung damage. Which response by the nurse is appropriate? a. "The infection spread through the circulation from the urinary tract to the lungs." b. "The urinary tract infection produced toxins that damaged the lungs." c. "The infection caused generalized inflammation that damaged the lungs." d. "The fever associated with the infection led to scar tissue formation in the lungs."

C Rationale: The pathophysiologic changes that occur in ARDS are thought to be caused by inflammatory and immune reactions that lead to changes at the alveolar-capillary membrane. ARDS is not directly caused by infection, toxins, or fever. Cognitive Level: Application Text Reference: p. 1813 Nursing Process: Implementation

Physiological Integrity 21. A patient's sinus rhythm rate is 62. The PR interval is 0.18 seconds at 1:00 AM, 0.20 seconds at 12:30 PM, and 0.23 seconds at 4:00 PM. Which action should the nurse take? a. Document the patient's rhythm and continue to monitor. b. Prepare for possible pacemaker insertion. c. Hold the ordered metoprolol (Lopressor) and call the health care provider. d. Give the PRN dose of lidocaine (Xylocaine).

C Rationale: The patient has progressive first-degree AV block, and the -blocker should be held until discussing the medication with the health care provider. Documentation and continued monitoring are not adequate responses because the block is progressively longer. The patient with first-degree AV block is asymptomatic, and a pacemaker is not indicated. Lidocaine is used to suppress ventricular dysrhythmias and is not appropriate to treat heart block. Cognitive Level: Application Text Reference: p. 853 Nursing Process: Implementation

Physiological Integrity 11. The nurse teaches the patient being evaluated for rhythm disturbances with a Holter monitor to a. remove the electrodes when taking a shower or tub bath. b. exercise more than usual while the monitor is in place. c. keep a diary of daily activities while the monitor is worn. d. connect the recorder to a telephone transmitter once daily.

C Rationale: The patient is instructed to keep a diary describing daily activities while Holter monitoring is being accomplished to help correlate any rhythm disturbances with patient activities. Patients are taught that they should not take a shower or bath during Holter monitoring and that they should continue with their usual daily activities. The recorder stores the information about the patient's rhythm until the end of the testing, when it is removed and the data are analyzed. Cognitive Level: Application Text Reference: pp. 753, 757 Nursing Process: Implementation

Physiological Integrity 6. Which electrocardiographic (ECG) change will be of most concern to the nurse when admitting a patient with chest pain? a. Sinus tachycardia b. Inverted T wave c. ST-segment elevation d. Frequent PACs

C Rationale: The patient is likely to be experiencing an ST-segment-elevation myocardial infarction (STEMI) and immediate therapy with PCI or fibrinolytic medications is indicated to minimize the amount of myocardial damage. The other ECG changes also suggest a need for therapy, but not as rapidly. Cognitive Level: Application Text Reference: p. 802 Nursing Process: Assessment

Psychosocial Integrity 26. An hour after a left upper lobectomy, a patient complains of incisional pain at a level 7 out of 10 and has decreased left-sided breath sounds. The pleural drainage system has 100 ml of bloody drainage and a large air leak. Which action should the nurse take first? a. Assist the patient to deep breathe and cough. b. Milk the chest tube gently to remove any clots. c. Medicate the patient with the ordered morphine. d. Notify the surgeon about the large air leak.

C Rationale: The patient is unlikely to take deep breaths or cough until the pain level is lower. A chest tube output of 100 ml is not unusual in the first hour after thoracotomy and would not require milking of the chest tube. An air leak is expected in the initial postoperative period after thoracotomy. Cognitive Level: Application Text Reference: p. 594 Nursing Process: Implementation

Physiological Integrity 24. After the nurse teaches a patient with chronic stable angina about how to use the prescribed nitrates, which statement by the patient indicates that the teaching has been effective? a. "I will be sure to remove the nitroglycerin patch before using any sublingual nitroglycerin." b. "I will put on the nitroglycerin patch as soon as I develop any chest pain." c. "I will stop what I am doing and sit down before I put the nitroglycerin under my tongue." d. "I will keep the nitroglycerin in my kitchen window where I can find it quickly."

C Rationale: The patient should sit down before taking the nitroglycerin to decrease cardiac workload and prevent orthostatic hypotension. Transdermal nitrates are used prophylactically rather than to treat acute pain and can be used concurrently with sublingual nitroglycerin. Nitrates should be stored in a dark place to avoid deterioration of the medication. Cognitive Level: Application Text Reference: p. 798 Nursing Process: Evaluation

Physiological Integrity 6. A patient hospitalized with IE develops sharp left flank pain and hematuria. The nurse notifies the health care provider, recognizing that these symptoms may indicate a. septicemia. b. acute pyelonephritis. c. vegetative embolization. d. glomerulonephritis.

C Rationale: The patient's clinical manifestations and history of IE indicate embolization. Sudden onset flank pain is not typical of pyelonephritis, septicemia, or glomerulonephritis. Cognitive Level: Application Text Reference: p. 869 Nursing Process: Assessment

Physiological Integrity 9. During a visit to an elderly patient with chronic heart failure, the home care nurse finds that the patient has severe dependent edema and that the legs appear to be weeping serous fluid. Based on these data, the best nursing diagnosis for the patient is a. activity intolerance related to venous congestion. b. disturbed body image related to massive leg swelling. c. impaired skin integrity related to peripheral edema. d. impaired gas exchange related to chronic heart failure.

C Rationale: The patient's findings of severe dependent edema and weeping serous fluid from the legs support the nursing diagnosis of impaired skin integrity. There is less evidence for the nursing diagnoses of activity intolerance, disturbed body image, and impaired gas exchange, although the nurse will further assess the patient to determine whether there are other clinical manifestations of heart failure to indicate that these diagnoses are appropriate. Cognitive Level: Application Text Reference: p. 836 Nursing Process: Diagnosis

Physiological Integrity 10. A patient recovering on a general surgical unit from an aortic valve replacement 1 week ago develops sudden severe pain, pulselessness, pallor, and coolness in the left leg. The nurse should notify the health care provider and a. elevate the left leg on a pillow. b. apply an elastic wrap to the leg. c. keep the patient in bed in the supine position. d. assist the patient in gently exercising the leg.

C Rationale: The patient's history and clinical manifestations are consistent with acute arterial occlusion, and resting the leg will decrease the oxygen demand of the tissues and minimize ischemic damage until circulation can be restored. Elevating the leg or applying an elastic wrap will further compromise blood flow to the leg. Exercise with increase oxygen demand for the tissues of the leg. Cognitive Level: Application Text Reference: p. 907 Nursing Process: Implementation

Physiological Integrity 31. A 26-year-old patient has had CF since birth and has severe lung changes and cor pulmonale as a result of the disease. An appropriate expected outcome is that the patient will a. engage in aerobic exercise without dyspnea. b. be weaned from home oxygen use. c. achieve a realistic attitude toward treatment. d. develop no CF-related complications.

C Rationale: The patient's severe lung disease and cor pulmonale are late complications of CF, and a realistic attitude about what outcome can be expected from treatment is an appropriate outcome. Exercising without dyspnea and weaning from home oxygen therapy are not realistic outcomes for this patient with end-stage disease. The patient already has multiple CF-related complications and is likely to continue to develop complications. Cognitive Level: Application Text Reference: pp. 657-658 Nursing Process: Planning

Physiological Integrity 23. Three days after an MI, the patient develops chest pain that radiates to the back and left arm and is relieved by sitting in a forward position. On auscultation of the patient's chest, the nurse would expect to hear a a. splitting of the S1 heart sound. b. S3 or S4 gallop rhythm. c. pericardial friction rub. d. holosystolic apical murmur.

C Rationale: The patient's symptoms are consistent with the development of pericarditis, a possible complication of MI. The other assessments listed are not consistent with the description of the patient's symptoms. Cognitive Level: Application Text Reference: p. 805 Nursing Process: Assessment

Physiological Integrity 21. When prone positioning is used in the care of a patient with acute respiratory distress syndrome (ARDS), which information obtained by the nurse indicates that the positioning is effective? a. The skin on the patient's back is intact and without redness. b. Sputum and blood cultures show no growth after 24 hours. c. The patient's PaO2 is 90 mm Hg, and the SaO2 is 92%. d. Endotracheal suctioning results in minimal mucous return.

C Rationale: The purpose of prone positioning is to improve the patient's oxygenation as indicated by the PaO2 and SaO2. The other information will be collected but does not indicate whether prone positioning has been effective. Cognitive Level: Application Text Reference: pp. 1817-1818 Nursing Process: Evaluation

Physiological Integrity 18. The nurse is observing a student who is auscultating a patient's lungs. Which action by the student indicates that the nurse should intervene? a. The student compares breath sounds from side to side. b. The student starts at the base of the posterior lung and moves to the apices. c. The student places the stethoscope over the scapulae and then auscultates. d. The student listens only over the posterior part of the chest.

C Rationale: The stethoscope should be placed over lung tissue, not over bony structures. Breath sounds should be compared from side to side. The techniques of starting at the lung base and then moving toward the apices and listening only over the posterior chest are acceptable. Cognitive Level: Comprehension Text Reference: p. 522 Nursing Process: Assessment

Physiological Integrity 9. During assessment of a patient with a history of asthma, the nurse notes wheezing and dyspnea. The nurse will anticipate giving medications to reduce a. laryngospasm. b. pulmonary edema. c. airway narrowing. d. alveolar distention.

C Rationale: The symptoms of asthma are caused by inflammation and spasm of the bronchioles, leading to airway narrowing. Treatment for laryngospasm or pulmonary edema would not be appropriate. There are no medications used to treat alveolar distention. Cognitive Level: Comprehension Text Reference: pp. 608, 611-612 Nursing Process: Assessment

Physiological Integrity 29. The nurse has completed teaching a patient about MDI use. Which patient statement indicates that further patient teaching is needed? a. "I will shake the MDI each time before giving the medication." b. "I will take a slow, deep breath in after pushing down on the MDI." c. "I will float the canister in water to decide whether I need to get a new MDI." d. "I will attach a spacer to the MDI to make it easier for me to use."

C Rationale: This method is no longer recommended as a means of determining whether the medication needs replacement. The other patient statements are accurate and indicate the patient understands how to use the MDI. Cognitive Level: Application Text Reference: p. 622 Nursing Process: Evaluation

Psychosocial Integrity 16. A patient who has been successfully resuscitated after developing ventricular fibrillation asks the nurse about what happened. The most appropriate response by the nurse is, a. "You almost died, but we were able to save you with electrical therapy." b. "You had an episode of some cardiac dysrhythmias that are common after a heart attack." c. "You had a serious abnormal heart rhythm, which treatment was able to reverse." d. "Your heart stopped beating, and we shocked you to get it started again."

C Rationale: This response honestly describes what happened to the patient while avoiding unnecessarily increasing the patient's anxiety level. More information may be given by the nurse if the patient asks further questions. The response "You had an episode of some cardiac dysrhythmias that are common after a heart attack" is not as honest and might lead to distrust of the nurse or health care system. The remaining two responses are accurate but would increase the anxiety level for many patients. Cognitive Level: Application Text Reference: p.855 Nursing Process: Implementation

Physiological Integrity 10. When admitting a patient who has a pleural effusion, which technique will the nurse use to assess for tactile fremitus? a. Percuss over the entire posterior chest. b. Use the fingertips to assess for vibration. c. Place the palms of the hands on the chest wall. d. Auscultate while the patient says "ninety-nine."

C Rationale: To assess for tactile fremitus, the nurse should use the palms of the hands to assess for vibration when the patient repeats a word or phrase such as "99." Percussion, palpation with the fingertips, and auscultation are also used during the respiratory assessment but will not assess for fremitus. Cognitive Level: Application Text Reference: p. 522 Nursing Process: Assessment

Physiological Integrity 5. A patient hospitalized with a streptococcal infective endocarditis tells the nurse," I know that I need antibiotics, but I do not want to be hospitalized for very long." The nurse explains that a. after 2 weeks of IV antibiotic therapy, the patient may be discharged with oral antibiotics to take for another 4 weeks. b. hospitalization for 4 to 6 weeks will be necessary to prevent a relapse while receiving IV antibiotic therapy. c. the patient will be able to receive outpatient IV antibiotic therapy if complications such as heart failure do not develop. d. hospitalization for IV antibiotics is necessary until the fever is resolved, but then the patient can be discharged on oral antibiotics.

C Rationale: Treatment for IE involves 4 to 6 weeks of IV antibiotic therapy in order to eradicate the bacteria, but patients frequently receive IV antibiotics on an outpatient basis. Oral antibiotics do not result in the consistently high antibiotic level that is needed to eradicate the infective bacteria. The resolution of the fever is not an indication that the patient can be discharged or that oral antibiotics are adequate for treatment. Cognitive Level: Application Text Reference: p. 871 Nursing Process: Implementation

Physiological Integrity 20. A patient with supraventricular tachycardia (SVT) is hemodynamically stable and requires cardioversion. The nurse will plan to a. turn the synchronizer switch to the "off" position. b. set the level of joules to 300 to convert the SVT. c. administer a sedative before the procedure is begun. d. check the incision for bleeding after the procedure.

C Rationale: When a patient has a non-emergency cardioversion, sedation is used just before the procedure. The synchronizer switch is turned on for cardioversion. A low level of joules (e.g., 50) is first selected for cardioversion. There is no incision after cardioversion. Cognitive Level: Application Text Reference: p. 857 Nursing Process: Implementation

Which of the following factors would a nurse identify as a most likely cause of diabetic ketoacidosis (DKA) in a client with diabetes? a) The client has been exercising more than usual. b) The client has not consumed sufficient calories. c) The client has eaten and has not taken or received insulin. d) The client continues medication therapy despite adequate food intake

C) The client has eaten and has not taken or received insulin

Integrated Process: Nursing Process (Implementation) 20. What statement indicates that a client needs further teaching regarding therapy with salmeterol (Serevent)? a. "I will be certain to shake the inhaler well before I use it." b. "It may take a while before I notice a change in my asthma." c. "I will use the drug when I have an asthma attack." d. "I will be careful not to let the drug escape out of my nose and mouth."

C Salmeterol is designed to prevent an asthma attack; it does not relieve or reverse symptoms. The client does not have to keep this inhaler with him or her always because it is not used as a rescue medication. Salmeterol (Serevent) has a slow onset of action; therefore it should not be used as a rescue drug. The drug must be shaken well because it has a tendency to separate easily. Poor technique on the client's part allows the drug to escape through the nose and mouth. DIF: Cognitive Level: Application/Applying or higher REF: N/A TOP: Client Needs Category: Health Promotion and Maintenance (Self-Care)

Integrated Process: Nursing Process (Implementation) 13. A client is undergoing radiation therapy as treatment for lung cancer and has developed esophagitis. Which is the best diet selection for this client? a. Spaghetti with meat sauce, ice cream b. Scrambled eggs, bacon, toast c. Omelet, whole wheat bread d. Pasta salad, custard, orange juice

C Side effects of radiation therapy may include inflammation of the esophagus. Clients should be taught that bland, soft, high-calorie foods are best, along with liquid nutritional supplements. Tomato sauce may prove too spicy for a client with esophagitis. Toast is too difficult to swallow with this condition, and orange juice and other foods with citric acid are too caustic. DIF: Cognitive Level: Comprehension/Understanding REF: p. 633 TOP: Client Needs Category: Physiological Integrity (Basic Care and Comfort—Nutrition and Oral Hydration)

Physiological Integrity 4. IV sodium nitroprusside (Nipride) is ordered for a patient with acute pulmonary edema. During the first hours of administration, the nurse will need to titrate the nitroprusside rate if the patient develops a. ventricular ectopy. b. a dry, hacking cough. c. a systolic BP <90 mm Hg. d. a heart rate <50 beats/minute.

C Sodium nitroprusside is a potent vasodilator, and the major adverse effect is severe hypotension. Coughing and bradycardia are not adverse effects of this medication. Nitroprusside does not cause increased ventricular ectopy. DIF: Cognitive Level: Apply (application) REF: 774 TOP: Nursing Process: Evaluation MSC:

Integrated Process: Nursing Process (Implementation) 3. A client diagnosed with asthma has not responded well to medication. The client is concerned and asks the nurse, "What is wrong with me, and why am I not getting better?" What is the nurse's best response? a. "You just weren't used to the medication yet." b. "The medication dose has to be increased." c. "It is possible that genetic testing may help." d. "You should try homeopathic medicine."

C Some genetic variations may cause the activity of beta-adrenergic receptors to change, meaning that the client would not respond as expected to beta agonists. Genetic testing may help to determine why the drug therapy is not working and may help the clinician to identify new therapy that will work. DIF: Cognitive Level: Application/Applying or higher REF: N/A TOP: Client Needs Category: Physiological Integrity (Pharmacological and Parenteral Therapies—Expected Actions/Outcomes)

Physiological Integrity 7. The health care provider writes an order for bacteriologic testing for a patient who has a positive tuberculosis skin test. Which action should the nurse take? a. Teach about the reason for the blood tests. b. Schedule an appointment for a chest x-ray. c. Teach about the need to get sputum specimens for 2 to 3 consecutive days. d. Instruct the patient to expectorate three specimens as soon as possible.

C Sputum specimens are obtained on 2 to 3 consecutive days for bacteriologic testing for M. tuberculosis. The patient should not provide all the specimens at once. Blood cultures are not used for tuberculosis testing. A chest x-ray is not bacteriologic testing. Although the findings on chest x-ray examination are important, it is not possible to make a diagnosis of TB solely based on chest x-ray findings because other diseases can mimic the appearance of TB. DIF: Cognitive Level: Apply (application) REF: 530 TOP: Nursing Process: Implementation MSC:

Integrated Process: Teaching/Learning 18. Which statement indicates that a client needs additional teaching about using an inhaler? a. "I will not exhale into the inhaler." b. "I will store the inhaler in a drawer in my bedroom." c. "I will soak my inhaler in water to clean it." d. "I will inhale and hold my breath."

C Submerging an inhaler in water to wash it is not necessary and may cause the medication in the inhaler to clump together if it is a dry powder inhaler. The other statements are all correct—the client should not exhale into the inhaler, can store the inhaler in his or her bedroom, and will need to inhale and hold breath slightly when using the inhaler. DIF: Cognitive Level: Application/Applying or higher REF: N/A TOP: Client Needs Category: Health Promotion and Maintenance (Self-Care)

Integrated Process: Teaching/Learning 16. The nurse assesses a client with asthma and finds wheezing throughout the lung fields and decreased pulse oxygen saturation. In addition, the nurse notes suprasternal retraction on inhalation. What is the nurse's best action? a. Perform peak expiratory flow readings. b. Assess for a midline trachea. c. Administer oxygen and a rescue inhaler. d. Call a code.

C Suprasternal retraction caused by inhalation usually indicates that the client is using accessory muscles and is having difficulty moving air into the respiratory passages because of airway narrowing. Wheezing indicates a narrowed airway; a decreased pulse oxygen saturation also supports this finding. The asthma is not responding to the medication, and intervention is needed. Administration of a rescue inhaler is indicated, probably along with administration of oxygen. The nurse would not do a peak flow reading at this time, nor would a code be called. Midline trachea is a normal and expected finding. DIF: Cognitive Level: Application/Applying or higher REF: N/A TOP: Client Needs Category: Physiological Integrity (Physiological Adaptation—Illness Management)

A client with type 1 diabetes mellitus is receiving short-acting insulin to maintain control of blood glucose levels. In providing glucometer instructions, the nurse would instruct the client to use which site for most accurate findings? a) Forearm b) Upper arm c) Finger d) Thigh

C) finger

The diabetic client asks the nurse why shoes and socks are removed at each office visit. Which assessment finding is most significant in determining the protocol for inspection of feet? a) Retinopathy b) Autonomic neuropathy c) Sensory neuropathy d) Nephropathy

C) sensory neuropathy

Physiological Integrity 7. The nurse working on the heart failure unit knows that teaching an older female patient with newly diagnosed heart failure is effective when the patient states that a. she will take furosemide (Lasix) every day at bedtime. b. the nitroglycerin patch is applied when any chest pain develops. c. she will call the clinic if her weight goes from 124 to 128 pounds in a week. d. an additional pillow can help her sleep if she is feeling short of breath at night.

C Teaching for a patient with heart failure includes information about the need to weigh daily and notify the health care provider about an increase of 3 pounds in 2 days or 3 to 5 pounds in a week. Nitroglycerin patches are used primarily to reduce preload (not to prevent chest pain) in patients with heart failure and should be used daily, not on an "as needed" basis. Diuretics should be taken earlier in the day to avoid nocturia and sleep disturbance. The patient should call the clinic if increased orthopnea develops, rather than just compensating by further elevating the head of the bed. DIF: Cognitive Level: Apply (application) REF: 771 TOP: Nursing Process: Evaluation MSC:

Physiological Integrity 18. A patient with chronic heart failure who is taking a diuretic and an angiotensin-converting enzyme (ACE) inhibitor and who is on a low-sodium diet tells the home health nurse about a 5-pound weight gain in the last 3 days. The nurse's priority action will be to a. have the patient recall the dietary intake for the last 3 days. b. ask the patient about the use of the prescribed medications. c. assess the patient for clinical manifestations of acute heart failure. d. teach the patient about the importance of restricting dietary sodium.

C The 5-pound weight gain over 3 days indicates that the patient's chronic heart failure may be worsening. It is important that the patient be assessed immediately for other clinical manifestations of decompensation, such as lung crackles. A dietary recall to detect hidden sodium in the diet, reinforcement of sodium restrictions, and assessment of medication compliance may be appropriate interventions but are not the first nursing actions indicated. DIF: Cognitive Level: Apply (application) REF: 771 OBJ: Special Questions: Prioritization TOP: Nursing Process: Assessment MSC:

Physiological Integrity 16. During the administration of the thrombolytic agent to a patient with an acute myocardial infarction (AMI), the nurse should stop the drug infusion if the patient experiences a. bleeding from the gums. b. increase in blood pressure. c. a decrease in level of consciousness. d. a nonsustained episode of ventricular tachycardia.

C The change in level of consciousness indicates that the patient may be experiencing intracranial bleeding, a possible complication of thrombolytic therapy. Some bleeding of the gums is an expected side effect of the therapy but not an indication to stop infusion of the thrombolytic medication. A decrease in blood pressure could indicate internal bleeding. A nonsustained episode of ventricular tachycardia is a common reperfusion dysrhythmia and may indicate that the therapy is effective. DIF: Cognitive Level: Apply (application) REF: 752 TOP: Nursing Process: Evaluation MSC:

The nurse is caring for a patient whose blood glucose level is 55mg/dL. What is the likely nursing response? "A. Administer a glucagon injection B. Give a small meal C. Administer 10-15 g of a carbohydrate D. Give a small snack of high protein food"

C The client has low hypoglycemia. This is generally treated with a small snack.

Integrated Process: Nursing Process (Assessment) 6. The nurse is evaluating a client's response to medication therapy for asthma. The client has a peak flowmeter reading in the yellow zone. What does the nurse do next? a. Nothing; this is an acceptable range. b. Teach the client to take deeper breaths. c. Assist the client to use a rescue inhaler. d. Assess the client's lungs.

C The client with a peak flow reading in the yellow zone needs to use a rescue inhaler, then have a reading taken again within a few minutes. The nurse has no reason to assess the client's lungs at this point in time, nor would the nurse take the time to teach at this moment. DIF: Cognitive Level: Application/Applying or higher REF: N/A TOP: Client Needs Category: Physiological Integrity (Pharmacological and Parenteral Therapies—Expected Actions/Outcomes)

Integrated Process: Teaching/Learning 24. The nurse is assessing a client with lung disease. Which symptom does the nurse intervene for first? a. The client's anterior-posterior chest diameter is 2:2. b. Clubbing of the finger tips is noted. c. The client has bilateral dependent leg edema. d. The client is pale.

C The client with bilateral dependent edema may be developing right-sided heart failure in response to respiratory disease. This symptom should be investigated right away and reported to the health care provider. Further assessment is needed. The client with chronic lung disease may develop increased anterior-posterior diameter and clubbing as responses to chronic hypoxia. These symptoms do not require immediate intervention. The client is often pale or has a dusky appearance; this also would not warrant immediate intervention. DIF: Cognitive Level: Application/Applying or higher REF: N/A TOP: Client Needs Category: Physiological Integrity (Reduction of Risk Potential—Potential for Complications from Surgical Procedures and Health Alterations)

Physiological Integrity 3. Which topic will the nurse plan to include in discharge teaching for a patient with systolic heart failure and an ejection fraction of 33%? a. Need to begin an aerobic exercise program several times weekly b. Use of salt substitutes to replace table salt when cooking and at the table c. Benefits and side effects of angiotensin-converting enzyme (ACE) inhibitors d. Importance of making an annual appointment with the primary care provider

C The core measures for the treatment of heart failure established by The Joint Commission indicate that patients with an ejection fraction (EF) <40% receive an ACE inhibitor to decrease the progression of heart failure. Aerobic exercise may not be appropriate for a patient with this level of heart failure, salt substitutes are not usually recommended because of the risk of hyperkalemia, and the patient will need to see the primary care provider more frequently than annually. DIF: Cognitive Level: Apply (application) REF: 767 | 776 | 779 TOP: Nursing Process: Planning MSC:

Physiological Integrity 35. The nurse obtains the following data when assessing a patient who experienced an ST-segment-elevation myocardial infarction (STEMI) 2 days previously. Which information is most important to report to the health care provider? a. The troponin level is elevated. b. The patient denies ever having a heart attack. c. Bilateral crackles are auscultated in the mid-lower lobes. d. The patient has occasional premature atrial contractions (PACs).

C The crackles indicate that the patient may be developing heart failure, a possible complication of myocardial infarction (MI). The health care provider may need to order medications such as diuretics or angiotensin-converting enzyme (ACE) inhibitors for the patient. Elevation in troponin level at this time is expected. PACs are not life-threatening dysrhythmias. Denial is a common response in the immediate period after the MI. DIF: Cognitive Level: Apply (application) REF: 748 OBJ: Special Questions: Prioritization TOP: Nursing Process: Assessment MSC:

Physiological Integrity 10. While admitting an 82-year-old with acute decompensated heart failure to the hospital, the nurse learns that the patient lives alone and sometimes confuses the "water pill" with the "heart pill." When planning for the patient's discharge the nurse will facilitate a a. consult with a psychologist. b. transfer to a long-term care facility. c. referral to a home health care agency. d. arrangements for around-the-clock care.

C The data about the patient suggest that assistance in developing a system for taking medications correctly at home is needed. A home health nurse will assess the patient's home situation and help the patient develop a method for taking the two medications as directed. There is no evidence that the patient requires services such as a psychologist consult, long-term care, or around-the-clock home care. DIF: Cognitive Level: Apply (application) REF: 783 TOP: Nursing Process: Planning MSC:

Physiological Integrity 8. While caring for a patient with respiratory disease, the nurse observes that the patient's SpO2 drops from 93% to 88% while the patient is ambulating in the hallway. What is the priority action of the nurse? a. Notify the health care provider. b. Document the response to exercise. c. Administer the PRN supplemental O2. d. Encourage the patient to pace activity.

C The drop in SpO2 to 85% indicates that the patient is hypoxemic and needs supplemental oxygen when exercising. The other actions are also important, but the first action should be to correct the hypoxemia. DIF: Cognitive Level: Apply (application) REF: 480 OBJ: Special Questions: Prioritization TOP: Nursing Process: Implementation MSC:

Safe and Effective Care Environment 22. The nurse is reviewing the laboratory results for newly admitted patients on the cardiovascular unit. Which patient laboratory result is most important to communicate as soon as possible to the health care provider? a. Patient whose triglyceride level is high b. Patient who has very low homocysteine level c. Patient with increase in troponin T and troponin I level d. Patient with elevated high-sensitivity C-reactive protein level

C The elevation in troponin T and I indicates that the patient has had an acute myocardial infarction. Further assessment and interventions are indicated. The other laboratory results are indicative of increased risk for coronary artery disease but are not associated with acute cardiac problems that need immediate intervention. DIF: Cognitive Level: Apply (application) REF: 698 OBJ: Special Questions: Prioritization; Multiple Patients TOP: Nursing Process: Assessment MSC:

Physiological Integrity 5. After the nurse has finished teaching a patient about the use of sublingual nitroglycerin (Nitrostat), which patient statement indicates that the teaching has been effective? a. "I can expect some nausea as a side effect of nitroglycerin." b. "I should only take the nitroglycerin if I start to have chest pain." c. "I will call an ambulance if I still have pain after taking 3 nitroglycerin 5 minutes apart." d. "Nitroglycerin helps prevent a clot from forming and blocking blood flow to my heart."

C The emergency medical services (EMS) system should be activated when chest pain or other symptoms are not completely relieved after 3 sublingual nitroglycerin tablets taken 5 minutes apart. Nitroglycerin can be taken to prevent chest pain or other symptoms from developing (e.g., before intercourse). Gastric upset (e.g., nausea) is not an expected side effect of nitroglycerin. Nitroglycerin does not impact the underlying pathophysiology of coronary artery atherosclerosis. DIF: Cognitive Level: Apply (application) REF: 743 TOP: Nursing Process: Evaluation MSC:

Physiological Integrity 41. The nurse is caring for a patient with idiopathic pulmonary arterial hypertension (IPAH) who is receiving epoprostenol (Flolan). Which assessment information requires the most immediate action by the nurse? a. The oxygen saturation is 94%. b. The blood pressure is 98/56 mm Hg. c. The patient's central IV line is disconnected. d. The international normalized ratio (INR) is prolonged.

C The half-life of this drug is 6 minutes, so the nurse will need to restart the infusion as soon as possible to prevent rapid clinical deterioration. The other data also indicate a need for ongoing monitoring or intervention, but the priority action is to reconnect the infusion. DIF: Cognitive Level: Apply (application) REF: 555 OBJ: Special Questions: Prioritization TOP: Nursing Process: Assessment MSC:

Physiological Integrity 44. Which action by the nurse will be most effective in decreasing the spread of pertussis in a community setting? a. Providing supportive care to patients diagnosed with pertussis b. Teaching family members about the need for careful hand washing c. Teaching patients about the need for adult pertussis immunizations d. Encouraging patients to complete the prescribed course of antibiotics

C The increased rate of pertussis in adults is thought to be due to decreasing immunity after childhood immunization. Immunization is the most effective method of protecting communities from infectious diseases. Hand washing should be taught, but pertussis is spread by droplets and contact with secretions. Supportive care does not shorten the course of the disease or the risk for transmission. Taking antibiotics as prescribed does assist with decreased transmission, but patients are likely to have already transmitted the disease by the time the diagnosis is made. DIF: Cognitive Level: Apply (application) REF: 522 TOP: Nursing Process: Implementation MSC:

A physician treating a client in the cardiac care unit for atrial arrhythmia orders metoprolol (Lopressor), 25 mg P.O. two times per day. Metoprolol inhibits the action of sympathomimetics at beta1-receptor sites. Where are these sites mainly located? a) Bronchi b) Uterus c) Heart d) Blood vessels

C. Heart

Integrated Process: Teaching/Learning 39. A client infected with Burkholderia cepacia is admitted to the unit. What is the nurse's priority action when caring for this client? a. Instruct the client to wash his or her hands after contact with other people. b. Place the client on strict isolation. c. Keep the client isolated from other clients with cystic fibrosis. d. Administer IV vancomycin daily.

C The infection is spread through casual contact between cystic fibrosis clients, thus the need for isolation of these clients from each other. Strict isolation measures will not be necessary. Although the client should wash his or her hands frequently, the most important measure that can be implemented on the unit is isolation of the client from other cystic fibrosis clients. DIF: Cognitive Level: Application/Applying or higher REF: N/A TOP: Client Needs Category: Safe and Effective Care Environment (Safety and Infection Control—Standard Precautions/Transmission-Based Precautions/Surgical Asepsis)

1. While assessing a 68-year-old with ascites, the nurse also notes jugular venous distention (JVD) with the head of the patient's bed elevated 45 degrees. The nurse knows this finding indicates a. decreased fluid volume. b. jugular vein atherosclerosis. c. increased right atrial pressure. d. incompetent jugular vein valves.

C The jugular veins empty into the superior vena cava and then into the right atrium, so JVD with the patient sitting at a 45-degree angle reflects increased right atrial pressure. JVD is an indicator of excessive fluid volume (increased preload), not decreased fluid volume. JVD is not caused by incompetent jugular vein valves or atherosclerosis. DIF: Cognitive Level: Understand (comprehension) REF: 769 TOP: Nursing Process: Assessment MSC:

Physiological Integrity 6. A patient with right lower-lobe pneumonia has been treated with IV antibiotics for 3 days. Which assessment data obtained by the nurse indicates that the treatment has been effective? a. Bronchial breath sounds are heard at the right base. b. The patient coughs up small amounts of green mucus. c. The patient's white blood cell (WBC) count is 9000/µL. d. Increased tactile fremitus is palpable over the right chest.

C The normal WBC count indicates that the antibiotics have been effective. All the other data suggest that a change in treatment is needed. DIF: Cognitive Level: Apply (application) REF: 524-525 TOP: Nursing Process: Evaluation MSC:

Physiological Integrity 14. Employee health test results reveal a tuberculosis (TB) skin test of 16-mm induration and a negative chest x-ray for a staff nurse working on the pulmonary unit. The nurse has no symptoms of TB. Which information should the occupational health nurse plan to teach the staff nurse? a. Standard four-drug therapy for TB b. Need for annual repeat TB skin testing c. Use and side effects of isoniazid (INH) d. Bacille Calmette-Guérin (BCG) vaccine

C The nurse is considered to have a latent TB infection and should be treated with INH daily for 6 to 9 months. The four-drug therapy would be appropriate if the nurse had active TB. TB skin testing is not done for individuals who have already had a positive skin test. BCG vaccine is not used in the United States for TB and would not be helpful for this individual, who already has a TB infection. DIF: Cognitive Level: Apply (application) REF: 532 TOP: Nursing Process: Planning MSC:

Physiological Integrity 3. The nurse discusses management of upper respiratory infections (URI) with a patient who has acute sinusitis. Which statement by the patient indicates that additional teaching is needed? a. "I can take acetaminophen (Tylenol) to treat my discomfort." b. "I will drink lots of juices and other fluids to stay well hydrated." c. "I can use my nasal decongestant spray until the congestion is all gone." d. "I will watch for changes in nasal secretions or the sputum that I cough up."

C The nurse should clarify that nasal decongestant sprays should be used for no more than 3 days to prevent rebound vasodilation and congestion. The other responses indicate that the teaching has been effective. DIF: Cognitive Level: Apply (application) REF: 502 TOP: Nursing Process: Evaluation MSC:

Safe and Effective Care Environment 15. Which action will the nurse implement for a patient who arrives for a calcium-scoring CT scan? a. Insert an IV catheter. b. Administer oral sedative medications. c. Teach the patient about the procedure. d. Confirm that the patient has been fasting.

C The nurse will need to teach the patient that the procedure is rapid and involves little risk. None of the other actions are necessary. DIF: Cognitive Level: Apply (application) REF: 703 TOP: Nursing Process: Implementation MSC:

Physiological Integrity 42. After reviewing information shown in the accompanying figure from the medical records of a 43-year-old, which risk factor modification for coronary artery disease should the nurse include in patient teaching? a. Importance of daily physical activity b. Effect of weight loss on blood pressure c. Dietary changes to improve lipid levels d. Ongoing cardiac risk associated with history of tobacco use

C The patient has an elevated low-density lipoprotein (LDL) cholesterol and low high-density lipoprotein (HDL) cholesterol, which will increase the risk of coronary artery disease. Although the blood pressure is in the prehypertensive range, the patient's waist circumference and body mass index (BMI) indicate an appropriate body weight. The risk for coronary artery disease a year after quitting smoking is the same as a nonsmoker. The patient's occupation indicates that daily activity is at the levels suggested by national guidelines. DIF: Cognitive Level: Analyze (analysis) REF: 737 TOP: Nursing Process: Planning MSC:

Safe and Effective Care Environment 30. Which electrocardiographic (ECG) change is most important for the nurse to report to the health care provider when caring for a patient with chest pain? a. Inverted P wave b. Sinus tachycardia c. ST-segment elevation d. First-degree atrioventricular block

C The patient is likely to be experiencing an ST-segment-elevation myocardial infarction (STEMI). Immediate therapy with percutaneous coronary intervention (PCI) or thrombolytic medication is indicated to minimize myocardial damage. The other ECG changes may also suggest a need for therapy, but not as rapidly. DIF: Cognitive Level: Apply (application) REF: 747-748 OBJ: Special Questions: Prioritization TOP: Nursing Process: Assessment MSC:

Anemia (HF)

Caused by poor nutrition, renal disease or drug therapy

hypercalcemia

Causes for this include hyperparathyroidism and malignancy (hypercalcemia, hypocalcemia)

1. The nurse teaches a patient with chronic bronchitis about a new prescription for Advair Diskus (combined fluticasone and salmeterol). Which action by the patient would indicate to the nurse that teaching about medication administration has been successful? a. The patient shakes the device before use. b. The patient attaches a spacer to the Diskus. c. The patient rapidly inhales the medication. d. The patient performs huff coughing after inhalation.

C The patient should inhale the medication rapidly. Otherwise the dry particles will stick to the tongue and oral mucosa and not get inhaled into the lungs. Advair Diskus is a dry powder inhaler; shaking is not recommended. Spacers are not used with dry powder inhalers. Huff coughing is a technique to move mucus into larger airways to expectorate. The patient should not huff cough or exhale forcefully after taking Advair in order to keep the medication in the lungs. DIF: Cognitive Level: Apply (application) REF: 574 TOP: Nursing Process: Evaluation MSC:

Physiological Integrity 18. In preparation for discharge, the nurse teaches a patient with chronic stable angina how to use the prescribed short-acting and long-acting nitrates. Which patient statement indicates that the teaching has been effective? a. "I will check my pulse rate before I take any nitroglycerin tablets." b. "I will put the nitroglycerin patch on as soon as I get any chest pain." c. "I will stop what I am doing and sit down before I put the nitroglycerin under my tongue." d. "I will be sure to remove the nitroglycerin patch before taking any sublingual nitroglycerin."

C The patient should sit down before taking the nitroglycerin to decrease cardiac workload and prevent orthostatic hypotension. Transdermal nitrates are used prophylactically rather than to treat acute pain and can be used concurrently with sublingual nitroglycerin. Although the nurse should check blood pressure before giving nitroglycerin, patients do not need to check the pulse rate before taking nitrates. DIF: Cognitive Level: Apply (application) REF: 743 TOP: Nursing Process: Evaluation MSC:

Physiological Integrity 23. An outpatient who has chronic heart failure returns to the clinic after 2 weeks of therapy with metoprolol (Toprol XL). Which assessment finding is most important for the nurse to report to the health care provider? a. 2+ pedal edema b. Heart rate of 56 beats/minute c. Blood pressure (BP) of 88/42 mm Hg d. Complaints of fatigue

C The patient's BP indicates that the dose of metoprolol may need to be decreased because of hypotension. Bradycardia is a frequent adverse effect of -adrenergic blockade, but the rate of 56 is not unusual with â-adrenergic blocker therapy. -Adrenergic blockade initially will worsen symptoms of heart failure in many patients, and patients should be taught that some increase in symptoms, such as fatigue and edema, is expected during the initiation of therapy with this class of drugs. DIF: Cognitive Level: Apply (application) REF: 777 OBJ: Special Questions: Prioritization TOP: Nursing Process: Assessment MSC:

Physiological Integrity 6. A patient seen in the asthma clinic has recorded daily peak flows that are 75% of the baseline. Which action will the nurse plan to take next? a. Increase the dose of the leukotriene inhibitor. b. Teach the patient about the use of oral corticosteroids. c. Administer a bronchodilator and recheck the peak flow. d. Instruct the patient to keep the next scheduled follow-up appointment.

C The patient's peak flow reading indicates that the condition is worsening (yellow zone). The patient should take the bronchodilator and recheck the peak flow. Depending on whether the patient returns to the green zone, indicating well-controlled symptoms, the patient may be prescribed oral corticosteroids or a change in dosing of other medications. Keeping the next appointment is appropriate, but the patient also needs to be taught how to control symptoms now and use the bronchodilator. DIF: Cognitive Level: Apply (application) REF: 580 TOP: Nursing Process: Planning MSC:

Physiological Integrity 17. A patient is recovering from a myocardial infarction (MI) and develops chest pain on day 3 that increases when taking a deep breath and is relieved by leaning forward. Which action should the nurse take next? a. Assess the feet for pedal edema. b. Palpate the radial pulses bilaterally. c. Auscultate for a pericardial friction rub. d. Check the heart monitor for dysrhythmias.

C The patient's symptoms are consistent with the development of pericarditis, a possible complication of MI. The other assessments listed are not consistent with the description of the patient's symptoms. DIF: Cognitive Level: Apply (application) REF: 749 TOP: Nursing Process: Implementation MSC:

Safe and Effective Care Environment 23. When the nurse is screening patients for possible peripheral arterial disease, indicate where the posterior tibial artery will be palpated. a. 1 b. 2 c. 3 d. 4

C The posterior tibial site is located behind the medial malleolus of the tibia. DIF: Cognitive Level: Understand (comprehension) REF: 696 TOP: Nursing Process: Assessment MSC:

Safe and Effective Care Environment 37. A patient who has chest pain is admitted to the emergency department (ED) and all of the following are ordered. Which one should the nurse arrange to be completed first? a. Chest x-ray b. Troponin level c. Electrocardiogram (ECG) d. Insertion of a peripheral IV

C The priority for the patient is to determine whether an acute myocardial infarction (AMI) is occurring so that reperfusion therapy can begin as quickly as possible. ECG changes occur very rapidly after coronary artery occlusion, and an ECG should be obtained as soon as possible. Troponin levels will increase after about 3 hours. Data from the chest x-ray may impact the patient's care but are not helpful in determining whether the patient is experiencing a myocardial infarction (MI). Peripheral access will be needed but not before the ECG. DIF: Cognitive Level: Apply (application) REF: 749 OBJ: Special Questions: Prioritization TOP: Nursing Process: Implementation MSC:

Safe and Effective Care Environment 41. A patient with diabetes mellitus and chronic stable angina has a new order for captopril (Capoten). The nurse should teach the patient that the primary purpose of captopril is to a. lower heart rate. b. control blood glucose levels. c. prevent changes in heart muscle. d. reduce the frequency of chest pain.

C The purpose for angiotensin-converting enzyme (ACE) inhibitors in patients with chronic stable angina who are at high risk for a cardiac event is to decrease ventricular remodeling. ACE inhibitors do not directly impact angina frequency, blood glucose, or heart rate. DIF: Cognitive Level: Apply (application) REF: 745 TOP: Nursing Process: Planning MSC:

Integrated Process: Nursing Process (Implementation) 7. Which statement indicates that the client understands teaching about the use of long-acting beta2 agonist medications? a. "I will not have to take this medication every day." b. "I will take this medication when I have an asthma attack." c. "I will take this medication daily to prevent an acute attack." d. "I will eventually be able to stop using this medication."

C This medication will help prevent an acute asthma attack because it is long acting. The client will take this medication every day for best effect. This is not the medication the client will use during an acute asthma attack because it does not have an immediate onset of action. The client will not be weaned off this medication because this is likely to be one of his or her daily medications. DIF: Cognitive Level: Application/Applying or higher REF: N/A TOP: Client Needs Category: Physiological Integrity (Pharmacological and Parenteral Therapies—Expected Actions/Outcomes)

Physiological Integrity 4. A nurse who is caring for patient with a tracheostomy tube in place has just auscultated rhonchi bilaterally. If the patient is unsuccessful in coughing up secretions, what action should the nurse take? a. Encourage increased incentive spirometer use. b. Encourage the patient to increase oral fluid intake. c. Put on sterile gloves and use a sterile catheter to suction. d. Preoxygenate the patient for 3 minutes before suctioning.

C This patient needs suctioning now to secure a patent airway. Sterile gloves and a sterile catheter are used when suctioning a tracheostomy. Preoxygenation for 3 minutes is not necessary. Incentive spirometer (IS) use opens alveoli and can induce coughing, which can mobilize secretions. However, the patient with a tracheostomy may not be able to use an incentive spirometer. Increasing oral fluid intake would not moisten and help mobilize secretions in a timely manner. DIF: Cognitive Level: Apply (application) REF: 510 TOP: Nursing Process: Implementation MSC:

Physiological Integrity 17. A patient with a history of chronic heart failure is admitted to the emergency department (ED) with severe dyspnea and a dry, hacking cough. Which action should the nurse do first? a. Auscultate the abdomen. b. Check the capillary refill. c. Auscultate the breath sounds. d. Assess the level of orientation.

C This patient's severe dyspnea and cough indicate that acute decompensated heart failure (ADHF) is occurring. ADHF usually manifests as pulmonary edema, which should be detected and treated immediately to prevent ongoing hypoxemia and cardiac/respiratory arrest. The other assessments will provide useful data about the patient's volume status and also should be accomplished rapidly, but detection (and treatment) of pulmonary complications is the priority. DIF: Cognitive Level: Apply (application) REF: 769-770 OBJ: Special Questions: Prioritization TOP: Nursing Process: Assessment MSC:

Physiological Integrity 14. A patient with ST-segment elevation in three contiguous electrocardiographic (ECG) leads is admitted to the emergency department (ED) and diagnosed as having an ST-segment-elevation myocardial infarction (STEMI). Which question should the nurse ask to determine whether the patient is a candidate for thrombolytic therapy? a. "Do you have any allergies?" b. "Do you take aspirin on a daily basis?" c. "What time did your chest pain begin?" d. "Can you rate your chest pain using a 0 to 10 scale?"

C Thrombolytic therapy should be started within 6 hours of the onset of the myocardial infarction (MI), so the time at which the chest pain started is a major determinant of the appropriateness of this treatment. The other information will also be needed, but it will not be a factor in the decision about thrombolytic therapy. DIF: Cognitive Level: Apply (application) REF: 743 TOP: Nursing Process: Assessment MSC:

Physiological Integrity 34. The nurse assesses a patient with a history of asthma. Which assessment finding indicates that the nurse should take immediate action? a. Pulse oximetry reading of 91% b. Respiratory rate of 26 breaths/minute c. Use of accessory muscles in breathing d. Peak expiratory flow rate of 240 L/minute

C Use of accessory muscle indicates that the patient is experiencing respiratory distress and rapid intervention is needed. The other data indicate the need for ongoing monitoring and assessment but do not suggest that immediate treatment is required. DIF: Cognitive Level: Apply (application) REF: 564-565 OBJ: Special Questions: Prioritization TOP: Nursing Process: Assessment MSC:

Physiological Integrity OTHER 1. While listening to the posterior chest of a patient who is experiencing acute shortness of breath, the nurse hears these sounds. How should the nurse document the lung sounds? Click here to listen to the audio clip a. Pleural friction rub b. Low-pitched crackles c. High-pitched wheezes d. Bronchial breath sounds

C Wheezes are continuous high-pitched or musical sounds heard initially with expiration. The other responses are typical of other adventitious breath sounds. DIF: Cognitive Level: Understand (comprehension) REF: 483 TOP: Nursing Process: Assessment MSC:

In caring for the patient with angina, the patient said, "I walked to the bathroom. While I was having a bowel movement, I started having the worst chest pain ever, like before I was admitted. I called for a nurse, but the pain is gone now." What further assessment data should the nurse obtain from the patient?

C "In what areas did you feel this pain?"

Physiological Integrity MULTIPLE RESPONSE 1. When assessing a 76-year-old woman, the nurse finds the following results: BP 146/102, resting HR 104, slightly irregular S4 heart sound, and a grade I/VI aortic systolic murmur. The nurse recognizes that common effects of aging may be responsible for the (Select all that apply.) a. HR. b. irregular pulse. c. S4 heart sound. d. systolic BP. e. diastolic BP. f. grade I/VI aortic systolic murmur.

C, D, F Rationale: An S4 gallop, increased systolic BP, and aortic stenosis are associated with aging, although all these findings require further assessment or intervention. Increases in HR, irregular heart rhythms, and diastolic BP increases are not associated with increased age. Cognitive Level: Comprehension Text Reference: p. 744 Nursing Process: Assessment

A nurse is educating a community group about coronary artery disease. One member asks about how to avoid coronary artery disease. Which of the following items are considered modifiable risk factors for coronary artery disease? Choose all that apply. a) Gender b) Race c) Hyperlipidemia d) Tobacco use e) Obesity

C, D, and E

The nurse is educating a patient diagnosed with angina pectoris about the difference between the pain of angina and a myocardial infarction (MI). How should the nurse describe the pain experienced during an MI? (Select all that apply.) a) It is relieved by rest and inactivity. b) It subsides after taking nitroglycerin. c) It is substernal in location. d) It is sudden in onset and prolonged in duration. e) It is viselike and radiates to the shoulders and arms.

C, D, and E

Integrated Process: Nursing Process (Assessment) 5. Which symptoms in chronic lung disease require nursing intervention? (Select all that apply.) a. Clubbed fingers b. Increased residual volume c. Decreased peak flow d. Increased anterior-posterior diameter e. Elevated platelets f. Expiratory wheezing g. Stridor h. Change in sputum color and amount

C, F, G, H Decreased peak flow could indicate worsening of symptoms of airflow occlusion. Likewise, expiratory wheezing and stridor can indicate inflammation and fluid accumulation leading to airway occlusion. A change in the amount and color of sputum can indicate infection. The other symptoms normally occur with chronic disease. DIF: Cognitive Level: Application/Applying or higher REF: N/A TOP: Client Needs Category: Physiological Integrity (Physiological Adaptation—Pathophysiology)

A nurse is caring for a client who is exhibiting signs and symptoms characteristic of a myocardial infarction (MI). Which statement describes priorities the nurse should establish while performing the physical assessment? a) Prepare the client for pulmonary artery catheterization. b) Ensure that the client's family is kept informed of his status. c) Assess the client's level of pain and administer prescribed analgesics. d) Assess the client's level of anxiety and provide emotional support.

C. Assess the client's level of pain and administer prescribed analgesics

The nurse is caring for a client with Raynaud's disease. What is an important instruction for a client who is diagnosed with this disease to prevent an attack? a) Take over-the-counter decongestants. b) Report changes in the usual pattern of chest pain. c) Avoid situations that contribute to ischemic episodes. d) Avoid fatty foods and exercise.

C. Avoid situations that contribute to ischemic episodes

A 56-year-old male client with CAD kept his appointment to see a cardiologist for posthospitalization follow up. The client indicates that he thinks the diltiazem (Cardizem) has been making him nauseous. The nurse is aware that diltiazem (Cardizem) is categorized as which type of drug? a) Diuretics b) Beta-adrenergic blockers c) Calcium-channel blockers d) Nitrates

C. Calcium-channel blockers

A middle-aged male presents to the ED complaining of severe chest discomfort. Which of the following patient findings is most indicative of a possible MI? a) Cool, clammy, diaphoretic, and pale appearance b) Anxiousness, restlessness, and lightheadedness c) Chest discomfort not relieved by rest or nitroglycerin d) Intermittent nausea and emesis for 3 days

C. Chest discomfort not relieve by rest or nitroglycerin

The nurse is reviewing the laboratory results for a patient having a suspected myocardial infarction (MI). What cardiac-specific isoenzyme does the nurse observe for myocardial cell damage? a) Alkaline phosphatase b) Troponin c) Creatine kinase MB d) Myoglobin

C. Creatine kinase MB

A patient presents to the ED complaining of anxiety and chest pain after shoveling heavy snow that morning. The patient says that he has not taken nitroglycerin for months but did take three nitroglycerin tablets and although the pain is less, "They did not work all that well. " The patient shows the nurse the nitroglycerin bottle and the prescription was filled 12 months ago. The nurse anticipates which of the following physician orders? a) Ativan 1 mg orally b) Serum electrolytes c) Nitroglycerin SL d) Chest x-ray

C. Nitroglycerin SL

A patient presents to the emergency room with characteristics of atherosclerosis. What characteristics would the patient display? a) Emboli in the veins b) Cholesterol plugs in the lumen of veins c) Fatty deposits in the lumen of arteries d) Blood clots in the arteries

C. fatty deposits in the lumen of arteries

A new surgical patient who has undergone a coronary artery bypass graft (CABG) is receiving opioids for pain control. The nurse must be alert to adverse effects of opioids. Which of the following effects would be important for the nurse to document? a) Hypertension b) Hyperactive bowel sounds c) Hypotension d) Urinary incontinence

C. hypotension

Dilated Cardiomyopathy

Can cause mixed systolic and diastolic HF -poor systolic function worsened by dilated ventricle walls that are unable to relax -extremely low EF -high pulmonary pressure -biventricular failure

Angina Pectoris

Chest pain due to lack of oxygen to heart. Diabetes have high risk. Give nitrogylcerine up to 3 times every 5 minutes. Protect pills from light in brown bottle and throw away old ones every 6 months.

hypomagnesemia

Chronic alcoholism commonly causes this: (hypermagnesemia, hypomagnesemia)

Dyspnea (symptom of HF)

Common manifestation Caused by increased pulmonary pressure secondary to interstitial and alveolar edema. Mild exertion, at rest Adaptive behaviors, sleeping with more pillows

Edema (symptom of HF)

Common sign, pitting edema, weight gain more than 3 lb in 2 days -Dependent body parts (peripheral edema) -Liver (hepatomegaly) -Abdominal cavity (acites) -Lungs (Pulmonary edema and effusion)

Cardiac decompensation

Compensatory mechanisms can no longer maintain adequate CO, inadequate tissue perfusion

Cardiac compensation

Compensatory mechanisms succeed and maintain adequate CO, perfuse tissues

23. A nurse is preparing to perform an admission assessment on a patient with COPD. It is most important for the nurse to review which of the following? A) Social work assessment B) Insurance coverage C) Chloride levels D) Available diagnostic tests

D Feedback: In addition to the patient's history, the nurse reviews the results of available diagnostic tests. Social work assessment is not a priority for the majority of patients. Chloride levels are relevant to CF, not COPD. Insurance coverage is not normally the domain of the nurse.

Pericarditis

Inflammation of pericardium-membrane sac around the heart. S/S: chest pain, fever, orthpnea, breathing difficulties. Monitor vital signs and heart sounds. Place on prophylactic antibiotics for dental work.

The nurse administers IV nitroglycerin to a patient with an MI. In evaluating the effect of this intervention, the nurse should monitor for a.relief of chest discomfort. b.a decreased heart rate. c.an increase in BP. d.fewer cardiac dysrhythmias.

Correct Answer: A Rationale: The goal of IV nitroglycerin administration in AMI is relief of chest pain by improving the balance between myocardial oxygen supply and demand. Increases in heart rate and a drop in BP are common side effects of nitroglycerin. Nitroglycerin does not directly impact cardiac dysrhythmias.

f

Match the following characteristics with their methods of oxygen administration: May cause aspiration of condensed fluid A. Nasal cannula B. Simple face mask C. Partial rebreathing mask D. Nonrebreathing mask E. Venturi mask F. Tracheostomy collar

While admitting a patient with an AMI, which action should the nurse carry out first? a.Assess peripheral pulses. b.Check the oxygen saturation. c.Attach the cardiac monitor. d.Obtain the BP.

Correct Answer: C Rationale: Because dysrhythmias are the most common complication of MI, the first action should be to place the patient on a cardiac monitor. The other actions are also important and should be accomplished as quickly as possible.

Which information given by a patient admitted with chronic stable angina will help the nurse confirm this diagnosis? a.The patient rates the pain at a level 3 to 5 (0-10 scale). b.The patient states that the pain "wakes me up at night." c.The patient indicates that the pain is resolved after taking one sublingual nitroglycerin tablet. d.The patient says that the frequency of the pain has increased over the last few weeks.

Correct Answer: C Rationale: Chronic stable angina is typically relieved by rest or nitroglycerin administration. The level of pain is not a consistent indicator of the type of angina. Pain occurring at rest or with increased frequency is typical of unstable angina.

When developing a health teaching plan for a 65-year-old patient with all these risk factors for coronary artery disease (CAD), the nurse will focus on the a.family history of heart disease. b.increased risk associated with the patient's ethnicity. c.high incidence of cardiovascular disease in older people. d.low activity level the patient reports.

Correct Answer: D Rationale: Because family history, ethnicity, and age are nonmodifiable risk factors, the nurse should focus on the patient's activity level. An increase in activity will help reduce the patient's risk for developing CAD.

14. A nursing is planning the care of a patient with emphysema who will soon be discharged. What teaching should the nurse prioritize in the plan of care? A) Taking prophylactic antibiotics as ordered B) Adhering to the treatment regimen in order to cure the disease C) Avoiding airplanes, buses, and other crowded public places D) Setting realistic short-term and long-range goals

D Feedback: A major area of teaching involves setting and accepting realistic short-term and long-range goals. Emphysema is not considered curable and antibiotics are not used on a preventative basis. The patient does not normally need to avoid public places.

a

Identify at least one observation made by the nurse that would indicate the presence of the following complications of aortic aneurysm repair. Graft thrombosis A. a decreased or absent pulses in conjunction with cool, painful extremities below the level of repair B. cardiac dysrhythmias, chest pain C. absent bowel sounds, abdominal distention, diarrhea, bloody stools D. increased temperature and WBC; surgical sit inflammation or drainage

The nurse is admitting a patient who is complaining of chest pain to the emergency department (ED). Which information collected by the nurse suggests that the pain is caused by an acute myocardial infarction (AMI)? a.The pain worsens when the patient raises the arms. b.The pain increases with deep breathing. c.The pain is relieved after the patient takes nitroglycerin. d.The pain has persisted longer than 30 minutes.

Correct Answer: D Rationale: Chest pain that lasts for 20 minutes or more is characteristic of AMI. Changes in pain that occur with raising the arms or with deep breathing are more typical of pericarditis or musculoskeletal pain. Stable angina is usually relieved when the patient takes nitroglycerin.

A 37-year-old forklift operator presents with shakiness, sweating, anxiety, and palpitations and tells the nurse he has type 1 diabetes mellitus. Which of the follow actions should the nurse do first? A. Inject 1 mg of glucagon subcutaneously. B. Administer 50 mL of 50% glucose I.V. C. Give 4 to 6 oz (118 to 177 mL) of orange juice. D. Give the client four to six glucose tablets.

Correct answer: C Because the client is awake and complaining of symptoms, the nurse should first give him 15 grams of carbohydrate to treat hypoglycemia. This could be 4 to 6 oz of fruit juice, five to six hard candies such as Lifesavers, or 1 tablespoon of sugar. When a client has worsening symptoms of hypoglycemia or is unconscious, treatment includes 1 mg of glucagon subcutaneously or intramuscularly, or 50 mL of 50% glucose I.V. The nurse may also give two to three glucose tablets for a hypoglycemic reaction.

20. The nurse is caring for a patient who is postoperative day 2 following a total laryngectomy for supraglottic cancer. The nurse should prioritize what assessment? A) Assessment of body image B) Assessment of jugular venous pressure C) Assessment of carotid pulse D) Assessment of swallowing ability

D Feedback: A common postoperative complication from this type of surgery is difficulty in swallowing, which creates a potential for aspiration. Cardiovascular complications are less likely at this stage of recovery. The patient's body image should be assessed, but dysphagia has the potential to affect the patient's airway, and is a consequent priority.

35. The home care nurse is planning to begin breathing retraining exercises with a client newly admitted to the home health service. The home care nurse knows that breathing retraining is especially indicated if the patient has what diagnosis? A) Asthma B) Pneumonia C) Lung cancer D) COPD

D Feedback: Breathing retraining is especially indicated in patients with COPD and dyspnea. Breathing retraining may be indicated in patients with other lung pathologies, but not to the extent indicated in patients with COPD.

9. The campus nurse at a university is assessing a 21-year-old student who presents with a severe nosebleed. The site of bleeding appears to be the anterior portion of the nasal septum. The nurse instructs the student to tilt her head forward and the nurse applies pressure to the nose, but the student's nose continues to bleed. Which intervention should the nurse next implement? A) Apply ice to the bridge of her nose B) Lay the patient down on a cot C) Arrange for transfer to the local ED D) Insert a tampon in the affected nare

D Feedback: A cotton tampon may be used to try to stop the bleeding. The use of ice on the bridge of the nose has no scientific rationale for care. Laying the client down on the cot could block the client's airway. Hospital admission is necessary only if the bleeding becomes serious.

33. A nurse is providing health education to the family of a patient with bronchiectasis. What should the nurse teach the patient's family members? A) The correct technique for chest palpation and auscultation B) Techniques for assessing the patient's fluid balance C) The technique for providing deep nasotracheal suctioning D) The correct technique for providing postural drainage

D Feedback: A focus of the care of bronchiectasis is helping patients clear pulmonary secretions; consequently, patients and families are taught to perform postural drainage. Chest palpation and auscultation and assessment of fluid balance are not prioritized over postural drainage. Nasotracheal suctioning is not normally necessary.

Which statement by the patient with type 2 diabetes is accurate. a. "I am supposed to have a meal or snack if I drink alcohol" b. "I am not allowed to eat any sweets because of my diabetes." c. "I do not need to watch what I eat because my diabetes is not the bad kind." d. "The amount of fat in my diet is not important; it is just the carbohydrates that raise my blood sugar."

Correct Answer: A Alcohol should be consumed with food to reduce the risk of hypoglycemia.

a, d

Cromolyn inhaler is associated with A. Long-term control B. Quick-relief C. B2-adrenergic agonist D. Mast-cell stabilizer E. Leukotriene inhibitor F. Steroid antiinflammatory G. Methylxanthine bronchodilator H. Anticholinergic I. Anti-IgE

A patient with type 1 diabetes has received diet instruction as part of the treatment plan. The nurse determines a need for additional instruction when the patient says, a. "I may have an occasional alcoholic drink if I include it in my meal plan." b. "I will need a bedtime snack because I take an evening dose of NPH insulin." c. "I will eat meals as scheduled, even if I am not hungry, to prevent hypoglycemia." d. "I may eat whatever I want, as long as I use enough insulin to cover the calories."

D Rationale: Most patients with type 1 diabetes need to plan diet choices very carefully. Patients who are using intensified insulin therapy have considerable flexibility in diet choices but still should restrict dietary intake of items such as fat, protein, and alcohol. The other patient statements are correct and indicate good understanding of the diet instruction.

a

Match the following characteristics with their methods of oxygen administration: Most comfortable and causes the least restriction on activities A. Nasal cannula B. Simple face mask C. Partial rebreathing mask D. Nonrebreathing mask E. Venturi mask F. Tracheostomy collar

15. A nurse is documenting the results of assessment of a patient with bronchiectasis. What would the nurse most likely include in documentation? A) Sudden onset of pleuritic chest pain B) Wheezes on auscultation C) Increased anterior-posterior (A-P) diameter D) Clubbing of the fingers

D Feedback: Characteristic symptoms of bronchiectasis include chronic cough and production of purulent sputum in copious amounts. Clubbing of the fingers also is common because of respiratory insufficiency. Sudden pleuritic chest pain is a common manifestation of a pulmonary embolism. Wheezes on auscultation are common in patients with asthma. An increased A-P diameter is noted in patients with COPD.

6. A nurse is educating a patient in anticipation of a procedure that will require a water-sealed chest drainage system. What should the nurse tell the patient and the family that this drainage system is used for? A) Maintaining positive chest-wall pressure B) Monitoring pleural fluid osmolarity C) Providing positive intrathoracic pressure D) Removing excess air and fluid

D Feedback: Chest tubes and closed drainage systems are used to re-expand the lung involved and to remove excess air, fluid, and blood. They are not used to maintain positive chest-wall pressure, monitor pleural fluid, or provide positive intrathoracic pressure.

26. The nurse is performing patient education for a patient who is being discharged on mini-nebulizer treatments. What information should the nurse prioritize in the patient's discharge teaching? A) How to count her respirations accurately B) How to collect serial sputum samples C) How to independently wean herself from treatment D) How to perform diaphragmatic breathing

D Feedback: Diaphragmatic breathing is a helpful technique to prepare for proper use of the small-volume nebulizer. Patient teaching would not include counting respirations and the patient should not wean herself from treatment without the involvement of her primary care provider. Serial sputum samples are not normally necessary.

40. The nurse is assessing a patient who has a chest tube in place for the treatment of a pneumothorax. The nurse observes that the water level in the water seal rises and falls in rhythm with the patient's respirations. How should the nurse best respond to this assessment finding? A) Gently reinsert the chest tube 1 to 2 cm and observe if the water level stabilizes. B) Inform the physician promptly that there is in imminent leak in the drainage system. C) Encourage the patient to do deep breathing and coughing exercises. D) Document that the chest drainage system is operating as it is intended.

D Feedback: Fluctuation of the water level in the water seal shows effective connection between the pleural cavity and the drainage chamber and indicates that the drainage system remains patent. No further action is needed.

2. A patient visiting the clinic is diagnosed with acute sinusitis. To promote sinus drainage, the nurse should instruct the patient to perform which of the following? A) Apply a cold pack to the affected area. B) Apply a mustard poultice to the forehead. C) Perform postural drainage. D) Increase fluid intake.

D Feedback: For a patient diagnosed with acute sinusitis, the nurse should instruct the patient that hot packs, increasing fluid intake, and elevating the head of the bed can promote drainage. Applying a mustard poultice will not promote sinus drainage. Postural drainage is used to remove bronchial secretions.

1. The nurse is providing patient teaching to a young mother who has brought her 3-month-old infant to the clinic for a well-baby checkup. What action should the nurse recommend to the woman to prevent the transmission of organisms to her infant during the cold season? A) Take preventative antibiotics, as ordered. B) Gargle with warm salt water regularly. C) Dress herself and her infant warmly. D) Wash her hands frequently.

D Feedback: Handwashing remains the most effective preventive measure to reduce the transmission of organisms. Taking prescribed antibiotics, using warm salt-water gargles, and dressing warmly do not suppress transmission. Antibiotics are not prescribed for a cold.

4. A patient comes to the ED and is admitted with epistaxis. Pressure has been applied to the patient's midline septum for 10 minutes, but the bleeding continues. The nurse should anticipate using what treatment to control the bleeding? A) Irrigation with a hypertonic solution B) Nasopharyngeal suction C) Normal saline application D) Silver nitrate application

D Feedback: If pressure to the midline septum does not stop the bleeding for epistaxis, additional treatment of silver nitrate application, Gelfoam, electrocautery, or vasoconstrictors may be used. Suction may be used to visualize the nasal septum, but it does not alleviate the bleeding. Irrigation with a hypertonic solution is not used to treat epistaxis.

12. A patient recovering from thoracic surgery is on long-term mechanical ventilation and becomes very frustrated when he tries to communicate. What intervention should the nurse perform to assist the patient? A) Assure the patient that everything will be all right and that remaining calm is the best strategy. B) Ask a family member to interpret what the patient is trying to communicate. C) Ask the physician to wean the patient off the mechanical ventilator to allow the patient to speak freely. D) Express empathy and then encourage the patient to write, use a picture board, or spell words with an alphabet board.

D Feedback: If the patient uses an alternative method of communication, he will feel in better control and likely be less frustrated. Assuring the patient that everything will be all right offers false reassurance, and telling him not to be upset minimizes his feelings. Neither of these methods helps the patient to communicate. In a patient with an endotracheal or tracheostomy tube, the family members are also likely to encounter difficulty interpreting the patient's wishes. Making them responsible for interpreting the patient's gestures may frustrate the family. The patient may be weaned off a mechanical ventilator only when the physiologic parameters for weaning have been met.

11. A 42-year-old patient is admitted to the ED after an assault. The patient received blunt trauma to the face and has a suspected nasal fracture. Which of the following interventions should the nurse perform? A) Administer nasal spray and apply an occlusive dressing to the patient's face. B) Position the patient's head in a dependent position. C) Irrigate the patient's nose with warm tap water. D) Apply ice and keep the patient's head elevated.

D Feedback: Immediately after the fracture, the nurse applies ice and encourages the patient to keep the head elevated. The nurse instructs the patient to apply ice packs to the nose to decrease swelling. Dependent positioning would exacerbate bleeding and the nose is not irrigated. Occlusive dressings are not used.

d

Match the following characteristics with their methods of oxygen administration: Provides the highest oxygen concentrations A. Nasal cannula B. Simple face mask C. Partial rebreathing mask D. Nonrebreathing mask E. Venturi mask F. Tracheostomy collar

40. A patient's total laryngectomy has created a need for alaryngeal speech which will be achieved through the use of tracheoesophageal puncture. What action should the nurse describe to the patient when teaching him about this process? A) Training on how to perform controlled belching B) Use of an electronically enhanced artificial pharynx C) Insertion of a specialized nasogastric tube D) Fitting for a voice prosthesis

D Feedback: In patients receiving transesophageal puncture, a valve is placed in the tracheal stoma to divert air into the esophagus and out the mouth. Once the puncture is surgically created and has healed, a voice prosthesis (Blom-Singer®) is fitted over the puncture site. A nasogastric tube and belching are not required. An artificial pharynx is not used.

8. A patient's plan of care specifies postural drainage. What action should the nurse perform when providing this noninvasive therapy? A) Administer the treatment with the patient in a high Fowler's or semi-Fowler's position. B) Perform the procedure immediately following the patient's meals. C) Apply percussion firmly to bare skin to facilitate drainage. D) Assist the patient into a position that will allow gravity to move secretions.

D Feedback: Postural drainage is usually performed two to four times per day. The patient uses gravity to facilitate postural draining. The skin should be covered with a cloth or a towel during percussion to protect the skin. Postural drainage is not administered in an upright position or directly following a meal.

37. A nurse is explaining to a patient with asthma what her new prescription for prednisone is used for. What would be the most accurate explanation that the nurse could give? A) To ensure long-term prevention of asthma exacerbations B) To cure any systemic infection underlying asthma attacks C) To prevent recurrent pulmonary infections D) To gain prompt control of inadequately controlled, persistent asthma

D Feedback: Prednisone is used for a short-term (3-10 days) "burst" to gain prompt control of inadequately controlled, persistent asthma. It is not used to treat infection or to prevent exacerbations in the long term.

16. A patient is having pulmonary-function studies performed. The patient performs a spirometry test, revealing an FEV1/FVC ratio of 60%. How should the nurse interpret this assessment finding? A) Strong exercise tolerance B) Exhalation volume is normal C) Respiratory infection D) Obstructive lung disease

D Feedback: Spirometry is used to evaluate airflow obstruction, which is determined by the ratio of forced expiration volume in 1 second to forced vital capacity. Obstructive lung disease is apparent when an FEV1/FVC ratio is less than 70%.

4. The medical nurse is creating the care plan of an adult patient requiring mechanical ventilation. What nursing action is most appropriate? A) Keep the patient in a low Fowler's position. B) Perform tracheostomy care at least once per day. C) Maintain continuous bedrest. D) Monitor cuff pressure every 8 hours.

D Feedback: The cuff pressure should be monitored every 8 hours. It is important to perform tracheostomy care at least every 8 hours because of the risk of infection. The patient should be encouraged to ambulate, if possible, and a low Fowler's position is not indicated.

1. A clinic nurse is caring for a patient who has just been diagnosed with chronic obstructive pulmonary disease (COPD). The patient asks the nurse what he could have done to minimize the risk of contracting this disease. What would be the nurse's best answer? A) "The most important risk factor for COPD is exposure to occupational toxins." B) "The most important risk factor for COPD is inadequate exercise." C) "The most important risk factor for COPD is exposure to dust and pollen." D) "The most important risk factor for COPD is cigarette smoking."

D Feedback: The most important risk factor for COPD is cigarette smoking. Lack of exercise and exposure to dust and pollen are not risk factors for COPD. Occupational risks are significant but are far exceeded by smoking.

31. The home care nurse is visiting a patient newly discharged home after a lobectomy. What would be most important for the home care nurse to assess? A) Resumption of the patient's ADLs B) The family's willingness to care for the patient C) Nutritional status and fluid balance D) Signs and symptoms of respiratory complications

D Feedback: The nurse assesses the patient's adherence to the postoperative treatment plan and identifies acute or late postoperative complications. All options presented need assessment, but respiratory complications are the highest priority because they affect the patient's airway and breathing.

18. The nurse is discussing activity management with a patient who is postoperative following thoracotomy. What instructions should the nurse give to the patient regarding activity immediately following discharge? A) Walk 1 mile 3 to 4 times a week. B) Use weights daily to increase arm strength. C) Walk on a treadmill 30 minutes daily. D) Perform shoulder exercises five times daily.

D Feedback: The nurse emphasizes the importance of progressively increased activity. The nurse also instructs the patient on the importance of performing shoulder exercises five times daily. The patient should ambulate with limits and realize that the return of strength will likely be gradual and likely will not include weight lifting or lengthy walks.

15. As a clinic nurse, you are caring for a patient who has been prescribed an antibiotic for tonsillitis and has been instructed to take the antibiotic for 10 days. When you do a follow-up call with this patient, you are informed that the patient is feeling better and is stopping the medication after taking it for 4 days. What information should you provide to this patient? A) Keep the remaining tablets for an infection at a later time. B) Discontinue the medications if the fever is gone. C) Dispose of the remaining medication in a biohazard receptacle. D) Finish all the antibiotics to eliminate the organism completely.

D Feedback: The nurse informs the patient about the need to take the full course of any prescribed antibiotic. Antibiotics should be taken for the entire 10-day period to eliminate the microorganisms. A patient should never be instructed to keep leftover antibiotics for use at a later time. Even if the fever or other symptoms are gone, the medications should be continued. Antibiotics do not need to be disposed of in a biohazard receptacle, though they should be discarded appropriately.

28. The decision has been made to discharge a ventilator-dependent patient home. The nurse is developing a teaching plan for this patient and his family. What would be most important to include in this teaching plan? A) Administration of inhaled corticosteroids B) Assessment of neurologic status C) Turning and coughing D) Signs of pulmonary infection

D Feedback: The nurse teaches the patient and family about the ventilator, suctioning, tracheostomy care, signs of pulmonary infection, cuff inflation and deflation, and assessment of vital signs. Neurologic assessment and turning and coughing are less important than signs and symptoms of infection. Inhaled corticosteroids may or may not be prescribed.

39. A nurse is teaching a patient how to perform flow type incentive spirometry prior to his scheduled thoracic surgery. What instruction should the nurse provide to the patient? A) "Hold the spirometer at your lips and breathe in and out like you normally would." B) "When you're ready, blow hard into the spirometer for as long as you can." C) "Take a deep breath and then blow short, forceful breaths into the spirometer." D) "Breathe in deeply through the spirometer, hold your breath briefly, and then exhale."

D Feedback: The patient should be taught to lace the mouthpiece of the spirometer firmly in the mouth, breathe air in through the mouth, and hold the breath at the end of inspiration for about 3 seconds. The patient should then exhale slowly through the mouthpiece.

14. The nurse is explaining the safe and effective administration of nasal spray to a patient with seasonal allergies. What information is most important to include in this teaching? A) Finish the bottle of nasal spray to clear the infection effectively. B) Nasal spray can only be shared between immediate family members. C) Nasal spray should be administered in a prone position. D) Overuse of nasal spray may cause rebound congestion.

D Feedback: The use of topical decongestants is controversial because of the potential for a rebound effect. The patient should hold his or her head back for maximal distribution of the spray. Only the patient should use the bottle.

2. The nurse caring for a patient with an endotracheal tube recognizes several disadvantages of an endotracheal tube. What would the nurse recognize as a disadvantage of endotracheal tubes? A) Cognition is decreased. B) Daily arterial blood gases (ABGs) are necessary. C) Slight tracheal bleeding is anticipated. D) The cough reflex is depressed.

D Feedback: There are several disadvantages of an endotracheal tube. Disadvantages include suppression of the patient's cough reflex, thickening of secretions, and depressed swallowing reflexes. Ulceration and stricture of the larynx or trachea may develop, but bleeding is not an expected finding. The tube should not influence cognition and daily ABGs are not always required.

27. The case manager for a group of patients with COPD is providing health education. What is most important for the nurse to assess when providing instructions on self-management to these patients? A) Knowledge of alternative treatment modalities B) Family awareness of functional ability and activities of daily living (ADLs) C) Knowledge of the pathophysiology of the disease process D) Knowledge about self-care and their therapeutic regimen

D Feedback: When providing instructions about self-management, it is important for the nurse to assess the knowledge of patients and family members about self-care and the therapeutic regimen. This supersedes knowledge of alternative treatments or the pathophysiology of the disease, neither of which is absolutely necessary for patients to know. The patient's own knowledge is more important than that of the family.

Integrated Process: Nursing Process (Implementation) 2. A client with asthma has been having frequent asthma attacks. What is the nurse's best action? a. Teach the client to stay away from pets. b. Assist the client in using an incentive spirometer. c. Administer aspirin for its anti-inflammatory properties. d. Administer montelukast (Singulair).

D A client who has been having increased attacks can have some chronic inflammation occurring. This inflammation is probably stimulated by mediators such as histamine and leukotriene and can be blocked by drugs like diphenhydramine (Benadryl) and montelukast (Singulair). DIF: Cognitive Level: Application/Applying or higher REF: N/A TOP: Client Needs Category: Physiological Integrity (Pharmacological and Parenteral Therapies—Expected Actions/Outcomes)

Integrated Process: Nursing Process (Assessment) 35. A client was diagnosed with lung cancer and appears distressed. The client states, "I am so afraid." What is the best action for the nurse to take? a. Provide comfort by holding the client's hand. b. Offer to give the client a back rub for relaxation. c. Offer the client a PRN antianxiety medication. d. Ask the client what is causing the most fear right now.

D A diagnosis of lung cancer often causes fear for many reasons, usually poor prognosis, fear of pain, and fear of dyspnea. The nurse should assess what is worrying the client most at the moment so appropriate interventions can be planned. Touch is often a powerful tool, but the nurse should assess whether this is acceptable to the client. The nurse should assess the client further and provide assistance with coping before offering to medicate him. DIF: Cognitive Level: Application/Applying or higher REF: N/A TOP: Client Needs Category: Psychosocial Integrity (Stress Management)

Physiological Integrity 2. The nurse teaches a patient how to administer formoterol (Perforomist) through a nebulizer. Which action by the patient indicates good understanding of the teaching? a. The patient attaches a spacer before using the inhaler. b. The patient coughs vigorously after using the inhaler. c. The patient activates the inhaler at the onset of expiration. d. The patient removes the facial mask when misting has ceased.

D A nebulizer is used to administer aerosolized medication. A mist is seen when the medication is aerosolized, and when all of the medication has been used, the misting stops. The other options refer to inhaler use. Coughing vigorously after inhaling and activating the inhaler at the onset of expiration are both incorrect techniques when using an inhaler. DIF: Cognitive Level: Apply (application) REF: 574 TOP: Nursing Process: Evaluation MSC:

Physiological Integrity 11. Following an acute myocardial infarction, a previously healthy 63-year-old develops clinical manifestations of heart failure. The nurse anticipates discharge teaching will include information about a. digitalis preparations. b. -adrenergic blockers. c. calcium channel blockers. d. angiotensin-converting enzyme (ACE) inhibitors.

D ACE inhibitor therapy is currently recommended to prevent the development of heart failure in patients who have had a myocardial infarction and as a first-line therapy for patients with chronic heart failure. Digoxin therapy for heart failure is no longer considered a first-line measure, and digoxin is added to the treatment protocol when therapy with other medications such as ACE-inhibitors, diuretics, and -adrenergic blockers is insufficient. Calcium channel blockers are not generally used in the treatment of heart failure. The -adrenergic blockers are not used as initial therapy for new onset heart failure. DIF: Cognitive Level: Apply (application) REF: 776 TOP: Nursing Process: Planning MSC:

Physiological Integrity 17. In reviewing the results of a patient's pulmonary function test, the nurse recognizes that a patient with COPD is likely to have an increased a. forced vital capacity. b. peak expiratory flow. c. tidal volume. d. residual volume.

D Rationale: Because elastic recoil of the lungs is decreased with COPD, the residual volume is increased. Tidal volume, forced vital capacity, and peak expiratory flow rate are likely to be decreased. Cognitive Level: Comprehension Text Reference: p. 531 Nursing Process: Assessment

Integrated Process: Nursing Process (Implementation) 38. A client has recently been placed on prednisone (Deltasone). What is the highest priority instruction the nurse will provide? a. "Expect to experience weight gain." b. "Watch your diet while on this medication." c. "Take the drug with food or milk." d. "Report any abdominal pain or dark-colored vomit."

D All of these directions are appropriate to give the client; however, telling the client to report abdominal pain and dark-colored vomit is most important because these could signal gastric ulceration. DIF: Cognitive Level: Application/Applying or higher REF: N/A TOP: Client Needs Category: Physiological Integrity (Pharmacological and Parenteral Therapies—Adverse Effects/Contraindications/Interactions/Side Effects)

Physiological Integrity 32. A patient admitted to the coronary care unit (CCU) with an ST-segment-elevation myocardial infarction (STEMI) is restless and anxious. The blood pressure is 86/40 and heart rate is 123. Based on this information, which nursing diagnosis is a priority for the patient? a. Acute pain related to myocardial infarction b. Anxiety related to perceived threat of death c. Stress overload related to acute change in health d. Decreased cardiac output related to cardiogenic shock

D All the nursing diagnoses may be appropriate for this patient, but the hypotension and tachycardia indicate decreased cardiac output and shock from the damaged myocardium. This will result in decreased perfusion to all vital organs (e.g., brain, kidney, heart) and is a priority. DIF: Cognitive Level: Apply (application) REF: 746-747 OBJ: Special Questions: Prioritization TOP: Nursing Process: Diagnosis MSC:

Physiological Integrity 15. The nurse analyzes the results of a patient's arterial blood gases (ABGs). Which finding would require immediate action? a. The bicarbonate level (HCO3-) is 31 mEq/L. b. The arterial oxygen saturation (SaO2) is 92%. c. The partial pressure of CO2 in arterial blood (PaCO2) is 31 mm Hg. d. The partial pressure of oxygen in arterial blood (PaO2) is 59 mm Hg.

D All the values are abnormal, but the low PaO2 indicates that the patient is at the point on the oxyhemoglobin dissociation curve where a small change in the PaO2 will cause a large drop in the O2 saturation and a decrease in tissue oxygenation. The nurse should intervene immediately to improve the patient's oxygenation. DIF: Cognitive Level: Apply (application) REF: eTable 26-1 OBJ: Special Questions: Prioritization TOP: Nursing Process: Assessment MSC:

Physiological Integrity 34. A patient with pneumonia has a fever of 101.4° F (38.6° C), a nonproductive cough, and an oxygen saturation of 88%. The patient complains of weakness, fatigue, and needs assistance to get out of bed. Which nursing diagnosis should the nurse assign as the highest priority? a. Hyperthermia related to infectious illness b. Impaired transfer ability related to weakness c. Ineffective airway clearance related to thick secretions d. Impaired gas exchange related to respiratory congestion

D All these nursing diagnoses are appropriate for the patient, but the patient's oxygen saturation indicates that all body tissues are at risk for hypoxia unless the gas exchange is improved. DIF: Cognitive Level: Apply (application) REF: 527 OBJ: Special Questions: Prioritization TOP: Nursing Process: Diagnosis MSC:

Integrated Process: Nursing Process (Implementation) 33. Which is the highest priority problem for a client with late-stage lung cancer? a. Malnutrition b. Constipation c. Weakness and fatigue d. Pain

D Although all of these problems are important issues, effective pain management is the most important issue for this client and family. The nurse must serve as a client advocate and must ensure that all appropriate measures for management of intractable, severe pain are implemented. DIF: Cognitive Level: Application/Applying or higher REF: N/A TOP: Client Needs Category: Safe and Effective Care Environment (Management of Care—Illness Management)

Physiological Integrity 16. A patient who has just been admitted with pulmonary edema is scheduled to receive the following medications. Which medication should the nurse question before giving? a. Furosemide (Lasix) 60 mg b. Captopril (Capoten) 25 mg c. Digoxin (Lanoxin) 0.125 mg d. Carvedilol (Coreg) 3.125 mg

D Although carvedilol is appropriate for the treatment of chronic heart failure, it is not used for patients with acute decompensated heart failure (ADHF) because of the risk of worsening the heart failure. The other medications are appropriate for the patient with ADHF. DIF: Cognitive Level: Analyze (analysis) REF: 774 | 771 TOP: Nursing Process: Implementation MSC:

Physiological Integrity 6. Which statement made by a patient with coronary artery disease after the nurse has completed teaching about therapeutic lifestyle changes (TLC) diet indicates that further teaching is needed? a. "I will switch from whole milk to 1% milk." b. "I like salmon and I will plan to eat it more often." c. "I can have a glass of wine with dinner if I want one." d. "I will miss being able to eat peanut butter sandwiches."

D Although only 30% of the daily calories should come from fats, most of the fat in the TLC diet should come from monosaturated fats such as are found in nuts, olive oil, and canola oil. The patient can include peanut butter sandwiches as part of the TLC diet. The other patient comments indicate a good understanding of the TLC diet. DIF: Cognitive Level: Apply (application) REF: 738 TOP: Nursing Process: Evaluation MSC:

Integrated Process: Nursing Process (Implementation) 12. The nurse assesses a client receiving chemotherapy for lung cancer and notes red swollen mucous membranes and open sores in the mouth. The client reports mouth pain and difficulty swallowing. Which action does the nurse perform first? a. Document the size of the sores. b. Perform mouth hygiene. c. Have the client rinse his or her mouth. d. Call the health care provider and hold chemotherapy.

D Although the nurse should perform all interventions for mucositis, the priority is to call the health care provider and hold the chemotherapy. Mucositis may be a dose-limiting condition in chemotherapy. The nurse should call the provider, then should assist the client with mouth hygiene, rinsing the mouth, and obtaining pain relief. Documenting the size and location of ulcers is also important. DIF: Cognitive Level: Application/Applying or higher REF: N/A TOP: Client Needs Category: Physiological Integrity (Pharmacological and Parenteral Therapies—Adverse Effects/Contraindications/Interactions/Side Effects)

1. When developing a teaching plan for a 61-year-old man with the following risk factors for coronary artery disease (CAD), the nurse should focus on the a. family history of coronary artery disease. b. increased risk associated with the patient's gender. c. increased risk of cardiovascular disease as people age. d. elevation of the patient's low-density lipoprotein (LDL) level.

D Because family history, gender, and age are nonmodifiable risk factors, the nurse should focus on the patient's LDL level. Decreases in LDL will help reduce the patient's risk for developing CAD. DIF: Cognitive Level: Apply (application) REF: 736 TOP: Nursing Process: Planning MSC:

Physiological Integrity 20. A patient arrives in the ear, nose, and throat clinic complaining of a piece of tissue being "stuck up my nose" and with foul-smelling nasal drainage from the right nare. Which action should the nurse take first? a. Notify the clinic health care provider. b. Obtain aerobic culture specimens of the drainage. c. Ask the patient about how the cotton got into the nose. d. Have the patient occlude the left nare and blow the nose.

D Because the highest priority action is to remove the foreign object from the nare, the nurse's first action should be to assist the patient to remove the object. The other actions are also appropriate but should be done after attempting to clear the nose. DIF: Cognitive Level: Apply (application) REF: 506 OBJ: Special Questions: Prioritization TOP: Nursing Process: Implementation MSC:

Physiological Integrity 11. Nadolol (Corgard) is prescribed for a patient with chronic stable angina and left ventricular dysfunction. To determine whether the drug is effective, the nurse will monitor for a. decreased blood pressure and heart rate. b. fewer complaints of having cold hands and feet. c. improvement in the strength of the distal pulses. d. the ability to do daily activities without chest pain.

D Because the medication is ordered to improve the patient's angina, effectiveness is indicated if the patient is able to accomplish daily activities without chest pain. Blood pressure and heart rate may decrease, but these data do not indicate that the goal of decreased angina has been met. The noncardioselective -adrenergic blockers can cause peripheral vasoconstriction, so the nurse would not expect an improvement in distal pulse quality or skin temperature. DIF: Cognitive Level: Apply (application) REF: 753 TOP: Nursing Process: Evaluation MSC:

Physiological Integrity 2. The nurse is caring for a patient who is receiving IV furosemide (Lasix) and morphine for the treatment of acute decompensated heart failure (ADHF) with severe orthopnea. Which clinical finding is the best indicator that the treatment has been effective? a. Weight loss of 2 pounds in 24 hours b. Hourly urine output greater than 60 mL c. Reduction in patient complaints of chest pain d. Reduced dyspnea with the head of bed at 30 degrees

D Because the patient's major clinical manifestation of ADHF is orthopnea (caused by the presence of fluid in the alveoli), the best indicator that the medications are effective is a decrease in dyspnea with the head of the bed at 30 degrees. The other assessment data also may indicate that diuresis or improvement in cardiac output has occurred, but are not as specific to evaluating this patient's response. DIF: Cognitive Level: Apply (application) REF: 769-770 | 773-774 TOP: Nursing Process: Evaluation MSC:

Physiological Integrity 19. Postural drainage with percussion and vibration is ordered twice daily for a patient with chronic bronchitis. Which intervention should the nurse include in the plan of care? a. Schedule the procedure 1 hour after the patient eats. b. Maintain the patient in the lateral position for 20 minutes. c. Perform percussion before assisting the patient to the drainage position. d. Give the ordered albuterol (Proventil) before the patient receives the therapy.

D Bronchodilators are administered before chest physiotherapy. Postural drainage, percussion, and vibration should be done 1 hour before or 3 hours after meals. Patients remain in each postural drainage position for 5 minutes. Percussion is done while the patient is in the postural drainage position. DIF: Cognitive Level: Apply (application) REF: 594 TOP: Nursing Process: Planning MSC:

Physiological Integrity 4. Which information given by a patient admitted with chronic stable angina will help the nurse confirm this diagnosis? a. The patient states that the pain "wakes me up at night." b. The patient rates the pain at a level 3 to 5 (0 to 10 scale). c. The patient states that the pain has increased in frequency over the last week. d. The patient states that the pain "goes away" with one sublingual nitroglycerin tablet.

D Chronic stable angina is typically relieved by rest or nitroglycerin administration. The level of pain is not a consistent indicator of the type of angina. Pain occurring at rest or with increased frequency is typical of unstable angina. DIF: Cognitive Level: Understand (comprehension) REF: 742 TOP: Nursing Process: Assessment MSC:

Physiological Integrity 16. When assessing the respiratory system of an older patient, which finding indicates that the nurse should take immediate action? a. Weak cough effort b. Barrel-shaped chest c. Dry mucous membranes d. Bilateral crackles at lung bases

D Crackles in the lower half of the lungs indicate that the patient may have an acute problem such as heart failure. The nurse should immediately accomplish further assessments, such as oxygen saturation, and notify the health care provider. A barrel-shaped chest, hyperresonance to percussion, and a weak cough effort are associated with aging. Further evaluation may be needed, but immediate action is not indicated. An older patient has a less forceful cough and fewer and less functional cilia. Mucous membranes tend to be drier. DIF: Cognitive Level: Apply (application) REF: 489 OBJ: Special Questions: Prioritization TOP: Nursing Process: Assessment MSC:

Integrated Process: Teaching/Learning 28. The nurse is caring for a client with bronchiolitis obliterans organizing pneumonia (BOOP) and assesses decreased vital capacity during pulmonary function testing. What is the nurse's best action? a. Administer intermittent positive-pressure breathing treatments. b. Administer a short-acting beta-adrenergic medication. c. Prepare to administer IV antibiotics. d. Document the finding in the client's chart.

D Decreased vital capacity is a common finding with this disorder because the white blood cells clump and obliterate airways. The nurse should note the finding and should assist the client in activities that help him or her maintain quality of life. DIF: Cognitive Level: Application/Applying or higher REF: N/A TOP: Client Needs Category: Physiological Integrity (Physiological Adaptation—Pathophysiology)

1. A client with asthma reports "not being able to take deep breaths." The nurse auscultates decreased breath sounds in the bases, and no wheezes. What is the nurse's best action? a. Encourage the client to stay calm and take deep breaths. b. Document the findings and continue to monitor. c. Have the client cough forcefully. d. Assess the client's oxygen saturation.

D Decreased wheezing accompanied by decreased breath sounds can mean airway occlusion from mucus and from inflammation. The nurse should assess the client's oxygenation and determine whether additional interventions are needed. Coughing forcefully may cause the smaller airways to collapse and may not help the client. Encouraging the client to remain calm and to try to take deep breaths is not helpful. Although providing documentation is important, the nurse needs to do more than that. DIF: Cognitive Level: Application/Applying or higher REF: N/A TOP: Client Needs Category: Physiological Integrity (Reduction of Risk Potential—Potential for Complications from Surgical Procedures and Health Alterations)

Physiological Integrity 12. An alcoholic and homeless patient is diagnosed with active tuberculosis (TB). Which intervention by the nurse will be most effective in ensuring adherence with the treatment regimen? a. Arrange for a friend to administer the medication on schedule. b. Give the patient written instructions about how to take the medications. c. Teach the patient about the high risk for infecting others unless treatment is followed. d. Arrange for a daily noon meal at a community center where the drug will be administered.

D Directly observed therapy is the most effective means for ensuring compliance with the treatment regimen, and arranging a daily meal will help ensure that the patient is available to receive the medication. The other nursing interventions may be appropriate for some patients but are not likely to be as helpful for this patient. DIF: Cognitive Level: Apply (application) REF: 531 TOP: Nursing Process: Implementation MSC:

Physiological Integrity 17. When the nurse is monitoring a patient who is undergoing exercise (stress) testing on a treadmill, which assessment finding requires the most rapid action by the nurse? a. Patient complaint of feeling tired b. Pulse change from 87 to 101 beats/minute c. Blood pressure (BP) increase from 134/68 to 150/80 mm Hg d. Newly inverted T waves on the electrocardiogram

D ECG changes associated with coronary ischemia (such as T-wave inversions and ST segment depression) indicate that the myocardium is not getting adequate oxygen delivery and that the exercise test should be terminated immediately. Increases in BP and heart rate (HR) are normal responses to aerobic exercise. Feeling tired is also normal as the intensity of exercise increases during the stress testing. DIF: Cognitive Level: Apply (application) REF: 701 OBJ: Special Questions: Prioritization TOP: Nursing Process: Assessment MSC:

Physiological Integrity 32. A patient who has just been admitted with community-acquired pneumococcal pneumonia has a temperature of 101.6° F with a frequent cough and is complaining of severe pleuritic chest pain. Which prescribed medication should the nurse give first? a. Codeine b. Guaifenesin (Robitussin) c. Acetaminophen (Tylenol) d. Piperacillin/tazobactam (Zosyn)

D Early initiation of antibiotic therapy has been demonstrated to reduce mortality. The other medications are also appropriate and should be given as soon as possible, but the priority is to start antibiotic therapy. DIF: Cognitive Level: Apply (application) REF: 523 | 525 OBJ: Special Questions: Prioritization TOP: Nursing Process: Implementation MSC:

Physiological Integrity 20. The nurse develops a teaching plan to help increase activity tolerance at home for an older adult with severe chronic obstructive pulmonary disease (COPD). Which instructions would be most appropriate for the nurse to include in the plan of care? a. Stop exercising when short of breath. b. Walk until pulse rate exceeds 130 beats/minute. c. Limit exercise to activities of daily living (ADLs). d. Walk 15 to 20 minutes daily at least 3 times/week.

D Encourage the patient to walk 15 to 20 minutes a day at least three times a week with gradual increases. Shortness of breath is normal with exercise and not an indication that the patient should stop. Limiting exercise to ADLs will not improve the patient's exercise tolerance. A 70-year-old patient should have a pulse rate of 120 or less with exercise (80% of the maximal heart rate of 150). DIF: Cognitive Level: Apply (application) REF: 599 TOP: Nursing Process: Planning MSC:

Integrated Process: Nursing Process (Implementation) 26. The nurse observes hematuria in a client receiving IV cyclophosphamide (Cytoxan). After notifying the health care provider, what intervention is the nurse's priority? a. Obtain a urine specimen. b. Assess laboratory studies. c. Increase hydration. d. Stop the medication.

D Hemorrhagic cystitis is a frequent side effect of cyclophosphamide therapy. The physician should be notified to prescribe co-administration of a bladder-protecting agent. The nurse then should stop the medication. Other actions would be to further assess the client and provide hydration to flush the medication. DIF: Cognitive Level: Application/Applying or higher REF: N/A TOP: Client Needs Category: Physiological Integrity (Pharmacological and Parenteral Therapies—Adverse Effects/Contraindications/Interactions/Side Effects)

Safe and Effective Care Environment 11. A patient who has a history of chronic obstructive pulmonary disease (COPD) was hospitalized for increasing shortness of breath and chronic hypoxemia (SaO2 levels of 89% to 90%). In planning for discharge, which action by the nurse will be most effective in improving compliance with discharge teaching? a. Start giving the patient discharge teaching on the day of admission. b. Have the patient repeat the instructions immediately after teaching. c. Accomplish the patient teaching just before the scheduled discharge. d. Arrange for the patient's caregiver to be present during the teaching.

D Hypoxemia interferes with the patient's ability to learn and retain information, so having the patient's caregiver present will increase the likelihood that discharge instructions will be followed. Having the patient repeat the instructions will indicate that the information is understood at the time, but it does not guarantee retention of the information. Because the patient is likely to be distracted just before discharge, giving discharge instructions just before discharge is not ideal. The patient is likely to be anxious and even more hypoxemic than usual on the day of admission, so teaching about discharge should be postponed. DIF: Cognitive Level: Apply (application) REF: 484 TOP: Nursing Process: Planning MSC:

Integrated Process: Nursing Process (Assessment) 9. A client is using omalizumab (Xolair) for the first time. What is the priority nursing action? a. Make sure the client takes the medication with water. b. Administer ibuprofen (Motrin) because Xolair often causes headaches. c. Teach the client how to use a syringe. d. Remain with the client and assess for anaphylaxis.

D Immune modulators are monoclonal antibodies that prevent allergens from binding to receptor sites on mast cells and basophils. The risk of anaphylaxis is high; the nurse should assess and stay with the client. DIF: Cognitive Level: Application/Applying or higher REF: N/A TOP: Client Needs Category: Physiological Integrity (Pharmacological and Parenteral Therapies—Adverse Effects/Contraindications/Interactions/Side Effects)

Integrated Process: Teaching/Learning 11. A client is undergoing lung reduction surgery. What is the nurse's highest priority preoperatively? a. Administer medications. b. Discuss the possibility of ventilator dependency. c. Teach how to cough and deep breathe. d. Teach about preoperative testing.

D In addition to standard preoperative testing, the client who will undergo lung reduction surgery is tested to determine the location of greatest lung hyperinflation and poorest lung blood flow. These tests include pulmonary plethysmography, gas dilution, and perfusion scans. The other interventions are lower priorities. DIF: Cognitive Level: Application/Applying or higher REF: N/A TOP: Client Needs Category: Health Promotion and Maintenance (Principles of Teaching/Learning)

Health Promotion and Maintenance 5. To determine the effects of therapy for a patient who is being treated for heart failure, which laboratory result will the nurse plan to review? a. Troponin b. Homocysteine (Hcy) c. Low-density lipoprotein (LDL) d. B-type natriuretic peptide (BNP)

D Increased levels of BNP are a marker for heart failure. The other laboratory results would be used to assess for myocardial infarction (troponin) or risk for coronary artery disease (Hcy and LDL). DIF: Cognitive Level: Apply (application) REF: 698-699 TOP: Nursing Process: Evaluation MSC:

Physiological Integrity 36. A patient had a non-ST-segment-elevation myocardial infarction (NSTEMI) 3 days ago. Which nursing intervention included in the plan of care is most appropriate for the registered nurse (RN) to delegate to an experienced licensed practical/vocational nurse (LPN/LVN)? a. Evaluation of the patient's response to walking in the hallway b. Completion of the referral form for a home health nurse follow-up c. Education of the patient about the pathophysiology of heart disease d. Reinforcement of teaching about the purpose of prescribed medications

D LPN/LVN education and scope of practice include reinforcing education that has previously been done by the RN. Evaluating the patient response to exercise after a NSTEMI requires more education and should be done by the RN. Teaching and discharge planning/ documentation are higher level skills that require RN education and scope of practice. DIF: Cognitive Level: Apply (application) REF: 15-16 OBJ: Special Questions: Delegation TOP: Nursing Process: Planning MSC:

Physiological Integrity 30. The nurse completes an admission assessment on a patient with asthma. Which information given by patient is most indicative of a need for a change in therapy? a. The patient uses albuterol (Proventil) before any aerobic exercise. b. The patient says that the asthma symptoms are worse every spring. c. The patient's heart rate increases after using the albuterol (Proventil) inhaler. d. The patient's only medications are albuterol (Proventil) and salmeterol (Serevent).

D Long-acting 2-agonists should be used only in patients who also are using an inhaled corticosteroid for long-term control. Salmeterol should not be used as the first-line therapy for long-term control. Using a bronchodilator before exercise is appropriate. The other information given by the patient requires further assessment by the nurse, but is not unusual for a patient with asthma. DIF: Cognitive Level: Apply (application) REF: 572 TOP: Nursing Process: Assessment MSC:

Safe and Effective Care Environment 46. Which intervention will the nurse include in the plan of care for a patient who is diagnosed with a lung abscess? a. Teach the patient to avoid the use of over-the-counter expectorants. b. Assist the patient with chest physiotherapy and postural drainage. c. Notify the health care provider immediately about any bloody or foul-smelling sputum. d. Teach about the need for prolonged antibiotic therapy after discharge from the hospital.

D Long-term antibiotic therapy is needed for effective eradication of the infecting organisms in lung abscess. Chest physiotherapy and postural drainage are not recommended for lung abscess because they may lead to spread of the infection. Foul smelling and bloody sputum are common clinical manifestations in lung abscess. Expectorants may be used because the patient is encouraged to cough. DIF: Cognitive Level: Apply (application) REF: 534-535 TOP: Nursing Process: Planning MSC:

Physiological Integrity 29. The nurse completes discharge teaching for a patient who has had a lung transplant. The nurse evaluates that the teaching has been effective if the patient makes which statement? a. "I will make an appointment to see the doctor every year." b. "I will stop taking the prednisone if I experience a dry cough." c. "I will not worry if I feel a little short of breath with exercise." d. "I will call the health care provider right away if I develop a fever."

D Low-grade fever may indicate infection or acute rejection so the patient should notify the health care provider immediately if the temperature is elevated. Patients require frequent follow-up visits with the transplant team. Annual health care provider visits would not be sufficient. Home oxygen use is not an expectation after lung transplant. Shortness of breath should be reported. Low-grade fever, fatigue, dyspnea, dry cough, and oxygen desaturation are signs of rejection. Immunosuppressive therapy, including prednisone, needs to be continued to prevent rejection. DIF: Cognitive Level: Apply (application) REF: 556 TOP: Nursing Process: Evaluation MSC:

1. The nurse teaches a patient about discharge instructions after a rhinoplasty. Which statement, if made by the patient, indicates that the teaching was successful? a. "I can take 800 mg ibuprofen for pain control." b. "I will safely remove and reapply nasal packing daily." c. "My nose will look normal after 24 hours when the swelling goes away." d. "I will keep my head elevated for 48 hours to minimize swelling and pain."

D Maintaining the head in an elevated position will decrease the amount of nasal swelling. NSAIDs, such as ibuprofen, increase the risk for postoperative bleeding and should not be used postoperatively. The patient would not be taught to remove or reapply nasal packing, which is usually removed by the surgeon on the day after surgery. Although return to a preinjury appearance is the goal of the surgery, it is not always possible to achieve this result, especially in the first few weeks after surgery. DIF: Cognitive Level: Apply (application) REF: 498 TOP: Nursing Process: Implementation MSC:

Physiological Integrity 8. When teaching the patient with newly diagnosed heart failure about a 2000-mg sodium diet, the nurse explains that foods to be restricted include a. canned and frozen fruits. b. fresh or frozen vegetables. c. eggs and other high-protein foods. d. milk, yogurt, and other milk products.

D Milk and yogurt naturally contain a significant amount of sodium, and intake of these should be limited for patients on a diet that limits sodium to 2000 mg daily. Other milk products, such as processed cheeses, have very high levels of sodium and are not appropriate for a 2000-mg sodium diet. The other foods listed have minimal levels of sodium and can be eaten without restriction. DIF: Cognitive Level: Apply (application) REF: 778 TOP: Nursing Process: Implementation MSC:

Physiological Integrity Lewis: Medical-Surgical Nursing, 7th Edition Test Bank Chapter 34: Nursing Management: Coronary Artery Disease and Acute Coronary Syndrome MULTIPLE CHOICE 1. When developing a health teaching plan for a 65-year-old patient with all these risk factors for coronary artery disease (CAD), the nurse will focus on the a. family history of heart disease. b. increased risk associated with the patient's ethnicity. c. high incidence of cardiovascular disease in older people. d. low activity level the patient reports.

D Rationale: Because family history, ethnicity, and age are nonmodifiable risk factors, the nurse should focus on the patient's activity level. An increase in activity will help reduce the patient's risk for developing CAD. Cognitive Level: Application Text Reference: pp. 790-792 Nursing Process: Planning

Health Promotion and Maintenance 18. A lobectomy is scheduled for a patient with stage I non-small cell lung cancer. The patient tells the nurse, "I would rather have chemotherapy than surgery." Which response by the nurse is most appropriate? a. "Are you afraid that the surgery will be very painful?" b. "Did you have bad experiences with previous surgeries?" c. "Surgery is the treatment of choice for stage I lung cancer." d. "Tell me what you know about the various treatments available."

D More assessment of the patient's concerns about surgery is indicated. An open-ended response will elicit the most information from the patient. The answer beginning, "Surgery is the treatment of choice" is accurate, but it discourages the patient from sharing concerns about surgery. The remaining two answers indicate that the nurse has jumped to conclusions about the patient's reasons for not wanting surgery. Chemotherapy is the primary treatment for small cell lung cancer. In non-small cell lung cancer, chemotherapy may be used in the treatment of nonresectable tumors or as adjuvant therapy to surgery. DIF: Cognitive Level: Apply (application) REF: 540 TOP: Nursing Process: Implementation MSC:

Physiological Integrity 8. A patient is admitted with active tuberculosis (TB). The nurse should question a health care provider's order to discontinue airborne precautions unless which assessment finding is documented? a. Chest x-ray shows no upper lobe infiltrates. b. TB medications have been taken for 6 months. c. Mantoux testing shows an induration of 10 mm. d. Three sputum smears for acid-fast bacilli are negative.

D Negative sputum smears indicate that Mycobacterium tuberculosis is not present in the sputum, and the patient cannot transmit the bacteria by the airborne route. Chest x-rays are not used to determine whether treatment has been successful. Taking medications for 6 months is necessary, but the multidrug-resistant forms of the disease might not be eradicated after 6 months of therapy. Repeat Mantoux testing would not be done because the result will not change even with effective treatment. DIF: Cognitive Level: Apply (application) REF: 533 TOP: Nursing Process: Implementation MSC:

Physiological Integrity 21. The nurse is caring for a patient who has acute pharyngitis caused by Candida albicans. Which action is appropriate for the nurse to include in the plan of care? a. Avoid giving patient warm liquids to drink. b. Assess patient for allergies to penicillin antibiotics. c. Teach the patient about the need to sleep in a warm, dry environment. d. Teach patient to "swish and swallow" prescribed oral nystatin (Mycostatin).

D Oral or pharyngeal fungal infections are treated with nystatin solution. The goal of the "swish and swallow" technique is to expose all of the oral mucosa to the antifungal agent. Warm liquids may be soothing to a sore throat. The patient should be taught to use a cool mist humidifier. There is no need to assess for penicillin/cephalosporin allergies because Candida albicans infection is treated with antifungals. DIF: Cognitive Level: Apply (application) REF: 506-507 TOP: Nursing Process: Planning MSC:

Safe and Effective Care Environment 38. The nurse is performing tuberculosis (TB) skin tests in a clinic that has many patients who have immigrated to the United States. Which question is most important for the nurse to ask before the skin test? a. "Is there any family history of TB?" b. "How long have you lived in the United States?" c. "Do you take any over-the-counter (OTC) medications?" d. "Have you received the bacille Calmette-Guérin (BCG) vaccine for TB?"

D Patients who have received the BCG vaccine will have a positive Mantoux test. Another method for screening (such as a chest x-ray) will need to be used in determining whether the patient has a TB infection. The other information also may be valuable but is not as pertinent to the decision about doing TB skin testing. DIF: Cognitive Level: Apply (application) REF: 532 TOP: Nursing Process: Assessment MSC:

Physiological Integrity 35. The nurse supervises unlicensed assistive personnel (UAP) who are providing care for a patient with right lower lobe pneumonia. The nurse should intervene if which action by UAP is observed? a. UAP splint the patient's chest during coughing. b. UAP assist the patient to ambulate to the bathroom. c. UAP help the patient to a bedside chair for meals. d. UAP lower the head of the patient's bed to 15 degrees.

D Positioning the patient with the head of the bed lowered will decrease ventilation. The other actions are appropriate for a patient with pneumonia. DIF: Cognitive Level: Apply (application) REF: 527 OBJ: Special Questions: Delegation TOP: Nursing Process: Implementation MSC:

Physiological Integrity 47. The nurse provides discharge teaching for a patient who has two fractured ribs from an automobile accident. Which statement, if made by the patient, would indicate that teaching has been effective? a. "I am going to buy a rib binder to wear during the day." b. "I can take shallow breaths to prevent my chest from hurting." c. "I should plan on taking the pain pills only at bedtime so I can sleep." d. "I will use the incentive spirometer every hour or two during the day."

D Prevention of the complications of atelectasis and pneumonia is a priority after rib fracture. This can be ensured by deep breathing and coughing. Use of a rib binder, shallow breathing, and taking pain medications only at night are likely to result in atelectasis. DIF: Cognitive Level: Apply (application) REF: 543 TOP: Nursing Process: Evaluation MSC:

Integrated Process: Teaching/Learning 31. Which nursing intervention is an example of primary prevention for lung cancer? a. Teaching clients with lung cancer how to cough and deep breathe b. Teaching clients with lung cancer to avoid infection c. Teaching clients about prophylactic antibiotics d. Teaching people about smoking and secondhand smoke

D Primary prevention for lung cancer focuses on reducing tobacco smoking. The other examples are examples of secondary prevention. DIF: Cognitive Level: Comprehension/Understanding REF: p. 631 TOP: Client Needs Category: Health Promotion and Maintenance (Principles of Teaching/Learning)

Health Promotion and Maintenance 43. After reviewing a patient's history, vital signs, physical assessment, and laboratory data, which information shown in the accompanying figure is most important for the nurse to communicate to the health care provider? a. Q waves on ECG b. Elevated troponin levels c. Fever and hyperglycemia d. Tachypnea and crackles in lungs

D Pulmonary congestion and tachypnea suggest that the patient may be developing heart failure, a complication of myocardial infarction (MI). Mild fever and hyperglycemia are common after MI because of the inflammatory process that occurs with tissue necrosis. Troponin levels will be elevated for several days after MI. Q waves often develop with ST-segment-elevation MI. DIF: Cognitive Level: Analyze (analysis) REF: 737 OBJ: Special Questions: Prioritization TOP: Nursing Process: Assessment MSC:

Physiological Integrity 23. A 55-year-old patient with increasing dyspnea is being evaluated for a possible diagnosis of chronic obstructive pulmonary disease (COPD). When teaching a patient about pulmonary function testing (PFT) for this condition, what is the most important question the nurse should ask? a. "Are you claustrophobic?" b. "Are you allergic to shellfish?" c. "Do you have any metal implants or prostheses?" d. "Have you taken any bronchodilators in the past 6 hours?"

D Pulmonary function testing will help establish the COPD diagnosis. Bronchodilators should be avoided at least 6 hours before the test. PFTs do not involve being placed in an enclosed area such as for magnetic resonance imaging (MRI). Contrast dye is not used for PFTs. The patient may still have PFTs done if metal implants or prostheses are present, as these are contraindications for an MRI. DIF: Cognitive Level: Apply (application) REF: 566 TOP: Nursing Process: Planning MSC:

1. After noting a pulse deficit when assessing a 74-year-old patient who has just arrived in the emergency department, the nurse will anticipate that the patient may require a. emergent cardioversion. b. a cardiac catheterization. c. hourly blood pressure (BP) checks. d. electrocardiographic (ECG) monitoring.

D Pulse deficit is a difference between simultaneously obtained apical and radial pulses. It indicates that there may be a cardiac dysrhythmia that would best be detected with ECG monitoring. Frequent BP monitoring, cardiac catheterization, and emergent cardioversion are used for diagnosis and/or treatment of cardiovascular disorders but would not be as helpful in determining the immediate reason for the pulse deficit. DIF: Cognitive Level: Apply (application) REF: 697 | 700 TOP: Nursing Process: Planning MSC:

Physiological Integrity 10. A patient with chronic obstructive pulmonary disease (COPD) has a nursing diagnosis of impaired breathing pattern related to anxiety. Which nursing action is most appropriate to include in the plan of care? a. Titrate oxygen to keep saturation at least 90%. b. Discuss a high-protein, high-calorie diet with the patient. c. Suggest the use of over-the-counter sedative medications. d. Teach the patient how to effectively use pursed lip breathing.

D Pursed lip breathing techniques assist in prolonging the expiratory phase of respiration and decrease air trapping. There is no indication that the patient requires oxygen therapy or an improved diet. Sedative medications should be avoided because they decrease respiratory drive. DIF: Cognitive Level: Apply (application) REF: 578 TOP: Nursing Process: Planning MSC:

Physiological Integrity 20. Following an AMI, a patient ambulates in the hospital hallway. When the nurse is evaluating the patient's response, which of these assessment data would indicate that the exercise level should be decreased? a. BP rises from 118/60 to 126/68 mm Hg. b. Respiratory rate goes from 14 to 22 breaths/min. c. Oxygen saturation drops from 100% to 98%. d. Heart rate increases from 66 to 90 beats/min.

D Rationale: A change in heart rate of more than 20 beats or more indicates that the patient should stop and rest. The increases in BP and respiratory rate, and the slight decrease in oxygen saturation, are normal responses to exercise. Cognitive Level: Application Text Reference: pp. 815-816 Nursing Process: Evaluation

Physiological Integrity 7. A patient with chronic obstructive pulmonary disease (COPD) arrives in the emergency department complaining of acute respiratory distress. When monitoring the patient, which assessment by the nurse will be of most concern? a. The patient is sitting in the tripod position. b. The patient has bibasilar lung crackles. c. The patient's pulse oximetry indicates an O2 saturation of 91%. d. The patient's respiratory rate has decreased from 30 to 10/min.

D Rationale: A decrease in respiratory rate in a patient with respiratory distress suggests the onset of fatigue and a high risk for respiratory arrest; therefore, the nurse will need to take immediate action. Patients who are experiencing respiratory distress frequently sit in the tripod position because it decreases the work of breathing. Crackles in the lung bases may be the baseline for a patient with COPD. An oxygen saturation of 91% is common in patients with COPD and will provide adequate gas exchange and tissue oxygenation. Cognitive Level: Application Text Reference: p. 1804 Nursing Process: Assessment

Physiological Integrity 17. A patient is seen in the clinic with COPD. Which information given by the patient would help most in confirming a diagnosis of chronic bronchitis? a. The patient tells the nurse about a family history of bronchitis. b. The patient denies having any respiratory problems until the last 6 months. c. The patient's history indicates a 40 pack-year cigarette history. d. The patient complains about having a productive cough every winter for 2 months.

D Rationale: A diagnosis of chronic bronchitis is based on a history of having a productive cough for several months for at least 2 consecutive years. There is no familial tendency for chronic bronchitis. Although smoking is the major risk factor for chronic bronchitis, a smoking history does not confirm the diagnosis. Cognitive Level: Application Text Reference: p. 629 Nursing Process: Assessment

Physiological Integrity 4. A patient is diagnosed with a large pulmonary embolism. When explaining to the patient what has happened to cause respiratory failure, which information will the nurse include? a. "Oxygen transfer into your blood is slow because of thick membranes between the small air sacs and the lung circulation." b. "Thick secretions in your small airways are blocking air from moving into the small air sacs in your lungs." c. "Large areas of your lungs are getting good blood flow but are not receiving enough air to fill the small air sacs." d. "Blood flow though some areas of your lungs is decreased even though you are taking adequate breaths."

D Rationale: A pulmonary embolus limits blood flow but does not affect ventilation, leading to a ventilation-perfusion mismatch. The response beginning, "Oxygen transfer into your blood is slow because of thick membranes" describes a diffusion problem. The remaining two responses describe ventilation-perfusion mismatch with adequate blood flow but poor ventilation. Cognitive Level: Application Text Reference: p. 1802 Nursing Process: Implementation

Physiological Integrity 37. The nurse has received a change-of-shift report about these patients with COPD. Which patient should the nurse assess first? a. A patient with loud expiratory wheezes b. A patient who has a cough productive of thick, green mucus c. A patient with jugular vein distension and peripheral edema d. A patient with a respiratory rate of 38

D Rationale: A respiratory rate of 38 indicates severe respiratory distress, and the patient needs immediate assessment and intervention to prevent possible respiratory arrest. The other patients also need assessment as soon as possible, but they do not need to be assessed as urgently as the tachypneic patient. Cognitive Level: Application Text Reference: pp. 612, 626 Nursing Process: Assessment

Physiological Integrity 17. Following an acute myocardial infarction, a previously healthy 67-year-old patient develops clinical manifestations of heart failure. The nurse anticipates discharge teaching will include information about a. digitalis preparations, such as digoxin (Lanoxin). b. calcium-channel blockers, such as diltiazem (Cardizem). c. -adrenergic agonists, such as dobutamine (Dobutrex). d. angiotensin-converting enzyme (ACE) inhibitors, such as captopril (Capoten).

D Rationale: ACE-inhibitor therapy is currently recommended to prevent the development of heart failure in patients who have had a myocardial infarction and as a first-line therapy for patients with chronic heart failure. Digoxin therapy for heart failure is no longer considered a first-line measure, and digoxin is added to the treatment protocol when therapy with other medications such as ACE-inhibitors, diuretics, and -adrenergic blockers is insufficient. Calcium-channel blockers are not generally used in the treatment of heart failure. The -adrenergic agonists such as dobutamine are administered through the IV route and are not used as initial therapy for heart failure. Cognitive Level: Application Text Reference: p. 832 Nursing Process: Implementation

Health Promotion and Maintenance 20. During IV administration of amphotericin B ordered for treatment of coccidioidomycosis, the nurse increases the patient's tolerance of the drug by a. cooling the solution to 80° F before administration. b. keeping the patient flat in bed for 1 hour after the infusion is completed. c. diluting the amphotericin B in 500 ml of sterile water. d. giving diphenhydramine (Benadryl) 1 hour before starting the infusion.

D Rationale: Administration of an antihistamine before giving the amphotericin B will reduce the incidence of hypersensitivity reactions. Cooling the solution and keeping the patient flat after infusion are not indicated. Amphotericin B does not need to be diluted in 500 ml of fluid, although the nurse should ensure adequate hydration in the patient receiving this drug. Cognitive Level: Application Text Reference: p. 575 Nursing Process: Implementation

Physiological Integrity 39. After discharge teaching has been completed for a patient who has had a lung transplant, the nurse will evaluate that the teaching has been effective if the patient states a. "I will make an appointment to see the doctor every year." b. "I will not turn the home oxygen up higher than 2 L/minute." c. "I will be careful to use sterile technique with my central line." d. "I will write down my medications and spirometry in a journal."

D Rationale: After lung transplant, patients are taught to keep logs of medications, spirometry, and laboratory results. Patients require frequent follow-up visits with the transplant team; annual health care provider visits would not be sufficient. Home oxygen use is not an expectation after lung transplant and patients would not usually have a central IV line. Cognitive Level: Application Text Reference: p. 604 Nursing Process: Evaluation

Physiological Integrity 34. All of these orders are received for a patient having an acute asthma attack. Which one will the nurse administer first? a. IV methylprednisolone (Solu-Medrol) 60 mg b. triamcinolone (Azmacort) 2 puffs per MDI c. salmeterol (Serevent) 50 mcg per DPI d. albuterol (Ventolin) 2.5 mg per nebulizer

D Rationale: Albuterol is a rapidly acting bronchodilator and is the first-line medication to reverse airway narrowing in acute asthma attacks. The other medications work more slowly. Cognitive Level: Application Text Reference: p. 620 Nursing Process: Implementation

Physiological Integrity 6. A 77-year-old patient with pneumonia has a fever of 101.2° F (38.5° C), a nonproductive cough, and an oxygen saturation of 89%. The patient is very weak and needs assistance to get out of bed. The priority nursing diagnosis for the patient is a. hyperthermia related to infectious illness. b. ineffective airway clearance related to thick secretions. c. impaired transfer ability related to weakness. d. impaired gas exchange related to respiratory congestion.

D Rationale: All these nursing diagnoses are appropriate for the patient, but the patient's oxygen saturation indicates that all body tissues are at risk for hypoxia unless the gas exchange is improved. Cognitive Level: Application Text Reference: p. 566 Nursing Process: Diagnosis

Physiological Integrity 2. The nurse recognizes that intubation and mechanical ventilation are indicated for a patient in status asthmaticus when a. ventricular dysrhythmias and dyspnea occur. b. loud wheezes are audible throughout the lungs. c. pulsus paradoxus is greater than 40 mm Hg. d. fatigue and an O2 saturation of 88% develop.

D Rationale: Although all of the assessment data indicate the need for rapid intervention, the fatigue and hypoxia indicate that the patient is no longer able to maintain an adequate respiratory effort and needs mechanical ventilation. The initial treatment for the other clinical manifestations would initially be administration of rapidly acting bronchodilators and oxygen. Cognitive Level: Application Text Reference: pp. 612-613 Nursing Process: Assessment

Physiological Integrity 9. A 72-year-old patient is hospitalized for an aortic dissection of the abdominal aorta that stabilizes with treatment. The nurse develops a teaching plan for the patient's discharge that includes information about a. gradually increasing exercise to improve cardiac function and BP control. b. appropriate use of nonsteroidal antiinflammatory agents (NSAIDs) to control any abdominal pain. c. holding prescribed -blockers if dizziness or weakness occur to avoid injury. d. the use of antihypertensive medications to lower the risk of further dissection or bleeding.

D Rationale: Antihypertensive medications are prescribed to help control BP and prevent re-dissection, leaking, or rupture. Exercise will increase the BP and increase the risk for further dissection. NSAIDs decrease platelet function, and the patient should avoid use of this category of medications; in addition, the patient is taught to call the health care provider about any abdominal pain. The patient may experience some side effects of -blockade but should discuss any change in dose or medication with the health care provider. Cognitive Level: Application Text Reference: pp. 899-900 Nursing Process: Planning

Physiological Integrity 13. A patient with dilated cardiomyopathy has an atrial fibrillation that has been unresponsive to drug therapy for several days. The nurse anticipates that further treatment of the patient will require a. IV adenosine (Adenocard). b. electrical cardioversion. c. insertion of an implantable cardioverter-defibrillator (ICD). d. anticoagulant therapy with warfarin (Coumadin).

D Rationale: Atrial fibrillation therapy that has persisted for more than 48 hours requires anticoagulant treatment for 3 to 4 weeks before attempting cardioversion; this is done to prevent embolization of clots from the atria. Adenosine is not used to treat atrial fibrillation. Cardioversion may be done after several weeks of Coumadin therapy. ICDs are used for patients with recurrent ventricular fibrillation. Cognitive Level: Application Text Reference: p. 852 Nursing Process: Planning

Health Promotion and Maintenance 24. When developing a teaching plan for a patient with a 42 pack-year history of cigarette smoking, it will be most important for the nurse to include information about a. reasons for annual sputum cytology testing. b. CT screening for lung cancer. c. erlotinib (Tarceva) therapy to prevent tumor risk. d. options for smoking cessation.

D Rationale: Because smoking is the major cause of lung cancer, the most important role for the nurse is educating patients about the benefits of and means of smoking cessation. Early screening of at-risk patients using sputum cytology, chest x-ray, or CT scanning has not been effective in reducing mortality. Tarceva may be used to in patients who have lung cancer, but not to reduce risk for developing tumors. Cognitive Level: Application Text Reference: pp. 582, 584 Nursing Process: Planning

Physiological Integrity 4. The nurse is caring for a patient receiving IV furosemide (Lasix) 40 mg and enalapril (Vasotec) 5 mg PO bid for ADHF with severe orthopnea. When evaluating the patient response to the medications, the best indicator that the treatment has been effective is a. weight loss of 2 pounds overnight. b. improvement in hourly urinary output. c. reduction in systolic BP. d. decreased dyspnea with the head of the bed at 30 degrees.

D Rationale: Because the patient's major clinical manifestation of ADHF is orthopnea (caused by the presence of fluid in the alveoli), the best indicator that the medications are effective is a decrease in crackles. The other assessment data also may indicate that diuresis or improvement in cardiac output have occurred but are not as useful in evaluating this patient's response. Cognitive Level: Application Text Reference: p. 825 Nursing Process: Evaluation

Physiological Integrity 20. Which statement by the nurse when explaining the purpose of positive end-expiratory pressure (PEEP) to the family members of a patient with ARDS is correct? a. "PEEP will prevent fibrosis of the lung from occurring." b. "PEEP will push more air into the lungs during inhalation." c. "PEEP allows the ventilator to deliver 100% oxygen to the lungs." d. "PEEP prevents the lung air sacs from collapsing during exhalation."

D Rationale: By preventing alveolar collapse during expiration, PEEP improves gas exchange and oxygenation. PEEP will not prevent the fibrotic changes that occur with ARDS, push more air into the lungs, or change the fraction of inspired oxygen (FIO2) delivered to the patient. Cognitive Level: Comprehension Text Reference: p. 1817 Nursing Process: Planning

Safe and Effective Care Environment 23. A hospitalized patient with heart failure has a new order for captopril (Capoten) 12.5 mg PO. After administering the first dose and teaching the patient about captopril, which statement by the patient indicates that teaching has been effective? a. "I will need to include more high-potassium foods in my diet." b. "I will expect to feel more short of breath for the next few days." c. "I will be sure to take the medication after eating something." d. "I will call for help when I need to get up to the bathroom."

D Rationale: Captopril can cause hypotension, especially after the initial dose, so it is important that the patient not get up out of bed without assistance until the nurse has had a chance to evaluate the effect of the first dose. The ACE inhibitors are potassium sparring, and the nurse should not teach the patient to increase sources of dietary potassium. Increased shortness of breath is expected with initiation of -blocker therapy for heart failure, not for ACE-inhibitor therapy. ACE inhibitors are best absorbed when taken an hour before eating. Cognitive Level: Application Text Reference: p. 832 Nursing Process: Evaluation

Health Promotion and Maintenance 3. The nurse is admitting a patient who is complaining of chest pain to the emergency department (ED). Which information collected by the nurse suggests that the pain is caused by an acute myocardial infarction (AMI)? a. The pain worsens when the patient raises the arms. b. The pain increases with deep breathing. c. The pain is relieved after the patient takes nitroglycerin. d. The pain has persisted longer than 30 minutes.

D Rationale: Chest pain that lasts for 20 minutes or more is characteristic of AMI. Changes in pain that occur with raising the arms or with deep breathing are more typical of pericarditis or musculoskeletal pain. Stable angina is usually relieved when the patient takes nitroglycerin. Cognitive Level: Application Text Reference: p. 803 Nursing Process: Assessment

Physiological Integrity 22. A 42-year-old service-counter worker undergoes sclerotherapy for treatment of superficial varicose veins at an outpatient center. Before discharging the patient, the nurse teaches the patient that a. exercises such as walking or jogging cause recurrence of varicosities. b. sitting at the work counter, rather than standing, is recommended. c. taking one aspirin daily will help prevent clotting around venous valves. d. compression stockings should be applied before getting out of bed.

D Rationale: Compression stockings are applied with the legs elevated to reduce pressure in the lower legs. Walking is recommended to prevent recurrent varicosities. Sitting and standing are both risk factors for varicose veins and venous insufficiency. An aspirin a day is not adequate to prevent venous thrombosis and would not be recommended to the patient who had just had sclerotherapy. Cognitive Level: Application Text Reference: pp. 918-919 Nursing Process: Planning

Health Promotion and Maintenance 14. The nurse recognizes that the goals of teaching regarding the transmission of TB have been met when the patient with TB a. demonstrates correct use of a nebulizer. b. reports daily to the public health department. c. washes dishes and personal items after use. d. covers the mouth and nose when coughing.

D Rationale: Covering the mouth and nose will help decrease airborne transmission of TB. The other actions will not be effective in decreasing the spread of TB. Cognitive Level: Application Text Reference: p. 574 Nursing Process: Evaluation

Physiological Integrity 29. While working in the outpatient clinic, the nurse notes that the chart states that a patient has intermittent claudication. Which of these statements by the patient would be consistent with this information? a. "My fingers hurt when I go outside in cold weather." b. "Sometimes I get tired when I climb a lot of stairs." c. "When I stand too long, my feet start to swell up." d. "My legs cramp whenever I walk more than a block."

D Rationale: Cramping that is precipitated by a consistent level of exercise is descriptive of intermittent claudication. Finger pain associated with cold weather is typical of Raynaud's phenomenon. Fatigue that occurs sometimes with exercise is not typical of intermittent claudication, which is reproducible. Swelling associated with prolonged standing is typical of venous disease. Cognitive Level: Application Text Reference: p. 900 Nursing Process: Assessment

c

Match the following characteristics with their methods of oxygen administration: Reservoir bag conserves oxygen A. Nasal cannula B. Simple face mask C. Partial rebreathing mask D. Nonrebreathing mask E. Venturi mask F. Tracheostomy collar

Health Promotion and Maintenance 13. The nurse plans discharge teaching for a patient with chronic heart failure who has prescriptions for digoxin (Lanoxin), hydrochlorothiazide (HydroDIURIL), and a potassium supplement. Appropriate instructions for the patient include a. avoid dietary sources of potassium because too much can cause digitalis toxicity. b. take the pulse rate daily and never take digoxin if the pulse is below 60 beats/min. c. take the hydrochlorothiazide before bedtime to maximize activity level during the day. d. notify the health care provider immediately if nausea or difficulty breathing occurs.

D Rationale: Difficulty breathing is an indication of acute decompensated heart failure and suggests that the medications are not achieving the desired effect. Nausea is an indication of digoxin toxicity and should be reported so that the provider can assess the patient for toxicity and adjust the digoxin dose, if necessary. Digoxin toxicity is potentiated by hypokalemia, rather than hyperkalemia. Patients should be taught to check their pulse daily before taking the digoxin and, if the pulse is less than 60, to call their provider before taking the digoxin. Diuretics should be taken early in the day to avoid sleep disruption. Cognitive Level: Application Text Reference: p. 835 Nursing Process: Implementation

Physiological Integrity 2. The nurse is monitoring a patient with possible coronary artery disease who is undergoing exercise (stress) testing on a treadmill. The symptom that has the most immediate implications for the patient's care during the exercise testing is a. the BP rising from 134/68 to 150/80 mm Hg. b. the heart rate (HR) increasing from 80 to 96 beats/min. c. the patient complaining of feeling short of breath. d. the ECG indicating the presence of coronary ischemia.

D Rationale: ECG changes associated with coronary ischemia (such as T-wave inversions and ST segment depression) indicate that the myocardium is not getting adequate oxygen delivery and that the exercise test should be immediately terminated. Increases in BP and HR are normal responses to aerobic exercise. Shortness of breath is also normal as the intensity of exercise increases during the stress testing. Cognitive Level: Application Text Reference: pp. 753, 757 Nursing Process: Assessment

Physiological Integrity 19. A patient with ST-segment elevation in several ECG leads is admitted to the ED and diagnosed as having an AMI. Which question should the nurse ask to determine whether the patient is a candidate for fibrinolytic therapy? a. "Is there any family history of heart disease?" b. "Do you take aspirin on a daily basis?" c. "Can you describe the quality of your chest pain?" d. "What time did your chest pain begin?"

D Rationale: Fibrinolytic therapy should be started within 6 hours of the onset of the MI, so the time at which the chest pain started is a major determinant of the appropriateness of this treatment. The other information will also be needed, but it will not be a factor in the decision about fibrinolytic therapy. Cognitive Level: Application Text Reference: p. 807 Nursing Process: Assessment

Physiological Integrity 34. When providing preoperative instruction for a patient scheduled for a left pneumonectomy for cancer of the lung, the nurse informs the patient that the postoperative care includes a. positioning on the right side. b. chest tubes to water-seal chest drainage. c. bedrest for the first 24 hours. d. frequent use of an incentive spirometer.

D Rationale: Frequent deep breathing and coughing are needed after chest surgery to prevent atelectasis. To promote gas exchange, patients after pneumonectomy are positioned on the surgical side. Chest tubes are not usually used after pneumonectomy because the affected side is allowed to fill with fluid. Early mobilization decreases the risk for postoperative complications such as pneumonia and deep vein thrombosis. Cognitive Level: Application Text Reference: pp. 596-597 Nursing Process: Planning

Physiological Integrity 18. A patient with syncope is scheduled for Holter monitoring. When teaching the patient about the purpose of the procedure, the nurse will explain that Holder monitoring provides information about the a. ventricular ejection fraction during usual daily activities. b. cardiovascular response to high-intensity exercise. c. changes in cardiac output when the patient is resting. d. HR and rhythm during normal patient activities.

D Rationale: Holter monitoring is used to assess for possible changes in HR or rhythm over a 24- to 48-hour period. The patient is usually instructed to continue with usual daily activities rather than changing exercise or activity level. Because Holter monitoring is useful only for detecting rhythm changes, it is not useful in determining ejection fraction or cardiac output. Cognitive Level: Application Text Reference: pp. 753, 757 Nursing Process: Implementation

Physiological Integrity 9. A patient with diabetes mellitus is admitted unresponsive to the emergency department (ED). Initial laboratory findings are serum potassium 2.8 mEq/L (2.8 mmol/L), serum sodium 138 mEq/L (138 mmol/L), serum chloride 90 mEq/L (90 mmol/L), and blood glucose 628 mg/dl (34.9 mmol/L). Cardiac monitoring shows multifocal PVCs. The nurse understands that the patient's PVCs are most likely caused by a. hyperglycemia. b. hypoxemia. c. dehydration. d. hypokalemia.

D Rationale: Hypokalemia increases the risk for ventricular dysrhythmias such as PVCs, ventricular tachycardia, and ventricular fibrillation. Hyperglycemia and dehydration are not associated with increased PVC risk. There is no indication that the patient is hypoxemic. Cognitive Level: Analysis Text Reference: p. 854 Nursing Process: Assessment

Physiological Integrity 10. While caring for a patient who has been admitted with a pulmonary embolism, the nurse notes a change in the patient's arterial oxyhemoglobin saturation (SpO2) from 94% to 88%. The nurse will a. assist the patient to cough and deep-breathe. b. help the patient to sit in a more upright position. c. suction the patient's oropharynx. d. increase the oxygen flow rate.

D Rationale: Increasing oxygen flow rate will usually improve oxygen saturation in patients with ventilation-perfusion mismatch, as occurs with pulmonary embolism. Because the problem is with perfusion, actions that improve ventilation, such as deep-breathing and coughing, sitting upright, and suctioning, are not likely to improve oxygenation. Cognitive Level: Application Text Reference: pp. 1802, 1807 Nursing Process: Implementation

Physiological Integrity 28. A patient who is seen in the clinic tells the health care provider about experiencing cold, numb fingers when running during the winter and is diagnosed with Raynaud's phenomenon. The nurse will anticipate teaching the patient about tests for a. coronary artery disease. b. familial hyperlipidemia. c. high BP. d. immune disorders.

D Rationale: Secondary Raynaud's phenomenon may occur in conjunction with autoimmune diseases such as rheumatoid arthritis, and patients should be screened for autoimmune disorders. Raynaud's phenomenon is not associated with hyperlipidemia, hypertension, or coronary artery disease. Cognitive Level: Application Text Reference: p. 909 Nursing Process: Planning

Physiological Integrity 18. A 55-year-old patient with inoperable coronary artery disease and end-stage heart failure asks the nurse whether heart transplant is a possible therapy. The nurse's response to the patient will be based on the knowledge that a. heart transplants are experimental surgeries that are not covered by most insurance. b. the patient is too old to be placed on the transplant list. c. the diagnoses and symptoms indicate that the patient is not an appropriate candidate. d. candidacy for heart transplant depends on many factors.

D Rationale: Indications for a heart transplant include inoperable coronary artery disease and refractory end-stage heart failure, but other factors such as coping skills, family support, and patient motivation to follow the rigorous post-transplant regimen are also considered. Heart transplants are not considered experimental; rather, transplantation has become the treatment of choice for patients who meet the criteria. The patient is not too old for a transplant. The patient's diagnoses and symptoms indicate that the patient may be an appropriate candidate for a heart transplant. Cognitive Level: Comprehension Text Reference: p. 837 Nursing Process: Planning

Physiological Integrity 18. A 21-year-old woman is scheduled for an open mitral valve commissurotomy for treatment of mitral stenosis. When explaining the advantage of valve repair instead of valve replacement to the patient, the nurse will include the information that a. mechanical mitral valves require replacement about every 10 years. b. no antibiotic prophylaxis to prevent endocarditis is needed after valve repair. c. biologic replacement valves require the use of life-long immunosuppressive drugs. d. long-term anticoagulation is necessary after mechanical valve replacement.

D Rationale: Long-term anticoagulation therapy is needed after mechanical valve replacement, and this would restrict decisions about career and childbearing in this patient. Mechanical valves are durable and typically last longer than 10 years. Patients require prophylactic antibiotics when having invasive procedures after either valve repair or replacement. Biologic valves do not activate the immune system, and immunosuppressive therapy is not needed. Cognitive Level: Application Text Reference: p. 882 Nursing Process: Implementation

Physiological Integrity 20. Which information obtained by the nurse who is admitting the patient for magnetic resonance imaging (MRI) will be most important to report to the health care provider before the MRI? a. The patient has a history of coronary artery disease. b. The patient took all the prescribed cardiac medications today. c. The patient has an allergy to shellfish and iodine. d. The patient has a permanent ventricular pacemaker in place.

D Rationale: MRI is contraindicated for patients with implanted metallic devices such as pacemakers. The other information will also be reported to the health care provider but does not impact on whether or not the patient can have an MRI. Cognitive Level: Application Text Reference: p. 755 Nursing Process: Implementation

Physiological Integrity 12. When teaching the patient with heart failure about a 2000-mg sodium diet, the nurse explains that foods to be restricted include a. eggs and other high-cholesterol foods. b. canned and frozen fruits. c. fresh or frozen vegetables. d. milk, yogurt, and other milk products.

D Rationale: Milk and yogurt naturally contain a significant amount of sodium, and intake of these should be limited for patients on a diet that limits sodium to 2000 mg daily. Other milk products, such as processed cheeses, have very high levels of sodium and are not appropriate for a 2000-mg sodium diet. The other foods listed have minimal levels of sodium and can be eaten without restriction. Cognitive Level: Application Text Reference: p. 833 Nursing Process: Implementation

Physiological Integrity 19. While caring for a patient with mitral valve prolapse with mild valvular regurgitation, the nurse determines that discharge teaching has been effective when the patient tells the nurse she will a. take 1 aspirin a day to prevent embolization from the valve. b. limit physical activity to avoid stressing the heart valves. c. schedule an appointment with the doctor every 6 months. d. discuss the diagnosis of mitral valve prolapse with the dentist.

D Rationale: Mitral valve prolapse with regurgitation is a risk factor for infective endocarditis, and the patient needs to discuss the need for antibiotic prophylaxis with the provider before any invasive medical or dental procedure is done. Anticoagulation, restriction of physical activity, and frequent medical appointments are not required for mild mitral valve prolapse. Cognitive Level: Application Text Reference: p. 880 Nursing Process: Evaluation

Safe and Effective Care Environment 32. A patient who has recently started taking rosuvastatin (Crestor) and niacin (Nicobid) reports all the following symptoms to the nurse. Which is most important to communicate to the health care provider? a. Skin flushing after taking the medications b. Dizziness when changing positions quickly c. Nausea when taking the drugs before eating d. Generalized muscle aches and pains

D Rationale: Muscle aches and pains may indicate myopathy and rhabdomyolysis, which have caused acute renal failure and death in some patients who have taken the statin medications. These symptoms indicate that the rosuvastatin may need to be discontinued. The other symptoms are common side effects when taking niacin, and, although the nurse should follow up with the patient, do not indicate that a change in medication is needed. Cognitive Level: Application Text Reference: p. 793 Nursing Process: Evaluation

Physiological Integrity 3. During the assessment of a patient with IE, the nurse would expect to find a. substernal chest pain and pressure. b. splinter hemorrhages of the lips. c. dyspnea and a dry, hacking cough. d. a new regurgitant murmur.

D Rationale: New regurgitant murmurs occur in IE because vegetation on the valves prevents valve closure. Splinter hemorrhages occur on the nailbeds. Chest pain for pressure is not typical for the patient with IE and would be more consistent with angina or MI. Although dyspnea may occur as a result of heart failure, a moist cough would be expected rather than a dry, hacking cough. Cognitive Level: Comprehension Text Reference: p. 867 Nursing Process: Assessment

Physiological Integrity 30. A patient experiences a steering wheel injury as a result of an automobile accident. During the initial assessment, the emergency department nurse would be most concerned about a. complaints of severe pain. b. heart rate of 110 beats/min. c. a large bruised area on the chest. d. paradoxic chest movement.

D Rationale: Paradoxic chest movement indicates that the patient may have flail chest, which will severely compromise gas exchange and can rapidly lead to hypoxemia. Severe pain, a slightly elevated pulse rate, and chest bruising all require further assessment or intervention, but the priority concern is poor gas exchange. Cognitive Level: Application Text Reference: pp. 586, 588 Nursing Process: Assessment

Physiological Integrity 8. Following discharge teaching, the nurse evaluates that the patient who was admitted with pneumonia understands measures to prevent a reoccurrence of the pneumonia when the patient states, a. "I will increase my food intake to 3000 calories a day." b. "I will need to use home oxygen therapy for 3 months." c. "I will seek medical treatment for any upper respiratory infections." d. "I will do deep-breathing and coughing exercises for the next 6 weeks."

D Rationale: Patients at risk for recurrent pneumonia should use the incentive spirometer or do deep breathing and coughing exercises or both for 6 to 8 weeks after discharge. Although caloric needs are increased during the acute infection, 3000 calories daily will lead to obesity and increase the risk for pneumonia. Patients with acute lower respiratory infections do not usually require home oxygen therapy. Upper respiratory infections require medical treatment only when they fail to resolve in 7 days. Cognitive Level: Application Text Reference: p. 569 Nursing Process: Evaluation

Health Promotion and Maintenance 10. After the nurse teaches the patient about the use of atenolol (Tenormin) in preventing anginal episodes, which statement by a patient indicates that the teaching has been effective? a. "Atenolol will increase the strength of my heart muscle." b. "I can expect to feel short of breath when taking atenolol." c. "Atenolol will improve the blood flow to my coronary arteries." d. "It is important not to suddenly stop taking the atenolol."

D Rationale: Patients who have been taking -blockers can develop intense and frequent angina if the medication is suddenly discontinued. Atenolol (Tenormin) decreases myocardial contractility. Shortness of breath that occurs when taking -blockers for angina may be due to bronchospasm and should be reported to the health care provider. Atenolol is not a vasodilator and works by decreasing myocardial oxygen demand, not by increasing blood flow to the coronary arteries. Cognitive Level: Application Text Reference: p. 801 Nursing Process: Evaluation

Physiological Integrity 14. The nurse establishes the nursing diagnosis of ineffective therapeutic regimen management related to lack of knowledge concerning long-term management of rheumatic fever when a patient recovering from rheumatic fever says, a. "I will need to have monthly antibiotic injections for at 5 years or longer." b. "I will call the doctor if I develop excessive fatigue or difficulty breathing." c. "I will need to let my dentist know that I have had this rheumatic fever." d. "I will be immune to further episodes of rheumatic fever after this infection."

D Rationale: Patients with a history of rheumatic fever are more susceptible to a second episode. The other patient statements are correct and would not support the nursing diagnosis of ineffective therapeutic regimen management related to lack of knowledge. Cognitive Level: Application Text Reference: p. 878 Nursing Process: Diagnosis

Physiological Integrity 41. After teaching the patient with asthma about home care, the nurse will evaluate that the teaching has been successful if the patient states, a. "I will use my corticosteroid inhaler as soon as I start to get short of breath." b. "I will only turn the home oxygen level up after checking with the doctor first." c. "My medications are working if I wake up short of breath only once during the night." d. "No changes in my medications are needed if my peak flow is at 80% of normal."

D Rationale: Peak flows of 80% or greater indicate that the asthma is well controlled. Corticosteroids are long-acting, prophylactic therapy for asthma and are not used to treat acute dyspnea. Because asthma is an acute and intermittent process, home oxygen is not used. The patient who has effective treatment should sleep throughout the night without waking up with dyspnea. Cognitive Level: Application Text Reference: p. 628 Nursing Process: Evaluation

Physiological Integrity 18. After prolonged cardiopulmonary bypass, a patient develops increasing shortness of breath and hypoxemia. To determine whether the patient has acute respiratory distress syndrome (ARDS) or pulmonary edema caused by left ventricular failure, the nurse will anticipate assisting with a. positioning the patient for a chest radiograph. b. drawing blood for arterial blood gases. c. obtaining a ventilation-perfusion scan. d. inserting a pulmonary artery catheter.

D Rationale: Pulmonary artery wedge pressure will remain at normal levels in the patient with ARDS because the fluid in the alveoli is caused by increased permeability of the alveolar-capillary membrane rather than by the backup of fluid from the lungs (as occurs in cardiogenic pulmonary edema). The other tests will not help in differentiating cardiogenic from noncardiogenic pulmonary edema. Cognitive Level: Application Text Reference: p. 1815 Nursing Process: Implementation

Physiological Integrity 12. While obtaining an admission health history from a patient with possible rheumatic fever, which question will be most pertinent to ask? a. "Are you using any illegal IV drugs?" b. "Do you have any family history of congenital heart disease?" c. "Can you recall having any chest injuries in the last few weeks?" d. "Have you had a recent sore throat?"

D Rationale: Rheumatic fever occurs as a result of an abnormal immune response to a streptococcal infection. Although illicit IV drug use should be discussed with the patient before discharge, it is not a risk factor for rheumatic fever and would not be as pertinent when admitting the patient. Family history is not a risk factor for rheumatic fever. Chest injury would cause musculoskeletal chest pain rather than rheumatic fever. Cognitive Level: Application Text Reference: p. 875 Nursing Process: Assessment

Physiological Integrity 30. A 23-year-old with cystic fibrosis (CF) is admitted to the hospital. Which intervention will be included in the plan of care? a. Schedule sweat chloride test to evaluate the effectiveness of therapy. b. Arrange for a hospice nurse to visit with the patient regarding home care. c. Place the patient on a low-sodium diet to prevent cor pulmonale. d. Perform chest physiotherapy every 4 hours to mobilize secretions.

D Rationale: Routine scheduling of airway clearance techniques is an essential intervention for patients with CF. A sweat chloride test is used to diagnose CF, but it does not provide any information about the effectiveness of therapy. There is no indication that the patient is terminally ill. Patients with CF lose excessive sodium in their sweat and require high amounts of dietary sodium. Cognitive Level: Application Text Reference: p. 658 Nursing Process: Planning

The nurse knows that tachycardia that is a response of the sympathetic nervous system to the pain of ischemia is detrimental because not only does it increase oxygen demand, it also does which of the following to the patient? A) Increases CO B) Causes reflex hypotension C) May lead to ventricular dysrhythmias D) Impairs perfusion of the coronary arteries

D) Impairs perfusion of the coronary arteries

Physiological Integrity 30. After having an AMI, a 62-year-old patient tells the nurse, "I guess having sex again will be too hard on my heart." The nurse's best response is a. "Sexual intercourse may be too strenuous on your heart, but closeness and intimacy can be maintained with holding and cuddling." b. "You should discuss your questions about your sexual activity with your doctor because the activity it requires is a medical concern." c. "Sexual activity can be resumed whenever you feel like you are ready. Most sexual response is emotional rather than physical." d. "Sexual activity can be gradually resumed like other activity. A good comparison of energy expenditure is climbing two flights of stairs."

D Rationale: Sexual activity places about as much physical stress on the cardiovascular system as climbing two flights of stairs. The answer beginning "Sexual intercourse may be too strenuous" may be true; however there are no data in the stem to indicate that intercourse will be too stressful to the heart for this patient. The answer beginning, "You should discuss your questions" implies that there are serious medical concerns about sexual activity. And the answer beginning "Sexual activity can be resumed whenever" is incorrect because physiologic parameters such as heart rate and BP do increase during sexual activity. Cognitive Level: Application Text Reference: p. 817 Nursing Process: Implementation

Physiological Integrity 23. A patient is receiving 35% oxygen via a Venturi mask. To ensure the correct amount of oxygen delivery, it is most important that the nurse a. give a high enough flow rate to keep the bag from collapsing. b. use an appropriate adaptor to ensure adequate oxygen delivery. c. drain moisture condensation from the oxygen tubing every hour. d. keep the air entrainment ports clean and unobstructed.

D Rationale: The air entrainment ports regulate the oxygen percentage delivered to the patient, so they must be unobstructed. A high oxygen flow rate is needed when giving oxygen by partial rebreather or non-rebreather masks. The use of an adaptor can improve humidification but not oxygen delivery. Draining oxygen tubing is necessary when caring for a patient receiving mechanical ventilation. Cognitive Level: Comprehension Text Reference: p. 642 Nursing Process: Implementation

Physiological Integrity 21. When a patient with COPD is receiving oxygen, the best action by the nurse is to a. avoid administration of oxygen at a rate of more than 2 L/min. b. minimize oxygen use to avoid oxygen dependency. c. administer oxygen according to the patient's level of dyspnea. d. maintain the pulse oximetry level at 90% or greater.

D Rationale: The best way to determine the appropriate oxygen flow rate is by monitoring the patient's oxygenation either by ABGs or pulse oximetry; an oxygen saturation of 90% indicates adequate blood oxygen level without the danger of suppressing the respiratory drive. For patients with an exacerbation of COPD, an oxygen flow rate of 2 L/min may not be adequate. Because oxygen use improves survival rate in patients with COPD, there is not a concern about oxygen dependency. The patient's perceived dyspnea level may be affected by other factors (such as anxiety) besides blood oxygen level. Cognitive Level: Application Text Reference: p. 640 Nursing Process: Implementation

Psychosocial Integrity 26. The nurse obtains the following data when caring for a patient who experienced an AMI 2 days previously. Which information is most important to report to the health care provider? a. The oral temperature is 100.8° F (38.2° C). b. The white blood cell count (WBC) is 12,000/l. c. The patient denies ever having a heart attack. d. The lungs have crackles audible to the midline.

D Rationale: The crackles indicate that the patient may be developing heart failure, a possible complication of MI. The health care provider may need to order medications such as diuretics or angiotensin-converting enzyme (ACE) inhibitors for the patient. The fever and elevated WBC are normal occurrences after MI as a result of inflammation that occurs after tissue necrosis. Denial is a common response in the immediate period after the MI. Cognitive Level: Application Text Reference: p. 804 Nursing Process: Assessment

Psychosocial Integrity 28. A patient is admitted to the emergency department with a stab wound to the right chest. Air can be heard entering his chest with each inspiration. To decrease the possibility of a tension pneumothorax in the patient, the nurse should a. position the patient so that the right chest is dependent. b. administer high-flow oxygen using a non-rebreathing mask. c. cover the sucking chest wound with an occlusive dressing. d. tape a nonporous dressing on three sides over the chest wound.

D Rationale: The dressing taped on three sides will allow air to escape when intrapleural pressure increases during expiration, but it will prevent air from moving into the pleural space during inspiration. Placing the patient on the right side or covering the chest wound with an occlusive dressing will allow trapped air in the pleural space and cause tension pneumothorax. The patient should receive oxygen, but this will have no effect on the development of tension pneumothorax. Cognitive Level: Application Text Reference: p. 586 Nursing Process: Implementation

Physiological Integrity 25. The nurse has received change-of-shift report about all of these patients on the telemetry unit. Which patient should the nurse see first? a. A patient with atrial fibrillation, rate 88, who has a new warfarin (Coumadin) order b. A patient with type 1 second-degree AV block, rate 60, who is dizzy when ambulating c. A patient who is in a sinus rhythm, rate 98, after having electrical cardioversion 2 hours ago d. A patient whose ICD fired three times today who is scheduled for a dose of amiodarone (Cordarone)

D Rationale: The frequent firing of the ICD indicates that the patient's ventricles are very irritable, and the priority is to assess the patient and administer the amiodarone. The other patients may be seen after the amiodarone is administered. Cognitive Level: Application Text Reference: pp. 852, 857-858 Nursing Process: Implementation

Safe and Effective Care Environment 10. After a patient with right lower-lobe pneumonia has been treated with intravenous (IV) antibiotics for 2 days, which assessment data obtained by the nurse indicates that the treatment has been effective? a. Bronchial breath sounds are heard at the right base. b. Increased vocal fremitus is palpable over the right chest. c. The patient coughs up small amounts of green mucous. d. The patient's white blood cell (WBC) count is 9000/µl.

D Rationale: The normal WBC count indicates that the antibiotics have been effective. All the other data suggest that a change in treatment is needed. Cognitive Level: Application Text Reference: p. 569 Nursing Process: Evaluation

c

Identify at least one observation made by the nurse that would indicate the presence of the following complications of aortic aneurysm repair. Bowel infarction A. a decreased or absent pulses in conjunction with cool, painful extremities below the level of repair B. cardiac dysrhythmias, chest pain C. absent bowel sounds, abdominal distention, diarrhea, bloody stools D. increased temperature and WBC; surgical sit inflammation or drainage

Physiological Integrity 6. When assessing a patient with chronic lung disease, the nurse finds a sudden onset of agitation and confusion. Which action should the nurse take first? a. Monitor the patient every 10 to 15 minutes. b. Notify the patient's health care provider immediately. c. Attempt to calm and reassure the patient. d. Assess vital signs and pulse oximetry.

D Rationale: The nurse needs to collect additional clinical data to share with the health care provider and to start interventions quickly if appropriate (e.g., increased oxygen flow if hypoxic). The change in the patient's neurologic status may indicate deterioration in respiratory function, and the health care provider should be notified immediately but only after some additional information is obtained. Monitoring the patient and attempting to calm the patient are appropriate actions, but they will not prevent further deterioration of the patient's clinical status and may delay care. Cognitive Level: Application Text Reference: pp. 1804-1805 Nursing Process: Assessment

Physiological Integrity 11. During an assessment of a 63-year-old patient at the clinic, the patient says, "I have always taken an evening walk, but lately my leg cramps and hurts after just a few minutes of walking. The pain goes away after I stop walking, though." The nurse should a. ask about any skin color changes that occur in response to cold. b. check for the presence of tortuous veins bilaterally on the legs. c. assess for unilateral swelling, redness, and tenderness of either leg. d. attempt to palpate the dorsalis pedis and posterial tibial pulses.

D Rationale: The nurse should assess for other clinical manifestations of peripheral arterial disease in a patient who describes intermittent claudication. Changes in skin color that occur in response to cold are consistent with Raynaud's phenomenon. Tortuous veins on the legs suggest venous insufficiency. Unilateral leg swelling, redness, and tenderness point to deep vein thrombosis (DVT). Cognitive Level: Application Text Reference: pp. 900-901 Nursing Process: Assessment

Physiological Integrity 4. Several hours following a surgical repair of an abdominal aortic aneurysm, the patient develops left flank pain and a urinary output of 20 ml/hr for 2 hours. The nurse notifies the health care provider and anticipates orders for a(n) a. additional antibiotic. b. increase in IV rate. c. complete blood count. d. blood urea nitrogen (BUN) and creatinine.

D Rationale: The pain and decreased urine output suggest a renal artery embolism, and monitoring of renal function is needed. The data are not consistent with the complications of infection, hypovolemia, or bleeding. Cognitive Level: Application Text Reference: p. 898 Nursing Process: Assessment

Physiological Integrity 4. A patient who is complaining of a "racing" heart and nervousness comes to the emergency department. The patient's blood pressure (BP) is 102/68. The nurse places the patient on a cardiac monitor and obtains the following ECG tracing. Which action should the nurse take next? a. Have the patient perform the Valsalva maneuver. b. Prepare to administer -blocker medication to slow the heart rate. c. Get ready to perform electrical cardioversion. d. Obtain further information about possible causes for the heart rate.

D Rationale: The patient has sinus tachycardia, which may be caused by multiple stressors such as pain, dehydration, or myocardial ischemia; further assessment is needed before determining the treatment. Vagal stimulation and electrical cardioversion are not used to treat sinus tachycardia. -blockade may be used, but further assessment is needed first. Cognitive Level: Analysis Text Reference: p. 850 Nursing Process: Implementation

Physiological Integrity 43. In developing a teaching plan for a patient who is being discharged with a warfarin (Coumadin) prescription after having a pulmonary embolus, the nurse will include information about a. where to schedule activated partial thromboplastin time testing. b. avoidance of a high protein diet. c. how to obtain enteric-coated aspirin. d. foods that are high in vitamin K.

D Rationale: The patient who is taking Coumadin should have a consistent vitamin K intake, since vitamin K interferes with the effect of the medication. INR testing, rather than aPTT testing, is used to monitor for a therapeutic level of Coumadin. Aside from vitamin K, there are no other dietary requirements associated with Coumadin use. Aspirin should be avoided when taking anticoagulant medications because of the effect on platelet function. Cognitive Level: Application Text Reference: p. 600 Nursing Process: Planning

Physiological Integrity 12. A patient in acute respiratory failure as a complication of COPD has a PaCO2 of 65 mm Hg, rhonchi audible in the right lung, and marked fatigue with a weak cough. The nurse will plan to a. allow the patient to rest to help conserve energy. b. arrange for a humidifier to be placed in the patient's room. c. position the patient on the right side with the head of the bed elevated. d. assist the patient with augmented coughing to remove respiratory secretions.

D Rationale: The patient's assessment indicates that assisted coughing is needed to help remove secretions, which will improve PaCO2 and will also help to correct fatigue. If the patient is allowed to rest, the PaCO2 will increase. Humidification may help loosen secretions, but the weak cough effort will prevent the secretions from being cleared. The patient should be positioned with the good lung down to improve gas exchange. Cognitive Level: Application Text Reference: p. 1809 Nursing Process: Planning

Physiological Integrity 15. The nurse is caring for a patient who was hospitalized 2 days earlier with aspiration pneumonia. Which assessment information is most important to communicate to the health care provider? a. The patient has a cough that is productive of blood-tinged sputum. b. The patient has scattered crackles throughout the posterior lung bases. c. The patient's temperature is 101.5° F after 2 days of IV antibiotic therapy. d. The patient's SpO2 has dropped to 90%, although the O2 flow rate has been increased.

D Rationale: The patient's dropping SpO2 despite having an increase in FIO2 indicates the possibility of acute respiratory distress syndrome (ARDS). The patient's blood-tinged sputum and scattered crackles are not unusual in a patient with pneumonia, although they do require continued monitoring. The continued temperature elevation indicates a possible need to change antibiotics, but this is not as urgent a concern as the progression toward hypoxemia despite an increase in O2 flow rate. Cognitive Level: Application Text Reference: p. 1815 Nursing Process: Assessment

A nursing responsibility in the management of a patient with a hypertensive urgency often include which of the following? A) Monitoring hourly urine output for drug effectiveness B) Providing continuous ECG monitoring to detect effects of the drugs C) Titrating intravenous drug dosages based on BP measurements every 2-3 minutes D) Instructing the patient to follow up with a health care professional 24 hours after outpatient treatment

D) Instructing the patient to follow up with a health care professional 24 hours after outpatient treatment

Physiological Integrity 26. A patient with severe COPD tells the nurse, "I wish I were dead! I cannot do anything for myself anymore." Based on this information, the nurse identifies the nursing diagnosis of a. hopelessness related to presence of long-term stress. b. anticipatory grieving related to expectation of death. c. ineffective coping related to unknown outcome of illness. d. disturbed self-esteem related to physical dependence.

D Rationale: The patient's statement about not being able to do anything for himself or herself supports this diagnosis. Although hopelessness, anticipatory grieving, and ineffective coping may also be appropriate diagnoses for patients with COPD, the patient does not mention long-term stress, death, or an unknown outcome as being concerns. Cognitive Level: Application Text Reference: p. 655 Nursing Process: Diagnosis

Physiological Integrity 17. A patient has a permanent pacemaker inserted for treatment of chronic atrial fibrillation with slow ventricular response. The nurse teaches the patient that the pacemaker will a. prevent or minimize ventricular irritability. b. discharge if ventricular fibrillation occurs and prevent cardiac arrest. c. depolarize the atria and generate a P wave. d. stimulate a heart beat if the patient's own heart rate drops too low.

D Rationale: The permanent pacemaker will discharge when the ventricular rate drops below the set rate. The pacemaker will not decrease ventricular irritability or discharge if the patient develops ventricular fibrillation. A P wave will not be generated even with a dual-chamber pacemaker because the atria are already depolarizing in atrial fibrillation. Cognitive Level: Application Text Reference: pp. 858-859 Nursing Process: Implementation

Physiological Integrity 33. A patient who has chest pain is admitted to the ED, and all the following diagnostic tests are ordered. Which one will the nurse arrange to be completed first? a. Chest x-ray b. Troponin level c. CT scan d. ECG

D Rationale: The priority for the patient is to determine whether an AMI is occurring so that reperfusion therapy can begin as quickly as possible. ECG changes occur very rapidly after coronary artery occlusion. Troponin levels will increase after about 3 hours. Data from the CT scan and chest x-ray may impact the patient's care but are not helpful in determining whether the patient is experiencing an MI. Cognitive Level: Application Text Reference: p. 805 Nursing Process: Implementation

Health Promotion and Maintenance 17. A patient admitted to the hospital with DVT has health care provider's orders for bed rest with the feet elevated. The best method for the nurse to use in elevating the patient's feet is to a. place two pillows under the calf of the affected leg. b. place the patient in the Trendelenburg position. c. elevate the bed at the knee and put pillows under the feet. d. put one pillow under the thighs and two pillows under the lower legs.

D Rationale: The purpose of elevation of the feet is to enhance venous flow from the feet to the right atrium, which is best accomplished by placing two pillows under the feet and one under the thighs. Placing the patient in the Trendelenburg position will lower the head below heart level, which is not indicated for this patient. Placing pillows under the calf or elevating the bed at the knee may cause blood stasis at the calf level. Cognitive Level: Application Text Reference: p. 916 Nursing Process: Implementation

Physiological Integrity 15. While assessing the role-relationship health pattern in a patient with respiratory problems, the nurse should specifically ask about a. any history of cigarette smoking. b. recent alterations in sexual activity. c. the course of the patient's illness. d. work exposure to respiratory irritants.

D Rationale: The role-relationship pattern includes information about the occupational exposure to fumes and allergens. History of cigarette smoking and the course of the illness are assessed in the health perception-health management pattern. Alterations in sexuality are assessed in the sexuality-reproductive pattern. Cognitive Level: Application Text Reference: p. 520 Nursing Process: Assessment

1. Following assessment of a patient with pneumonia, the nurse identifies a nursing diagnosis of ineffective airway clearance. Which information best supports this diagnosis? a. Resting pulse oximetry (SpO2) of 85% b. Respiratory rate of 28 c. Large amounts of greenish sputum d. Weak, nonproductive cough effort

D Rationale: The weak, nonproductive cough indicates that the patient is unable to clear the airway effectively. The other data would be used to support diagnoses such as impaired gas exchange and ineffective breathing pattern. Cognitive Level: Application Text Reference: p. 568 Nursing Process: Diagnosis

Physiological Integrity 2. When needing to estimate the ventricular rate quickly for a patient with a regular heart rhythm using an ECG strip, the nurse will a. print a 1-minute ECG strip and count the number of QRS complexes. b. count the number of large squares in the R-R interval and divide by 300. c. calculate the number of small squares between one QRS complex and the next and divide into 1500. d. use the 3-second markers to count the number of QRS complexes in 6 seconds and multiply by 10.

D Rationale: This is the quickest way to determine the ventricular rate for a patient with a regular rhythm. All the other methods take longer. Cognitive Level: Comprehension Text Reference: pp. 843, 845 Nursing Process: Assessment

Physiological Integrity 4. A patient is admitted to the ED after an episode of severe chest pain, and the physician schedules the patient for coronary angiography and possible percutaneous coronary intervention (PCI). The nurse prepares the patient for the procedure by explaining that it is used to a. determine whether there are any structural defects in the chambers of the heart. b. locate any coronary artery obstructions and administer thrombolytic agents. c. measure the amount of blood being pumped from the heart with each contraction. d. visualize any coronary artery blockages and dilate any obstructed arteries.

D Rationale: Visualization of the coronary arteries and possible balloon dilation are scheduled for this patient. Thrombolytic therapy is an alternative treatment if the patient is experiencing acute coronary syndrome (ACS) but is not the ordered therapy for this patient. Although angiography might help to detect structural defects or changes in cardiac output, it is not the reason for the procedure in this patient with symptoms of CAD. Cognitive Level: Application Text Reference: p. 807 Nursing Process: Implementation

1. A patient in respiratory distress is admitted to the medical unit at the hospital. During the initial assessment of the patient, the nurse should a. obtain a comprehensive health history to determine the extent of any prior respiratory problems. b. complete a full physical examination to determine the systemic effect of the respiratory distress. c. delay the physical assessment and ask family members about any history of respiratory problems. d. perform a respiratory system assessment and ask specific questions about this episode of respiratory distress.

D Rationale: When a patient has severe respiratory distress, only information pertinent to the current episode is obtained, and a more thorough assessment is deferred until later. Obtaining a comprehensive health history or full physical examination is unnecessary until the acute distress has resolved. A focused physical assessment should be done rapidly to help determine the cause of the distress and suggest treatment. Although family members may know about the patient's history of medical problems, the patient is the best informant for these data. Cognitive Level: Application Text Reference: p. 517 Nursing Process: Assessment

Physiological Integrity 10. While assessing a patient with heart failure, the nurse notes that the patient has jugular venous distension (JVD) when lying flat in bed. The nurse's next action will be to a. have the patient perform the Valsalva maneuver and observe the jugular veins. b. palpate the jugular veins and compare the volume and pressure on the both sides. c. use a centimeter ruler to measure and document accurately the level of the JVD. d. elevate the patient gradually to an upright position and examine for continued JVD.

D Rationale: When assessing for and documenting JVD, the nurse should document the angle at which the patient is positioned. When the patient is lying flat, the jugular veins are at the level of the right atrium, so JVD is a common (but not clinically significant) finding. JVD that persists when the patient is sitting at a 30- to 45-degree angle or greater is significant. The nurse may use a ruler to determine the level of JVD above the heart if the JVD persists when the patient is at 30 to 45 degree angle or more. JVD is an expected finding when a patient performs the Valsalva maneuver because right atrial pressure increases. Comparison of the volume and pressure of the jugular veins is not included in jugular vein assessment. Cognitive Level: Application Text Reference: pp. 748-749 Nursing Process: Assessment

Physiological Integrity 7. The nurse teaches a patient who has asthma about peak flow meter use. Which action by the patient indicates that teaching was successful? a. The patient inhales rapidly through the peak flow meter mouthpiece. b. The patient takes montelukast (Singulair) for peak flows in the red zone. c. The patient calls the health care provider when the peak flow is in the green zone. d. The patient uses albuterol (Proventil) metered dose inhaler (MDI) for peak flows in the yellow zone.

D Readings in the yellow zone indicate a decrease in peak flow. The patient should use short-acting 2-adrenergic (SABA) medications. Readings in the green zone indicate good asthma control. The patient should exhale quickly and forcefully through the peak flow meter mouthpiece to obtain the readings. Readings in the red zone do not indicate good peak flow, and the patient should take a fast-acting bronchodilator and call the health care provider for further instructions. Singulair is not indicated for acute attacks but rather is used for maintenance therapy. DIF: Cognitive Level: Apply (application) REF: 568 | 580 TOP: Nursing Process: Evaluation MSC:

Physiological Integrity 24. A young adult patient with cystic fibrosis (CF) is admitted to the hospital with increased dyspnea. Which intervention should the nurse include in the plan of care? a. Schedule a sweat chloride test. b. Arrange for a hospice nurse visit. c. Place the patient on a low-sodium diet. d. Perform chest physiotherapy every 4 hours.

D Routine scheduling of airway clearance techniques is an essential intervention for patients with CF. A sweat chloride test is used to diagnose CF, but it does not provide any information about the effectiveness of therapy. There is no indication that the patient is terminally ill. Patients with CF lose excessive sodium in their sweat and require high amounts of dietary sodium. DIF: Cognitive Level: Apply (application) REF: 594 TOP: Nursing Process: Planning MSC:

Integrated Process: Nursing Process (Implementation) 4. The nurse is caring for four clients with asthma. Which client does the nurse assess first? a. Client with a barrel chest and clubbed fingernails b. Client with an SaO2 level of 92% at rest c. Client whose expiratory phase is longer than the inspiratory phase d. Client whose heart rate is 120 beats/min

D Tachycardia can indicate hypoxemia as the body tries to circulate the oxygen that is available. A barrel chest is not an emergency finding. Likewise a pulse oximetry level of 92% is not considered an acute finding. The expiratory phase is expected to be longer than the inspiratory phase in someone with airflow limitation. DIF: Cognitive Level: Application/Applying or higher REF: N/A TOP: Client Needs Category: Safe and Effective Care Environment (Management of Care—Establishing Priorities)

Physiological Integrity 3. During a physical examination of a 74-year-old patient, the nurse palpates the point of maximal impulse (PMI) in the sixth intercostal space lateral to the left midclavicular line. The most appropriate action for the nurse to take next will be to a. ask the patient about risk factors for atherosclerosis. b. document that the PMI is in the normal anatomic location. c. auscultate both the carotid arteries for the presence of a bruit. d. assess the patient for symptoms of left ventricular hypertrophy.

D The PMI should be felt at the intersection of the fifth intercostal space and the left midclavicular line. A PMI located outside these landmarks indicates possible cardiac enlargement, such as with left ventricular hypertrophy. Cardiac enlargement is not necessarily associated with atherosclerosis or carotid artery disease. DIF: Cognitive Level: Apply (application) REF: 697 TOP: Nursing Process: Assessment MSC:

Physiological Integrity 24. A patient with hyperlipidemia has a new order for colesevelam (Welchol). Which nursing action is most appropriate when giving the medication? a. Have the patient take this medication with an aspirin. b. Administer the medication at the patient's usual bedtime. c. Have the patient take the colesevelam with a sip of water. d. Give the patient's other medications 2 hours after the colesevelam.

D The bile acid sequestrants interfere with the absorption of many other drugs, and giving other medications at the same time should be avoided. Taking an aspirin concurrently with the colesevelam may increase the incidence of gastrointestinal side effects such as heartburn. An increased fluid intake is encouraged for patients taking the bile acid sequestrants to reduce the risk for constipation. For maximum effect, colesevelam should be administered with meals. DIF: Cognitive Level: Apply (application) REF: 739 TOP: Nursing Process: Implementation MSC:

Physiological Integrity 13. The nurse admits a patient who has a diagnosis of an acute asthma attack. Which statement indicates that the patient may need teaching regarding medication use? a. "I have not had any acute asthma attacks during the last year." b. "I became short of breath an hour before coming to the hospital." c. "I've been taking Tylenol 650 mg every 6 hours for chest-wall pain." d. "I've been using my albuterol inhaler more frequently over the last 4 days."

D The increased need for a rapid-acting bronchodilator should alert the patient that an acute attack may be imminent and that a change in therapy may be needed. The patient should be taught to contact a health care provider if this occurs. The other data do not indicate any need for additional teaching. DIF: Cognitive Level: Apply (application) REF: 482 TOP: Nursing Process: Assessment MSC:

Physiological Integrity 13. A nurse is caring for a patient who has had a total laryngectomy and radical neck dissection. During the first 24 hours after surgery what is the priority nursing action? a. Monitor for bleeding. b. Maintain adequate IV fluid intake. c. Suction tracheostomy every eight hours. d. Keep the patient in semi-Fowler's position.

D The most important goals after a laryngectomy and radical neck dissection are to maintain the airway and ensure adequate oxygenation. Keeping the patient in a semi-Fowler's position will decrease edema and limit tension on the suture lines to help ensure an open airway. Maintenance of IV fluids and monitoring for bleeding are important, but maintaining an open airway is the priority. Tracheostomy care and suctioning should be provided as needed. During the immediate postoperative period, the patient with a laryngectomy requires frequent suctioning of the tracheostomy tube. DIF: Cognitive Level: Apply (application) REF: 516 OBJ: Special Questions: Prioritization TOP: Nursing Process: Implementation MSC:

Physiological Integrity 2. The nurse plans to teach a patient how to manage allergic rhinitis. Which information should the nurse include in the teaching plan? a. Hand washing is the primary way to prevent spreading the condition to others. b. Use of oral antihistamines for 2 weeks before the allergy season may prevent reactions. c. Corticosteroid nasal sprays will reduce inflammation, but systemic effects limit their use. d. Identification and avoidance of environmental triggers are the best way to avoid symptoms.

D The most important intervention is to assist the patient in identifying and avoiding potential allergens. Intranasal corticosteroids (not oral antihistamines) should be started several weeks before the allergy season. Corticosteroid nasal sprays have minimal systemic absorption. Acute viral rhinitis (the common cold) can be prevented by washing hands. DIF: Cognitive Level: Apply (application) REF: 500 TOP: Nursing Process: Planning MSC:

Safe and Effective Care Environment 36. A patient with a possible pulmonary embolism complains of chest pain and difficulty breathing. The nurse finds a heart rate of 142 beats/minute, blood pressure of 100/60 mmHg, and respirations of 42 breaths/minute. Which action should the nurse take first? a. Administer anticoagulant drug therapy. b. Notify the patient's health care provider. c. Prepare patient for a spiral computed tomography (CT). d. Elevate the head of the bed to a semi-Fowler's position.

D The patient has symptoms consistent with a pulmonary embolism (PE). Elevating the head of the bed will improve ventilation and gas exchange. The other actions can be accomplished after the head is elevated (and oxygen is started). A spiral CT may be ordered by the health care provider to identify PE. Anticoagulants may be ordered after confirmation of the diagnosis of PE. DIF: Cognitive Level: Apply (application) REF: 553 OBJ: Special Questions: Prioritization TOP: Nursing Process: Implementation MSC:

Physiological Integrity 9. The nurse teaches the patient being evaluated for rhythm disturbances with a Holter monitor to a. connect the recorder to a computer once daily. b. exercise more than usual while the monitor is in place. c. remove the electrodes when taking a shower or tub bath. d. keep a diary of daily activities while the monitor is worn.

D The patient is instructed to keep a diary describing daily activities while Holter monitoring is being accomplished to help correlate any rhythm disturbances with patient activities. Patients are taught that they should not take a shower or bath during Holter monitoring and that they should continue with their usual daily activities. The recorder stores the information about the patient's rhythm until the end of the testing, when it is removed and the data are analyzed. DIF: Cognitive Level: Apply (application) REF: 700 TOP: Nursing Process: Implementation MSC:

Psychosocial Integrity 19. An hour after a thoracotomy, a patient complains of incisional pain at a level 7 (based on 0 to 10 scale) and has decreased left-sided breath sounds. The pleural drainage system has 100 mL of bloody drainage and a large air leak. Which action is best for the nurse to take next? a. Milk the chest tube gently to remove any clots. b. Clamp the chest tube momentarily to check for the origin of the air leak. c. Assist the patient to deep breathe, cough, and use the incentive spirometer. d. Set up the patient controlled analgesia (PCA) and administer the loading dose of morphine.

D The patient is unlikely to take deep breaths or cough until the pain level is lower. A chest tube output of 100 mL is not unusual in the first hour after thoracotomy and would not require milking of the chest tube. An air leak is expected in the initial postoperative period after thoracotomy. DIF: Cognitive Level: Apply (application) REF: 549 | eNCP 28-2 TOP: Nursing Process: Implementation MSC:

Physiological Integrity 31. When caring for a patient with acute coronary syndrome who has returned to the coronary care unit after having angioplasty with stent placement, the nurse obtains the following assessment data. Which data indicate the need for immediate action by the nurse? a. Heart rate 102 beats/min b. Pedal pulses 1+ bilaterally c. Blood pressure 103/54 mm Hg d. Chest pain level 7 on a 0 to 10 point scale

D The patient's chest pain indicates that restenosis of the coronary artery may be occurring and requires immediate actions, such as administration of oxygen and nitroglycerin, by the nurse. The other information indicates a need for ongoing assessments by the nurse. DIF: Cognitive Level: Apply (application) REF: 746 OBJ: Special Questions: Prioritization TOP: Nursing Process: Assessment MSC:

A female client returns for a follow-up visit to the cardiologist 4 days after a trip to the ED for sudden shortness of breath and abdominal pain. The nurse realizes the client had a myocardial infarction because the results from the blood work drawn in the hospital shows: a) Increased C-reactive protein levels b) Decreased LDH levels c) Decreased myoglobin levels d) Elevated troponin levels

D. Elevated troponin levels

e

Match the following characteristics with their methods of oxygen administration: Safest system to use in patient with COPD A. Nasal cannula B. Simple face mask C. Partial rebreathing mask D. Nonrebreathing mask E. Venturi mask F. Tracheostomy collar

Physiological Integrity 23. When assessing a patient who has just arrived after an automobile accident, the emergency department nurse notes tachycardia and absent breath sounds over the right lung. For which intervention will the nurse prepare the patient? a. Emergency pericardiocentesis b. Stabilization of the chest wall with tape c. Administration of an inhaled bronchodilator d. Insertion of a chest tube with a chest drainage system

D The patient's history and absent breath sounds suggest a right-sided pneumothorax or hemothorax, which will require treatment with a chest tube and drainage. The other therapies would be appropriate for an acute asthma attack, flail chest, or cardiac tamponade, but the patient's clinical manifestations are not consistent with these problems. DIF: Cognitive Level: Apply (application) REF: 542-543 TOP: Nursing Process: Planning MSC:

Safe and Effective Care Environment 49. After change-of-shift report, which patient should the nurse assess first? a. 72-year-old with cor pulmonale who has 4+ bilateral edema in his legs and feet b. 28-year-old with a history of a lung transplant and a temperature of 101° F (38.3° C) c. 40-year-old with a pleural effusion who is complaining of severe stabbing chest pain d. 64-year-old with lung cancer and tracheal deviation after subclavian catheter insertion

D The patient's history and symptoms suggest possible tension pneumothorax, a medical emergency. The other patients also require assessment as soon as possible, but tension pneumothorax will require immediate treatment to avoid death from inadequate cardiac output or hypoxemia. DIF: Cognitive Level: Analyze (analysis) REF: 543 OBJ: Special Questions: Prioritization; Multiple Patients TOP: Nursing Process: Assessment MSC:

Physiological Integrity 21. A patient with severe chronic obstructive pulmonary disease (COPD) tells the nurse, "I wish I were dead! I'm just a burden on everybody." Based on this information, which nursing diagnosis is most appropriate? a. Complicated grieving related to expectation of death b. Ineffective coping related to unknown outcome of illness c. Deficient knowledge related to lack of education about COPD d. Chronic low self-esteem related to increased physical dependence

D The patient's statement about not being able to do anything for himself or herself supports this diagnosis. Although deficient knowledge, complicated grieving, and ineffective coping may also be appropriate diagnoses for patients with COPD, the data for this patient do not support these diagnoses. DIF: Cognitive Level: Apply (application) REF: 599-600 TOP: Nursing Process: Diagnosis MSC:

Physiological Integrity 2. When reviewing the 12-lead electrocardiograph (ECG) for a healthy 79-year-old patient who is having an annual physical examination, what will be of most concern to the nurse? a. The PR interval is 0.21 seconds. b. The QRS duration is 0.13 seconds. c. There is a right bundle-branch block. d. The heart rate (HR) is 42 beats/minute.

D The resting HR does not change with aging, so the decrease in HR requires further investigation. Bundle-branch block and slight increases in PR interval or QRS duration are common in older individuals because of increases in conduction time through the AV node, bundle of His, and bundle branches. DIF: Cognitive Level: Apply (application) REF: 691 TOP: Nursing Process: Assessment MSC:

Physiological Integrity 21. Using the illustrated technique, the nurse is assessing for which finding in a patient with chronic obstructive pulmonary disease (COPD)? a. Hyperresonance b. Tripod positioning c. Accessory muscle use d. Reduced chest expansion

D The technique for palpation for chest expansion is shown in the illustrated technique. Reduced chest movement would be noted on palpation of a patient's chest with COPD. Hyperresonance would be assessed through percussion. Accessory muscle use and tripod positioning would be assessed by inspection. DIF: Cognitive Level: Understand (comprehension) REF: 486 TOP: Nursing Process: Assessment MSC:

Physiological Integrity 2. The nurse prepares a patient with a left-sided pleural effusion for a thoracentesis. How should the nurse position the patient? a. Supine with the head of the bed elevated 30 degrees b. In a high-Fowler's position with the left arm extended c. On the right side with the left arm extended above the head d. Sitting upright with the arms supported on an over bed table

D The upright position with the arms supported increases lung expansion, allows fluid to collect at the lung bases, and expands the intercostal space so that access to the pleural space is easier. The other positions would increase the work of breathing for the patient and make it more difficult for the health care provider performing the thoracentesis. DIF: Cognitive Level: Apply (application) REF: 492 TOP: Nursing Process: Implementation MSC:

Physiological Integrity 17. A patient hospitalized with chronic obstructive pulmonary disease (COPD) is being discharged home on oxygen therapy. Which instruction should the nurse include in the discharge teaching? a. Storage of oxygen tanks will require adequate space in the home. b. Travel opportunities will be limited because of the use of oxygen. c. Oxygen flow should be increased if the patient has more dyspnea. d. Oxygen use can improve the patient's prognosis and quality of life.

D The use of home oxygen improves quality of life and prognosis. Because increased dyspnea may be a symptom of an acute process such as pneumonia, the patient should notify the physician rather than increasing the oxygen flow rate if dyspnea becomes worse. Oxygen can be supplied using liquid, storage tanks, or concentrators, depending on individual patient circumstances. Travel is possible using portable oxygen concentrators. DIF: Cognitive Level: Apply (application) REF: 592 TOP: Nursing Process: Implementation MSC:

Safe and Effective Care Environment 19. The laboratory has just called with the arterial blood gas (ABG) results on four patients. Which result is most important for the nurse to report immediately to the health care provider? a. pH 7.34, PaO2 82 mm Hg, PaCO2 40 mm Hg, and O2 sat 97% b. pH 7.35, PaO2 85 mm Hg, PaCO2 45 mm Hg, and O2 sat 95% c. pH 7.46, PaO2 90 mm Hg, PaCO2 32 mm Hg, and O2 sat 98% d. pH 7.31, PaO2 91 mm Hg, PaCO2 50 mm Hg, and O2 sat 96%

D These ABGs indicate uncompensated respiratory acidosis and should be reported to the health care provider. The other values are normal or close to normal. DIF: Cognitive Level: Apply (application) REF: 479 OBJ: Special Questions: Prioritization TOP: Nursing Process: Implementation MSC:

Physiological Integrity 38. After receiving change-of-shift report about the following four patients, which patient should the nurse assess first? a. 39-year-old with pericarditis who is complaining of sharp, stabbing chest pain b. 56-year-old with variant angina who is to receive a dose of nifedipine (Procardia) c. 65-year-old who had a myocardial infarction (MI) 4 days ago and is anxious about the planned discharge d. 59-year-old with unstable angina who has just returned to the unit after having a percutaneous coronary intervention (PCI)

D This patient is at risk for bleeding from the arterial access site for the PCI, so the nurse should assess the patient's blood pressure, pulse, and the access site immediately. The other patients should also be assessed as quickly as possible, but assessment of this patient has the highest priority. DIF: Cognitive Level: Analyze (analysis) REF: 751 OBJ: Special Questions: Prioritization; Multiple Patients TOP: Nursing Process: Assessment MSC:

Physiological Integrity 5. The nurse palpates the posterior chest while the patient says "99" and notes absent fremitus. Which action should the nurse take next? a. Palpate the anterior chest and observe for barrel chest. b. Encourage the patient to turn, cough, and deep breathe. c. Review the chest x-ray report for evidence of pneumonia. d. Auscultate anterior and posterior breath sounds bilaterally.

D To assess for tactile fremitus, the nurse should use the palms of the hands to assess for vibration when the patient repeats a word or phrase such as "99." After noting absent fremitus, the nurse should then auscultate the lungs to assess for the presence or absence of breath sounds. Absent fremitus may be noted with pneumothorax or atelectasis. The vibration is increased in conditions such as pneumonia, lung tumors, thick bronchial secretions, and pleural effusion. Turning, coughing, and deep breathing is an appropriate intervention for atelectasis, but the nurse needs to first assess breath sounds. Fremitus is decreased if the hand is farther from the lung or the lung is hyperinflated (barrel chest).The anterior of the chest is more difficult to palpate for fremitus because of the presence of large muscles and breast tissue. DIF: Cognitive Level: Apply (application) REF: 486 TOP: Nursing Process: Assessment MSC:

Physiological Integrity 4. Which information will the nurse include in the asthma teaching plan for a patient being discharged? a. Use the inhaled corticosteroid when shortness of breath occurs. b. Inhale slowly and deeply when using the dry powder inhaler (DPI). c. Hold your breath for 5 seconds after using the bronchodilator inhaler. d. Tremors are an expected side effect of rapidly acting bronchodilators.

D Tremors are a common side effect of short-acting 2-adrenergic (SABA) medications and not a reason to avoid using the SABA inhaler. Inhaled corticosteroids do not act rapidly to reduce dyspnea. Rapid inhalation is needed when using a DPI. The patient should hold the breath for 10 seconds after using inhalers. DIF: Cognitive Level: Apply (application) REF: 572 TOP: Nursing Process: Implementation MSC:

Physiological Integrity 8. A patient who has had chest pain for several hours is admitted with a diagnosis of rule out acute myocardial infarction (AMI). Which laboratory test should the nurse monitor to help determine whether the patient has had an AMI? a. Myoglobin b. Homocysteine c. C-reactive protein d. Cardiac-specific troponin

D Troponin levels increase about 4 to 6 hours after the onset of myocardial infarction (MI) and are highly specific indicators for MI. Myoglobin is released within 2 hours of MI, but it lacks specificity and its use is limited. The other laboratory data are useful in determining the patient's risk for developing coronary artery disease (CAD) but are not helpful in determining whether an acute MI is in progress. DIF: Cognitive Level: Understand (comprehension) REF: 749 TOP: Nursing Process: Assessment MSC:

Physiological Integrity 6. While doing the admission assessment for a thin 76-year-old patient, the nurse observes pulsation of the abdominal aorta in the epigastric area. Which action should the nurse take? a. Teach the patient about aneurysms. b. Notify the hospital rapid response team. c. Instruct the patient to remain on bed rest. d. Document the finding in the patient chart.

D Visible pulsation of the abdominal aorta is commonly observed in the epigastric area for thin individuals. The nurse should simply document the finding in the admission assessment. Unless there are other abnormal findings (such as a bruit, pain, or hyper/hypotension) associated with the pulsation, the other actions are not necessary. DIF: Cognitive Level: Apply (application) REF: 695 | 697 TOP: Nursing Process: Assessment MSC:

Health Promotion and Maintenance 9. A patient scheduled for a total laryngectomy and radical neck dissection for cancer of the larynx asks the nurse, "Will I be able to talk normally after surgery?" What is the best response by the nurse? a. "You will breathe through a permanent opening in your neck, but you will not be able to communicate orally." b. "You won't be able to talk right after surgery, but you will be able to speak again after the tracheostomy tube is removed." c. "You won't be able to speak as you used to, but there are artificial voice devices that will give you the ability to speak normally." d. "You will have a permanent opening into your neck, and you will need to have rehabilitation for some type of voice restoration."

D Voice rehabilitation is planned after a total laryngectomy, and a variety of assistive devices are available to restore communication. Although the ability to communicate orally is changed, it would not be appropriate to tell a patient that this ability would be lost. Artificial voice devices do not permit normal-sounding speech. In a total laryngectomy, the vocal cords are removed, so normal speech is impossible. DIF: Cognitive Level: Apply (application) REF: 516 TOP: Nursing Process: Implementation MSC:

Physiological Integrity 8. While assessing a patient who was admitted with heart failure, the nurse notes that the patient has jugular venous distention (JVD) when lying flat in bed. Which action should the nurse take next? a. Document this finding in the patient's record. b. Obtain vital signs, including oxygen saturation. c. Have the patient perform the Valsalva maneuver. d. Observe for JVD with the patient upright at 45 degrees.

D When the patient is lying flat, the jugular veins are at the level of the right atrium, so JVD is a common (but not a clinically significant) finding. Obtaining vital signs and oxygen saturation is not warranted at this point. JVD is an expected finding when a patient performs the Valsalva maneuver because right atrial pressure increases. JVD that persists when the patient is sitting at a 30- to 45-degree angle or greater is significant. The nurse will document the JVD in the medical record if it persists when the head is elevated. DIF: Cognitive Level: Apply (application) REF: 694 | 696 TOP: Nursing Process: Assessment MSC:

b

Match the following characteristics with their methods of oxygen administration: Used to given oxygen quickly for short time A. Nasal cannula B. Simple face mask C. Partial rebreathing mask D. Nonrebreathing mask E. Venturi mask F. Tracheostomy collar

d

Identify at least one observation made by the nurse that would indicate the presence of the following complications of aortic aneurysm repair. Graft infection A. a decreased or absent pulses in conjunction with cool, painful extremities below the level of repair B. cardiac dysrhythmias, chest pain C. absent bowel sounds, abdominal distention, diarrhea, bloody stools D. increased temperature and WBC; surgical sit inflammation or drainage

The nurse instructs a 68-year-old woman with hypercholesterolemia about natural lipid-lowering therapies. The nurse determines further teaching is necessary if the patient makes which statement?

D "I will take garlic instead of my prescription medication to reduce my cholesterol."

The patient is being dismissed from the hospital after ACS and will be attending rehabilitation. What information does the patient need to be taught about the early recovery phase of rehabilitation?

D Activity level is gradually increased under cardiac rehabilitation team supervision and with ECG monitoring.

For which problem is percutaneous coronary intervention (PCI) most clearly indicated?

D Acute myocardial infarction

When evaluating a patient's knowledge regarding a low-sodium, low-fat cardiac diet, the nurse recognizes additional teaching is needed when the patient selects which food choice?

D Canned chicken noodle soup

A patient was admitted to the emergency department (ED) 24 hours earlier with complaints of chest pain that were subsequently attributed to ST-segment-elevation myocardial infarction (STEMI). What complication of MI should the nurse anticipate?

D Cardiac dysrhythmias

A patient experienced sudden cardiac death (SCD) and survived. What should the nurse expect to be used as preventive treatment for the patient?

D Implantable cardioverter-defibrillator (ICD)

A female patient who has type 1 diabetes mellitus has chronic stable angina that is controlled with rest. She states that over the past few months she has required increasing amounts of insulin. What goal should the nurse use to plan care that should help prevent cardiovascular disease progression?

D Keep Hgb A1C less than 7%.

A male patient who has coronary artery disease (CAD) has serum lipid values of LDL cholesterol 98 mg/dL and HDL cholesterol 47 mg/dL. What should the nurse include in the patient teaching?

D The lipid levels are normal.

Which of the following should be included in the teaching plan for a patient receiving glargine (Lantus),"peakless" basal insulin? a) It is rapidly absorbed, has a fast onset of action. b) Draw up the drug first, then add regular insulin. c) Administer the total daily dosage in two doses. d) Do not mix with other insulins.

D) Do not mix with other insulins

25. A nurse is caring for a patient with COPD. The patient's medication regimen has been recently changed and the nurse is assessing for therapeutic effect of a new bronchodilator. What assessment parameters suggest a consequent improvement in respiratory status? Select all that apply. A) Negative sputum culture B) Increased viscosity of lung secretions C) Increased respiratory rate D) Increased expiratory flow rate E) Relief of dyspnea

D, E Feedback: The relief of bronchospasm is confirmed by measuring improvement in expiratory flow rates and volumes (the force of expiration, how long it takes to exhale, and the amount of air exhaled) as well as by assessing the dyspnea and making sure that it has lessened. Increased respiratory rate and viscosity of secretions would suggest a worsening of the patient's respiratory status. Bronchodilators would not have a direct result on the patient's infectious process.

Shortly after being admitted to the coronary care unit with an acute myocardial infarction (MI), a client reports midsternal chest pain radiating down his left arm. The nurse notes that the client is restless and slightly diaphoretic, and measures a temperature of 99.6° F (37.6° C); a heart rate of 102 beats/minute; regular, slightly labored respirations at 26 breaths/minute; and a blood pressure of 150/90 mm Hg. Which nursing diagnosis takeshighest priority? a) Decreased cardiac output b) Risk for imbalanced body temperature c) Anxiety d) Acute pain

D. Acute pain

The principal goals of therapy for older patients who have poor glycemic control are: "A. Enhancing quality of life. B. Decreasing the chance of complications. C. Improving self-care through education. D. All of the above."

D. All of the above. Rationale: The principal goals of therapy for older persons with diabetes mellitus and poor glycemic control are enhancing quality of life, decreasing the chance of complications, improving self-care through education, and maintaining or improving general health status.

The nurse is caring for a patient who was admitted to the telemetry unit with a diagnosis of rule/out acute MI. The patient's chest pain began 3 hours ago. Which of the following laboratory tests would be most helpful in confirming the diagnosis of a current MI? a) Troponin C level b) Myoglobin level c) CK-MM d) Creatinine kinase-myoglobin (CK-MB) level

D. CK-MB level

"A nurse shoud recognize which symptom as a cardinal sign of diabetes mellitus? "a. Nausea b. Seizure c. Hyperactivity d. Frequent urination

D. Frequent Urination Polyphagia, polyuria, polydipsia, and weight loss are cardinal signs of DM. Other signs include irritability, shortened attention span, lowered frustration tolerance, fatigue, dry skin, blurred vision, sores that are slow to heal, and flushed skin."

In the treatment of coronary artery disease (CAD), medications are often ordered to control blood pressure in the client. Which of the following is a primary purpose of using beta-adrenergic blockers in the nursing management of CAD? a) To dilate coronary arteries b) To prevent angiotensin II conversion c) To decrease workload of the heart d) To decrease homocysteine levels

D. To decrease workload of the heart

The patient has had biomarkers drawn after complaining of chest pain. Which diagnostic of myocardial infarction remains elevated for as long as 3 weeks? a) Total CK b) Myoglobin c) CK-MB d) Troponin

D. Troponin

During the insertion of a rigid scope for bronchoscopy, a client experiences a vasovagal response. The nurse should expect: a) the client's pupils to become dilated. b) a decrease in the client's gastric secretions. c) the client to experience bronchodilation. d) a drop in the client's heart rate

D. a drop in the client's heart rate

When the postcardiac surgery patient demonstrates restlessness, nausea, weakness, and peaked T waves, the nurse reviews the patient's serum electrolytes anticipating which abnormality? a) Hyponatremia b) Hypercalcemia c) Hypomagnesemia d) Hyperkalemia

D. hyperkalemia

The nurse knows that women and the elderly are at greater risk for a fatal myocardial event. Which factor is the primary contributor of this cause? a) Chest pain is typical b) Gender bias c) Decreased sensation to pain d) Vague symptoms

D. vague symptoms

Neurohormonal response (compensatory mechanism) RAAS

Decrease in CO-> decrease in blood flow to kidneys -kidneys sense this as decreased volume, juxtaglomerular apparatus -renin-angiotensin-aldosterone-system (RAAS) response causes: *sodium and water retention *peripheral vasoconstriction-> increase in BP

Diuretics

Decrease volume overload, preload Promote sodium and water excretion Monitor K+ levels -Furosemide (Lasix) -Bumetanide (Bumex)

Frank-Starling Law

Degree of stretch is directly related to the face of the contraction (systole)

Endomyocardial biopsy

Diagnosis of HF -determining new HF that is unresponsive

Echocardiography, nuclear imaging

Diagnosis of HF What is the function? How are the valves working? Measure EF Ischemia, arrhythmias

Mixed HF, systolic and diastolic

Diseases with dilated cardiomyopathy (DCM) -poor EF (<35%) -high pulmonary pressure -biventricular failure, dilated, poor filling, emptying capacity

Coronary Artery Disease

Due to Atherosclerosis-overgrowth of muscle cells causing narrowing of blood vessels decreasing blood flow to the heart and major organs.

d (The tripod position with an elevated backrest and supported upper extremities to fix the shoulder girdle maximizes respiratory excursion and an effective breathing pattern.)

During an acute exacerbation of COPD, the patient is severely short of breathe and the nurse identifies a nursing diagnosis of ineffective breathing pattern related to obstruction of airflow and anxiety. The best action by the nurse is to: A. prepare to administer bronchodilator medications B. perform chest physiotherapy to promote removal of secretions C. administer oxygen at 5L/min until the shortness of breath is relieved D. position the patient upright with the elbows resting on the over-the-bed table

b (A second dose of the pneumonococcal vaccine should be provided to all persons 65y or older who have not received vaccine within 5 years or were younger than 65y at time of vaccination. Influenza vaccine should be taken each year by those older than 65yo. Antibiotic therapy is not appropriate for all upper respiratory infections unless secondary bacterial infections develop.)

During an annual health assessment of a 65yo clinic patient, the patient tells the nurse he had the pneumonia vaccine when he was 58. The nurse advises the patient that the best way for him to prevent pneumonia now is to: a)seek medical care and antibiotic therapy for all upper respiratory infections b)obtain the pneumococcal vaccine this year with an annual influenza vaccine c)obtain the pneumococcal vaccine if he is exposed to individuals with pneumonia d)obtain only the influenza vaccine every year because he has immunity to the pneumococcus

b (Increased tactile fremitus over the area of pulmonary consolidation is expected with bacterial pneumonias. Dullness to percussion would be expected. Pneumococcal pneumonia typically presents with a loose, productive cough. Adventitious breath sounds such as crackles and wheezes are typical.)

During assessment of the chest in a patient with pneumococcal pneumonia, the nurse would expect to find A. vesicular breath sounds. B. increased tactile fremitus. C. dry, nonproductive cough. D. hyperresonance to percussion.

c (Loss of palpable pulses and numbness and tingling of the extremity are indications of occlusion of the bypass graft and need immediate medical attention. Pain, redness, and serous drainage at the incision site are expected postop, but decreasing ankle-brachial indices may indicate graft obstruction.)

During care of the patient following femoral bypass graft surgery, the nurse immediately notifies the health care provider if the patient experiences... A. Fever and redness at the incision site B. 2+ edema of the extremity and pain at the incision site C. A loss of palpable pulses and numbness and tingling of the feet D. Decreasing ankle-brachial indices and serous drainage from the incision

b

During conventional aortic aneurysm repair, a ______ _______ is sutured to the aorta above and below the aneurysm, and the native aorta is replaced around the site. A. saccular B. synthetic graft C. all D. iliac E. endovascular graft f. renal

b (the cause of the aneurysm is a systemic vascular disease, because atherosclerosis is a systemic disease, the patient with an AAA is likely to have cardiac, pulmonary, cerebral, or lower-extremity vascular problems that should be noted and monitored throughout the perioperative period.)

During preop preparation of the patient scheduled for a AAA, the nurse establishes baseline data for the patient knowing that... A. All physiologic processes will be altered postop B. The cause of the aneurysm is a systemic vascular disease C. Surgery will be cancelled if any physiologic function is not normal D. BP and heart rate (HR) will be maintained well below the baseline levels during the postop peroid

4 (Rationale: Clinical manifestations of a pleural effusion include absent or distant breath sounds over the affected area, progressive dyspnea, decreased movement of the chest wall on the affected side, pleuritic pain, and dullness to percussion.)

During the assessment of a patient with pneumonia, the nurse suspects the development of a pleural effusion upon finding 1. a barrel chest. 2. paradoxical respirations. 3. hyperresonance on percussion. 4. localized absence of breath sounds.

d (The onset of aortic dissection involving the distal descending aorta is usually characterized by a sudden, severe, tearing pain in the back, and as it progresses down the aorta, the kidneys, abdominal organs, and lower extremities may begin to show evidence of ischemia. Aortic dissections of the ascending aorta and aortic arch may affect the heart and circulation to the head, with the development of murmurs, ventricular failure, and cerebral ischemia.)

During the nursing assessment of the patient with a distal descending aortic dissection, the nurse would expect the patient to manifest what? A. Cardiac murmur characteristic of aortic valve insufficiency B. Altered LOC with dizziness and weak carotid pulses C. Severe hypotension and orthopnea and dyspnea of pulmonary edema D. Severe "ripping" back or abdominal pain with decreasing urine output

b (the BP and all peripheral pulses are evaluated at least every hour in the acute postop period to ensure that BP is adequate and that extremities are being perfused. BP is kept within normal range. If it is too low, thrombosis of the graft may occur; if it is too high it may cause leaking or rupture at the suture line. Hypotheramis is induced during surgery, but the patient is rewarmed as soon as surgery is completed. Fluid replacement to maintain urine output at 100 mL/hr would increase the BP too much)

During the patient's acute post op period following repair of an aneurysm, the nurse should ensure that a) hypothermia is maintained to decrease oxygen need. b) the BP and all peripheral pulses are evaluated at least every hour. c) IV fluids are administered at a rate to maintain hourly urine output of 100 mL. d) the patient's BP is kept lower than baseline to prevent leaking at the suture line.

hypovolemia

ECF volume deficit (hypervolemia/hypovolemia)

hypervolemia

ECF volume excess (hypervolemia/hypovolemia)

Tachycardia (symptom of HF)

Early clinical symptom Compensate for failing ventricle-> increase HR Decrease CO activates SNS May be reduced or absent in pt on beta-blockers

Fatigue (symptom of HF)

Early symptom Unable to perform usual activities Caused by: decreased CO, impaired perfusion, decreased oxygenation, anemia

Acute Decompensated HF (ADHF)

Early: increased pulmonary venous pressure, increased RR, decreased PaO2 Later: interstitial edema, tachypnea, SOB End: alveolar edema, respiratory acidosis

a

Effect of smoking on the follow; acute, long-term: Alveolar macrophages A. Decreased Function; increased risk for infection B. Decreased taste; cancer C. Paralysis, sputum accumulation cough; chronic bronchitis, cancer D. Hoarseness; chronic cough, cancer E. Increased secretions and cough; hyperplasia of glands, chronic bronchitis F. Decreased smell; cancer G. Bronchospasm, cough; chronic bronchitis, asthma, cancer

g

Effect of smoking on the follow; acute, long-term: Bronchioles A. Decreased Function; increased risk for infection B. Decreased taste; cancer C. Paralysis, sputum accumulation cough; chronic bronchitis, cancer D. Hoarseness; chronic cough, cancer E. Increased secretions and cough; hyperplasia of glands, chronic bronchitis F. Decreased smell; cancer G. Bronchospasm, cough; chronic bronchitis, asthma, cancer

c

Effect of smoking on the follow; acute, long-term: Cilia A. Decreased Function; increased risk for infection B. Decreased taste; cancer C. Paralysis, sputum accumulation cough; chronic bronchitis, cancer D. Hoarseness; chronic cough, cancer E. Increased secretions and cough; hyperplasia of glands, chronic bronchitis F. Decreased smell; cancer G. Bronchospasm, cough; chronic bronchitis, asthma, cancer

e

Effect of smoking on the follow; acute, long-term: Mucous glands A. Decreased Function; increased risk for infection B. Decreased taste; cancer C. Paralysis, sputum accumulation cough; chronic bronchitis, cancer D. Hoarseness; chronic cough, cancer E. Increased secretions and cough; hyperplasia of glands, chronic bronchitis F. Decreased smell; cancer G. Bronchospasm, cough; chronic bronchitis, asthma, cancer

f

Effect of smoking on the follow; acute, long-term: Nasopharyngeal A. Decreased Function; increased risk for infection B. Decreased taste; cancer C. Paralysis, sputum accumulation cough; chronic bronchitis, cancer D. Hoarseness; chronic cough, cancer E. Increased secretions and cough; hyperplasia of glands, chronic bronchitis F. Decreased smell; cancer G. Bronchospasm, cough; chronic bronchitis, asthma, cancer

b

Effect of smoking on the follow; acute, long-term: Tongue A. Decreased Function; increased risk for infection B. Decreased taste; cancer C. Paralysis, sputum accumulation cough; chronic bronchitis, cancer D. Hoarseness; chronic cough, cancer E. Increased secretions and cough; hyperplasia of glands, chronic bronchitis F. Decreased smell; cancer G. Bronchospasm, cough; chronic bronchitis, asthma, cancer

d

Effect of smoking on the follow; acute, long-term: Vocal cords A. Decreased Function; increased risk for infection B. Decreased taste; cancer C. Paralysis, sputum accumulation cough; chronic bronchitis, cancer D. Hoarseness; chronic cough, cancer E. Increased secretions and cough; hyperplasia of glands, chronic bronchitis F. Decreased smell; cancer G. Bronchospasm, cough; chronic bronchitis, asthma, cancer

true

Indicate whether the statement is true or false: Cystic fibrosis has the highest incidence in whites

Pt's w/ varicosities should do what several times/day?

Elevate feet 15-20 minutes. - sleep w/ legs slightly above the heart

Catecholamines, Compensatory mechanism, HF

Epinephrine and norepinephrine -increase HR and myocardial contractility -peripheral vasoconstriction -increase in preload -increase venous return and peripheral vascular resistance on already failing heart -increases workload and O2 demand of heart

The most important measure in the tx of venous stasis ulcers is:

Extrinsic compression

Clinical sign of HF (9)

FACES: fatigue, limited activities, chest congestion, edema, SOB 1) fatigue 2) dyspnea 3) tachycardia 4) edema 5) nocturia 6) skin changes 7) behavioral changes 8) chest pain 9) weight changes 10) muscle wasting, thin arms and legs

false (african americans)

Indicate whether the statement is true or false: Female whites have the highest mortality rates from asthma

c (The initial action by the nurse will be to assess the circulation to the leg and to observe for any evidence of injury such as fractures or dislocations. After the initial assessment, the other actions may be appropriate based on what is observed during the assessment.)

Following a motor vehicle accident, a patient arrives in the emergency department with massive right lower leg swelling. Which action will the nurse take first? A. Elevate the leg on pillows. B. Apply a compression bandage. C. Check leg pulses and sensation. D. Place ice packs on the lower leg.

4 (Rationale: A decreased or absent pulse together with a cool, pale, mottled, or painful extremity may indicate embolization or graft occlusion.)

Following an aortic aneurysm repair, the patient suddenly develops severe pain in the right lower extremity. The right pedal pulse is decreased, and the right foot is cool and pale. The nurse suspects 1. hypothermia. 2. a wound infection. 3. bleeding from the graft site. 4. an embolization or graft occlusion.

a (A change in level of consciousness and ability to speak; because during a repair of an AAA, the blood supply to the carotid arteries may be interrupted, leading to neurologic complications manifested by a decreased LOC and altered pupil responses to light as well as changes in facial symmetry, speech, and movement of upper extremities. The thorax is opened is opened for ascending aortic surgery, and shallow breathing, poor cough, and decreasing chest drainage are expected. Often, lower limb pulses are normally decreased or absent for a short time following surgery.)

Following an ascending aortic aneurysm repair, which of the following findings should the nurse report immediately to the health care provider? a) a change in LOC and ability to speak b) shallow respirations and poor coughing c) decreased drainage from the chest tubes d) lower-extremity pulses that are decreased from preoperative baseline

a (Oxygen saturation obtained by pulse oximetry should be between 90-100%. An SpO₂ lower than 90% indicates a hypoxemia and impaired gas exchange. Crackles, purulent sputum, and fever are all symptoms but do not necessarily relate to impaired gas exchange.)

Following assessment of a patient with pneumonia, the nurse identifies a nursing diagnosis of impaired gas exchange is based on the findings of: a)SpO₂ of 86% b)crackles in both lower lobes c)temperature of 101.4°F (38.6°C) d)production of greenish purulent sputum

a (The weak, nonproductive cough indicates that the patient is unable to clear the airway effectively. The other data would be used to support diagnoses such as impaired gas exchange and ineffective breathing pattern.)

Following assessment of a patient with pneumonia, the nurse identifies a nursing diagnosis of ineffective airway clearance. Which information best supports this diagnosis? A. Weak, nonproductive cough effort B. Large amounts of greenish sputum C. Respiratory rate of 28 breaths/minute D. Resting pulse oximetry (SpO2) of 85%

d (I should take the pulses in my extremities and let the doctor know if they get too fast or too slow; patients are taught to palpate peripheral pulses to identify changes in their quality or strength, but the rate is not a significant factor in peripheral perfusion. The color and temperature of the extremities are also important for the patient to observe. The remaining statements are all true)

Following discharge teaching with a male patient with an AAA repair, the nurse determines that further instruction is needed when the patient says, a) I should avoid heavy lifting b) I may have some permanent sexual dysfunction as a result of the surgery c) I should maintain a low-fat and low-cholesterol diet to help keep the new graft open d) I should take the pulses in my extremities and let the doctor know if they get too fast or too slow.

c ( Oral anticoagulants, warfarin, are not recommended for treatment of peripheral artery disease, but all the other statements are correct in relation to treatment of peripheral artery disease)

Following teaching about medications for PAD, the nurse determines that additional instruction is necessary when the patient says, a) I should take one ASA a day to prevent clotting in my legs b) The lisinipril (Zestril) I use for my BP may help me walk further without pain c) I will need to have frequent blood tests to evaluate the effect of the oral anticoagulant I will be taking. d) Pletal should help me be able to increase my walking distance and keep clots from forming in my legs

What herbal products can help improve circulation?

Ginkgo, ginseng - decrease platelet aggregation.

Post-op, how often would you check weight and output to ax kidney perfusion status?

Hourly - >30mL/hour

true

Indicate whether the statement is true or false: The incidence of COPD is increasing in women

Ultrafiltration (aquapheresis)

HF pt with fluid overload, resistant to diuretics -get the pt out of crisis

Another term for Diastolic HF

HF with normal EF

false (lower)

Indicate whether the statement is true or false: The mortality rate from asthma is greater in men than in women

etiology of heart risk factors

Hypertension, CAD primary risk factors Other risk factors: diabetes, advanced age, tobacco use, obesity, high serum cholesterol

hypokalemia

IV infusions must be slowly diluted and given slowly with a rate not to exceed 10 to 20 mEq/hr for this electrolyte imbalance (Hyperkalemia, Hypokalemia)

b

Identify at least one observation made by the nurse that would indicate the presence of the following complications of aortic aneurysm repair. Myocardial ischemia A. a decreased or absent pulses in conjunction with cool, painful extremities below the level of repair B. cardiac dysrhythmias, chest pain C. absent bowel sounds, abdominal distention, diarrhea, bloody stools D. increased temperature and WBC; surgical sit inflammation or drainage

c

Identify the factor of Virchow's triad present in each of the following conditions associated with venous thromboembolism (VTE). Estrogen therapy A. damage to the endothelium B. venous stasis C. hypercoagulability

a

Identify the factor of Virchow's triad present in each of the following conditions associated with venous thromboembolism (VTE). IV therapy A. damage to the endothelium B. venous stasis C. hypercoagulability

b

Identify the factor of Virchow's triad present in each of the following conditions associated with venous thromboembolism (VTE). Orthopedic surgery A. damage to the endothelium B. venous stasis C. hypercoagulability

b

Identify the factor of Virchow's triad present in each of the following conditions associated with venous thromboembolism (VTE). Pregnancy A. damage to the endothelium B. venous stasis C. hypercoagulability

b

Identify the factor of Virchow's triad present in each of the following conditions associated with venous thromboembolism (VTE). Prolonged immobilization A. damage to the endothelium B. venous stasis C. hypercoagulability

c

Identify the factor of Virchow's triad present in each of the following conditions associated with venous thromboembolism (VTE). Smoking A. damage to the endothelium B. venous stasis C. hypercoagulability

A pt w/ a dissection of the arch of the aorta has a cecreased LOC and weak carotid pulses. The nurse anticipates that the initial tx will include:

Immediate surgery to replace the torn area w/ a graft.

d (Many patients with aortic aneurysms also have peripheral arterial disease, so the nurse should check the preoperative assessment to determine whether pulses were present before surgery before notifying the health care providers about the absent pulses. Because the patient's symptoms may indicate graft occlusion or multiple emboli and a possible need to return to surgery, it is not appropriate to wait 30 minutes before taking action. Warm blankets will not improve the circulation to the patient's legs.)

Immediately after repair of an abdominal aortic aneurysm, a patient has absent popliteal, posterior tibial, and dorsalis pedis pulses. The legs are cool and mottled. Which action should the nurse take first? A. Wrap both the legs in warm blankets. B. Notify the surgeon and anesthesiologist. C. Document that the pulses are absent and recheck in 30 minutes. D. Review the preoperative assessment form for data about the pulses.

Nursing diagnosis, HF

Impaired gas exchange Decreased cardiac output Excess fluid volume Activity intolerance

d (smoking cessation is one of the most important factors in preventing further damage to the lungs in COPD, but prevention of infections that further increase lung damage is also important. The pt is very susceptible to infections, and infections make the disease worse, creating a vicious cycle. Bronchodilators, inhaled steroids, and lung-volume-reduction surgery help to control symptoms, but these are symptomatic measures)

In addition to smoking cessation, treatment for COPD that is indicated to slow the progression of the disease includes A. use of bronchodilator drugs B. use of inhaled corticosteroids C. lung-volume-reduction surgery D. prevention of respiratory tract infections

d (A patient having an ORIF is at risk for problems such as wound infection and osteomyelitis. After an ORIF, patients typically are mobilized starting the first postoperative day, so problems caused by immobility are not as likely.)

In developing a care plan for a patient with an open reduction and internal fixation (ORIF) of an open, displaced fracture of the tibia, the priority nursing diagnosis is A. activity intolerance related to deconditioning. B. risk for constipation related to prolonged bed rest. C. risk for impaired skin integrity related to immobility. D. risk for infection related to disruption of skin integrity.

b (Rationale: Because the patient has impaired circulation and sensation to the feet, the use of a heating pad could lead to burns. The other patient statements are correct and indicate that teaching has been successful)

In evaluating the patient outcomes following teaching for a patient with chronic peripheral artery disease (PAD), the nurse determines a need for further instruction when the patient says, A. "I will have to buy some loose clothing that does not bind across my legs or waist." B. "I will use a heating pad on my feet at night to increase the circulation and warmth in my feet." C. "I will walk to the point of pain, rest, and walk again until I develop pain for a half hour daily." D. "I will change my position every hour and avoid long periods of sitting with my legs down."

d (Mucous production is increased in bronchiectasis and collects in the dilated, pouched bronchi. A major goal of treatment is to promote drainage and removal of the mucous, primarily through deep breathing, coughing, and postural drainage.)

In planning care for the patient with bronchiectasis, the nurse includes measures that will: A, Relieve or reduce pain B. Prevent paroxysmal coughing C. Prevent spread of the disease to others D. Promote drainage and removal of mucus

c (Many postural drainage positions require placement in Trendelenburg position, but patients with heart disease, hemoptysis, chest trauma, or severe dyspnea should not be place in these positions. Postural drainage should be done 1hr before and 3hrs after meals if possible. Coughing, percussion, and vibration are all performed after the patient has been positioned)

In planning for postural drainage for the patient with COPD, the nurse: A. schedules the procedure 1hr before and after meals B. has the patient cough before positioning to clear the lungs C. assesses the patient's tolerance for dependent (head-down) positions D. ensures that percussion and vibration are performed before positioning the patient

d (Compression is essential for treating chronic venous insufficiency, CVI, healing venous ulcers, and preventing ulcer recurrence. Use of custom-fitted graduated compression stockings is one option for compression therapy.)

In planning patient care and teaching for the patient with venous leg ulcers, the nurse recognizes that the most important intervention in healing and control of this condition is: A. sclerotherapy B. taking horse chestnut extract daily C. using moist environment dressing D. applying graduated compression stockings

c (SOB usually increases during exercise, but the activity is not being overdone if breathing returns to baselines within 5min after stopping)

In teaching the patient with COPD about the need for physical exercise, the nurse informs the patient that : A. All patients with COPD should be able to increase walking gradually up to 20min/day B. A bronchodilator inhaler should be used to relieve exercise-induced dyspnea immediately after exercise C. Shortness of breath is expected during exercise but should return to baseline within 5 minutes after the exercise D. Monitoring the HR before and after exercise is the best way to determine how much exercise can be tolerated

c, d, b, e, a, f (The initial actions should be to ensure that airway, breathing, and circulation are intact. This should be followed by checking the neurovascular status of the leg (before and after splint application). Application of a splint to immobilize the leg should be done before sending the patient for x-rays. The tetanus prophylaxis is the least urgent of the actions.)

In which order will the nurse take these actions when caring for a patient with left leg fractures after a motor vehicle accident? Put a comma and space between each answer choice (a, b, c, d, etc.). _____________________ a. Obtain x-rays. b. Check pedal pulses. c. Assess lung sounds. d. Take blood pressure. e. Apply splint to the leg. f. Administer tetanus prophylaxis.

Hypertrophy (compensatory mechanism)

Increase in muscle mass and cardiac wall thickness in response to overwork and strain -slow response to increased work and strain Causes: *poor contractility *requires more O2 *poor coronary artery circulation, heart can become ischemic

Positive iontropes

Increase myocardial contractility, therapeutic, sign that HF is worsening -beta-adrenergic antagonists= dopamine, dobutamine -Phosphodiesterase inhibitors -Digitalis

Precipitating causes of HF

Increase workload of the ventricles, acute condition, decreased heart function

Pleural effusion

Increased pressure in pulmonary capillaries -HF complication

Nocturia (symptom of HF)

Increased renal blood flow and diuresis When they lie down to sleep fluid moves from interstitial space back into circulatory system, cardiac workload decreases at rest

true

Indicate whether the statement is true or false: Women with COPD respond better to oxygen therapy than do men

b

Indicate the role or relationship of the following agent to asthma: B-adrenergic blocking agents A. Are associated with the asthma triad-people with nasal polyps, asthma, and sensitivity to salicylates and NSAIDs B. Contraindicated for patients with asthma because they prevent bronchodilation C. Contain sulfites that are common triggers of asthma

c

Indicate the role or relationship of the following agent to asthma: Beer and wine A. Are associated with the asthma triad-people with nasal polyps, asthma, and sensitivity to salicylates and NSAIDs B. Contraindicated for patients with asthma because they prevent bronchodilation C. Contain sulfites that are common triggers of asthma

a

Indicate the role or relationship of the following agent to asthma: Salicylates A. Are associated with the asthma triad-people with nasal polyps, asthma, and sensitivity to salicylates and NSAIDs B. Contraindicated for patients with asthma because they prevent bronchodilation C. Contain sulfites that are common triggers of asthma

b

Indicate whether the following clinical manifestations are most characteristic of Asthma, COPD , or Both: Barrel Chest A. Asthma B. COPD C. Both

b

Indicate whether the following clinical manifestations are most characteristic of Asthma, COPD , or Both: Cor pulmonale A. Asthma B. COPD C. Both

c

Indicate whether the following clinical manifestations are most characteristic of Asthma, COPD , or Both: Decreased Breath Sounds A. Asthma B. COPD C. Both

b

Indicate whether the following clinical manifestations are most characteristic of Asthma, COPD , or Both: Flattened diaphragm A. Asthma B. COPD C. Both

b

Indicate whether the following clinical manifestations are most characteristic of Asthma, COPD , or Both: Frequent Sputum production A. Asthma B. COPD C. Both

a

Indicate whether the following clinical manifestations are most characteristic of Asthma, COPD , or Both: Increased fractional exhaled nitric oxide (FENO) A. Asthma B. COPD C. Both

c

Indicate whether the following clinical manifestations are most characteristic of Asthma, COPD , or Both: Increased total lung capacity A. Asthma B. COPD C. Both

c

Indicate whether the following clinical manifestations are most characteristic of Asthma, COPD , or Both: Persistent cough A. Asthma B. COPD C. Both

b

Indicate whether the following clinical manifestations are most characteristic of Asthma, COPD , or Both: Polycythemia A. Asthma B. COPD C. Both

b

Indicate whether the following clinical manifestations are most characteristic of Asthma, COPD , or Both: Weight loss A. Asthma B. COPD C. Both

a

Indicate whether the following clinical manifestations are most characteristic of Asthma, COPD , or Both: Wheezing A. Asthma B. COPD C. Both

b

Indicate whether the following manifestations and treatments are characteristics of thromboangititis obliterans (Buerger's disease) (B) or arteriospastic disease (Raynaud's phenomenon) (R): Amputation of digits of legs below the knee may be necessary for ulceration and gangrene

r

Indicate whether the following manifestations and treatments are characteristics of thromboangititis obliterans (Buerger's disease) (B) or arteriospastic disease (Raynaud's phenomenon) (R): Episodes include white, blue, and red color of the finger tips

r

Indicate whether the following manifestations and treatments are characteristics of thromboangititis obliterans (Buerger's disease) (B) or arteriospastic disease (Raynaud's phenomenon) (R): Frequently associated with autoimmune disorders

b

Indicate whether the following manifestations and treatments are characteristics of thromboangititis obliterans (Buerger's disease) (B) or arteriospastic disease (Raynaud's phenomenon) (R): Inflammation of midsized arteries and veins

b

Indicate whether the following manifestations and treatments are characteristics of thromboangititis obliterans (Buerger's disease) (B) or arteriospastic disease (Raynaud's phenomenon) (R): Intermittent claudication of feet, arms, and hands may be present

r

Indicate whether the following manifestations and treatments are characteristics of thromboangititis obliterans (Buerger's disease) (B) or arteriospastic disease (Raynaud's phenomenon) (R): Involves small cutaneous arteries of the fingers and toes

r

Indicate whether the following manifestations and treatments are characteristics of thromboangititis obliterans (Buerger's disease) (B) or arteriospastic disease (Raynaud's phenomenon) (R): Precipitated by exposure to cold, caffeine, and tobacco

r

Indicate whether the following manifestations and treatments are characteristics of thromboangititis obliterans (Buerger's disease) (B) or arteriospastic disease (Raynaud's phenomenon) (R): Predominant in young females

b

Indicate whether the following manifestations and treatments are characteristics of thromboangititis obliterans (Buerger's disease) (B) or arteriospastic disease (Raynaud's phenomenon) (R): Strongly associated with smoking

r

Indicate whether the following manifestations and treatments are characteristics of thromboangititis obliterans (Buerger's disease) (B) or arteriospastic disease (Raynaud's phenomenon) (R): Treated with calcium-channel blockers, especially nifedipine (Procardia)

Chronic venous insuff. has what effect on tissues?

Inflammatory effect. Impairs wound healing.

d (Prompt treatment of pneumonia with appropriate antibiotics is important in treating bacterial and mycoplasma pneumonia, and antibiotics are often administered on the basis of the history, physical examination, and a chest x-ray showing a typical pattern characteristic of a particular organism without further testing. Sputum and blood cultures take 24-72h for results, and microorganisms often cannot be identified with either Gram stains or cultures. Whether the pneumonia is CAP or HAP is more significant than the severity of symptoms.)

Initial antibiotic treatment for pneumonia is usually based on: a)the severity of symptoms b)the presence of characteristic leukocytes c)Gram stains and cultures of sputum specimens d)history and physical examination and characteristics chest radiographic findings

What's the most common surgical tx for VTE?

Insertion of vena cava interruption device.

c

Match the following condition with their related mechanisms of pulmonary hypertension. Obstruction of Pulmonary blood flow A. COPD B. Pulmonary fibrosis C. Pulmonary embolism

b, h

Ipratropium inhaler is associated with A. Long-term control B. Quick-relief C. B2-adrenergic agonist D. Mast-cell stabilizer E. Leukotriene inhibitor F. Steroid antiinflammatory G. Methylxanthine bronchodilator H. Anticholinergic I. Anti-IgE

Fluid restriction, HF

Less than 2 L/day

Left-sided HF leading to Right-sided HF

LS HF-> pulmonary congestion-> increased pressure in blood vessels of lung, pulmonary hypertension-> increased right ventricular overload-> RS HF

Pulmonary hypertension

Left-sided HF causes increased pressure in blood vessels of lung, increased right ventricular after load, causes right-sided hypertrophy and HF

What ax finding characterizes an aortic rupture?

Line of demarcation: - cyanosis or redness on top - pallor on bottom

a

List the components of the four-drug therapy that is recommended for the initial 2-month treatment of clinically active TB: INH Rifadin PZA Myambutol A

a

List the processes that occur as venous stasis precipitates varicose veins leading to venous stasis ulcers. 1. Venous pressure increases 2. Vein dilate 3. Venous valves become incompetent 4. Venous blood flow reverses 5. Additional venous distention occurs 6. Capillary pressure increases 7. Edema forms 8. Blood supply to local tissues decreases 9. Ulceration occurs a

Pacemaker

Located on left-side. Avoid electrical devices. Report to doctor if pts has hiccups, dizziness, soreness at site, do not drive for 3 months.

Neurohormonal response (compensatory mechanism) ADH

Low CO-> decrease in cerebral perfusion pressure-> pituitary secretes ADH *increase water reabsorption, H2O retention *increase in blood volume

Nutritional therapy. HF

Low sodium diet, less than 2 g/day DASH

heparin

Match the following anticoagulant drugs with their characteristics. Can choose unfractionated heparin, LMWH, Hirudin derivatives, Warfarin (Coumadin), Thrombin inhibitor (argatroban (Acova)) May be administered IV or Subq

heparin

Match the following anticoagulant drugs with their characteristics. Can choose unfractionated heparin, LMWH, Hirudin derivatives, Warfarin (Coumadin), Thrombin inhibitor (argatroban (Acova)) Protamine sulfate is antidote

lmwh

Match the following anticoagulant drugs with their characteristics. Can choose unfractionated heparin, LMWH, Hirudin derivatives, Warfarin (Coumadin), Thrombin inhibitor (argatroban (Acova)) Routine coagulation tests not usually required

d

Major complications of aortic aneurysm repair are associated with involvement or obstruction of the _______ arteries. A. saccular B. synthetic graft C. all D. iliac E. endovascular graft F. renal

b (Peak expiratory flow rates, PEFRs, are normally up to 600L/min and instatus asthmaticus may be as low as 100 to 150L/min. An SaO2 of 85% and FEV1 of 85% of predicated are typical of mild to moderate asthma, and a flattened diaphragm may be presne tin the patient with long-standing asthma but does not reflect current bronchoconstriction)

Marked bronchoconstriction with air trapping and hyperinflation of the lungs in the patient with asthma is indicated by: A. SaO2 of 85% B. PEFR of <150L/min C. FEV1 of 85% of predicated D. chest xray showing a flattened diaphragm

Heparin, Hirudin derivatives, Thrombin inhibitor

Match the following anticoagulant drugs with their characteristics. Can choose unfractionated heparin, LMWH, Hirudin derivatives, Warfarin (Coumadin), Thrombin inhibitor (argatroban (Acova)) Activated partial thromboplastin time

warfarin

Match the following anticoagulant drugs with their characteristics. Can choose unfractionated heparin, LMWH, Hirudin derivatives, Warfarin (Coumadin), Thrombin inhibitor (argatroban (Acova)) International normalized ration (INR)

lmwh

Match the following anticoagulant drugs with their characteristics. Can choose unfractionated heparin, LMWH, Hirudin derivatives, Warfarin (Coumadin), Thrombin inhibitor (argatroban (Acova)) Is administered subcutaneously only

warfarin

Match the following anticoagulant drugs with their characteristics. Can choose unfractionated heparin, LMWH, Hirudin derivatives, Warfarin (Coumadin), Thrombin inhibitor (argatroban (Acova)) Is only administered orally

a (The presence of chronic disease that is present at birth and significantly lowers life span affects all relationships and development of those patients who live to young adulthood.Children of a parent with CF will either be carriers of CF or have the disease; many men with cf are sterile, and women have difficulty becoming pregnant.)

Meeting the developmental tasks of young adulthood becomes a major problem for young adults with cystic fibrosis primarily because: A. They have an expected shorted life span B. Any children they have will develop cystic fibrosis C. They must also adapt to a newly diagnosed chronic disease D. Their illness keeps them from becoming financially independent

a, e

Montelukast is associated with A. Long-term control B. Quick-relief C. B2-adrenergic agonist D. Mast-cell stabilizer E. Leukotriene inhibitor F. Steroid antiinflammatory G. Methylxanthine bronchodilator H. Anticholinergic I. Anti-IgE

a, d

Nedocromil inhaler is associated with A. Long-term control B. Quick-relief C. B2-adrenergic agonist D. Mast-cell stabilizer E. Leukotriene inhibitor F. Steroid antiinflammatory G. Methylxanthine bronchodilator H. Anticholinergic I. Anti-IgE

a, f

Oral prednisone is associated with A. Long-term control B. Quick-relief C. B2-adrenergic agonist D. Mast-cell stabilizer E. Leukotriene inhibitor F. Steroid antiinflammatory G. Methylxanthine bronchodilator H. Anticholinergic I. Anti-IgE

Are oral anticoagulants taken for PAD?

No - anti-platelets are

Should you elevate the leg after a PTCA or endarterectomy?

No!

Bueger's is a vascular disease caused by:

Occlusion - - arteries are inflamed and spastic - inflammation causes thrombus formation

Right-sided heart failure

Occurs when Blood backs into the periphery. S/S: Peripheral edema, fatigue, asites.

Left-sided heart failure

Occurs when fluid backs into the Lungs. S/S: crackles, distended neck vein,

a, b, d, e, g, h, i

Of the following instructions for patients about the use of asthma medications, check all those that are correct. A. When using Pirbuterol (Maxair) and fluticasone (Flovent) inhalers, it does not make any difference which you use first B. Cromolyn (Intal) inhalers should be used before exercise or when anticipating exposure to allergens known to cause asthma C. The mouth should be rinsed thoroughly after using the ipratropium (atrovent) inhaler to prevent oral candidiasis D. The salmeterol (servent) inhaler should not be used more than every 12 hrs E. The best way to use a metered-dose inhaler is to hold it about 1 to 2 inches in front of your mouth before depressing the inhaler F. You should wait 5min before taking a 2nd puff of any inhaled medication G. If you use a spacer with your inhaler, depress the inhaler before starting to inhale H. You should take zakfirlukast (Accolate) tablets on an empty stomach I. To use your dry powder inhaler (DPI), empty your lungs of air, close your lips around the mouthpiece, and inhale quickly and deeply

a, i

Omalizumab is associated with A. Long-term control B. Quick-relief C. B2-adrenergic agonist D. Mast-cell stabilizer E. Leukotriene inhibitor F. Steroid antiinflammatory G. Methylxanthine bronchodilator H. Anticholinergic I. Anti-IgE

b (Phantom limb pain is treated like any other type of postoperative pain would be treated. Explanations of the reason for the pain may be given, but the nurse should still medicate the patient. The compression bandage is left in place except during physical therapy or bathing. Although the pain may decrease over time, it still requires treatment now.)

On the first postoperative day, a patient with a below-the-knee amputation complains of pain in the amputated limb. Which action is best for the nurse to take? A. Explain the reasons for the phantom limb pain. B. Administer prescribed analgesics to relieve the pain. C. Loosen the compression bandage to decrease incisional pressure. D. Remind the patient that this phantom pain will diminish over time.

a, g

Oral theophylline is associated with A. Long-term control B. Quick-relief C. B2-adrenergic agonist D. Mast-cell stabilizer E. Leukotriene inhibitor F. Steroid antiinflammatory G. Methylxanthine bronchodilator H. Anticholinergic I. Anti-IgE

Where do chronic venous ulcers occur? List other characteristics: color, temperature, complications, pulses?

Over medial or anterior ankle - superficial/pink/brown (pale or red in arterials) - skin temp is normal (cold in arterials) - normally no gangrene (gangrene in arterials) - will have normal pulses (pulses dec. in arterials)

Purpose and Types of compensatory mechanisms

Overloaded heart tries to maintain adequate CO 1) SNS activation 2) Neurohormonal response 3) dilation 4) hypertrophy

What is *rest pain* and what causes it?

Pain of LE at rest, most often when elevated or at night. - insufficient blood flow to distal tissues

What ax finding indicates an AAA?

Palpable, pulsating, periumbilical mass to the left of the midline.

What is a very late sign of acute arterial ischemia?

Paralysis (may become permanent, even after revascularization)

Endothelin

Peptides released by vascular endothelial cells that cause vasoconstriction

Nitric oxide (counterregulatory mechanism)

Released from vascular endothelium in response to compensatory mechanisms activated by HF -relaxes arterial smooth muscle -vasodilation -decreased after load

Physician's orders for a client with acute pancreatitis include the following: strict NPO, NG tube to low intermittent suction. The nurse recognizes that these interventions will: a. Reduce the secretion of pancreatic enzymes b. Decrease the client's need for insulin c. Prevent secretion of gastric acid d. Eliminate the need for analgesia

Physician's orders for a client with acute pancreatitis include the following: strict NPO, NG tube to low intermittent suction. The nurse recognizes that these interventions will: a. Reduce the secretion of pancreatic enzymes b. Decrease the client's need for insulin c. Prevent secretion of gastric acid d. Eliminate the need for analgesia a. Reduce the secretion of pancreatic enzymes

Porcine valve/ Bovine valve

Porcine valve rejected by Jewish. Bovine valve rejected by Hindu.

a (Postural drainage, percussion, and vibration should be done 1 hour before or 3 hours after meals. Patients remain in each postural drainage position for 5 minutes. Percussion is done while the patient is in the postural drainage position. Bronchodilators are administered before chest physiotherapy.)

Postural drainage with percussion and vibration is ordered twice daily for a patient with chronic bronchitis. The nurse will plan to A. carry out the procedure 3 hours after the patient eats. B. maintain the patient in the lateral position for 20 minutes. C. perform percussion before assisting the patient to the drainage position. D. give the ordered albuterol (Proventil) after the patient has received the therapy.

A patient in the recovery room after cardiac surgery begins to have extremity paresthesia, peaked T waves, and mental confusion. What type of electrolyte imbalance does the nurse suspect this patient is having?

Potassium

Neurohormonal response (compensatory mechanism) Endothelin

Potent vasoconstrictor produced by: -vascular endothelial cells -stimulated by ADH, catecholamines, and angiotensin II Causes: -arterial vasoconstriction -increase in cardiac contractility and hypertrophy

Prediabetes is associated with all of the following except: a. Increased risk of developing type 2 diabetes b. Impaired glucose tolerance c. Increased risk of heart disease and stroke d. Increased risk of developing type 1 diabetes

Prediabetes is associated with all of the following except: a. Increased risk of developing type 2 diabetes b. Impaired glucose tolerance c. Increased risk of heart disease and stroke d. Increased risk of developing type 1 diabetes D: Increased risk of developing type 1 diabetes Persons with elevated glucose levels that do not yet meet the criteria for diabetes are considered to have prediabetes and are at increased risk of developing type 2 diabetes. Weight loss and increasing physical activity can help people with prediabetes prevent or postpone the onset of type 2 diabetes.

c (C-reactive protein is a marker for inflammation, and a decrease would indicate that the corticosteroid therapy was effective. Blood glucose and serum electrolyte levels also will be monitored to check for side effects of prednisone. Liver function is not routinely monitored for patients receiving steroids.)

Prednisone (Deltasone) is prescribed for a patient with an acute exacerbation of rheumatoid arthritis. Which laboratory result will the nurse monitor to determine whether the medication has been effective? A. Blood glucose test B. Liver function tests C. C-reactive protein level D. Serum electrolyte levels

b (The decreasing urine output is evidence that either the patient needs volume or there is reduced renal blood flow. The physician will want to be notified as soon as possible of this change in condition and may order labs)

Priority Decision: patient who is postoperative following repair of an AAA has been receiving intravenous fluids at 125 mL/hr continuously for the last 12 hours. Urine output for the last 4 hours has been 60 mL, 42 mL, 28 mL, and 20 mL the last hour. The priority action that the nurse should take is to.. A. Monitor for a couple more hours B. Contact the physician and report the decrease in urine output C. Check that the infusion device is set at the correct rate and infusing correctly D. Send blood for electrolytes, blood urea nitrogen (BUN), and creatinine

c

Repair of ______ aneurysms requires cross-clamping of the artery proximal and distal to the aneurysm. A. saccular B. synthetic graft C. all D. iliac E. endovascular graft f. renal

c (Rationale: Postoperative priorities include administration of intravenous, IV, fluids and maintenance of renal perfusion. An adequate blood pressure is important to maintain graft patency, and administration of IV fluids and blood components, as indicated, is essential for adequate blood flow. The nurse should evaluate renal function by hourly urine output measurements and monitoring daily blood urea nitrogen, BUN, and serum creatinine levels. Irreversible renal failure may occur after aortic surgery, particularly in high-risk individuals.)

Priority nursing measures after an abdominal aortic aneurism repair include: A. assessment of cranial nerves and mental status B. administration of IV heparin and monitoring aPTT C. administration of IV fluids and monitoring of kidney function D. elevation of legs and application of graduated compression stockings

What prep is needed for a transesophogeal echocardiogram?

Pt is NPO, sedated - watch for return of gag reflex

c

Pulmonary capillary/alveolar damage A. Pulmonary fibrosis B. Pulmonary embolism C. COPD

A diabetic patient is admitted with ketoacidosis and the health care provider writes all of the following orders. Which order should the nurse implement first? a. Start an infusion of regular insulin at 50 U/hr. b. Give sodium bicarbonate 50 mEq IV push. c. Infuse 1 liter of normal saline per hour. d. Administer regular IV insulin 30 U.

Rationale: The most urgent patient problem is the hypovolemia associated with DKA, and the priority is to infuse IV fluids. The other actions can be accomplished after the infusion of normal saline is initiated.

Paroxysmal nocturnal dyspnea (PND)

Reaborption of fluid from dependent body when pt is flat. Awake with panic, feeling of suffocation, impulse to sit or stand up. Cough is first clinical symptom: dry, nonproductive cough not relieved by position change or OTC medication

Vasodilators

Reduce circulating blood volume, improve coronary artery circulation -IV nitroglycerin (ICU setting) -Sodium nitroprusside (Nipride) -Nesiritide (Natrecor)

c (Rationale: Rest pain most often occurs in the forefoot or toes and is aggravated by limb elevation. Rest pain occurs when there is insufficient blood flow to meet basic metabolic requirements of the distal tissues. Rest pain occurs more often at night because cardiac output tends to drop during sleep and the limbs are at the level of the heart. Patients often try to achieve partial pain relief by dangling the leg over the side of the bed or sleeping in a chair to allow gravity to maximize blood flow.)

Rest pain is a manifestation of PAD that occurs due to a chronic: A. vasospasm of small cutaneous arteries in the feet B. increase in retrograde venous blood flow in the legs C. decrease in arterial blood flow to the nerves of the feet D. decrease in arterial blood flow to the leg muscles during exercise

If an acute arterial embolism lodges and creates demarcation, what must happen?

Restore blood flow: - arteriography - intra-arterial thrombolytics (sm. vessels) [keep it lower than heart until intervention]

Systolic HF causes

Results from inability to of heart to pump blood effectively -impaired contractile function (MI) -increased afterload (hypertension) -cardiomyopathy -mechanical abnormalities (valvular heart disease)

Right-sided HF side effects

Right ventricle does not contract effectively, blood backs up into right atrium and venous circulation causing: -jugular vein distention -hepatomegaly -splenomegaly -vascular congestion in GI tract -peripheral edema

a, c

Salmeterol inhaler is associated with A. Long-term control B. Quick-relief C. B2-adrenergic agonist D. Mast-cell stabilizer E. Leukotriene inhibitor F. Steroid antiinflammatory G. Methylxanthine bronchodilator H. Anticholinergic I. Anti-IgE

Daily weights, HF

Same time, same clothing, same scale Monitor fluid balance

Risk factors for type 2 diabetes include all of the following except: "a. Advanced age b. Obesity c. Smoking d. Physical inactivity"

Smoking

hypervolemia

Some causes for this are renal failure, heart failure, increased oral or IV sodium intake: (hypervolemia/hypovolemia)

d

Surgery for PAD is indicated when the patient has limb pain during _______. A. intermittent claudication B. non-healing ischemic ulcers and gangrene C. 0.77; mild D. rest

Hypertension

Systolic pressure greater than 140/90. Need diet, weight control, and exercise.

t

T or F A patient with VTE is scheduled for surgical treatment. The nurse recognizes that surgery is most commonly performed for this condition to insert a vena cava interruption device to prevent pulmonary embolism.

f (venous thromboembolism should be replaced with superficial vein thrombosis)

T or F A tender, red, inflamed induration along the course of a subcutaneous vein is chararcteristic of a venous thromboembolism (VTE).

f (thrombophlebitis should be varicose veins)

T or F The most common cause of superficial thrombophlebitis in the legs is IV therapy.

A patient having a sickle cell crisis asks the nurse why the sickling causes pain. What should the nurse explain about sickle cell anemia?

The capillaries become occluded, leading to tissue death

a

The classic ischemic pain of PAD is known as _______ _________. A. intermittent claudication B. non-healing ischemic ulcers and gangrene C. 0.77; mild D. rest

c (Pneumonia that has its onset in the community us usually caused by different microorganisms than pneumonia that develops during hospitalization and treatment can be empiric-based on observations and experience without knowing the exact cause. In at least half the cases of pneumonia, a causative organism cannot be identified from cultures, and treatment is based on experience.)

The classification of pneumonia as community-acquired pneumonia (CAP) or hospital-acquired pneumonia (HAP) is clinically useful because: a)atypical pneumonia syndrome is more likely to occur in HAP b)diagnostic testing does not have to be used to identify causative agents c)causative agents can be predicted, and empiric treatment is often effective d)IV antibiotic therapy is necessary for HAP, but oral therapy is adequate for CAP

a

The disruption of all layers of an artery with bleeding? A. Pseudoaneurysm B. Saccular Aneurysm C. Fusiform aneurysm

b (Rationale: The initial goals of therapy for acute aortic dissection without complications are blood pressure, BP, control and pain management. BP control reduces stress on the aortic wall by reducing systolic BP and myocardial contractility.)

The first priority of collaborative care of a patient with a suspected acute aortic dissection is to: A. reduce anxiety B. control blood pressure C. monitor for chest pain D. increase myocardial contractility

b (Albuterol is a rapidly acting bronchodilator and is the first-line medication to reverse airway narrowing in acute asthma attacks. The other medications work more slowly.)

The following medications are prescribed by the health care provider for a patient having an acute asthma attack. Which one will the nurse administer first? A. salmeterol (Serevent) 50 mcg per dry-powder inhaler (DPI) B. albuterol (Ventolin) 2.5 mg per nebulizer C. triamcinolone (Azmacort) 2 puffs per metered-dose inhaler (MDI) D. methylprednisolone (Solu-Medrol) 60 mg IV

hypovolemia

The goal of treatment is to replace water and electrolytes usually with LR. (hypervolemia/hypovolemia)

d (The purpose of elevating the feet is to enhance venous flow from the feet to the right atrium, which is best accomplished by placing two pillows under the feet and one under the thighs. Placing the patient in the Trendelenburg position will lower the head below heart level, which is not indicated for this patient. Placing pillows under the calf or elevating the bed at the knee may cause blood stasis at the calf level)

The health care provider has prescribed bed rest with the feet elevated for a patient admitted to the hospital with deep vein thrombosis. The best method for the nurse to use in elevating the patient's feet is to A. place the patient in the Trendelenburg position. B. place two pillows under the calf of the affected leg. C. elevate the bed at the knee and put pillows under the feet. D. put one pillow under the thighs and two pillows under the lower legs.

b (Live virus vaccines, such as varicella, are contraindicated in a patient taking immunosuppressive drugs. The other orders are appropriate for the patient.)

The health care provider has prescribed the following collaborative interventions for a 49-year-old who is taking azathioprine (Imuran) for systemic lupus erythematosus. Which order will the nurse question? A. Draw anti-DNA blood titer. B. Administer varicella vaccine. C. Use naproxen (Aleve) 200 mg BID. D. Take famotidine (Pepcid) 20 mg daily.

a (IM injections are avoided in patients receiving anticoagulation. A PTT of 50 seconds is withinthe therapeutic range. Vitamin K is used to reverse warfarin. Pulse quality is not affected by VTE.)

The health care provider prescribes an infusion of argatroban (Acova) and daily partial thromboplastin time (PTT) testing for a patient with venous thromboembolism (VTE). The nurse will plan to A. avoid giving any IM medications to prevent localized bleeding. B. discontinue the infusion for PTT values greater than 50 seconds. C. monitor posterior tibial and dorsalis pedis pulses with the Doppler. D. have vitamin K available in case reversal of the argatroban is needed.

c (Three consecutive sputum specimens are obtained on different days for bacteriologic testing for M. tuberculosis. The patient should not provide all the specimens at once. Blood cultures are not used for tuberculosis testing. Once skin testing is positive, it is not repeated.)

The health care provider writes an order for bacteriologic testing for a patient who has a positive tuberculosis skin test. Which action will the nurse take? A. Repeat the tuberculin skin testing. B. Teach about the reason for the blood tests. C. Obtain consecutive sputum specimens from the patient for 3 days. D. Instruct the patient to expectorate three specimens as soon as possible.

d (The joints should be maintained in an extended position to avoid contractures, so patients should use a small, flat pillow for sleeping. The other information is appropriate for a patient with RA and indicates that teaching has been effective.)

The home health nurse is doing a follow-up visit to a patient with recently diagnosed rheumatoid arthritis (RA). Which assessment made by the nurse indicates that more patient teaching is needed? A. The patient requires a 2-hour midday nap. B. The patient has been taking 16 aspirins daily. C. The patient sits on a stool when preparing meals. D. The patient sleeps with two pillows under the head.

d (Specific guidelines for sexual activity help to preserve energy and prevent dyspnea, and maintenance of sexual activity is important to the healthy psychologic well-being of the patient)

The husband of a patient with COPD tells the nurse that they have not had any sexual activity since the patient was diagnosed with COPD because she becomes too short of breath. The best response by the nurse is,: A. "You need to discuss your feelings and needs with your wife so she knows what you expect of her" B. "There are ways to maintain intimacy besides sexual intercourse that will not maker her SOB C. "You should explore other ways to meet your sexual needs since your wife is no longer capable of sexual activity" D. "Would you like for me to talk to you and your wife about some modifications that can be made to maintain sexual activity?"

b (Although leg elevation, moist dressings, and topical antibiotics are useful in treatment of venous stasis ulcers, the most important factor appears to be extrinsic compression to minimize venous stasis, venous hypertension, and edema. Extrinsic compression methods include compression gradient stockings, elastic bandages, and Unna's boot.)

The most important measure in the treatment of venous stasis ulcers is... A. elevation of the limb B. extrinsic compression C. application of moist dressings D. application of topical antibiotics

c (A sputum specimen for culture and Gram stain to identify the organism should be obtained before beginning antibiotic therapy. However, antibiotic administration should not be delayed if a specimen cannot be readily obtained because delays in antibiotic therapy can increase morbidity and mortality risks.)

The nurse cares for a patient who is diagnosed with bacterial pneumonia. Which task is most important for the nurse to complete before administering the prescribed antibiotic? a. Teach the patient to cough and deep breathe. b. Take the temperature, pulse,and respiratory rate. c. Obtain a sputum specimen for culture and Gram stain. d. Check the patient's oxygen saturation by pulse oximetry.

a (Relief of pain is an indication that the dissection has stabilized, and it may be treated conservatively for an extended time with drugs that lower BP and decrease myocardial contractility. Surgery is usually indicated for dissections of the ascending aorta or if complications occur)

The nurse evaluates that treatment for the patient with an uncomplicated aortic dissection is successful when.. A. Pain is relieved B. BP is within normal range C. Surgical repair is completed D. Renal output is maintained at 30 mL/hr

d (This response is consistent with the American Nurses Association, ANA, definition of nursing, which describes the role of nurses in promoting health. The other responses describe some of the dependent and collaborative functions of the nursing role but do not accurately describe the nurse's role in the health care system.)

The nurse has admitted a patient with a new diagnosis of pneumonia and explained to the patient that together they will plan the patient's care and set goals for discharge. The patient says, "How is that different from what the doctor does?" Which response by the nurse is most appropriate? A. "The role of the nurse is to administer medications and other treatments prescribed by your doctor." B. "The nurse's job is to help the doctor by collecting data and communicating when there are problems." C. "Nurses perform many of the procedures done by physicians, but nurses are here in the hospital for a longer time than doctors." D. "In addition to caring for you while you are sick, the nurses will assist you to develop an individualized plan to maintain your health."

a (Spacers can improve the delivery of medication to the lower airways. The other patient actions indicate a need for further teaching.)

The nurse has completed patient teaching about the administration of salmeterol (Serevent) using a metered-dose inhaler (MDI). Which action by the patient indicates good understanding of the teaching? A. The patient attaches a spacer before using the MDI. B. The patient coughs vigorously after using the inhaler. C. The patient floats the MDI in water to see if it is empty. D. The patient activates the inhaler at the onset of expiration.

a (Patients taking Coumadin are taught to follow a consistent diet with regard to foods that are highin vitamin K, such as green, leafy vegetables. The other patient statements are accurate)

The nurse has initiated discharge teaching for a patient who is to be maintained on warfarin (Coumadin) following hospitalization for venous thromboembolism (VTE). The nurse determines that additional teaching is needed when the patient says, A. "I should reduce the amount of green, leafy vegetables that I eat." B. "I should wear a Medic Alert bracelet stating that I take Coumadin." C. "I will need to have blood tests routinely to monitor the effects of the Coumadin." D. "I will check with my health care provider before I begin or stop any medication."

a (A respiratory rate of 38 indicates severe respiratory distress, and the patient needs immediate assessment and intervention to prevent possible respiratory arrest. The other patients also need assessment as soon as possible, but they do not need to be assessed as urgently as the tachypneic patient.)

The nurse has received a change-of-shift report about the following patients with chronic obstructive pulmonary disease (COPD). Which patient should the nurse assess first? A. A patient with a respiratory rate of 38 B. A patient with loud expiratory wheezes C. A patient with jugular vein distention and peripheral edema D. A patient who has a cough productive of thick, green mucus

a (Positioning and activities of daily living assistance are within the educational boundaries of NAP)

The nurse in caring for a patient with COPD. Which of these interventions could be delegated to nursing assistive personnel (NAP)? A. Assist the patient to get up out of bed B. Auscultate breath sounds every 4hrs C. Plant patient activities to minimize exertion D. Teach the patient pursed-lip breathing technique

a (The pH, PaCO2, and PaO2 indicate that the patient has severe uncompensated respiratory acidosis and hypoxemia. Rapid action will be required to prevent increasing hypoxemia and correct the acidosis. The other patients also should be assessed as quickly as possible, but do not require interventions as quickly as the 20-year-old.)

The nurse in the emergency department receives arterial blood gas results for four recently admitted patients with obstructive pulmonary disease. Which patient will require the most rapid action by the nurse? A. 20-year-old with ABG results: pH 7.28, PaCO2 60 mm Hg, and PaO2 58 mm Hg B. 32-year-old with ABG results: pH 7.50, PaCO2 30 mm Hg, and PaO2 65 mm Hg C. 40-year-old with ABG results: pH 7.34, PaCO2 33 mm Hg, and PaO2 80 mm Hg D. 64-year-old with ABG results: pH 7.31, PaCO2 58 mm Hg, and PaO2 64 mm Hg

c (Rationale: Enoxaparin is a low-molecular weight heparin that is administered for 10 to 14 days and prevents future clotting but does not dissolve existing clots. Fibrinolytic agents, e.g., tissue plasminogen activator or alteplase, will dissolve an existing clot. Enoxaparin is administered subcutaneously by injection.)

The nurse instructs a patient with a pulmonary embolism about enoxaparin (Lovenox). Which statement by the patient indicates understanding about the instructions? a. "The medicine will dissolve the clot in my lung." b. "I need to take this medicine with meals." c. "The medicine will be prescribed for 10 days." d. "I will inject this medicine into my abdomen."

c (Rationale: Warfarin is an anticoagulant that is used to prevent thrombi from forming on the walls of the atria during a fib. Once the medication is terminated, thrombi could again form. If one or more detach from the atrial wall, they could travel as cerebral emboli from the left atrium, or pulmonary emboli from the right atrium.)

The nurse is caring for a patient who has been receiving warfarin (Coumadin) and digoxin (Lanoxin) as treatment for atrial fibrillation. Because the warfarin has been discontinued before surgery, the nurse should diligently assess the patient for which complication early in the postoperative period until the medication is resumed? A. Decreased cardiac output B. Increased blood pressure C. Cerebral or pulmonary emboli D. Excessive bleeding from incision or IV sites

b (The nurse should be familiar with the weight-bearing orders for the patient before attempting the transfer. Mechanical lifts are not typically needed after this surgery. Pain medications should be given, since the movement is likely to be painful for the patient. The RN should supervise the patient during the initial transfer to evaluate how well the patient is able to accomplish this skill.)

The nurse is preparing to assist a patient who has had an open reduction and internal fixation (ORIF) of a hip fracture out of bed for the first time. Which action should the nurse take? A. Use a mechanical lift to transfer the patient from the bed to the chair. B. Check the postoperative orders for the patient's weight-bearing status. C. Avoid administration of pain medications before getting the patient up. D. Delegate the transfer of the patient out of bed to nursing assistive personnel (NAP).

a (Rationale: The most common etiology of descending abdominal aortic aneurysm, AAA, is atherosclerosis. Risk factors include male gender, age 65 years or older, and tobacco use are the major risk factors for AAAs of atherosclerotic origin. Other risk factors include the presence of coronary or peripheral artery disease, high blood pressure, and high cholesterol.)

The nurse is teaching a patient about risk factors for aortic abdominal aneurysms. Which risk factors should the nurse include in the teaching plan? A. Smoking, high cholesterol, and hypertension B. Female gender, hyperhomocysteinemia, and substance abuse C. Diabetes mellitus, obesity, and metabolic syndrome D. Physical inactivity, African American, and renal insufficiency

b (Rationale: Pursed-lip breathing, PLB, prolongs exhalation and prevents bronchiolar collapse and air trapping. PLB is simple and easy to teach and learn. It also gives the patient more control over breathing. Evidence from controlled studies does not support the use of diaphragmatic breathing in patients with COPD. Diaphragmatic breathing results in hyperinflation because of increased fatigue and dyspnea and abdominal paradoxical breathing rather than with normal chest wall motion. Chest physiotherapy, percussion and vibration, is primarily used for patients with excessive bronchial secretions who have difficulty clearing them. Huff coughing is a technique that helps patients with COPD to use a forced expiratory technique to clear secretions.)

The nurse notes that a patient with chronic obstructive pulmonary disease develops severe dyspnea with a change in respiratory rate from 26 breaths/minute to 44 breaths/minute. Which action by the nurse would be the most appropriate? a. Have the patient perform huff coughing b. Teach the patient to use pursed lip breathing c. Instruct the patient in diaphragmatic breathing d. Perform chest physiotherapy for 5 minutes

b (Plaquenil can cause retinopathy; the medication should be stopped. The other findings are not related to the medication, although they also will be reported.)

The nurse obtains this information when assessing a patient who is taking hydroxychloroquine (Plaquenil) to treat rheumatoid arthritis. Which symptom is most important to report to the health care provider? A. Abdominal cramping B. Complaint of blurry vision C. Phalangeal joint tenderness D. Blood pressure 170/84 mm Hg

c (Capillary refill is prolonged in PAD because of the slower and decreased blood flow to the periphery. The other listed clinical manifestations are consistent with chronic venous disease.)

The nurse performing an assessment with a patient who has chronic peripheral artery disease (PAD) of the legs and an ulcer on the left great toe would expect to find A. a positive Homans' sign. B. swollen, dry, scaly ankles. C. prolonged capillary refill in all the toes. D. a large amount of drainage from the ulcer.

a (Flexion contractures, especially of the hip, may be debilitating and delay rehabilitation of the patient with a leg amputation. To prevent hip flexion, the patient should avoid sitting in a chair with the hips flexed or having pillows under the surgical extremity for prolonged periods, and the patient should lie on the abdomen for 30min three to four times a day to extend the hip)

The nurse positions a patient with an above-the-knee amputation with delayed prosthetic fitting prone several times a day to: A. prevent flexion contractures B. assess the posterior skin flap C. reduce edema in the residual limb D. relieve pressure on the incision site

b (Nonprescription drugs should not be used by patients with asthma because of dangers associated with rebound bronchospasm, interactions with prescribed drugs, and undesirable side effects. All the other response are appropriate for the patient with asthma)

The nurse recognizes that additional teaching is needed when the patient with asthma says, A. "I should exercise every day if my symptoms are controlled" B. "I may use over-the-counter bronchodilator drugs occasionally if I develop chest tightness." C. "I should inform my spouse about my medications and how to get help if I have a severe asthma attack." D. "A diary to record my medication use, symptoms, peak expiratory flow rate (PEFR) levels, and activity level will help in adjusting my therapy

c (Covering the mouth and nose will help decrease airborne transmission of TB. The other actions will not be effective in decreasing the spread of TB.)

The nurse recognizes that the goals of teaching regarding the transmission of pulmonary tuberculosis (TB) have been met when the patient with TB A. demonstrates correct use of a nebulizer. B. washes dishes and personal items after use. C. covers the mouth and nose when coughing. D. reports daily to the public health department.

b (Constriction of the pulmonary vessels, leading to pulmonary hypertension, is cause by alveolar hypoxia and the acidosis that results from hypercapnia. Polycythemia is a contributing factor in cor pulmonale because it increases the viscosity of blood and the pressure needed to circulate the blood. Long-term low-flow oxygen therapy dilates pulmonary vessels and is used to treat cor pulmonale; high oxygen administration is not related to cor pulmonale)

The pulmonary vasoconstriction leading to the development of cor pulmonale in the patient with COPD results from: A. Increased viscosity of the blood B. Alveolar hypoxia and hypercapnia C. Long-term low-flow oxygen therapy D. Administration of high concentrations of oxygen

d (Rationale: It is within the scope of practice of the RN to assess, teach, and evaluate. The LPN provides care for stable patients and may adjust oxygen flow rates depending on desired oxygen saturation levels of stable patients. The NAP may obtain oxygen saturation levels, assist patients with adjustment of oxygen devices, and report changes in patient's level of consciousness or difficulty breathing.)

The nurse supervises a registered nurse (RN), a licensed practical/vocational nurse (LPN/LVN), and nursing assistive personnel (NAP) on a medical unit. The nurse should intervene with her team member if which of the following occurs? a. NAP report to the nurse that the patient complained of difficulty breathing. b. LPN/LVN obtained a pulse oximetry reading of 94%. c. RN taught the patient about the dangers of oxygen use at home. d. LPN/LVN changed the oxygen delivery method based on arterial blood gas results.

1 (Rationale: Discharge instructions should include the following: Wash residual limb thoroughly each night with warm water and a bacteriostatic soap. Do not use any substance such as lotions, alcohol, powders, or oil on residual limb unless prescribed by the health care provider. To prevent flexion contractures, have patients avoid sitting in a chair for longer than 1 hour with hips flexed or having pillows under the surgical extremity.)

The nurse teaches a 58-year-old patient who had an above-the-knee amputation how to care for the residual limb. Which of the following statements by the patient indicates that teaching was effective? 1. "I should use only mild soap and water to clean the limb." 2. "I can apply lotion to the residual limb to keep the skin from cracking." 3. "Tincture of iodine or alcohol could be used to toughen the limb." 4. "I can elevate the residual limb on a pillow to prevent swelling when I am sitting or lying down."

d (Rationale: Fluticasone, Flovent HFA, may cause oral candidiasis, thrush; the patient should rinse the mouth with water or mouthwash after use or use a spacer device to prevent oral fungal infections. Flutincasone is not recommended for an acute asthma attack; this medication is an inhaled corticosteroid and may take 2 weeks of regular use to see effects.)

The nurse teaches a patient how to administer fluticasone (Flovent HFA) by metered dose inhaler (MDI). Which statement by the patient to the nurse indicates understanding about the instructions? a. "My breathing will improve slowly over the next 2 to 3 days." b. "A spacer is used with this inhaler to prevent mouth dryness." c. "I should use this inhaler immediately if I have trouble breathing." d. "It is important to remember to rinse my mouth after using this inhaler."

a (During walking, the muscles of the legs continuously knead the veins, promoting movement of venous blood toward the heart, and walking is the best measure to prevent venous stasis. The other methods will help venous return, but they do not provide the benefit that ambulation does.)

The nurse teaches the patient with any venous disorder that the best way to prevent venous stasis and increase venous return is to... A. walk B. sit with the legs elevated C. frequently rotate the ankles D. continuously wear graduated compression stockings

d (Almost all forms of bronchiectasis are associated with bacterial infections that damage the bronchial walls. The incidence of bronchiectasis has decreased with use of measles and pertussis vaccines and better treatment of lower respiratory tract infections)

The nursing assessment of a patient with bronchiectasis is most likely to reveal a history of: A. Chest trauma B. Childhood asthma C. smoking or oral tobacco use D. Recurrent lower respiratory tract infections

c (CF is an autosomal-recessive, multisystem disease involving altered function of the exocrine glands of the lungs, pancreas, and sweat glands. Abnormally thick, abundant secretions from mucous glands lead to a chronic, diffuse, obstructive pulmonary disorder almost all patients)

The pathophysiologic mechanism of cystic fibrosis leading to obstruction lung disease is: A. Fibrosis of mucous glands and destruction of bronchial walls B. Destruction of lung parenchyma from inflammation and scarring C. Production of abnormally thick, copious secretions from mucous glands D. Increased serum levels of pancreatic enzymes that are deposited in the bronchial mucosa

b (Anticoagulation therapy with heparin or warfarin, coumadin, does not dissolve clots but prevents propagation of the clot, development of new thrombi, and embolization; lysis of the clot occurs through the action of the body's intrinsic fibrinolytic system or by the administration of fibrinolytic agents.)

The patient with VTE is receiving therapy with heparin and asks the nurse whether the drug will dissolve the clot in her leg. The best response by the nurse is... A. "The drug will break up and dissolve the clot so that circulation in the vein can be restored." B. "The purpose of the heparin is to prevent growth of the clot or formation of new clots where the circulation is slowed." C. "Heparin won't dissolve the clot, but it will inhibit the inflammation around the clot and delay the development of new clots." D. "The heparin will dilate the vein, preventing turbulence of blood flow around the clot that may cause it to break off and travel to the lungs."

d (The major objective of therapy in CF is to promote the removal of the secretions, and performance of postural drainage, vibration, and percussion, has been the mainstay of treatment)

The primary treatment for cystic fibrosis is: A. Heart-lung transplantation B. Administration of prophylactic antibiotics C. Administration of nebulized bronchodilators D. Vigorous and consistent chest physiotherapy

d (Rationale: Patients with confirmed VTE should receive initial treatment with low-molecular-weight heparin, LMWH, unfractionated heparin, UFH, or fondaparinux and with warfarin for at least 5 days or until the international normalized ratio, INR, is >2.0 for 24 hours. Patients with multiple co-morbidities, complex medical issues, or a very large VTE usually are hospitalized for treatment and typically receive intravenous UFH. LMWH is recommended over UFH for most patients with acute VTE. Depending on the clinical presentation, patients often can be safely and effectively managed as outpatients.)

The recommended treatment for an initial VTE in an otherwise healthy person with no significant comorbidities would include: A. IV agratoban (Acova) as an inpatient B. IV unfractionated heparin as an inpatient C. subcutaneous unfractionated heparin as an outpatient D. subcutaneous low-molecular weight heparin as an outpatient

b (Drug-resistant strains of TB have developed because TB patients' compliance to drug therapy has been poor and there has been general decreased vigilance in monitoring and follow-up of Tb treatment. Antitubercular drugs are almost exclusively used for Tb infections. TB can be effectively diagnosed with sputum cultures. The incidence of Tb is at epidemic proportions in patients with HIV, but this does not account for drug-resistant strains of TB.)

The resurgence in TB resulting from the emergence of multidrug-resistant strains of Mycobacterium tuberculosis was primarily the result of: a)a lack of effective means to diagnose TB b)poor compliance with drug therapy in patients with TB c)the increased population of immunosuppressed individuals with AIDS d)indiscriminate use of antitubercular drugs in treatment of other infections

d (The patient's history and clinical manifestations suggest a fat embolus. The most important assessment is oxygenation. The other actions also are appropriate but will be done after the nurse assesses gas exchange.)

The second day after admission with a fractured pelvis, a patient develops acute onset confusion. Which action should the nurse take first? A. Take the blood pressure. B. Assess patient orientation. C. Check pupil reaction to light. D. Assess the oxygen saturation.

Arteriosclerosis

Thickening and hardening of the walls.

hypocalcemia

This occues with a value <9.0 mg/dl (hypercalcemia, hypocalcemia)

hypernatremia

This occurs when values >145 mEq/L (hypernatremia/hyponatremia)

hyperkalemia

This occurs when values >5 mEq/L (Hyperkalemia, Hypokalemia)

"The client diagnosed with Type 1 diabetes has a glycosylated hemoglobin (A1 c) of 8.1%. Which interpretation should the nurse make based on this result? 1.This result is below normal levels. 2.This result is within acceptable levels. 3.This result is above recommended levels 4.This result is dangerously high.

This result is above recommended levels.

Deep, diffuse chest pain (possibly extending to interscapular area) is the most common sx of:

Thoracic aortic aneurysm - most often a-sx

Buergers Disease

Thromboangiitis obliteran- Spasms of the arteries and veins in lower extremities. S/S: Pallor, cyanosis, pain, paresthesia. Use Buerger-Allen Test and exercises.

Diff b/n thrombus and embolus?

Thrombus adheres to bv wall, builds in place. Embolus moved to get there or can move.

b (The patient in an acute asthma attack is very anxious and fearful. It is important to stay with the patient and interact in a calm, unhurried manner. Helping the patient breath with pursed lips will facilitate expiration of trapped air and help the patient gain control of breathing)

To decrease the patient's sense of panic during an acute asthma attack, the best action of the nurse is to A. Leave the patient alone to rest in a quiet, calm environment B. Stay with the patient and encourage slow, pursed-lip breathing C. Reassure the patient that the attack can be controlled with treatment D. Let the patient know his or her status is being closely monitored with frequent measurement of vital signs and sign SpO2

c (The anti-Sm is antibody found almost exclusively in SLE. The other blood tests also are used in screening but are not as specific to SLE.)

To determine whether a patient with joint swelling and pain has systemic lupus erythematosus, which test will be most useful for the nurse to review? A. Rheumatoid factor (RF) B. Antinuclear antibody (ANA) C. Anti-Smith antibody (Anti-Sm) D. Lupus erythematosus (LE) cell prep

c (Cor pulmonale causes clinical manifestations of right ventricular failure, such as jugular vein distention. The other clinical manifestations may occur in the patient with other complications of chronic obstructive pulmonary disease, COPD, but are not indicators of cor pulmonale.)

To evaluate the effectiveness of therapy for a patient with cor pulmonale, the nurse will monitor the patient for A. elevated temperature. B. clubbing of the fingers. C. jugular vein distention. D. complaints of chest pain.

d (Prevention of emboli formation can be achieved by bed rest and limiting movement of the involved extremity until the clot is table, inflammation has receded and anticoagualtion is achieved. Elevating the affected limb promote venous return, but it does not prevent embolization, and dangling the legs promotes venous stasis and further clot formation)

To help prevent embolization of the thrombus in a patient with VTE, the nurse teaches the patient to do what? A. dangle the feet over the edge of the bed q2-3hr B. ambulate for short periods three to four times a day C. keep the affected leg elevated above the level of the heart D. maintain bed rest until edema is relieved and anticoagulation is established

hyponatremia

Treatment is sodium replacement, water restriction, diuretic administration or fluid replacement. (hypernatremia, hyponatremia)

a, f

Triamcinolone inhaler is associated with A. Long-term control B. Quick-relief C. B2-adrenergic agonist D. Mast-cell stabilizer E. Leukotriene inhibitor F. Steroid antiinflammatory G. Methylxanthine bronchodilator H. Anticholinergic I. Anti-IgE

b

Two serious complications of PAD that frequently lead to lower limb amputation are ______________ and ___________. A. intermittent claudication B. non-healing ischemic ulcers and gangrene C. 0.77; mild D. rest

c

Uniform, circumferential dilation of artery? A. Pseudoaneurysm B. Saccular Aneurysm C. Fusiform aneurysm

Cardiac Catheterization

Used to detect blockages from MI. Assess for dye allergies. Best rest for 8 hrs after procedure. Pressure on site for 5 minutes after procedure. Assess distal pulse. Force fluids. Assess for Hematoma(diminished distal pulse).

The benefits of using an insulin pump include all of the following except: "a. By continuously providing insulin they eliminate the need for injections of insulin b. They simplify management of blood sugar and often improve A1C c. They enable exercise without compensatory carbohydrate consumption d. They help with weight loss

Using an insulin pump has many advantages, including fewer dramatic swings in blood glucose levels, increased flexibility about diet, and improved accuracy of insulin doses and delivery; however, the use of an insulin pump has been associated with weight gain.

The Trendelenberg test will show what?

Varicose veins will disappear, then become engorged on standing.

Raynauds phenomenon

Vascular vasospams. Avoid cold environment.

Varicose Veins/Thrombophlebitis

Vein becomes inflamed and a clot forms. Assess for Homan signs (positive if pain when dorsiflexion). Apply Stockings to legs.

Which ulcers create a dull ache in the calf or thigh and pruritis?

Venous

Which ulcers have heavy drainage?

Venous

Where might a superficial venous thrombosis occur?

Venous catheter - RBCs congregate and clot, you can see this

What is a complication of varicose veins?

Venous ulcers Thrombosis

c (Capsaicin cream blocks the transmission of pain impulses and is helpful for some patients in treating OA. The other medications would be used for patients with RA.)

When caring for a patient who has osteoarthritis, the nurse will anticipate the need to teach the patient about which of these medications? A. Adalimumab (Humira) B. Prednisone (Deltasone) C. Capsaicin cream (Zostrix) D. Sulfasalazine (Azulfidine)

What dressing is appropriate for stasis ulcers?

Wet to dry

b (The goal for exercise programs for patients with COPD is to increase exercise time gradually to a total of 20 minutes daily. Shortness of breath is normal with exercise and not an indication that the patient should stop. Limiting exercise to ADLs will not improve the patient's exercise tolerance. A 70-year-old patient should have a pulse rate of 120 or less with exercise, 80% of the maximal heart rate of 150.)

When developing a teaching plan to help increase activity tolerance at home for a 70-year-old with severe chronic obstructive pulmonary disease (COPD), the nurse should teach the patient that an appropriate exercise goal is to A. walk until pulse rate exceeds 130. B. walk for a total of 20 minutes daily. C. exercise until shortness of breath occurs. D. limit exercise to activities of daily living (ADLs).

d (Feedback: All of the factors contribute to the patient's risk, but only the hypertension can potentially be modified to decrease the patient's risk for further expansion of the aneurysm.)

When discussing risk factor modification for a 60-year-old patient who has a 4-cm abdominal aortic aneurysm, the nurse will focus patient teaching on which of these patient risk factors? A. Male gender B. Marfan syndrome C. Abdominal trauma history D. Uncontrolled hypertension

a (The jaw will be wired for stabilization, and the patient should know what emergency situations require that the wires be cut to protect the airway. There are no dressing changes for this procedure. The diet is liquid, and patients are not able to chew high fiber foods. Initially, the patient may receive nasogastric tube feedings, but by discharge the patient will swallow liquid through a straw.)

When doing discharge teaching for a patient who has had a repair of a fractured mandible, the nurse will include information about A. when and how to cut the immobilizing wires. B. self-administration of nasogastric tube feedings. C. the use of sterile technique for dressing changes. D. the importance of including high-fiber foods in the diet.

d (Explanation of the underlying mechanism usually helps calm anxiety about a phantom pain experience)

When a client who had an above-the-knee amputation (AKA) complains of phantom limb sensations the nursing staff should: A. Reassure the client that theses sensations will pass B. Explain the psychological component involved to the client C. Encourage the client to get involved in diversional activities D. Describe the neurological mechanisms in language that the client understands

b (The best way to determine the appropriate oxygen flow rate is by monitoring the patient's oxygenation either by arterial blood gases, ABGs, or pulse oximetry; an oxygen saturation of 90% indicates adequate blood oxygen level without the danger of suppressing the respiratory drive. For patients with an exacerbation of COPD, an oxygen flow rate of 2 L/min may not be adequate. Because oxygen use improves survival rate in patients with COPD, there is not a concern about oxygen dependency. The patient's perceived dyspnea level may be affected by other factors, such as anxiety, besides blood oxygen level.)

When a hospitalized patient with chronic obstructive pulmonary disease (COPD) is receiving oxygen, the best action by the nurse is to A. minimize oxygen use to avoid oxygen dependency. B. maintain the pulse oximetry level at 90% or greater. C. administer oxygen according to the patient's level of dyspnea. D. avoid administration of oxygen at a rate of more than 2 L/min.

d (Initial drug therapy for acute respiratory distress involves the use of aerosolized albuterol or other B-adrenergic agonists by nebulization every 20mins to 4hrs as neccessary. The other medications may be added if the patient does not respond to inhaled B-adrenergic agonists)

When a patient with asthma is admitted to the emergency department in severe respiratory distress, the nurse anticipates that initial drug treatment will most likely include administration of: A. IV aminophylline B. IV hydrocortisone C. Inhaled ipratropium D. Aerosolized albuterol

c (A high-efficiency particulate-absorbing, HEPA, mask, rather than a standard surgical mask, should be used when entering the patient's room because the HEPA mask can filter out 100% of small airborne particles. Hand washing before visiting the patient is not necessary, but there is no reason for the nurse to stop the family member from doing this. Because anorexia and weight loss are frequent problems in patients with TB, bringing food from outside the hospital is appropriate. The family member should wash the hands after handling a tissue that the patient has used, but no precautions are necessary when giving the patient an unused tissue.)

When caring for a patient who is hospitalized with active tuberculosis (TB), the nurse observes a family member who is visiting the patient. The nurse will need to intervene if the family member A. washes the hands before entering the patient's room. B. hands the patient a tissue from the box at the bedside. C. puts on a surgical face mask before visiting the patient. D. brings food from a "fast-food" restaurant to the patient.

c (Use of accessory muscle indicates that the patient is experiencing respiratory distress and rapid intervention is needed. The other data indicate the need for ongoing monitoring and assessment but do not suggest that immediate treatment is required.)

When caring for a patient with a history of asthma, which assessment finding should the nurse communicate immediately to the health care provider? A. Pulse oximetry reading of 91% B. Respiratory rate of 26 breaths/minute C. Use of accessory muscles in breathing D. Peak expiratory flow rate of 240 mL/min

c (Adequate rest helps decrease the fatigue and pain that are associated with rheumatoid arthritis. Patients are taught to avoid stressing joints, to use warm baths to relieve stiffness, and to use a firm mattress.)

When caring for a patient with a new diagnosis of rheumatoid arthritis, which action will the nurse include in the plan of care? A. Instruct the patient to purchase a soft mattress. B. Teach patient to use lukewarm water when bathing. C. Suggest that the patient take a nap in the afternoon. D. Suggest exercise with light weights several times daily.

a (Bleeding is a possible complication after catheterization of the femoral artery, so the nurse's first action should be to assess for changes in vital signs that might indicate hemorrhage. The other actions also are appropriate but can be done after determining that bleeding is not occurring.)

When caring for a patient with critical limb ischemia who has just arrived on the nursing unit after having percutaneous transluminal balloon angioplasty, which action should the nurse take first? A. Take the blood pressure and pulse rate. B. Check for the presence of pedal pulses. C. Assess the appearance of any ischemic ulcers. D. Start discharge teaching about antiplatelet drugs.

c (Since any touch on the area of inflammation may increase pain, bedding should be held away from the toe and touching the toe will be avoided. Elevation of the foot will not reduce the pain, which is caused by the urate crystals. Acetaminophen can be used for pain relief.)

When caring for a patient with gout and a red and painful left great toe, which nursing action will be included in the plan of care? A. Gently palpate the toe to assess swelling. B. Use pillows to keep the left foot elevated. C. Use a footboard to hold bedding away from the toe. D. Teach patient to avoid use of acetaminophen (Tylenol).

c (Comfortable shoes with good support will help decrease the risk for falls. Scatter rugs should be eliminated, not just tacked down. Activities of daily living provide range of motion exercise; these do not need to be taught by a physical therapist. Falls inside the home are responsible for many injuries.)

When counseling an older patient about ways to prevent fractures, which information will the nurse include? A. Tack down scatter rugs in the home. B. Most falls happen outside the home. C. Buy shoes that provide good support and are comfortable to wear. D. Range-of-motion exercises should be taught by a physical therapist.

b (Smoking cessation is essential for slowing the progression of PAD to critical limb ischemia and reducing the risk of myocardial infarction and death. Circulation to the legs will decrease if the legs are elevated. Patients with PAD are taught to exercise to the point of feeling pain, rest, and then resume walking. Support hose are not used for patients with PAD.)

When developing a teaching plan for a patient newly diagnosed with peripheral artery disease (PAD), which information should the nurse include? A. "Exercise only if you do not experience any pain." B. "It is very important that you stop smoking cigarettes." C. "Try to keep your legs elevated whenever you are sitting." D. "Put on support hose early in the day before swelling occurs."

d (Increased swelling or numbness may indicate increased pressure at the injury, and the health care provider should be notified immediately to avoid damage to nerves and other tissues. The patient should be encouraged to move the joints above and below the cast to avoid stiffness. There is no need to elevate the shoulder, although the forearm should be elevated to reduce swelling. NSAIDs are appropriate to treat pain after a fracture.)

When giving home care instructions to a patient who has multiple forearm fractures and a long-arm cast on the right arm, which information should the nurse include? A. Keep the hand immobile to prevent soft tissue swelling. B. Keep the right shoulder elevated on a pillow or cushion. C. Avoid the use of nonsteroidal anti-inflammatory drugs (NSAIDs) for the first 48 hours after the injury. D. Call the health care provider for increased swelling or numbness.

Cardiac Shock Symptoms

When heart fails to pump enough blood to perfuse the tissue adequately. S/S:Frothy-Pink Tinged sputum, hypotension, tachycardia, tachypnea, Orthopnea, Oliguria.

a (Taking a warm shower or bath is recommended to relieve joint stiffness, which is worse in the morning. Isometric exercises would place stress on joints and would not be recommended. Stretching and ROM should be done later in the day, when joint stiffness is decreased.)

When helping a patient with rheumatoid arthritis (RA) plan a daily routine, the nurse informs the patient that it is most helpful to start the day with A. a warm bath followed by a short rest. B. a short routine of isometric exercises. C. active range-of-motion (ROM) exercises. D. stretching exercises to relieve joint stiffness.

d (Regardless of the location, atherosclerosis is responsible for peripheral arterial disease and is related to other cardiovascular disease and its risk factors.)

When obtaining a health history from a 72-year-old man with peripheral arterial disease(PAD) of the lower extremities, the nurse asks about a history of related conditions such as: a) Venous thrombosis b) Venous stasis ulcers c) Pulmonary embolism d) Carotid artery disease

c (Community-acquired pneumonia, CAP, is most commonly caused by Staph... pneumonia and is associated with an acute onset with fever, chills, productive cough with purulent or bloody sputum, and pleuritic chest pain. Other causes of pneumonia have a more gradual onset with dry, hacking cough; headache; and sore throat. A recent loss of consciousness or altered consciousness is common in those pneumonias associated with aspiration, such as anaerobic bacterial pneumonia.)

When obtaining a health history from a patient at the clinic with suspected CAP, the nurse expects the patient to report: a)a dry, hacking cough b)a recent loss of consciousness c)an abrupt onset of fever and chills d)a gradual onset of headache and sore throat

b (TB usually develops insidiously with fatigue, malaise, low-grade fevers, and night sweats. Chest pain and a productive cough may also occur, but hemoptysis is a late symptom.)

When obtaining a health history from a patient suspected of having early TB, the nurse asks the patient about experiencing: a)chest pain, hemoptysis, and weight loss b)fatigue, low-grade fever, and night sweats c)cough with purulent mucus and fever with chills d)pleuritic pain, non-productive cough, and temperature elevation at night

b (Repositioning of patients is within the scope of practice of NAP, after they have been trained and evaluated in this skill. The other actions should be done by licensed nursing staff members.)

When planning care for a patient who has had hip replacement surgery, which nursing action can the nurse delegate to experienced nursing assistive personnel (NAP)? A. Teach quadriceps-setting exercises. B. Reposition the patient every 1 to 2 hours. C. Assess for skin irritation on the patient's back. D. Determine the patient's pain level and tolerance.

d (Bronchodilators are held before pulmonary function testing so that a baseline assessment of airway function can be determined. Testing is repeated after bronchodilator use to determine whether the decrease in lung function is reversible. There is no need for the patient to be NPO. Oral corticosteroids also should be held before the examination and corticosteroids given 2 hours before the examination would be at a high level. Rescue medications, which are bronchodilators, would not be given until after the baseline pulmonary function was assessed.)

When preparing a patient with possible asthma for pulmonary function testing, the nurse will teach the patient to A. avoid eating or drinking for several hours before the testing. B. use rescue medications immediately before the tests are done. C. take oral corticosteroids at least 2 hours before the examination. D. withhold bronchodilators for 6 to 12 hours before the examination.

b (Patients are advised to avoid repetitious movements. Sitting during household chores is recommended to decrease stress on joints. Wringing water out of sponges would increase the joint stress. Patients are encouraged to position joints in the extended position, and sleeping with a pillow behind the knees would decrease the ability of the knee to extend and also decrease knee range of motion, ROM.)

When teaching a patient who has rheumatoid arthritis (RA) about how to manage activities of daily living, the nurse instructs the patient to A. stand rather than sit when performing household chores. B. avoid activities that require continuous use of the same muscles. C. strengthen small hand muscles by wringing sponges or washcloths. D. protect the knee joints by sleeping with a small pillow under the knees.

3 (Rationale: Patients should be taught to exercise to the point of discomfort, stop and rest, and then resume walking until the discomfort recurs. Smoking cessation and proper foot care are also important interventions for patients with peripheral arterial disease.)

When teaching a patient with peripheral arterial disease, the nurse determines that further teaching is needed when the patient says, 1. "I should not use heating pads to warm my feet." 2. "I will examine my feet every day for any sores or red areas." 3. "I should cut back on my walks if they cause pain in my legs." 4. "I think I can quit smoking with the use of short-term nicotine replacement and support groups."

a (Noninfectious hepatitis is a toxic effect of isoniazid, INH, rifampin, and pyrazinamide, and patients who develop hepatotoxicity will need to use other medications. Changes in hearing and nail thickening are not expected with the four medications used for initial TB drug therapy. Orange discoloration of body fluids is an expected side effect of rifampin and not an indication to call the health care provider.)

When teaching the patient who is receiving standard multidrug therapy for tuberculosis (TB) about possible toxic effects of the antitubercular medications, the nurse will give instructions to notify the health care provider if the patient develops A. yellow-tinged skin. B. changes in hearing. C. orange-colored sputum. D. thickening of the fingernails.

b ( A yellow zone indicated on the peak flow meter indicates that the patient's asthma is getting worse, and quick-relief medication should be used. The meter is routinely used only each morning before taking medications and does not have to be on hand at all times. The meter measures the ability to empty the lungs and involves blowing through the meter)

When teaching the patient with asthma about use of the peak flow meter, the nurse instructs the patient to A. carry the flow meter with the patient at all times in case an asthma attack occurs B. follow written asthma action plan (eg increasing quick relief drugs) if the park expiratory flow rate is in the yellow zone C. use the flow meter to check the status of the patient's asthma every time the patient takes quick-relief medication D. use the flow meter by emptying the lungs, closing the mouth around the mouthpiece, and inhaling through the meter as quickly as possible

b (Use of a bronchodilator before exercise improves airflow for some patients and is recommended. Shortness of breath is normal with exercise and not a reason to stop. Patients should be taught to breathe in through the nose and out through the mouth, using a pursed lip technique. Upper-body exercise can improve the mechanics of breathing in patients with COPD.)

When teaching the patient with chronic obstructive pulmonary disease (COPD) about exercise, which information should the nurse include? A. "Stop exercising if you start to feel short of breath." B. "Use the bronchodilator before you start to exercise." C. "Breathe in and out through the mouth while you exercise." D. "Upper body exercise should be avoided to prevent dyspnea."

b (PAD occurs as a result of atherosclerosis, and the risk factors are the same as for other diseases associated with atherosclerosis, such as CAD, cerebral vascular disease, and aneurysms. Major risk factors are hypertension, cigarette smoking, and hyperlipidemia. The risk for amputation is high in patients with severe occlusive disease, but it is not the best approach to encourage patients to make lifestyle modifications)

When teaching the patient with peripheral artery disease about modifying risk factors associated with the condition, the nurse emphasizes that... A. amputation is the ultimate outcome if the patient does not alter lifestyle behaviors B. modifications will reduce the risk of other atherosclerotic conditions such as stroke C. risk-reducing behaviors initiated after angioplasty can stop the progression of the disease D. maintenance of normal body weight is the most important factor in controlling arterial disease

b (Loose, bloody stools at this time may indicate intestinal ischemia or infarction and should bereported immediately because the patient may need an emergency bowel resection. The otherfindings are normal on the first postoperative day after abdominal surgery.)

When the nurse is caring for a patient on the first postoperative day after an abdominal aortic aneurysm repair, which assessment finding is most important to communicate to the health care provider? A. Absence of flatus B. Loose, bloody stools C. Hypotonic bowel sounds D. Abdominal pain with palpation

b (The abdominal distention and absent bowel tones may be due to complications of pelvic fractures such as paralytic ileus or hemorrhage or trauma to the bladder, urethra, or colon. Pelvic instability, abdominal pain with palpation, and abdominal bruising would be expected with this type of injury.)

When the nurse is caring for a patient who is on bed rest after having a complex pelvic fracture, which assessment finding is most important to report to the health care provider? A. The patient states that the pelvis feels unstable. B. Abdominal distention is present and bowel tones are absent. C. There are ecchymoses on the abdomen and hips. D. The patient complains of pelvic pain with palpation.

b (The goal for treatment of an asthma attack is to keep the oxygen saturation >90%. The other patient data may occur when the patient is too fatigued to continue with the increased work of breathing required in an asthma attack.)

When the nurse is evaluating the effectiveness of therapy for a patient who has received treatment during an asthma attack, which finding is the best indicator that the therapy has been effective? A. No wheezes are audible. B. Oxygen saturation is >90%. C. Accessory muscle use has decreased. D. Respiratory rate is 16 breaths/minute.

d (A diagnosis of chronic bronchitis is based on a history of having a productive cough for 3 months for at least 2 consecutive years. There is no familial tendency for chronic bronchitis. Although smoking is the major risk factor for chronic bronchitis, a smoking history does not confirm the diagnosis.)

When the nurse is interviewing a patient with a new diagnosis of chronic obstructive pulmonary disease (COPD), which information will help most in confirming a diagnosis of chronic bronchitis? A. The patient tells the nurse about a family history of bronchitis. B. The patient's history indicates a 40 pack-year cigarette history. C. The patient denies having any respiratory problems until the last 6 months. D. The patient complains about a productive cough every winter for 3 months.

c (Bone marrow suppression is a possible side effect of methotrexate, and the patient's low WBC count places the patient at high risk for infection. The elevated erythrocyte sedimentation rate and positive rheumatoid factor are expected in rheumatoid arthritis. The blood glucose is normal.)

When the nurse is reviewing laboratory data for a patient who is taking methotrexate (Rheumatrex) to treat rheumatoid arthritis, which information is most important to communicate to the health care provider? A. The blood glucose is 75 mg/dL. B. The rheumatoid factor is positive. C. The white blood cell (WBC) count is 1500/mL. D. The erythrocyte sedimentation rate is elevated.

b (The elevated BUN and creatinine levels indicate possible lupus nephritis and a need for a change in therapy to avoid further renal damage. The positive lupus erythematosus, LE, cell prep and ANA would be expected in a patient with SLE. A drop in CRP shows an improvement in the inflammatory process.)

When the nurse is reviewing laboratory results for a patient with systemic lupus erythematosus (SLE), which result is most important to communicate to the health care provider? A. Decreased C-reactive protein (CRP) B. Elevated blood urea nitrogen (BUN) C. Positive antinuclear antibodies (ANA) D. Positive lupus erythematosus cell prep

c (b-blockers such as propranolol can cause bronchospasm in some patients. The other information will be documented in the health history but does not indicate a need for a change in therapy.)

When the nurse takes an admission history for a patient with possible asthma who has new-onset wheezing and shortness of breath, which information may indicate a need for a change in therapy? A. The patient has a history of pneumonia 2 years ago. B. The patient has chronic inflammatory bowel disease. C. The patient takes propranolol (Inderal) for hypertension. D. The patient uses acetaminophen (Tylenol) for headaches.

d (Repetitive strain injuries caused by prolonged times working at a keyboard can be prevented by the use of a pad that will keep the wrists in a straight position. Stretching exercises during the day may be helpful, but these would not be needed before starting. Use of a compression bandage is not needed, although a splint may be used for carpal tunnel syndrome. NSAIDs are appropriate to use to decrease swelling.)

When working with a patient whose job involves many hours of word processing, the nurse will teach the patient about the need to A. do stretching and warm-up exercises before starting work. B. wrap the wrists with a compression bandage every morning. C. use acetaminophen (Tylenol) instead of nonsteroidal anti-inflammatory drugs (NSAIDs) for wrist pain. D. obtain a keyboard pad to support the wrist while word processing.

Loop Diuretics

bumetanide, ethacrynic acid, furosemide (Lasix), torsemide- inhibit resorption of sodium and chloride- Check potassium and give in Morning.

d ( The air bubble is not ejected before giving Arixtra. The other actions by the nurse areappropriate.)

Which action by a nurse who is administering fondaparinux (Arixtra) to a patient with venous thromboembolism (VTE) indicates that more education about the medication is needed? A. The nurse avoids rubbing the injection site after giving the medication. B. The nurse injects the medication into the abdominal subcutaneous tissue. C. The nurse fails to assess the partial thromboplastin time (PTT) before administration of the medication. D. The nurse ejects the air bubble in the syringe before administering the Arixtra.

c (Readings in the yellow zone indicate a decrease in peak flow; the patient should use short-acting b2-adrenergic, SABA, medications. The best of three peak flow readings should be recorded. Readings in the green zone indicate good asthma control. The patient should exhale quickly and forcefully through the peak flow meter mouthpiece to obtain the readings)

Which action by a patient who has asthma indicates a good understanding of the nurse's teaching about peak flow meter use? A. The patient records an average of three peak flow readings every day. B. The patient inhales rapidly through the peak flow meter mouthpiece. C. The patient uses the albuterol (Proventil) metered-dose inhaler (MDI) for peak flows in the yellow zone. D. The patient calls the health care provider when the peak flow is in the green zone.

a (The patient's assessment indicates impending respiratory failure, and the nurse should prepare to assist with intubation and mechanical ventilation. IV corticosteroids require several hours before having any effect on respiratory status. The patient will not be able to cough or deep breathe effectively. Documentation is not a priority at this time.)

Which action should the nurse anticipate taking first when a patient who is experiencing an asthma attack develops bradycardia and a decrease in wheezing? A. Assist with endotracheal intubation. B. Document changes in respiratory status. C. Encourage the patient to cough and deep breathe. D. Administer IV methylprednisolone (SoluMedrol).

d (Routine scheduling of airway clearance techniques is an essential intervention for patients with CF. A sweat chloride test is used to diagnose CF, but it does not provide any information about the effectiveness of therapy. There is no indication that the patient is terminally ill. Patients with CF lose excessive sodium in their sweat and require high amounts of dietary sodium.)

Which action will be included in the plan of care for a 23-year-old with cystic fibrosis (CF) who is admitted to the hospital with increased dyspnea? A. Schedule a sweat chloride test. B. Arrange for a hospice nurse visit. C. Place the patient on a low-sodium diet. D. Perform chest physiotherapy every 4 hours.

d (Buck's traction keeps the leg immobilized and reduces painful muscle spasm. Hip contractures and dislocation are unlikely to occur in this situation. The peripheral pulses will be assessed, but this does not help in evaluating the effectiveness of Buck's traction.)

Which action will the nurse take in order to evaluate the effectiveness of Buck's traction for a patient who has an intracapsular fracture of the left femur? A. Assess for hip contractures. B. Monitor for hip dislocation. C. Check the peripheral pulses. D. Ask about left hip pain level.

b, d (Rationale: The patient with lower extremity venous thromboembolism, VTE, may or may not have unilateral leg edema, extremity pain, a sense of fullness in the thigh or calf, paresthesias, warm skin, erythema, or a systemic temperature >100.4F, 38 C. If the calf is involved, it may be tender to palpation. A positive Homans' sign, pain on forced dorsiflexion of the foot when the leg is raised, is a classic but very unreliable sign with frequent false positives.)

Which are probable clinical findings in a person with an acute VTE (select all that apply): A. pallor and coolness of the foot and calf B. mild to moderate calf pain and tenderness C. grossly diminished or absent pedal pulses D. unilateral edema and induration of the thigh E. palpable cord along a superficial varicose vein

a (Dark colored stools may indicate that the patient is experiencing gastrointestinal bleeding caused by the naproxen. The information about the patient's ongoing pain and weight gain also will be reported and may indicate a need for a different treatment and/or counseling about avoiding weight gain, but these are not as large a concern as the possibility of gastrointestinal bleeding. Use of capsaicin cream with oral medications is appropriate.)

Which assessment finding about a patient who has been using naproxen (Naprosyn) for 3 weeks to treat osteoarthritis is most important for the nurse to report to the health care provider? A. The patient has dark colored stools. B. The patient's pain has not improved. C. The patient is using capsaicin cream (Zostrix). D. The patient has gained 3 pounds over 3 weeks.

b, c, d (Rationale: Both Buerger's disease and Raynaud's phenomenon have the following clinical manifestations in common: cold sensitivity, ischemic and gangrenous ulcers on fingertips, and color changes of the distal extremity, fingers or toes.)

Which clinical manifestation are seen in both patients with Buerger's disease and Raynaud's phenomenon (select all that apply): A. intermittent fevers B. sensitivity to cold temperatures C. gangrenous ulcers of fingertips D. color changes of fingers and toes E. episodes of superficial vein thrombosis

b (Pulmonary function testing will help establish the COPD diagnosis. The other tests would be used to test for an allergic component for asthma, but will not be used in the diagnosis of COPD.)

Which diagnostic test will the nurse plan to discuss with a 54-year-old patient with progressively increasing dyspnea who is being evaluated for a possible diagnosis of chronic obstructive pulmonary disease (COPD)? A. Eosinophil count B. Pulmonary function testing C. Immunoglobin E (IgE) levels D. Radioallergosorbent test (RAST)

a (For the nursing diagnosis of impaired gas exchange, the best data for evaluation are arterial blood gases, ABGs, or pulse oximetry. The other data may indicate either improvement or impending respiratory failure caused by fatigue.)

Which finding by the nurse for a patient with a nursing diagnosis of impaired gas exchange will be most useful in evaluating the effectiveness of treatment? A. Pulse oximetry reading of 91% B. Absence of wheezes or crackles C. Decreased use of accessory muscles D. Respiratory rate of 22 breaths/minute

b (Flushing and dizziness may indicate that the patient is experiencing an anaphylactic reaction; immediate intervention is needed. The other information also should be reported, but do not indicate possibly life-threatening complications of the omalizumab therapy.)

Which finding in a patient who has received omalizumab (Xolair) is most important to report immediately to the health care provider? A. Pain at injection site B. Flushing and dizziness C. Respiratory rate 22 breaths/minute D. Peak flow reading 75% of normal

b (Initial symptoms of OA include pain with joint movement. Heberden's nodules occur on the fingers. Redness of the joint is more strongly associated with rheumatoid arthritis, RA, and stiffness in OA is worse right after the patient rests and decreases with joint movement.)

Which finding will the nurse expect when assessing a 60-year-old patient who has osteoarthritis (OA) of the left knee? A. Heberden's nodules B. Pain upon joint movement C. Redness and swelling of the knee joint D. Stiffness that increases with movement

d (Cimetidine interferes with the metabolism of theophylline, and concomitant administration may lead rapidly to theophylline toxicity. The other patient information would not impact on whether the theophylline should be administered or not.)

Which information about a newly admitted patient with chronic obstructive pulmonary disease (COPD) indicates that the nurse should consult with the health care provider before administering the prescribed theophylline? A. The patient has had a recent 10-pound weight gain. B. The patient has a cough productive of green mucus. C. The patient denies any shortness of breath at present. D. The patient takes cimetidine (Tagamet) 150 mg daily.

d (Negative sputum smears indicate that M. tuberculosis is not present in the sputum, and the patient cannot transmit the bacteria by the airborne route. Chest x-rays are not used to determine whether treatment has been successful. Taking medications for 6 months is necessary, but the multidrug-resistant forms of the disease might not be eradicated after 6 months of therapy. Repeat Mantoux testing would not be done since it will not change even with effective treatment.)

Which information about a patient who has a recent history of tuberculosis (TB) indicates that the nurse can discontinue airborne isolation precautions? A. Chest x-ray shows no upper lobe infiltrates. B. TB medications have been taken for 6 months. C. Mantoux testing shows an induration of 10 mm. D. Three sputum smears for acid-fast bacilli are negative.

b (New onset dyspnea suggests a pulmonary embolus, which will require rapid actions such as oxygen administration and notification of the health care provider. The other findings are typical of VTE.)

Which information about a patient who has been admitted with a right calf venous thromboembolism (VTE) requires immediate action by the nurse? A. Complaint of left calf pain B. New onset shortness of breath C. Red skin color of left lower leg D. Temperature of 100.4° F (38° C)

d (Long-acting b2-agonists should be used only in patients who also are using an inhaled cortico-steroid for long-term control. Salmeterol should not be used as the first-line therapy for long-term control. The other information given by the patient requires further assessment by the nurse but is not unusual for a patient with asthma.)

Which information given by an asthmatic patient while the nurse is doing the admission assessment is most indicative of a need for a change in therapy? A. The patient uses cromolyn (Intal) before any aerobic exercise. B. The patient says that the asthma symptoms are worse every spring. C. The patient's heart rate increases after using the albuterol (Proventil) inhaler. D. The patient's only medications are albuterol (Proventil) and salmeterol (Serevent).

d (Prolonged capillary refill may indicate complications such as arterial damage or compartment syndrome. The other findings are typical with a left femur fracture.)

Which information obtained by the emergency department nurse when admitting a patient with a left femur fracture is most important to report to the health care provider? A. Bruising of the left thigh B. Complaints of left thigh pain C. Outward pointing toes on the left foot D. Prolonged capillary refill of the left foot

d (Research supports the use of home oxygen to improve quality of life and prognosis. Since increased dyspnea may be a symptom of an acute process such as pneumonia, the patient should notify the physician rather than increasing the oxygen flow rate if dyspnea becomes worse. Oxygen can be supplied using liquid, storage tanks, or concentrators, depending on individual patient circumstances. Travel is possible by using portable oxygen concentrators.)

Which information will the nurse include in teaching a patient with chronic obstructive pulmonary disease (COPD) who has a new prescription for home oxygen therapy? A. Storage of oxygen tanks will require adequate space in the home. B. Travel opportunities will be limited because of the use of oxygen. C. Oxygen flow should be increased if the patient has more dyspnea. D. Oxygen use can improve the patient's prognosis and quality of life.

a (Orange-colored body secretions are a side effect of rifampin. The other adverse effects are associated with other antituberculosis medications.)

Which information will the nurse include in the patient teaching plan for a patient who is receiving rifampin (Rifadin) for treatment of tuberculosis? A. "Your urine, sweat, and tears will be orange colored." B. "Read a newspaper daily to check for changes in vision." C. "Take vitamin B6 daily to prevent peripheral nerve damage." D. "Call the health care provider if you notice any hearing loss."

c (Elevation of the arm will reduce the amount of swelling and pain. Compression bandages are used to decrease swelling. For the first 24 to 48 hours, cold packs are used to reduce swelling. The wrist should be rested and kept immobile to prevent further swelling or injury.)

Which information will the nurse include when discharging a patient with a sprained wrist from the emergency department? A. Keep the wrist loosely wrapped with gauze. B. Apply a heating pad to reduce muscle spasms. C. Use pillows to elevate the arm above the heart. D. Gently move the wrist through the range of motion.

b (Cold application is helpful in reducing pain during periods of exacerbation of RA. Because the joint pain is chronic, patients are instructed to exercise even when joints are painful. ROM exercises are intended to strengthen joints as well as improve flexibility, so passive ROM alone is not sufficient. Recreational exercise is encouraged but is not a replacement for ROM exercises.)

Which information will the nurse include when teaching range-of-motion exercises to a patient with an exacerbation of rheumatoid arthritis? A. Affected joints should not be exercised when pain is present. B. Application of cold packs before exercise may decrease joint pain. C. Exercises should be performed passively by someone other than the patient. D. Walking may substitute for range-of-motion (ROM) exercises on some days.

Blood sugar is well controlled when Hemoglobin A1C is... "a. Below 7% b. Between 12%-15% c. Less than 180 mg/dL d. Between 90 and 130 mg/dL"

a. Below 7% A1c measures the percentage of hemoglobin that is glycated and determines average blood glucose during the 2 to 3 months prior to testing. Used as a diagnostic tool, A1C levels of 6.5% or higher on two tests indicate diabetes. A1C of 6% to 6.5% is considered prediabetes."

d (Tremors are a common side effect of short-acting b2-adrenergic, SABA, medications and not a reason to avoid using the SABA inhaler. Inhaled corticosteroids do not act rapidly to reduce dyspnea. Rapid inhalation is needed when using a DPI. The patient should hold the breath for 10 seconds after using inhalers.)

Which information will the nurse include when teaching the patient with asthma about the prescribed medications? A. Utilize the inhaled corticosteroid when shortness of breath occurs. B. Inhale slowly and deeply when using the dry-powder inhaler (DPI). C. Hold your breath for 5 seconds after using the bronchodilator inhaler. D. Tremors are an expected side effect of rapidly acting bronchodilators.

d (Skim milk and yogurt are high in calcium. The other choices do not contain any high calcium foods.)

Which menu choice by a patient with osteoporosis indicates that the nurse's teaching about appropriate diet has been effective? A. Pancakes with syrup and bacon B. Whole wheat toast and fruit jelly C. Two-egg omelet and a half grapefruit D. Oatmeal with skim milk and fruit yogurt

d (Assisting a patient who has already been taught how to cough is part of routine postoperative care and within the education and scope of practice for an experienced NAP. Patient teaching and assessment of essential postoperative functions such as circulation and movement should be done by RNs)

Which nursing action in the care plan for a patient who had an open repair of an abdominal aortic aneurysm 3 days previously is appropriate for the nurse to delegate to experienced nursing assistive personnel (NAP)? A. Check the lower extremity strength and movement. B. Monitor the quality and presence of the pedal pulses. C. Teach the patient the signs of possible wound infection. D. Help the patient to use a pillow to splint while coughing.

b (Because renal artery occlusion can occur after endovascular repair, the nurse should monitorparameters of renal function such as intake and output. Chest tubes will not be needed forendovascular surgery, the recovery period will be short, and there will not be an abdominalwound.)

Which nursing action will be included in the plan of care after endovascular repair of an abdominal aortic aneurysm? A. Record hourly chest tube drainage. B. Monitor fluid intake and urine output. C. Check the abdominal wound for redness or swelling. D. Teach the reason for a prolonged rehabilitation process.

b (The risk for aspiration is decreased when patients with a decreased level of consciousness are placed in a side-lying or upright position. Frequent turning prevents pooling of secretions in immobilized patients but will not decrease the risk for aspiration in patients at risk. Monitoring of parameters such as breath sounds and oxygen saturation will help detect pneumonia in immunocompromised patients, but it will not decrease the risk for aspiration. Continuous subglottic suction is recommended for intubated patients but not for all patients receiving enteral feedings.)

Which nursing action will be most effective in preventing aspiration pneumonia in patients who are at risk? A. Turn and reposition immobile patients at least every 2 hours. B. Place patients with altered consciousness in side-lying positions. C. Monitor for respiratory symptoms in patients who are immunosuppressed. D. Provide for continuous subglottic aspiration in patients receiving enteral feedings.

c (After knee arthroplasty, active or passive flexion exercises are used to obtain a 90-degree flexion of the knee. The goal for extension of the knee will be 180 degrees. A compression bandage is used to hold the knee in an extended position after surgery. Full weight bearing is expected before discharge.)

Which nursing action will the nurse include in the plan of care for a patient who has had a total knee arthroplasty? A. Avoid extension of the knee beyond 120 degrees. B. Use a compression bandage to keep the knee flexed. C. Start progressive knee exercises to obtain 90-degree flexion. D. Teach about the need to avoid weight bearing for 4 weeks.

b (When using crutches, patients are usually taught to move the assistive device and the injured leg forward at the same time and then to move the unaffected leg. Patients are discouraged from using furniture to assist with ambulation. The patient is taught to place weight on the hands, not in the axilla, to avoid nerve damage. If the 2- or 4-point gaits are to be used, the crutch and leg on opposite sides move forward, not the crutch and same-side leg.)

Which of the following observations made by the nurse who is evaluating the crutch-walking technique of a patient who is to have no weight bearing on the right leg indicates that the patient can safely ambulate independently? A. The patient keeps the padded area of the crutch firmly in the axillary area when ambulating. B. The patient advances the right leg and both crutches together and then advances the left leg. C. The patient moves the left crutch with the left leg and then the right crutch with the right leg. D. The patient uses the bedside chair to assist in balance as needed when ambulating in the room.

c (Venturi deliver the most precise oxygen concentrations and can be set to give 24%, 28%, 31%, 35%, 40%, or 50%. In COPD patients the venturi mask can be set to deliver low, constant, oxygen.)

Which of the following oxygen delivery systems delivers a precise oxygen concentration and is often used for COPD patients? A. Nasal cannula B. Simple face mask C. Venturi face mask D. Nonrebreather face mask

a (NAP can obtain oxygen saturation, after being trained and evaluated in the skill. The other actions require more education scope of practice and should be done by LPN/LVNs or by RNs.)

Which of these nursing actions included in the care plan for a patient with chronic obstructive pulmonary disease (COPD) should the nurse delegate to experienced nursing assistive personnel (NAP)? A. Obtain oxygen saturation using pulse oximetry. B. Monitor for increased oxygen need with exercise. C. Teach the patient about safe use of oxygen at home. D. Adjust oxygen to keep saturation in prescribed parameters

b (The patient's presentation is consistent with dissecting thoracic aneurysm, which will require rapid intervention. The other patients do not need urgent interventions.)

Which of these patients admitted to the emergency department should the nurse assess first? A. 62-year-old who has gangrenous ulcers on both feet B. 50-year-old who is complaining of "tearing" chest pain C. 45-year-old who is taking anticoagulants and has bloody stools D. 36-year-old who has right calf tenderness, redness, and swelling

c (OA is more likely to occur in women as a result of estrogen reduction at menopause and in individuals whose work involves repetitive movements and lifting. Moderate exercise, such as softball, reduces risk for OA. Diabetes is not a risk factor for OA. Working on a construction crew would involve nonrepetitive work and thus would not be as risky.)

Which of these patients seen by the nurse in the outpatient clinic is most likely to require teaching about ways to reduce risk for osteoarthritis (OA)? A. A 56-year-old man who is a member of a construction crew B. A 24-year-old man who participates in a summer softball team C. A 49-year-old woman who works on an automotive assembly line D. A 36-year-old woman who is newly diagnosed with diabetes mellitus

a (Rationale: Conditions that increase the risk of aspiration include decreased level of consciousness, difficulty swallowing, dysphagia, and nasogastric intubation with or without tube feeding. With loss of consciousness, the gag and cough reflexes are depressed, and aspiration is more likely to occur. Dysphasia is difficulty with speech. Absent bowel sounds and coarse crackles do not increase the risk for aspiration.)

Which patient is at highest risk of aspiration? a. A 26-year-old patient with continuous enteral tube feedings through a nasogastric tube b. A 67-year-old patient who had a cerebrovascular accident with expressive dysphasia c. A 58-year-old patient with absent bowel sounds immediately after abdominal surgery d. A 92-year-old patient with viral pneumonia and coarse crackles throughout lung fields

a (Severe skin reactions can occur in patients with SLE who are exposed to the sum. Patients should avoid fatigue by balancing exercise with rest periods as needed. Oral contraceptives can exacerbate lupus. Aspirin and nonsteroidal anti-inflammatory drugs are used to treat the musculoskeletal manifestations of SLE.)

Which statement by a 24-year-old woman with systemic lupus erythematosus (SLE) indicates that the patient has understood the nurse's teaching about management of the condition? A. "I will use a sunscreen whenever I am outside." B. "I will try to keep exercising even if I am tired." C. "I should take birth control pills to keep from getting pregnant." D. "I should not take aspirin or nonsteroidal anti-inflammatory drugs."

c (Patients should continue to cough and deep breathe after discharge. Fatigue for several weeks is expected. Home oxygen therapy is not needed with successful treatment of pneumonia. The pneumovax and influenza vaccines can be given at the same time.)

Which statement by a patient who has been hospitalized for pneumonia indicates a good understanding of the discharge instructions given by the nurse? A. "I will call the doctor if I still feel tired after a week." B. "I will need to use home oxygen therapy for 3 months." C. "I will continue to do the deep breathing and coughing exercises at home." D. "I will schedule two appointments for the pneumonia and influenza vaccines."

a (The patient lies in the prone position several times daily to prevent flexion contractures of the hip. The limb sock should be changed daily. Lotion should not be used on the stump. The residual limb should not be elevated because this would encourage flexion contracture.)

Which statement by a patient who has had an above-the-knee amputation indicates that the nurse's discharge teaching has been effective? A. "I should lay on my abdomen for 30 minutes 3 or 4 times a day." B. "I should elevate my residual limb on a pillow 2 or 3 times a day." C. "I should change the limb sock when it becomes soiled or stretched out." D. "I should use lotion on the stump to prevent drying and cracking of the skin."

c (Inhaled corticosteroids are more effective in improving asthma than any other drug and are indicated for all patients with persistent asthma. The other therapies would not typically be first-line treatments for newly diagnosed asthma.)

Which topic will the nurse include in medication teaching for a patient with newly-diagnosed persistent asthma? A. Use of long-acting b-adrenergic medications B. Side effects of sustained-release theophylline C. Self-administration of inhaled corticosteroids D. Complications associated with oxygen therapy

c (Compression of the leg is essential to healing of venous stasis ulcers. High dietary intake of protein, rather than carbohydrates, is needed. Prophylactic antibiotics are not routinely used forvenous ulcers. Moist environment dressings are used to hasten wound healing)

Which topic will the nurse include in patient teaching for a patient with a venous stasis ulcer on the right lower leg? A. Adequate carbohydrate intake B. Prophylactic antibiotic therapy C. Application of compression to the leg D. Methods of keeping the wound area dry

d (Respiratory infections are one of the most common precipitating factors of an acute asthma attack. Sensitivity to food and drugs may also precipitate attacks, and exercise-induced asthma probably occurs to some extent in all patients with asthma. Psychologic factors can interact with the asthmatic response to worsen the disease, but it is not a psychosomatic disease)

While assisting a patient with asthma to identify specific triggers of the asthma, the nurse explains that A. food and drug allergies do not manifest in respiratory symptoms B. exercise-induced asthma is seen only in individuals with sensitivity to cold air C. asthma attack are psychogenic in origin and can be controlled with relaxation techniques D. viral upper respiratory infections are a common precipitating factor in acute asthma attacks

d (Cramping that is precipitated by a consistent level of exercise is descriptive of intermittent claudication. Finger pain associated with cold weather is typical of Raynaud's phenomenon.Fatigue that occurs sometimes with exercise is not typical of intermittent claudication, which is reproducible. Swelling associated with prolonged standing is typical of venous disease.)

While working in the outpatient clinic, the nurse notes that the medical record states that a patient has intermittent claudication. Which of these statements by the patient would be consistent with this information? A. "When I stand too long, my feet start to swell up." B. "Sometimes I get tired when I climb a lot of stairs." C. "My fingers hurt when I go outside in cold weather." D. "My legs cramp whenever I walk more than a block."

a, e

Zileuton is associated with A. Long-term control B. Quick-relief C. B2-adrenergic agonist D. Mast-cell stabilizer E. Leukotriene inhibitor F. Steroid antiinflammatory G. Methylxanthine bronchodilator H. Anticholinergic I. Anti-IgE

Physiological Integrity 4. When teaching the client about the procedure of a bone marrow aspiration, the nurse should explain that it is used to a. identify blood abnormalities. b. determine long-term prognosis. c. assess for presence of infection. d. determine the red blood cell (RBC) indices.

a Bone marrow aspiration and biopsy are used to assess and identify most blood dyscrasias (e.g., aplastic anemia, leukemias, pernicious anemia, thrombocytopenia). DIF: Cognitive Level: Comprehension REF: Text Reference: 2265 TOP: Nursing Process Step: Intervention MSC:

Which one of the following methods/techniques will the nurse use when giving insulin to a thin person? [Hint] A. Pinch the skin up and use a 90 degree angle B. Use a 45 degree angle with the skin pinched up C. Massage the area of injection after injecting the insulin D. Warm the skin with a warmed towel or washcloth prior to the injection

a. Pinch the skin up and use a 90 degree angle The best angle for a thin person is 90 degrees with the skin pinched up. The area is not massaged and it is not necessary to warm it.

Physiological Integrity Black & Hawks: Medical-Surgical Nursing: Clinical Management for Positive Outcomes, 7th Edition Chapter 77: Management of Clients with Hematologic Disorders MULTIPLE CHOICE 1. The nurse assessing a female client with a hemoglobin of 11 g/100 ml would expect the client to report a. no significant manifestations. b. shortness of breath. c. tachycardia. d. chronic fatigue.

a Clients with mild anemia (hemoglobin of 10 to 12 g/ml) are usually asymptomatic. DIF: Cognitive Level: Knowledge REF: Text Reference: 2273 TOP: Nursing Process Step: Assessment MSC:

Physiological Integrity 35. The nurse would include in the care of the client with multiple myeloma the intervention of a. forcing fluids. b. encouraging ingestion of dairy products. c. maintaining reverse isolation. d. administering frequent mouth care.

a Clients with multiple myeloma usually require about 3 L of fluid per day. The client needs sufficient fluid not only to dilute the calcium overload but also prevent protein from precipitating in the renal tubules, even after being effectively treated with chemotherapy. DIF: Cognitive Level: Application REF: Text Reference: 2303 TOP: Nursing Process Step: Intervention MSC:

Physiological Integrity 10. The nurse explains that the drug essential for a client with pernicious anemia is a. vitamin B12. b. ferrous gluconate. c. vitamin K. d. ferritin.

a Clients with pernicious anemia need both immediate care and lifelong therapy with maintenance vitamin B12. DIF: Cognitive Level: Knowledge REF: Text Reference: 2290 TOP: Nursing Process Step: Intervention MSC:

Physiological Integrity 32. The nurse recognizes a common historical finding in a client with multiple myeloma, which is a. employment in a farm industry. b. a diet that is high in fat. c. long-term smoking. d. excessive exposure to the sun.

a Etiologic factors include exposure to ionizing radiation and occupational chemical exposure. DIF: Cognitive Level: Application REF: Text Reference: 2302 TOP: Nursing Process Step: Assessment MSC:

Physiological Integrity 7. Following a client's bone marrow aspiration, the nursing intervention should be to a. apply firm pressure to the area until bleeding stops. b. provide a warm, moist compress to the site. c. encourage the client to assume a supine position. d. place an occlusive dressing on the area for 24 hours.

a Following the procedure, apply pressure until the bleeding stops. A pressure dressing or a sandbag might be necessary. DIF: Cognitive Level: Application REF: Text Reference: 2265 TOP: Nursing Process Step: Intervention MSC:

Physiological Integrity 3. A client has a hematocrit (Hct) of 30%. The nurse interprets this to mean a. the individual has fewer red blood cells than normal. b. the blood is viscous and concentrated. c. bleeding disorders are possible. d. 30% of the blood will be plasma and plasma products.

a Hematocrit measures the percent volume of red cells in whole blood. The normal value in adult women is 37% to 45%. DIF: Cognitive Level: Comprehension REF: Text Reference: 2262 TOP: Nursing Process Step: Assessment MSC:

Physiological Integrity 2. The nurse should anticipate an elevated hemoglobin level in a. a client who lives in Colorado. b. a dehydrated elderly gentleman being treated with intravenous (IV) fluids. c. a 40-year-old woman with congestive heart failure. d. a client with iron deficiency anemia.

a Hemoglobin levels are frequently elevated in people who live in high altitudes. DIF: Cognitive Level: Application REF: Text Reference: 2258 TOP: Nursing Process Step: Assessment MSC:

Health Promotion and Maintenance 26. A client who has hemophilia A and his wife, who is not a carrier of the disease, wish to start a family. In discussing the risk factors of transmitting hemophilia to his children, it is important to explain that a. none of his children are likely to have hemophilia. b. all of his children will be carriers. c. all of his sons will have hemophilia. d. 50% of his children are at risk for developing the disease.

a Hemophilia is genetically transmitted in a sex-linked (X chromosome) recessive pattern. Females usually transmit the defective gene, but males express the bleeding disorder. Males with hemophilia transmit the gene to all of their daughters but to none of their sons. DIF: Cognitive Level: Analysis REF: Text Reference: 2311, Table 77-12; TOP: Nursing Process Step: Intervention MSC:

Physiological Integrity 30. When a client experiences an adverse reaction to a blood transfusion, the nurse should initially a. discontinue the transfusion. b. notify the physician. c. administer oxygen via nasal prongs. d. raise the head of the bed.

a In all cases of transfusion reaction, stop the transfusion and keep the intravenous (IV) line open with normal saline. DIF: Cognitive Level: Application REF: Text Reference: 2284 TOP: Nursing Process Step: Intervention MSC:

Physiological Integrity 6. The manifestation the nurse would expect to find on laboratory test results on a 52-year-old who has had severe anemia for many years is a. microcytic and hypochromic red blood cells (RBCs). b. a hemoglobin level of 9.5 g/dl. c. serum iron level of 58 mg/dl. d. total iron-binding capacity reduced to less than 250 mg/dl.

a In severe cases, peripheral blood smears reveal microcytic and pale (hypochromic) RBCs. DIF: Cognitive Level: Analysis REF: Text Reference: 2286 TOP: Nursing Process Step: Assessment MSC:

Physiological Integrity 34. The nurse reassures a client newly diagnosed with multiple myeloma and is asymptomatic that he will initially be treated by a. close monitoring. b. alkalating agents. c. the VAD regimen: vincristine (Oncovin), doxorubicin (Adriamycin), and dexamethasone (Decadron). d. interferon.

a In the initial phases when clients are asymptomatic, they are often carefully monitored until the disease progresses and then are treated with chemotherapy. DIF: Cognitive Level: Comprehension REF: Text Reference: 2303 TOP: Nursing Process Step: Intervention MSC:

Physiological Integrity 15. The nursing diagnosis that would have priority in the care of a client with aplastic anemia is a. Risk for Infection due to decreased leukocyte count. b. Impaired Gas Exchange due to low RBC count. c. potential for Impaired Skin Integrity due to poor nutritional status. d. alteration in bowel elimination: Constipation due to iron overload.

a The client suffers from an increased susceptibility to infection because without leukocytes the body cannot adequately battle bacteria and other invading organisms. DIF: Cognitive Level: Analysis REF: Text Reference: 2293 TOP: Nursing Process Step: Assessment MSC:

Physiological Integrity 38. When a child with sickle cell anemia contracts gastroenteritis with vomiting and diarrhea, the nurse cautions the family about the development of the complication of a. hemoconcentration. b. gastric bleeding. c. increased need for oxygen. d. decreased nutritional intake.

a Vomiting and diarrhea can cause a fluid volume deficit that leads to hemoconcentration, a factor in the onset of sickle cell crisis. DIF: Cognitive Level: Application REF: Text Reference: 2298 TOP: Nursing Process Step: Intervention MSC:

Physiological Integrity 25. When a client with sickle cell anemia confides that she wishes to have a child, the nurse should inform her that a. pregnancy could result in vascular complications. b. her child will have sickle cell anemia. c. she has an increased chance of miscarriage. d. there is no reason not to have a child.

a Warn young women with sickle cell anemia that pregnancy carries a very high risk for them. Explain that pulmonary or renal complications, or both, may develop. DIF: Cognitive Level: Application REF: Text Reference: 2299 TOP: Nursing Process Step: Intervention MSC:

Physiological Integrity 20. The manifestation that would require immediate investigation in a client with infectious mononucleosis is a. abdominal pain. b. joint discomfort. c. sore throat. d. leukocyte count of 12,000/mm3.

a When infectious mononucleosis is severe, the client may develop splenic rupture resulting from the infiltration of the spleen by massive numbers of lymphocytes. DIF: Cognitive Level: Application REF: Text Reference: 2304 TOP: Nursing Process Step: Assessment MSC:

A client is ordered a nitroglycerine transdermal patch for treatment of CAD and asks the nurse why the patch is removed at bedtime. Which is the best response by the nurse? a) "Removing the patch at night prevents drug tolerance while keeping the benefits." b) "Contact dermatitis and skin irritations are common when the patch remains on all day." c) "Nitroglycerine causes headaches, but removing the patch decreases the incidence." d) "You do not need the effects of nitroglycerine while you sleep."

a) "Removing the patch at night prevents drug tolerance while keeping the benefits."

A client with diabetes comes to the clinic for a follow-up visit. The nurse reviews the client's glycosylated hemoglobin test results. Which result would indicate to the nurse that the client's blood glucose level has been well controlled? a) 6.5% b) 8.0% c) 7.5% d) 8.5%

a) 6.5%

A client in the emergency department complains of squeezing substernal pain that radiates to the left shoulder and jaw. He also complains of nausea, diaphoresis, and shortness of breath. What should the nurse do? a) Administer oxygen, attach a cardiac monitor, take vital signs, and administer sublingual nitroglycerin. b) Alert the cardiac catheterization team, administer oxygen, attach a cardiac monitor, and notify the physician. c) Gain I.V. access, give sublingual nitroglycerin, and alert the cardiac catheterization team. d) Complete the client's registration information, perform an electrocardiogram, gain I.V. access, and take vital signs.

a) Administer oxygen, attach a cardiac monitor, take vital signs, and administer sublingual nitroglycerin.

The nurse assisting in the admission of a client with diabetic ketoacidosis will anticipate the physician ordering which of the following types of intravenous solution if the client cannot take fluids orally? a. 0.45% normal saline solution b. Lactated Ringer's solution c. 0.9 normal saline solution d. 5% dextrose in water (D5W)

a. 0.45% normal saline solution

"A client who is started on metformin and glyburide would have initially presented with which symptoms? "a. Polydispisa, polyuria, and weight loss b. weight gain, tiredness, & bradycardia c. irritability, diaphoresis, and tachycardia d. diarrhea, abdominal pain, and weight loss

a. Polydispisa, polyuria, and weight loss"Symptoms of hyperglycemia include polydipsia, polyuria, and weight loss. Metformin and sulfonylureas are commonly ordered medications. Weight gain, tiredness, and bradycardia are symptoms of hypothyroidism. Irritability, diaphoresis, and tachycardia are symptoms of hypoglycemia. Symptoms of Crohn's disease include diarrhea, abdominal pain, and weight loss."

A patient newly diagnosed with Type I DM is being seen by the home health nurse. The doctors orders include: 1200 calorie ADA diet, 15 units NPH insulin before breakfast, and check blood sugar qid. When the nurse visits the patient at 5 pm, the nurse observes the man performing blood sugar analysis. The result is 50 mg/dL. The nurse would expect the patient to be a. confused with cold, clammy skin an pulse of 110 b. lethargic with hot dry dkin and rapid deep respirations c. alert and cooperative with BP of 130/80 and respirations of 12 d. short of breath, with distended neck veins and bounding pulse of 96.

a. confused with cold, clammy skin an pulse of 110 hypoglycemia

Physiological Integrity 8. The nurse notes that a client has a higher than normal reticulocyte count. This would indicate a. polycythemia vera. b. increased erythrocyte production. c. dehydration. d. bone marrow depression.

b An increase in the reticulocyte count indicates an increase in erythrocyte production, probably because of excessive RBC destruction (hemolytic anemia) or loss (hemorrhage). DIF: Cognitive Level: Analysis REF: Text Reference: 2264 TOP: Nursing Process Step: Assessment MSC:

Physiological Integrity 21. The nursing order that would be counterproductive for a client with infectious mononucleosis is a. maintain bed rest. b. limit fluids. c. provide throat irrigations. d. use sponge baths for fever.

b Bed rest is recommended until fever is resolved. Acetaminophen, cool sponge baths, and a large fluid intake help control fever. Warm saline throat irrigations may relieve the sore throat. Aspirin is avoided because of the risk of Reye's syndrome. DIF: Cognitive Level: Application REF: Text Reference: 2304 TOP: Nursing Process Step: Intervention MSC:

Physiological Integrity 7. The food the nurse would include in the discussion relative to foods that are high in iron is a. citrus fruits. b. green leafy vegetables. c. milk products. d. grains.

b Encourage foods cooked in iron pots and ingestion of foods such as liver (the richest source), oysters, lean meats, kidney beans, whole wheat bread, kale, spinach, egg yolk, turnip tops, beet greens, carrots, apricots, and raisins. DIF: Cognitive Level: Application REF: Text Reference: 2285 TOP: Nursing Process Step: Intervention MSC:

Physiological Integrity 11. The manifestation the nurse would question the client about that is characteristically associated with anemia is a. pruritus. b. fatigue. c. rash. d. ruddy skin color.

b Fatigue is the most common manifestation of anemia. DIF: Cognitive Level: Analysis REF: Text Reference: 2255 TOP: Nursing Process Step: Assessment MSC:

Physiological Integrity 16. The nurse is monitoring the laboratory test results for a client receiving anticoagulation therapy. The nurse is aware that the International Normalized Ratio (INR) for most clinical conditions, requiring anticoagulation is a. less than 1. b. 1 to 2. c. 2 to 3.5. d. 3 to 5.5.

b For most clinical conditions that necessitate anticoagulation, the recommended INR is 2 to 3.5. DIF: Cognitive Level: Knowledge REF: Text Reference: 2265 TOP: Nursing Process Step: Assessment MSC:

Physiological Integrity 23. The nurse assessing a client with sickle cell anemia would recognize the common manifestation characteristic of the disease is a. confusion. b. leg ulcers. c. diarrhea. d. hypertension.

b Leg ulcers are found in about 75% of older children and adults with the disease. DIF: Cognitive Level: Knowledge REF: Text Reference: 2297 TOP: Nursing Process Step: Assessment MSC:

1. In evaluating a young woman, the following laboratory result the nurse recognizes as abnormal is a. hemoglobin 13 g/dl. b. platelet count 20,000/mm3. c. red blood cell count 5 million/mm3. d. white blood cell count 6000/mm3.

b Normal platelet count is 150,000/mm3. DIF: Cognitive Level: Application REF: Text Reference: 2262, Table 76-3; TOP: Nursing Process Step: Assessment MSC:

Physiological Integrity 33. When assessing the client with multiple myeloma, the nurse would expect to find the manifestation of a. shortness of breath. b. bone pain. c. ecchymosis of the skin. d. painless enlarged lymph nodes.

b Once manifestations appear, they typically involve the skeletal system, particularly the pelvis, spine, and ribs. Some clients have backache or bone pain that worsens with movement. DIF: Cognitive Level: Application REF: Text Reference: 2302 TOP: Nursing Process Step: Assessment MSC:

Physiological Integrity 36. When caring for a client with multiple myeloma, the nurse should take appropriate precautions to a. alleviate respiratory difficulty. b. prevent fractures. c. prevent seizures. d. alleviate diarrhea.

b Some clients have backache or bone pain that worsens with movement. Others suffer sudden pathologic fractures accompanied by severe pain. Because of skeletal complications, care should be taken when moving the client. Family members should institute safety measures to prevent falls. DIF: Cognitive Level: Application REF: Text Reference: 2303 TOP: Nursing Process Step: Intervention MSC:

Physiological Integrity 4. The statement about dietary iron made by a client with iron deficiency anemia that indicates understanding of the dietary concepts is a. "I know that iron from animal sources is not absorbed well." b. "I will not be able to obtain enough iron by just increasing my dietary intake." c. "I should be able to change my diet so that I can get sufficient iron." d. "I know that dairy products are the best source of iron."

b The amount of iron normally absorbed daily is sufficient for meeting the needs of women past the childbearing age and healthy men, but it does not meet the greater needs of menstruating and pregnant women, adolescents, children, and infants. DIF: Cognitive Level: Application REF: Text Reference: 2274 TOP: Nursing Process Step: Evaluation MSC:

Physiological Integrity 21. A client with polycythemia vera will have a characteristic skin color that is a. pale. b. ruddy. c. bronzed. d. jaundiced.

b The client with polycythemia vera has a ruddy complexion. DIF: Cognitive Level: Knowledge REF: Text Reference: 2259 TOP: Nursing Process Step: Assessment MSC:

Physiological Integrity 37. The nurse informs the client that the confirming finding for perncious anemia from a gastric secretion analysis is that after the administration of histamine, the gastric secretion shows a a. rebound increase in volume. b. a low volume with a high pH. c. reduction of free hydrochloric acid. d. a characteristic blue color.

b The gastric secretion analysis shows the same low volume with a high pH and high free hydrochloric acid levels even after the administration of histamine. DIF: Cognitive Level: Comprehension REF: Text Reference: 2290 TOP: Nursing Process Step: Intervention MSC:

Physiological Integrity 9. The laboratory test result that would be most helpful to the nurse in the assessment of a client with a bleeding disorder is a. RBC count. b. platelet count. c. hematocrit. d. differential count.

b The platelet count is valuable in assessing the severity of thrombocytopenia (abnormally low platelet count), which can result in spontaneous bleeding. DIF: Cognitive Level: Analysis REF: Text Reference: 2265 TOP: Nursing Process Step: Assessment MSC:

Physiological Integrity 24. The nurse explains that the definitive laboratory findng confirming the diagnosis of sickle cell anemia is a. hemoglobin of less than 9 g/dl. b. presence of hemoglobin S (Hgb S). c. increase in hemoglobin G (Hgb G). d. folate deficiency.

b The presence of Hgb S is the definitive finding that confirms the diagnosis of sickle cell anemia. DIF: Cognitive Level: Knowledge REF: Text Reference: 2295 TOP: Nursing Process Step: Intervention MSC:

Physiological Integrity 27. Using Y-set with a filter, the nurse prepares to initiate a blood transfusion with the intravenous solution of a. D5W. b. normal saline. c. lactated Ringer's. d. dextran.

b To prevent hemolysis, add no solution other than normal saline to blood components. DIF: Cognitive Level: Knowledge REF: Text Reference: 2283 TOP: Nursing Process Step: Intervention MSC:

Integrated Process: Nursing Process (Assessment) OTHER 1. The nurse is teaching a client to cough productively. Put the actions in proper sequence. a. Have the client flex the head and hold a pillow to the stomach. b. Assist the client to a sitting position with feet on the floor. c. Instruct the client to bend forward and to cough two or three times. d. Have the client return to an upright position and take a deep breath. e. Encourage the client to take several deep breaths.

b, a, e, c, d When the client can tolerate it, the best position for effective coughing and secretion removal is sitting with the shoulders turned inward and the head bent slightly down while hugging a pillow. The client should take several deep breaths followed by holding the breath slightly before coughing two or three times in a row. Then the client should cough at the end of exhalation; this should be followed by taking several deep breaths. DIF: Cognitive Level: Application/Applying or higher REF: N/A TOP: Client Needs Category: Health Promotion and Maintenance (Self-Care)

The nurse instructs a 22-year-old female patient with diabetes mellitus about a healthy eating plan. Which statement made by the patient indicates that teaching was successful? a. "I plan to lose 25 pounds this year by following a high-protein diet." b. "I may have a hypoglycemic reaction if I drink alcohol on an empty stomach." c. "I should include more fiber in my diet than a person who does not have diabetes." d. "If I use an insulin pump, I will not need to limit the amount of saturated fat in my diet."

b. "I may have a hypoglycemic reaction if I drink alcohol on an empty stomach."

One of the benefits of Glargine (Lantus) insulin is its ability to: a. Release insulin rapidly throughout the day to help control basal glucose. b. Release insulin evenly throughout the day and control basal glucose levels. c. Simplify the dosing and better control blood glucose levels during the day. d. Cause hypoglycemia with other manifestation of other adverse reactions.

b. Release insulin evenly throughout the day and control basal glucose levels. Glargine (Lantus) insulin is designed to release insulin evenly throughout the day and control basal glucose levels.

Creatine kinase-MB isoenzyme (CK-MB) can increase as a result of: a) skeletal muscle damage due to a recent fall. b) myocardial necrosis. c) I.M. injection. d) cerebral bleeding.

b. myocardial necrosis

Physiological Integrity 19. The nurse schedules the time to access delayed type hypersensitivity (DTH) skin testing, which is a. 24 hours postinjection. b. 36 hours postinjection. c. 48 hours postinjection. d. 72 hours postinjection.

c A DTH should be read after 48 hours for the most reliable results. DIF: Cognitive Level: Application REF: Text Reference: 2268 TOP: Nursing Process Step: Intervention MSC:

Physiological Integrity 6. A client expresses concern over the discomfort expected during the bone marrow aspiration. The nurse can best address these concerns by informing the client a. "You will be asleep during the procedure." b. "There is no pain associated with bone marrow aspirations." c. "The pain during marrow aspiration is of short duration." d. "A local anesthetic will make you comfortable during the procedure."

c Because the marrow space itself cannot be anesthetized, removal of the marrow usually produces moderate to severe pain of short duration. It stops as soon as suction on the marrow space is stopped. DIF: Cognitive Level: Comprehension REF: Text Reference: 2265 TOP: Nursing Process Step: Intervention MSC:

Physiological Integrity 5. The nurse explains to a client who is to undergo a bone marrow aspiration that the most common site for withdrawal of bone marrow is a. antecubital fossa. b. long bones. c. posterior iliac crests. d. acetabulum.

c Bone marrow samples are most commonly taken from the posterior iliac crests. Other sampling sites include the sternum and the anterior iliac crests. DIF: Cognitive Level: Comprehension REF: Text Reference: 2265 TOP: Nursing Process Step: Intervention MSC:

Physiological Integrity 2. While performing an admission assessment on a moderately anemic client, the nurse would expect to find a history of a. blurred vision. b. increased appetite. c. cardiac palpitations. d. warm, flushing sensations.

c Clients with moderate anemia may suffer from dyspnea, palpitations, diaphoresis with exertion, and chronic fatigue. DIF: Cognitive Level: Knowledge REF: Text Reference: 2273 TOP: Nursing Process Step: Assessment MSC:

Physiological Integrity 12. A client presents to the ambulatory care center seeking treatment for allergies. When questioning the client about contact agents as allergic triggers, the nurse would inquire about a. pollen. b. molds and spores. c. types of clothing fibers. d. food additives.

c Contact agents as allergic triggers include dyes in clothing, fibers, and cosmetics; metal in jewelry; plant oils and secretions; topical drugs; and numerous chemicals. DIF: Cognitive Level: Analysis REF: Text Reference: 2255 TOP: Nursing Process Step: Assessment MSC:

Physiological Integrity 14. A client reports getting allergy manifestations when in the workplace. Further assessment about airborne allergens should focus on a. lighting. b. food service vendor. c. heating and cooling systems. d. water supply.

c Exposure to allergens at work may trigger reactions. Ask about the heating and cooling systems if airborne allergens are suspected. DIF: Cognitive Level: Application REF: Text Reference: 2258 TOP: Nursing Process Step: N/A MSC:

Physiological Integrity 13. The nurse should teach a client with pernicious anemia who is being treated with folic acid to report any manifestations related to the a. cardiovascular system. b. gastrointestinal system. c. neurologic system. d. respiratory system.

c Folic acid can be dangerous because it may intensify neurologic problems, and large doses of folate may obscure a vitamin B12 deficiency. DIF: Cognitive Level: Comprehension REF: Text Reference: 2290 TOP: Nursing Process Step: Intervention MSC:

Physiological Integrity 16. When obtaining a nursing history from an anemic male client, the nurse would recognize a factor significant in the development of anemia is a. smoking one pack of cigarettes daily. b. a diet high in cholesterol. c. large hemorrhoids. d. oral contraceptive intake.

c Gastrointestinal (GI) tract bleeding is a common etiologic factor in men; it may result from peptic ulcers, hiatal hernia, gastritis, cancer, hemorrhoids, diverticula, ulcerative colitis, or salicylate poisoning. DIF: Cognitive Level: Application REF: Text Reference: 2286 TOP: Nursing Process Step: Assessment MSC:

Physiological Integrity 22. The nurse counsels a client with idiopathic thrombocytopenic purpura (ITP) that if medication therapy is not effective, the surgical procedure is most likely to be used in the treatment is a. hepatic shunt. b. exploratory laparotomy. c. splenectomy. d. bone marrow transplant.

c If the client does not have a sustained remission, splenectomy may be needed. In 60% to 80% of cases, removal of the spleen results in complete and permanent remission. DIF: Cognitive Level: Knowledge REF: Text Reference: 2306 TOP: Nursing Process Step: Intervention MSC:

Health Promotion and Maintenance 8. The nurse explaining aspects of pernicious anemia to the client would include information regarding a. frequent bouts of dyspnea. b. the risks relative to dehydration. c. deficiency of intrinsic factor. d. lack of any effective treatment for this disorder.

c Pernicious anemia is a type of anemia due to failure of absorption of vitamin B12 (cobalamin). The most common cause is lack of intrinsic factor, a glucoprotein produced by the parietal cells of the gastric lining. DIF: Cognitive Level: Knowledge REF: Text Reference: 2289 TOP: Nursing Process Step: Intervention MSC:

Physiological Integrity 10. A client has severe anemia and is being treated with transfusion therapy. The nurse should be alert for a complicaton of transfusion, such as a. hearing loss. b. liver damage. c. flank pain. d. sore throat.

c Reactions to blood products include fever, chills, back or flank pain, shock, wheezing, headache, vomiting, or urticaria (hives). DIF: Cognitive Level: Application REF: Text Reference: 2257 TOP: Nursing Process Step: Assessment MSC:

Health Promotion and Maintenance 5. The nurse would instruct the client with iron deficiency anemia to avoid a. citrus fruits. b. poultry. c. tea. d. leafy green vegetables.

c Tannates (in tea and coffee), carbonates, the chelating agent ethylenediaminetetraacetic acid (EDTA), and the medicinal antacid magnesium trisilicate all hinder non-heme iron absorption. DIF: Cognitive Level: Application REF: Text Reference: 2286 TOP: Nursing Process Step: Intervention MSC:

Physiological Integrity 17. The laboratory result to which the nurse would look to confirm a diagnosis of disseminated intravascular coagulation (DIC) is a. prothrombin time. b. partial thromboplastin time. c. D-dimer. d. reticulocyte count.

c The D-dimer confirms the diagnosis of DIC. DIF: Cognitive Level: Knowledge REF: Text Reference: 2265 TOP: Nursing Process Step: Assessment MSC:

Physiological Integrity 12. The nurse is aware that the situation that would warrant administration of iron supplements to a client with pernicious anemia is a. poor appetite. b. increase in the total erythrocyte count in the peripheral circulation. c. discrepancy between hemoglobin and erythrocyte levels. d. paresthesia in the fingers.

c The client may need oral iron supplementation if the hemoglobin level fails to rise in proportion to an increased RBC count. DIF: Cognitive Level: Analysis REF: Text Reference: 2290 TOP: Nursing Process Step: Assessment MSC:

Physiological Integrity 31. The nurse recognizes the laboratory finding that is characteristic of granulocytosis as a. elevated granulocytes. b. hypoprothrombinemia. c. profound neutropenia. d. thrombocytosis.

c The manifestations of agranulocytosis are a result of the neutropenia. DIF: Cognitive Level: Knowledge REF: Text Reference: 2301 TOP: Nursing Process Step: Assessment MSC:

Physiological Integrity 28. The precautionary nursing intervention that is of primary importance when preparing to administer blood is a. establishing baseline vital signs. b. administration of pretransfusion antihistamines. c. asking a second health care professional to confirm blood acceptability. d. obtaining a written order for the transfusion.

c The most critical phase of transfusion is confirming product compatibility and verifying client identity. DIF: Cognitive Level: Application REF: Text Reference: 2283 TOP: Nursing Process Step: Intervention MSC:

Physiological Integrity 20. A client with thrombocytopenia complains of a visual disturbance. The nurse is aware this visual problem may be due to a. occlusive emboli in the visual center of the brain. b. severe anemia causing altered perfusion to the brain. c. minute retinal hemorrhages. d. bile pigment accumulation in the eye.

c The reduced number of platelets make the thrombocytopenic clients at risk for hemorrhages. DIF: Cognitive Level: Application REF: Text Reference: 2259 TOP: Nursing Process Step: Assessment MSC:

A client comes to the physician's office for a follow-up visit 4 weeks after suffering a myocardial infarction (MI). The nurse takes this opportunity to evaluate the client's knowledge of the ordered cardiac rehabilitation program. Which evaluation statement suggests that the client needs more instruction? a) "Client performs relaxation exercises three times per day to reduce stress." b) "Client verbalizes an understanding of the need to seek emergency help if his heart rate increases markedly while at rest." c) "Client walks 4 miles in 1 hour every day." d) "Client's 24-hour dietary recall reveals low intake of fat and cholesterol."

c) "Client walks 4 miles in 1 hour every day."

Which condition most commonly results in coronary artery disease (CAD)? a) Diabetes mellitus b) Myocardial infarction c) Atherosclerosis d) Renal failure

c) Atherosclerosis

Which of the following diabetes drugs acts by decreasing the amount of glucose produced by the liver? a. Sulfonylureas b. Meglitinides c. Biguanides d. Alpha-glucosidase inhibitors

c. Biguanides Biguanides, such as metformin, lower blood glucose by reducing the amount of glucose produced by the liver. Sulfonylureas and Meglitinides stimulate the beta cells of the pancreas to produce more insulin. Alpha-glucosidase inhibitors block the breakdown of starches and some sugars, which helps to reduce blood glucose levels

The nurse is having difficulty obtaining a capillary blood sample from a client's finger to measure blood glucose using a blood glucose monitor. Which procedure will increase the blood flow to the area to ensure an adequate specimen? a. Raise the hand on a pillow to increase venous flow. b. Pierce the skin with the lancet in the middle of the finger pad. c. Wrap the finger in a warm cloth for 30-60 seconds. d. Pierce the skin at a 45-degree angle

c. Wrap the finger in a warm cloth for 30-60 seconds. The hand is lowered to increase venous flow. The finger is pierced lateral to the middle of the pad perpendicular to the skin surface.

A patient received 6 units of REGULAR INSULIN 3 hours ago. The nurse would be MOST concerned if which of the following was observed? a. kussmaul respirations and diaphoresis b. anorexia and lethargy c. diaphoresis and trembling d. headache and polyuria

c. diaphoresis and trembling indicates hypoglycemia

Non- selective Beta Adrenergic blocking agent

carvedilol (Coreg)- treat CHF and hypertension

Physiological Integrity 13. The nurse assesses the client who underwent partial removal of the stomach a year ago for the manifestations of a. shortened bleeding times. b. high white blood cell count. c. low platelet count. d. anemia.

d Anemia may also occur following partial or total gastrectomy or removal of the terminal portion of the ileum because of the consequent reduction in absorption of vitamin B12. DIF: Cognitive Level: Analysis REF: Text Reference: 2256 TOP: Nursing Process Step: Assessment MSC:

Thiazide Diuretics

chlorpthiazide (Diuril)/ hydrochlorothiazide (Esidrix,HCTZ)- Increases excretion of water and sodium- Check potassium level, take in morning.

FACES

clinical signs of HF: -Fatigues -Limitation of Activities -Chest congestion/cough -Edema -SOB

Peripheral artery disease (Intermittent Claudication)

condition of the blood vessels that leads to narrowing and hardening of the arteries that supply the legs and feet. The narrowing of the blood vessels leads to decreased blood flow, which can injure nerves and other tissues.

Health Promotion and Maintenance 15. The nurse reads that an assigned client has an immunodeficiency. The nurse reads further in the medical record, anticipating that the client also most likely has a history of a. skin eruptions. b. conjunctivitis. c. severe headaches. d. unexplained weight loss.

d Clients with immunodeficiencies have a history of recurrent infections, especially of mucous membranes (e.g., oral cavity, anorectal area, genitourinary [GU] tract, respiratory tract); poor wound healing; diarrhea; and manifestations of systemic activation of the immune response. DIF: Cognitive Level: Analysis REF: Text Reference: 2258 TOP: Nursing Process Step: Assessment MSC:

Physiological Integrity 17. The nurse outlines methods to increase iron in diets that include a. including tea or coffee at mealtime. b. cooking food quickly with high heat setting. c. boiling foods in at least 3 cups of water. d. cooking in iron cookware.

d Cooking in iron cookware, cooking foods at a moderate heat in a minimum of water, and excluding caffeine will improve the iron intake. DIF: Cognitive Level: Comprehension REF: Text Reference: 2285 TOP: Nursing Process Step: Intervention MSC:

Health Promotion and Maintenance 18. The nurse recognizes that the laboratory finding indicative of polycythemia vera is a. erythrocyte count of 5 million/mm3. b. leukocyte count of 6000/mm3. c. platelets of 50,000/mm3. d. hemoglobin level of 22 g/100 ml.

d Diagnostic findings include RBC count as high as 8 to 12 million/mm3, hemoglobin level of 18 to 25 g/dl, and a hematocrit level greater than 54% in men and 49% in women. DIF: Cognitive Level: Application REF: Text Reference: 2300 TOP: Nursing Process Step: Assessment MSC:

Physiological Integrity 29. The nurse can decrease the danger of transfusion reactions in a client by a. forcing fluids. b. adding sterile saline to the blood transfusion. c. monitoring the urine output. d. infusing the blood slowly during the first 15 minutes.

d It is recommended that the transfusion begin slowly and that the client be closely monitored. If no evidence of a reaction is noted within the first 15 minutes, flow can then be increased to the prescribed rate. DIF: Cognitive Level: Application REF: Text Reference: 2284 TOP: Nursing Process Step: Intervention MSC:

Physiological Integrity 9. To determine if the client has a risk factor related to iron deficiency anemia, the nurse could ask a. "Have you ever had a cardiac catheterization?" b. "Have you had a pregnancy terminated within the past 6 months?" c. "Have you had a blood transfusion recently?" d. "Have you ever had any surgery involving your stomach?"

d Malabsorption of iron may result from alterations in the mucosa of the duodenum and proximal jejunum, gastrectomy, or removal of the proximal small bowel, resulting in iron deficiency anemia. DIF: Cognitive Level: Application REF: Text Reference: 2286 TOP: Nursing Process Step: Assessment MSC:

Physiological Integrity 19. The nurse caring for a client with polycythemia vera explains the objective of phlebotomies is to decrease the hematocrit to a. 15%. b. 25%. c. 35%. d. 45%.

d Phlebotomies can be used to normalize red cell mass until the hematocrit reaches 45%. DIF: Cognitive Level: Application REF: Text Reference: 2300 TOP: Nursing Process Step: Intervention MSC:

Physiological Integrity 11. The nurse informs a client suspected of pernicious anemia that the lab study that will be helpful in the diagnosis is a. clotting studies. b. hemoglobin levels. c. endoscopy. d. Schilling test.

d The Schilling test measures the absorption of orally administered radioactive vitamin B12 (tagged with cobalt 60) before and after parenteral administration of intrinsic factor. This procedure detects lack of intrinsic factor and is the definitive test for pernicious anemia. DIF: Cognitive Level: Knowledge REF: Text Reference: 2290 TOP: Nursing Process Step: Intervention MSC:

Physiological Integrity 18. The nurse discovers a client is taking the herb St. John's wort. The nurse cautions that this herb reduces the effectiveness of a. prednisone. b. theophylline. c. lanoxin. d. warfarin.

d The anticoagulation properties of warfarin are diminished if taken in conjunction with St. John's wort. DIF: Cognitive Level: Application REF: Text Reference: 2257 TOP: Nursing Process Step: Intervention MSC:

Physiological Integrity 3. The nurse points out that nursing management of all individuals with anemia is primarily directed toward a. genetic counseling. b. identifying complications. c. rehabilitative measures. d. managing manifestations.

d The goals of care for clients with anemia include (1) alleviating or controlling the causes, (2) relieving the manifestations, and (3) preventing complications. DIF: Cognitive Level: Comprehension REF: Text Reference: 2273 TOP: Nursing Process Step: Intervention MSC:

Physiological Integrity 14. The statement made by a client with pernicious anemia that would indicate to the nurse a need for further teaching is a. "I promise to have a checkup every 6 months." b. "I'm glad my nervous problems will not get worse." c. "Monthly injections are not so bad." d. "I need physical therapy to get rid of my palpitations."

d The response to injections is quick and dramatic. By the end of the first week the total RBC count rises significantly. Cardiovascular involvement usually lessens with improved erythropoiesis. Peripheral nerve function may improve with treatment. DIF: Cognitive Level: Analysis REF: Text Reference: 2290 TOP: Nursing Process Step: Evaluation MSC:

An adolescent client with type I diabetes mellitus is admitted to the emergency department for treatment of diabetic ketoacidosis. Which assessment findings should the nurse expect to note? "a) sweating and tremors b) hunger and hypertension c) cold, clammy skin and irritability d) fruity breath and decreasing level of consciousness

d) fruity breath and decreasing level of consciousness"Hyperglycemia occurs with diabetic ketoacidosis. Signs of hyperglycemia include fruity breath and a decreasing level of consciousness. Hunger can be a sign of hypoglycemia or hyperglycemia, but hypertension is not a sign of diabetic ketoacidosis. Instead, hypotension occurs because of a decrease in blood volume related to the dehydrated state that occurs during diabetic ketoacidosis. Cold, clammy skin, irritability, sweating, and tremors are all signs of hypoglycemia."

Integrated Process: Teaching/Learning 2. Place the steps for obtaining a peak expiratory flow rate in the order in which they should occur. a. Take as deep a breath as possible. b. Stand up (unless you have a physical disability). c. Place the meter in your mouth, and close your lips around the mouthpiece. d. Make sure the device reads zero or is at base level. e. Blow out as hard and as fast as possible for 1 to 2 seconds. f. Write down the value obtained. g. Repeat the process two additional times, and record the highest number in your chart.

d, b, a, c, e, f, g The proper order for obtaining a peak expiratory flow rate is as follows: Make sure the device reads zero or is at base level. Stand up (unless you have a physical disability). Take as deep a breath as possible. Place the meter in your mouth, and close your lips around the mouthpiece. Blow out as hard and as fast as possible for 1 to 2 seconds. Write down the value obtained. Repeat the process two more times, and record the highest of the three numbers in your chart. DIF: Cognitive Level: Comprehension/Understanding REF: Chart 32-4, p. 605 TOP: Client Needs Category: Physiological Integrity (Pharmacological and Parenteral Therapies—Medication Administration)

Proliferative retinopathy is often treated using: a. Tonometry b. Fluorescein angiogram c. Antibiotics d. Laser surgery

d. Laser surgery Scatter laser treatment is used to shrink abnormal blood vessels in an effort to preserve vision. When there is significant bleeding in the eye, it is removed in a procedure known as vitrectomy. Tonometry is a diagnostic test that measures pressure inside the eye. A fluorescein angiogram is a diagnostic test that traces the flow of dye through the blood vessels in the retina; it is used to detect macular edema.

The nurse is performing discharge teaching for a patient with Addison's disease. It is MOST important for the nurse to instruct the patient about: a. signs and symptoms of infection b. fluid and electrolyte balance c. seizure precautions d. steroid replacement

d. steroid replacement steroid replacement is the most important information the client needs to know. Steroid replacement can cause the patient to develop diabetes

EF in systolic HF

decreases, <45% normally 55-60%

Myocardial Infarction

heart attack. disruption of blood supply to heart from emboli,thrombus, shock, hemorrhage. Cause Acidosis. MI in left ventricle. S/S:radiating pain down left arm,neck, jaw, not relieved by nitroglycerine or rest. ECG test. Elevated Creatine Kinase.

heart failure definition

inadequate pumping and/or filling of the heart -perfusion insufficient to meet O2 needs of tissues

Osmotic Diuretics

mannitol (Mannitol, Osmitrol, Resectisol)- increase osmotic pressure, decrease absorption of sodium- Check potassium level and give in Morning.

Beta-Adrenergic Blockers

propanolol (Inderal), metopolol (Lopressor), nadolol (Corgard) - decrease blood pressure- rise slowly, report signs of edema.


Kaugnay na mga set ng pag-aaral

日本語三・ユニット三・単語二(新しい動詞)

View Set

Cognitive Psychology Exam 3 Chapters 10-13

View Set

Module 2 Content Activities (3370)

View Set

Unit 4: Hola, ¡estoy bien! (Hi, I'm well!)

View Set

Unit Test: Periodic Table Locations+Characteristics (Halogens, Metals, Alkali Metals, etc.)

View Set

Managerial vs. Financial Accounting

View Set

Module 5 : Do We Think Like Computers?

View Set

Marketing (Chapter 7) Market Segmentation

View Set